Sunteți pe pagina 1din 719

IV 1 2 3 4 5 6 7 8 9 10 11 12 13 14 15 16 17 18 19

1 X X X X X X X X X X X
2 X X X X X X X X
3 X X X X X X X
4 X X X X
5 X X X X
6 X X X X
7 X X X X X X X X X X X
8 X X X X X X X X

1
I.1.1
Identify and sketch the set of points satisfying.
(a) jz 1 ij = 1 (f) 0 < Im z <
(b) 1 < j2z 6j < 2 (g) < Re z <
(c) jz 1j2 + jz + 1j2 < 8 (h) jRe zj < jzj
(d) jz 1j + jz + 1j 2 (i) Re (iz + 2) > 0
(e) jz 1j < jzj (j) jz ij2 + jz + ij2 < 2

Solution
Let z = x + iy, where x; y 2 R.
(a) Circle, centre 1 + i, radius 1.

jz 1 ij = 1 , j(x 1) + i (y 1)j = 1 , (x 1)2 + (y 1)2 = 12

(b) Annulus with centre 3, inner radius 1=2, outer radius 1.

1 < j2z 6j < 2 , 1 < 2 jz 3j < 2 ,


, 1=2 < jz 3j < 1 , (1=2)2 < (x 3)2 + y 2 < 12
p
(c) Disk, centre 0, radius 3.

jx + iy 1j2 + jx + iy + 1j2 < 8 ,


p 2
, (x 1)2 + y 2 + (x + 1)2 + y 2 < 8 , x2 + y 2 < 3

(d) Interval [ 1; 1].

q q
2
jz 1j + jz + 1j 2 , (x 1) + y2 2 (x + 1)2 + y 2 ,
q 2 q 2
2 2
, (x 1) + y2 2 (x + 1) + y 2 ,
q q 2
, (x + 1)2 + y 2 x+1, (x + 1) +2
y2 (x + 1)2 , y = 0

Now, take y = 0 in the inequality, and compute the three intervals

2
x < 1; then jx 1j + jx + 1j = (x 1) (x + 1) = 2x 2;
1 x 1 then jx 1j + jx + 1j = (x 1) + (x + 1) = 2 2
x > 1; then jx 1j + jx + 1j = (x 1) + (x + 1) = 2x 2:

(e) Halfplane x > 1=2.

jz 1j < jzj , jz 1j2 < jzj2 , jx + iy 1j2 < jx + iyj2 ,


, (x 1)2 + y 2 < x2 + y 2 , x > 1=2

(f) Horizontal strip, 0 < y < .


(g) Vertical strip, <x< .
(h) CnR.

jRe zj < jzj , jRe (x + iy)j2 < jx + iyj2 , x2 < x2 + y 2 , jyj > 0
(i) Half plane y < 2.

Re (iz + 2) > 0 , Re (i (x + iy) + 2) > 0 , y+2>0,y <2


(j) Empty set.

jz ij2 + jz + ij2 < 2 , jx + iy ij2 + jx + iy + ij2 < 2 ,


, x2 + (y 1)2 + x2 + (y + 1)2 < 2 , x2 + y 2 < 0

3
I.1.2
Verify from the denitions each of the identities
(a) z + w = z + w (b) zw = z w (c) jzj = jzj (d) jzj2 = z z
Draw sketches to illustrate (a) and (c).

Solution
Substitute z = x + iy and w = u + iv, and use the denitions.
(a)

z + w = (x + iy) + (u + iv) = (x + u) + (y + v) i =
= (x + u) (y + v) i = (x iy) + (u iv) = z + w:

(b)

zw = (x + iy) (u + iv) = (xu yv) + (xv + yu) i =


= (xu yv) (xv + yu) i = (x iy) (u iv) = z w:

(c)

q p
jzj = x + iy = jx iyj = x2 + ( y)2 = x2 + y 2 = jx + iyj = jzj :

(d)

p 2
jzj2 = jx + iyj2 = x2 + y 2 = x2 + y 2 =
= x2 i2 y 2 = (x + iy) (x iy) = z z:

4
I.1.3
Show that the equation jzj2 2 Re (az) + jaj2 = 2
represents a circle
centered at a with radius .

Solution
Let z = x + iy and a = + i , we have

jzj2 2 Re (az) + jaj2 =


2
= x2 + y 2 2 Re (( i ) (x + iy)) + 2
+ =
2 2 2 2
=x +y 2 Re (( x + y) + i ( y x)) + + =
2 2 2 2
=x +y 2 ( x + y) + + =
= (x )2 + (y )2 :

Thus the equation jzj2 2 Re (az) + jaj2 = 2


, becomes

(x )2 + (y )2 = 2
;
which is the equation for a circle of radius centered at ( ; ), which in
complex notation is the point a = + i .

5
I.1.4
Show that jzj jRe zj + jIm zj, and sketch the set of points for which
equality holds.

Solution
Apply triangle inequality to z = Re z + i Im z, to obtain jzj 6 jRe zj + jIm zj.
Now set z = x + iy, and see then equality holds

p p p p 2 p p 2
jzj = jRe zj+jIm zj , x2 + y 2 = x2 + y 2 , x2 + y 2 = x2 + y 2 ,
p 2 p 2 p p
, x2 +y 2 = x2 + x2 +2 x2 y 2 , x2 y 2 = 0 , x = 0 or y = 0:

Equality holds only when z is real or pure imaginary, which are all the points
on the real and imaginary axis.

6
I.1.5
Show that

jRe zj jzj ; jIm zj jzj :


Show that

jz + wj2 = jzj2 + jwj2 + 2 Re (z w) :


Use this to prove the triangle inequality jz + wj jzj + jwj.

Solution
Let z = x + iy. Then since the square root function is monotone, we have
p p
jRe zj = jxj = x2 6 x2 + y 2 = jzj ;
p p
jIm zj = jyj = y 2 6 x2 + y 2 = jzj :

Now for z; w we have

jz + wj2 =
= (z + w) (z + w) = (z + w) (z + w) = z z + z w + wz + ww =
= jzj2 + 2 Re (z w) + jwj2 ;

where we have used the fact that 2 Re (z w) = z w +z w = z w + wz = z w +wz.


We use both of the above facts and the trivial identities jzj = jzj and jzwj =
jzj jwj to prove the triangle inequality for z; w. We have

jz + wj2 = jzj2 + 2 Re (z w) + jwj2 jzj2 + 2 jRe (z w)j + jwj2


jzj2 + 2 jz wj + jwj2 = jzj2 + 2 jzj jwj + jwj2 = (jzj + jwj)2 :

The desired inequality now follows by taking square root of both sides.

7
I.1.6
For xed a 2 C, show that jz aj = j1 azj = 1 if jzj = 1 and 1 az 6= 0.

Solution
If jzj = 1, then jzj = 1 and z z = 1. Use this and get

jz aj = jz aj jzj = jz z azj = j1 azj = j1 azj :


We have

jz aj
= 1;
j1 azj
as was to be shown.

8
I.1.7
Fix > 0, = 6 1, and x z0 ; z1 2 C. Show that the set of z satisfying
jz z0 j = jz z1 j is a circle. Sketch it for = 12 and = 2, with
z0 = 0 and z1 = 1. What happens when = 1?

Solution
Recall that a circle in R2 centered at (a; b) with radius r is given by the
equation

(x a)2 + (y b)2 = r2 :
We manipulate the equation

jz z0 j = jz z1 j :
The solutions set of the equation above remains the same if we square both
sides,

jz z0 j2 = 2
jz z1 j2 :
Let z = x + iy, z0 = x0 + iy0 and z1 = x1 + iy1 . Thus our equation becomes

(x x0 )2 + (y y0 )2 = 2
(x x1 )2 + (y y1 )2 :
Expanding the squares, and grouping terms, we have

2
1 x2 2 x0 2
x1 x+ x20 2 2
x1 + 1 2
y 2 2 y0 2
y1 y+ y02 2 2
y1 =0
2
Dividing both sides by (1 ), we have

2
(x0 x1 ) (x20 2 2
x1 ) (y0 2
y1 ) (y02 2 2
y1 )
x2 2 2
x+ 2
+ y2 2 2
y+ 2
=0
1 1 1 1
Now complete the squares for both the x and y terms. Recall that

x2 2ax + b = (x a)2 a2 + b:
So we have

9
2 2 2 2
x0 x1 (1 ) (x20 2 2
x1 ) (x0 2
x1 )
x + +
1 2
(1 2 )2

2 2 2 2
y0 y1 (1 ) (y02 2 2
y1 ) (y0 2
y1 )
+ y + = 0:
1 2
(1 2 )2

This becomes

2 2 2 2
x0 x1 y0 y1
x 2
+ y 2
=
1 1
2 2 2 2 2
(x0 x1 ) + (y0 y1 ) (1 ) (x20 + y02 2
(x21 + y12 ))
= =
(1 2 )2

2
(x1 x0 )2 + (y1 y0 )2
= ;
(1 2 )2

which is the equation for a circle. If z0 = 0, and z1 = 1, we have


2 2 2
2
x 2
+y = 2
:
1 1
When = 21 we have a circle of radius 23 centered at 1
3
; 0 , and when = 2,
2 4
we have a circle of radius 3 centered at 3 ; 0 . When = 1, we have the
equation

jz z0 j = jz z1 j ;
which is the line bisecting the two points. When z0 = 0; z1 = 1, this is the
line Re z = 21 .

10
I.1.7 I.1.7 I.1.7
2 2 2
Im z Im z Im z
1 1 1

-1 1 2 -1 1 2 -1 1 2
-1
Re z -1
Re z -1
Re z

-2 -2 -2

1
= 2
=1 =2

11
I.1.8
Let p (z) be a polynomial of degree n 1 and let z0 2 C. Show
that there is a polynomial h (z) of degree n 1 such that p (z) =
(z z0 ) h (z)+p (z0 ). In particular, if p (z0 ) = 0, then p (z) = (z z0 ) h (z).

Solution
Set

p (z) = an z n + an 1 z n 1
+ an 2 z n 2
+ + a2 z 2 + a1 z + a0 :
and

h (z) = bn 1 z n 1
+ bn 2 z n 2
+ bn 3 z n 3
+ : : : + b2 z 2 + b1 z + b0 :
We equate coe cients in the polynomial identity p (z) = (z z0 ) h (z) +
p (z0 ), and get

an z n + an 1 z n 1
+ an 2 z n 2
+ + a2 z 2 + a1 z + a0 =
= (z z0 ) bn 1 z n 1 + bn 2 z n 2 + bn 3 z n 3 + : : : + b2 z 2 + b1 z + b0 +p (z0 ) =
= bn 1 z n + bn 2 z n 1 + bn 3 z n 2 + : : : + b2 z 3 + b1 z 2 + b0 z + p (z0 )
bn 1 z0 z n 1 bn 2 z0 z n 2 bn 3 z0 z n 3 : : : b2 z0 z 2 b1 z0 z b0 z0 =
bn 1 z n +(bn 2 bn 1 z0 ) z n 1 +(bn 3 bn 2 z0 ) z n 2 +(bn 4 bn 3 z0 ) z n 3 +
+ (b2 b3 z0 ) z 3 + (b1 b2 z0 ) z 2 + (b0 b1 z0 ) z + p (z0 ) b0 z0 :

Equate and solve for the bj s in terms of aj s.

8
8 >
> b n 1 = an
>
> nP k 1
< an = b n 1 <
bk = ak+1+i z0i ; 0 k n 2
ak = bk 1 bk z0 ; 0 k n 1 ) i=0
: >
> Pn
a0 = p (z0 ) b0 z0 >
>
: p (z0 ) = ai z0i
i=0

Proof by induction on degree n of p (z), set

p (z) = an z n + an 1 z n 1
: : : + a0 ;
where an 6= 0.

12
Fix z0 and write

p (z) = an (z z0 ) z n 1
+ r (z) ;
where deg r (z) n 1.
By using the induction hypothesis, we can assume that

r (z) = q (z) (z z0 ) + c;
where deg q (z) deg r (z). Then

p (z) = an z n 1
+ q (z) (z z0 ) + c = h (z) (z z0 ) + c
Since deg q (z) n 2, deg r (z) n 1.
Plug in z0 , get p (z0 ) = c.

13
I.1.9
Find the polynomial h (z) in the preceding exercise for the following
choices of p (z) and z0
(a) p (z) = z 2 and z0 = i
(b) p (z) = z 3 + z 2 + z and z0 = 1
(c) p (z) = 1 + z + z 2 + + z m and z0 = 1

Solution
From the preceding exercise we have

p (z) = (z z0 ) h (z) + p (z0 ) :


We solve for h (z) then

p (z) p (z0 )
h (z) = :
z z0
(a)
We have that p (z) = z 2 and z0 = i, thus p (z0 ) = p (i) = 1.
Thus

p (z) p (z0 ) z2 + 1
h (z) = = = z + i;
z z0 z i
and

z 2 = (z i) (z + i) 1:
(b)
We have that p (z) = z 3 + z 2 + z and z0 = 1, thus p (z0 ) = p ( 1) = 1.
Thus

p (z) p (z0 ) z3 + z2 + z + 1 (z + 1) (z 2 + 1)
h (z) = = = = z 2 + 1;
z z0 z+1 z+1
and

z 3 + z 2 + z = (z + 1) z 2 + 1 1:
(c)
We have that p (z) = 1 + z + z 2 + + z m and z0 = 1, thus

14
0; m odd
p (z0 ) = p ( 1) =
1; m even
Thus

p (z) p (z0 )
h (z) = =
8 z z0
< (zm 1 +z m 3 +z 2 +1)(z+1)
= zm 1
+ zm 3
+ + z2 + 1 if m is odd
= z+1
: (zm 1 +z m 3 +z 3 +z )(z+1)+1
= zm 1
+ zm 3
+ : : : + z 3 + z if m is even
z+1

and

zm + zm 1
+ z2 + z + 1 =
(z + 1) (z m 1
+ zm 3
+ + z 2 + 1) if m is odd
=
(z + 1) (z m 1
+ zm 3
+ 3
+ z + z) + 1 if m is even

15
I.1.10
Let q (z) be a polynomial of degree m 1. Show that any polynomial
p (z) can be expressed in the form

p (z) = h (z) q (z) + r (z) ;


where h (z) and r (z) are polynomials and the degree of the remain-
der r (z) is strictly less than m.
Hint. Proceed by induction on the degree of p (z). The resulting
method is called the division algorithm.

Solution
First suppose p (z) is the zero polynomial. (So, the degree of p (z) is 1.)
The degree of r (z) must be less than the degree of q (z). If h (z) 6= 0, it
follows that the degree of h (z) q (z) is greater than the degree of r (z). This
then implies that h (z) q (z)+r (z) 6= 0. So, h (z) q (z)+r (z) = 0 implies that
h (z) = 0, and thus r (z) = 0. So, the polynomials are h (z) = 0 and r (z) = 0,
and these polynomials are the only ones that satisfy both conditions.

Now assume that the division algorithm is true for all polynomials p (z) of
degree less than n. (Where n 0.) If the degree of q (z) is greater than the
degree of p (z), and h (z) is nonzero, then h (z) q (z) + r (z) has degree greater
than p (z). So, if the degree of q (z) is greater than the degree of p (z), then
h (z) = 0 and thus r (z) = p (z). This proves both existence and uniqueness
of h (z) and r (z), in this case.

Now, suppose that the degree of q (z) is less than or equal to the degree of
p (z). Set

p (z) = an z n + an 1 z n 1
+ + a1 z + a0
and

q (z) = bm z m + bm 1 z m 1
+ + b1 z + b0 ;
where an 6= 0, bm 6= 0 and m n.
Let
an n m
p1 (z) = z q (z) p (z) ;
bm

16
then

an n m
p1 (z) = z bm z m + bm 1 z m 1
+ + b1 z + b0 an z n + an 1 z n 1
+ + a1 z + a0 :
bm
The monomials of degree n cancel, and therefore p1 (z) is a polynomial of de-
gree at most n 1. It follows, by assumption, that p1 (z) = h1 (z) q (z)+r1 (z),
where h1 (z) and r1 (z) are the unique polynomials satisfying the conditions
above.
Let
an n m
h (z) = z h1 (z) ;
bm
and let

r (z) = r1 (z) :
Then

h (z) q (z) + r (z) =


an n m
= z h1 (z) q (z) r1 (z) =
bm
an n m
= z q (z) (h1 (z) q (z) + r1 (z)) =
bm
an n m
= z q (z) p1 (z) =
bm
= p (z) :

Thus given p (z) and q (z), there exist polynomials h (z) and r (z) satisfying
the above two conditions.

17
I.1.11
Find the polynomials h (z) and r (z) in the preceding exercise for
p (z) = z n and q (z) = z 2 1.

Solution
Require

z n = h (z) z 2 1 + r (z) ; deg r (z) 1:


If n is even

zn = zn 2
+ zn 4
+ zn 6
+ + z2 + 1 z2 1 + 1:
If n is odd

zn = zn 2
+ zn 4
+ zn 6
+ + z3 + z z2 1 + z:
Thus

zn 2
+ zn 4
+ zn 6
+ + z 2 + 1; if n is even,
h (z) = ;
zn 2
+ zn 4
+ zn 6
+ + z 3 + z; if n is odd.
and

1; if n is even,
r (z) = :
z; if n is odd.

18
I.2.1
Express all values of the following expressions in both polar and
Cartesian
p coordinates,
p and plot them.
(a) i (c) 4
1 (e) ( 8)1=3 (g) (1 + i)8
p p 25
(b) i 1 (d) 4 i (f) (3 4i)1=8 (h) 1+i
p
2

Solution
(a)

p n o n p o
i( =2+2k ) 1=2 i( =4+k )
i= e =e ; k = 0; 1 = ei =4 i5 =4
;e = (1 + i) = 2 :

(b)

p p 1=2
i 1 = 2ei(3 =4+2k )
= 21=4 ei(3 =8+k )
; k = 0; 1 =

= 21=4 ei3 =8
; 21=4 ei11 =8
= 21=4 (cos (3 =8) + i sin (3 =8)) :

(c)

p n o
+2k ) 1=4
4
1 = ei( = ei( =4+k =2)
; k = 0; 1; 2; 3 =
n p p o
i =4 i3 i=4 i5 =4 i7 =4
= e ;e ;e ;e = (1 i)= 2; ( 1 i) = 2 :

(d)
p n o
4 =2+2k ) 1=4 i( =8+k =2)
i= ei( e ; k = 0; 1; 2; 3 = ei =8
; ei5 =8
; ei9 =8
; ei13 =8
=
= f (cos ( =8) + i sin ( =8)) ; (cos (5 =8) + i sin (5 =8))g :

(e)
n o
+2k ) 1=3
( 8)1=3 = 23 ei( = 2ei( =3+2k =3)
; k = 0; 1; 2 =
n p p o
= 2ei =3
; 2ei ; 2ei5 =3
= 1 + i 3; 2; 1 i 3 :

(f)

19
1 4
Draw gure and get tan 0 = 4=3 ) 0 = tan 3
.

n o
) 1=8
(3 4i)1=8 = 5ei( 0 +2k
= 51=8 ei( 0 =8+k =4)
; k = 0; 1; : : : ; 7 =
= 51=8 ei( 0 =8)
; 51=8 ei( 0 =8+ =4)
; 51=8 ei( 0 =8+ =2)
; 51=8 ei( 0 =8+3 =4)
=
1=8 1=8
5 (cos ( 0 =8) + i sin ( 0 =8)) ; 5 (cos ( 0 =8 + =4) + i sin ( 0 =8 + =4)) ;
51=8 (cos ( 0 =8 + =2) + i sin ( 0 =8 + =2)) ; 51=8 (cos ( 0 =8 + 3 =4) + i sin ( 0 =8 + 3 =4)) :
(g)
p 8
(1 + i)8 = 2ei( =4+2k )
= 16ei(2 +16k )
= 16ei2 = 16:
(h)

25
1+i =4+2k ) 25 1+i
p = ei( = ei(25 =4+50k )
= ei( =4+6 +50k )
= ei =4
= p :
2 2

I.2.1a I.2.1b I.2.1c

1 1 1

-1 1 -1 1 -1 1

-1 -1 -1

I.2.1d I.2.1e I.2.1f


2
1 1

-1 1 -2 2 -1 1

-1 -1
-2

20
I.2.1g I.2.1h
20
1

-20 20 -1 1

-1
-20

21
I.2.2
Sketch the following sets
(a) jarg zj < =4 (c) jzj = arg z
(b) 0 < arg (z 1 i) < =3 (d) log jzj = 2 arg z

Solution
a) Sector. b) Sector. c) Is a spiral curve starting at 0, spiraling to 1. (d) Is
a spiral curve, spiraling to 0, and to 1.

I.1.2a I.1.2b I.2.2c


5 5

/3 (/2,0)
/4
/4 (,0) (2,0)
-5 5 -5 5

(3/2,0)

-5 -5

I.2.2d (-3 < x < 3) I.2.2d (-30 < x < 30) I.2.2d (-800 < x < 800)

(1,0)

22
I.2.3
For a xed complex number b, sketch the curve ei + be i : 0 2 .
Dierentiate between the cases jbj < 1; jbj = 1 and jbj < 1.
Hint. First consider the case b > 0, and then reduce the general
case to this case by a rotation.

Solution
For 0 < b < 1, an ellipse x2 = (1 + b)2 + y 2 = (1 b)2 = 1, traversed in positive
direction with increasing . For b = 1, an interval [ 2; 2]. For 1 < b < +1,
an ellipse traversed in negative direction. For b = ei' , express equation as
ei'=2 ei( '=2) + e i( '=2) to see that curve is rotate of ellipse or interval
by '=2.

I.2.7 (b = 0.5) I.2.7 (b = 1) I.2.7 (b = 1.5)


3 3 3
y y y
2 2 2
1 1 1

-3 -2 -1 1 2 3 -3 -2 -1 1 2 3 -3 -2 -1 1 2 3
-1 x -1 x -1 x
-2 -2 -2
-3 -3 -3

23
I.2.4
For which n is i an n-th root of unity?

Solution.
i is an nth root of unity for in = 1, n = 4k, k = 1; 2; 3; : : :

24
I.2.5
For n 1, show that
(a) 1 + z + z 2 + + z n = (1 z n+1 ) = (1 z), z 6= 1
1
sin n+
1 2
(b) 1 + cos + cos 2 + + cos n = 2
+ 2 sin =2
.

Solution
(a)
Set
sn = 1 + z + z 2 + + zn;
and multiply sn with z and have

zsn = z + z 2 + z 3 + + z n+1 :
Now subtract zsn from the sum sn , and have

sn (1 z) = 1 z n+1 :
If z 6= 1 we have after division by 1 z;

1 z n+1
sn = :
1 z
(b)
Apply (a) to z = ei and to z = e i
;

1ei(n+1)
1 + ei + ei2 + + ein = ;
1 ei
i i2 in 1 e i(n+1)
1+e +e + +e = :
1 e i
Add the identities, and use the denitions of sine and cosine.

25
2 (1 + cos + cos 2 + + cos n ) =
1ei(n+1) 1 e i(n+1)
= + =
1 ei 1 e i
1 ei(n+1) e i =2 1 e i(n+1) ei =2
= + =
e i =2 ei =2 ei =2 e i =2
1 e i =2 1 ei(n+1) 1 ei =2 1 e i(n+1)
= + =
2i sin ( =2) 2i sin ( =2)
" 1 1
#
1 ei =2 e i =2 + ei(n+ 2 ) e i(n+ 2 ) sin ( =2) + sin n + 1
2
= = =
2i sin ( =2) sin ( =2)
sin n + 12
=1+ :
sin ( =2)

Divide both sides with 2 and get

1 sin n + 21
1 + cos + cos 2 + + cos n = + :
2 2 sin =2

26
I.2.6
Fix n 1. Show that the n th roots of unity w0 ; : : : ; wn 1 satisfy
(a) (z w0 ) (z w1 ) : : : (z wn 1 ) = z n 1:
(b) w0 + + wn 1 = 0 if n 2:
(c) w0 wn 1 = ( 1)n 1 :
nP1
0; 1 k n 1;
(d) wjk =
j=0 n; k = n:

Solution
(a)
Let w0 ; : : : ; wn 1 be the n th roots of unity then wj = e2 ji=n
, and wj are
a roots of z n 1 since
ji=n n
wjn 1 = e2 1 = e2 ji
1 = 0:

By the fundamental theorem of algebra, and since the root are simple and
the coe cient for the z n term is 1 it follows that

zn 1 = (z w0 ) (z w1 ) (z wn 1 ) :
(b)
Using exercise (a) and multiply the factors in the product we get

zn 1 = zn (w0 + w1 + wn 1 ) z n 1
+c2 z n 2
+ +( 1)n w0 w1 wn 1 :

By identication of the coe cients we see that

w0 + w1 + + wn 1 =0 if n 2:
(c)
From (b) follows that

1 = ( 1)n w0 w1 wn 1 , w0 w1 wn 1 = ( 1)n 1
:
(d)
If 1 k n 1

n
X
n 1 X
n 1 X
n 1
1 e2 ki=n 1 e2 ki
2 ji=n k 2 ki=n j
wjk = e = e = = = 0:
j=0 j=0 j=0
1 e2 ki=n 1 e2 ki=n

27
If k = n

X
n 1 X
n 1 X
n 1 X
n 1
2 ji=n n
wjk = e = 2 ji
e = 1 = n:
j=0 j=0 j=0 j=0

28
I.2.7
Fix R > 1 and n 1, m 0. Show that

zm Rm
; jzj = R:
zn + 1 Rn 1
Sketch the set where equality holds. Hint. See (1.1) p.2.

Solution

I.2.7 I.2.7 (n = 1) I.2.7 (n = 2)

1 1

a+b b
-1 1 -1 1
a
-1 -1

I.2.7 (n = 3) I.2.7 (n = 4) I.2.7 (n = 5)

1 1 1

-1 1 -1 1 -1 1

-1 -1 -1

We use that jz wj jzj jwj see (1.1) on page 2 in CA, and have that

jz n + 1j > jz n j 1 = Rn 1;
where the last equaltity is due to that jzj = R.
Some rearrangement gives
1 1
;
jz n + 1j (Rn 1)

29
and mulitplication by jz m j = Rm gives

zm Rm
; jzj = R:
zn + 1 Rn 1
For equality, we must have

jz n + 1j = Rn 1;
because jzjm = Rm .
We rearrarange this equality to

j 1j + jz n + 1j = jz n j

and we conclude that 1 and z n +1 must lie on the same ray. (We have used
the fact that jaj+jbj = ja + bj implies that a; b lie on the same ray, see rst gure I.2.7)
If

z = Rei
then

z n = Rn ein :
Since R > 1, we require that ein = 1 thus

i=n
z = wk Re ;
where wk is an n th root of unity.
If

i=n
z = wk Re ;
then

zn + 1 = Rn + 1;
and

zm Rm
= :
zn + 1 Rn 1

30
I.2.8
Show that cos 2 = cos2 sin2 and sin 2 = 2 cos sin using de
Moivres formulae. Find formulae for cos 4 and sin 4 in terms of
cos and sin .

Solution
Let 2 R be given. Then by de Moivres formulae (on page 8 in CA) we
have for all n 2 Z

cos n + i sin n = (cos + i sin )n


Hence for n = 2 we get

cos 2 + i sin 2 =
= (cos + i sin )2 = cos2 + i2 cos sin sin2 =
= cos2 sin2 + i (2 cos sin ) ;

then, by setting the real and imaginary parts equal to each other we obtain

cos 2 = cos2 sin2


sin 2 = 2 cos sin :

Similarly, applying de Moivres formulae for n = 4 we get (using the Binomial


Theorem)

cos 4 + i sin 4 = (cos + i sin )4 =


= cos4 + i4 cos3 sin 6 cos2 sin2 i4 cos sin3 + sin4 =
= cos4 6 cos2 sin2 + sin4 + i 4 cos3 sin 4 cos sin3 ;

then, by setting the real and imaginary parts equal to each other we obtain

cos 4 = cos4 6 cos2 sin2 + sin4


sin 4 = 4 cos3 sin 4 cos sin3 :

as required.

31
I.3.1
Sketch the image under the spherical projection of the following
sets on the sphere
(a) the lower hemisphere Z 0
(b) the polar cap 43 Z 1
p p
(c) lines of lattitude X = 1 Z 2 cos , Y = 1 Z 2 sin , for Z xed
and 0 2 p p
(d) lines of longitude X = 1 Z 2 cos , Y = 1 Z 2 sin , for xed
and 1 Z 1
(e) the spherical cap A X 1, with center lying on the equator,
for xed A. Separate into cases, according to various ranges of A.

Solution

I.3.1a I.3.1b I.3.1c


2 4 2
y y y
1 2 1

-2 -1 1 2 -4 -2 2 4 -2 -1 1 2
-1 -2 -1

-2 -4 -2

I.3.1d I.3.1e (A = -1/2) I.3.1e (A = 0)


2 4 2
y y y
1 2 1

-2 -1 1 2 -6 -4 -2 2 -2 -1 1 2
-1 -2 -1

-2 -4 -2

32
I.3.1e (A = 1/2)
4
y
2

-2 2 4 6
-2

-4

(a)
Image is the unit disk.
(b) p
Set Y = 0 and Z = 3=4 in X 2 + Y 2 + Zp 2
= 1, we have X = 7=4. We have
the formula x = X= (1 p Z), thus x = 7. Image is the exterior of a disk,
with centre 0 and radius 7. p p
(c) Image
p is the disk, with centre
p 0 and radius 1 + z= 1 z. We have that
2
X = 1 Z cos and 2
Y = p 1 Z sin , we can take the radius to image
p
1 Z2
for = 0, thus r = 1 Z = p1+Z 1 Z
.
(d) Image is lines of longitude issuing from 0.
(e)
Case 1: 1 < A < 0 p
Image is the exterior of the disk centered at 1=A with radius 1 A2 = jAj.
Case 2: A = 0
Image is the left halfplane.
Case 3: 0 < A < 1 p
Image is a disk centered at 1=A p with radius p 1 A2 = jAj.
Set Y = 0 then Z goes from 1 A2 top 1 A2 . We have p the formula
x = X= (1 Z) thus x goes from A= 1 + 1 A = 1 2 1 A2 =A to
p p
A= 1 1 A2 = 1 + 1 A2 =A.

33
I.3.2
If the point P on the sphere corresponds to z under the stereo-
graphic projection, show that the antipodal point P on the sphere
corresponds to 1=z.

Solution
For z = x + iy the corresponding point on the sphere under transformation
given on p. 12 in CA is given by (X; Y; Z) where

2x
X = ;
jzj2 + 1
2y
Y = 2 ;
jzj + 1
jzj2 1
Z = :
jzj2 + 1
For z 6= 0, we have
1 1 x + iy x + iy
x y
= = = 2 = i 2
z x iy (x iy) (x + iy) x2 + y 2
jzj jzj
which corresponds to the point on the sphere given by (X 0 ; Y 0 ; Z 0 ) where

2x
0 jzj2 2x
X = = = X;
1 2 2
jzj + 1
z
+ 1
2y
jzj2 2y
Y0 = = = Y;
1 2 2
jzj + 1
z
+ 1
1 2
0 z
1 jzj2 1
Z = = = Z:
1 2 jzj2 + 1
z
+ 1
Hence, the map (X; Y; Z) 7 ! ( X; Y; Z) on the sphere corresponds to
z 7 ! 1=z of C.

34
I.3.3
Show that as z travers a small circle in the complex plane in the
positive (counterclockwise) direction, the corresponding point P
on the sphere traverses a small circle in the negative (clockwise)
direction with respect to someone standing at the center of the
circle and with body outside the sphere. (Thus the stereographic
projection is orientation reversing, as a map from the sphere with
orientation determined by the unit outer normal vector to the com-
plex plane with the usual orientation.)

Solution
Draw the picture. Or argue as follows. The orientation of the image circle
is the same for all circles on the sphere orientated so that N is outside the
circle. This can be seen by moving one circle continuously to the other, and
seeing that the image circles moves continuously. Thus we need to shrink it
only for the equator of the sphere oriented as indicated ( ), if the South Pole
is inside it. And the image circle is the unity and positive direction of the
unit circle ( ) is the converse. So the orientation of the image is clockwise
(negative).

35
I.3.4
Show that a rotation of the sphere of 180 about the X-axis corre-
sponds under stereographic projection to the inversion z 7 ! 1=z of
C.

Solution
For z = x + iy the corresponding point on the sphere under transformation
given on p. 12 in CA is given by (X; Y; Z) where

2x
X = ;
jzj2 + 1
2y
Y = 2 ;
jzj + 1
jzj2 1
Z = :
jzj2 + 1

For z 6= 0, we have
1 1 x iy x iy x y
= = = 2 = i
z x + iy (x + iy) (x iy) x + y2 jzj2 jzj2
which corresponds to the point on the sphere given by (X 0 ; Y 0 ; Z 0 ) where

2x
0 jzj2 2x
X = = = X;
1 2 jzj2 + 1
z
+ 1
2y
jzj2 2y
Y0 = = = Y;
1 2 2
jzj + 1
z
+ 1
1
jzj2
1 jzj2 1
Z0 = 1 = = Z:
jzj 2 + 1 jzj2 + 1

Hence, the map (X; Y; Z) 7 ! (X; Y; Z) on the sphere corresponds to


inversion z 7 ! 1=z of C. Moreover, the map (X; Y; Z) 7 ! (X; Y; Z) is
given by rotation of the sphere by 180 about the X-axis.

36
I.3.5
Suppose (x; y; 0) is the spherical projection of (X; Y; Z). Show that
the product of the distances from the north pole N to (X; Y; Z) and
from N to (x; y; 0) is 2. What is the situation when (X; Y; Z) lies on
the equator on the sphere?

Solution
The distance from from the north pole N = (0; 0; 1) to (X; Y; Z) is
q
X 2 + Y 2 + (Z 1)2 ;
X
and the distance from the north pole N = (0; 0; 1) to (x; y; 0) = ; Y ;0
1 Z 1 Z
is
s
X2 Y2
+ + 1:
(1 Z)2 (1 Z)2
The product of distances is

1=2
1=2 X2 Y2 X 2 + Y 2 + (Z 1)2
X 2 + Y 2 + (Z 1)2 + +1 = = 2:
(1 Z)2 (1 Z)2 1 Z

When (X; Y; Z) lie on the equator, the product is simply the square of the
distance from N to a point on the equator. By the Pythagorean Law, this is
1 + 1 = 2.

37
I.3.6
We dene the chordal distance d (z; w) between two points z; w 2 C
to be the length of the straight line segment joining to the points
P and Q on the unit sphere whose stereographic projections are z
and w, respectively.
(a) Show that the chordal distance is a metric, that is, it is sym-
metric, d (z; w) = d (w; z); satises the triangle inequality d (z; w)
d (z; ) + d ( ; w); and d (z; w) = 0 if and only if z = w.
(b) Show that the chordal distance from z to w is given by

2 jz wj
d (z; w) = q q ; z; w 2 C:
2 2
1 + jzj 1 + jwj
(c) What is d (z; 1)? Remark. The expression for d (z; w) shows
that innitesimal arc length corresponding to the chordal metric
is given by
2ds
d (z) = ;
1 + jzj2
where ds = jdzj is the usual Euclidean innitesimal arc length. The
innitesimal arc length d (z) determines another metric, the spher-
ical metric (z; w), on the extended complex plane. See Section
IX.3.

Solution
(a)
Follows from fact that Euclidean distance in R3 is a metric on the sphere.
(b)
Set z = x1 + iy1 ; w = x2 + iy2 , we have

jz wj2 =
= (z w) (z w) = (z w) (z w) =
2 2
= zz zw zw + ww = jzj + jwj zw zw =
2 2
= jzj + jwj (x1 + iy1 ) (x2 iy2 ) (x1 iy1 ) (x2 + iy2 ) =
= jzj + jwj2
2
2x1 x2 2y1 y2 : (1)

38
We take square of distance between P and Q

2 2 2
d (z; w)2 = (X X 0 ) + (Y Y 0 ) + (Z Z 0) =
= X 2 + Y 2 + Z 2 + X 02 + Y 02 + Z 02
2 (XX 0 + Y Y 0 + ZZ 0 ) =
!
4x1 x2 + 4y1 y2 + jzj2 1 jwj2 1
=2 2 =
jzj2 + 1 jwj2 + 1
!
jzj2 + 1 jwj2 + 1 4x1 x2 4y1 y2 jzj2 1 jwj2 1
=2 =
jzj2 + 1 jwj2 + 1
!
2jzj2 + 2 jwj2 4x1 x2 4y1 y2 (1) 4 jz wj2
=2 = :
jzj2 + 1 2
jwj + 1 2
jzj + 1 jwj2 + 1

Taking the positive square root we have

2 jz wj
d (z; w) = q q ; z; w 2 C:
1 + jzj2 1 + jwj2
(c)

2 jz wj 2 jz=w 1j 2
d (z; 1) = lim q q = lim q q =q :
w!1 2 2 w!1 2 2 2
1 + jzj 1 + jwj 1 + jzj 1= jwj + 1 1 + jzj

39
I.3.7
Consider the sphere of radius 21 in (X; Y; Z) space, resting on the
(X; Y; 0) plane, with south pole at the origin (0; 0; 0) and north pole
at (0; 0; 1). We dene a stereographic projection of the sphere onto
the complex plane as before, so that corresponding points (X; Y; Z)
and z (x; y; 0) lie on the same line through the north pole. Find
the equations for z = x + iy in terms of X; Y; Z, and the equations
for X; Y; Z in terms of z. What is the corresponding formula for the
chordal distance? Note. In this case, the equation of the sphere is
2
X 2 + Y 2 + Z 12 = 41 .

Solution
The line through P = (X; Y; Z) and N = (0; 0; 1) is on the form N +
t (P N ), and it meats the xy plane when

(x; y; 0) = (0; 0; 1) + t ((X; Y; Z) (0; 0; 1)) = (tX; tY; 1 + t (Z 1)) :

By simultaneous equations we have, and solve for X, Y and Z


8 8
< x = tX < X = xt
y = tY ) Y = yt
: :
1 + t (Z 1) = 0 Z = 1 1t
1 2 1
We solve for the t that is a point on the sphere X 2 + Y 2 + Z 2
= 4
we
have

2 2
x2 y 2 1 1 1 t t2
+ 2 + = , jzj2 + 1 = ,
t2 t 2 t 4 2 4
, jzj2 t + 1 = 0 , t = jzj2 + 1

thus we have
8 x
> X = 1+jzj
< y
2

Y = 1+jzj2
>
: Z = jzj2 :
1+jzj2

40
Set z = x1 + iy1 ; w = x2 + iy2 , we have

jz wj2 =
= (z w) (z w) = (z w) (z w) =
2 2
= zz zw zw + ww = jzj + jwj zw zw =
2 2
= jzj + jwj (x1 + iy1 ) (x2 iy2 ) (x1 iy1 ) (x2 + iy2 ) =
= jzj2 + jwj2 2 (x1 x2 + y1 y2 ) : (1)

And the coordical distance

41
d (z; w)2 = (X1 X 2 )2 + (Y1 Y 2 )2 + (Z1 Z 2 )2 =
!2
2 2
x1 x2 y1 y2 jz1 j2 jz2 j2
= + + =
1 + jzj2 1 + jwj2 1 + jzj2 1 + jwj2 1 + jzj2 1 + jwj2
2 2 2
x1 1 + jwj2 x2 1 + jzj2 + y1 1 + jwj2 y2 1 + jzj2 + jzj2 jwj2
= 2 2 =
1 + jzj2 1 + jwj2
2 2
(x21 + y12 ) 1 + jwj2 + (x22 + y22 ) 1 + jzj2 + jzj4 2 jzj2 jwj2 + jwj4
= 2 2 +
1 + jzj2 1 + jwj2
2x1 x2 1 + jwj2 1 + jzj2 2y1 y2 1 + jwj2 1 + jzj2
+ 2 2 =
1 + jzj2 1 + jwj2
2 2
jzj2 1 + jwj2 + jwj2 1 + jzj2 + jzj4 2 jzj2 jwj2 + jwj4
= 2 2 +
1 + jzj2 1 + jwj2
2 (x1 x2 + y1 y2 ) 1 + jwj2 1 + jzj2
+ 2 2 =
1 + jzj2 1 + jwj2
jzj2 + 2 jzj2 jwj2 + jzj2 jwj4 + jwj2 + 2 jwj2 jzj2 + jwj2 jzj4 + jzj4 2 jzj2 jwj2 + jwj4
= 2 2 +
1 + jzj2 1 + jwj2
2 (x1 x2 + y1 y2 ) 1 + jwj2 1 + jzj2
+ 2 2 =
1 + jzj2 1 + jwj2
jzj2 + jwj2 1 + jwj2 1 + jzj2 2 (x1 x2 + y1 y2 ) 1 + jwj2 1 + jzj2
= 2 2 =
1 + jzj2 1 + jwj2
jzj2 + jwj2 2 (x1 x2 + y1 y2 ) 1 + jwj2 1 + jzj2
= 2 2 =
1 + jzj2 1 + jwj2
jzj2 + jwj2 2 (x1 x2 + y1 y2 ) (1)
= 2 2 =
1 + jzj 1 + jwj
(1) jz wj2
= :
jzj2 + 1 jwj2 + 1

This gives

42
jzwj
d (z; w) = q q :
2 2
jzj + 1 jwj + 1

43
I.4.1
Sketch each curve and its image under w = z 2 .
(a) jz 1j = 1 (c) y = 1 (e) y 2 = x2 1; x > 0
(b) x = 1 (d) y = x + 1 (f) y = 1=x; x 6= 0

Solution

I.4.1a z-plane I.4.1b z-plane I.4.1c z-plane


y 4 y 4 y 4

2 2 2

-4 -2 2 4 -4 -2 2 4 -4 -2 2 4
-2 x -2 x -2 x
-4 -4 -4

I.4.1a w-plane I.4.1b w-plane I.4.1c w-plane


y 4 y 4 y 4

2 2 2

-4 -2 2 4 -4 -2 2 4 -4 -2 2 4
-2 x -2 x -2 x
-4 -4 -4

I.4.1d z-plane I.4.1e z-plane I.4.1f z-plane


y 4 y 4 y 4

2 2 2

-4 -2 2 4 -4 -2 2 4 -4 -2 2 4
-2 x -2 x -2 x
-4 -4 -4

44
I.4.1d w-plane I.4.1e w-plane I.4.1f w-plane
y 4 y 4 y 4

2 2 2

-4 -2 2 4 -4 -2 2 4 -4 -2 2 4
-2 x -2 x -2 x
-4 -4 -4

45
I.4.2
Sketch the image of eachpcurve in the preceding problem under the
principal branch of w = z, and also sketch, on the same grid but
in apdierent color, the image of each curve under the other branch
of z.

Solution
1=2 n 1=2
p
w = z 1=2 = jzj ei argjzj = jzj ei Argjzj+i2 = jzjei Arg z=2

I.4.2a z-plane I.4.2b z-plane I.4.2c z-plane


y 4 y 4 y 4

2 2 2

-4 -2 2 4 -4 -2 2 4 -4 -2 2 4
-2 x -2 x -2 x
-4 -4 -4

I.4.2a w-plane I.4.2b w-plane I.4.2c w-plane


y 4 y 4 y 4

2 2 2

-4 -2 2 4 -4 -2 2 4 -4 -2 2 4
-2 x -2 x -2 x
-4 -4 -4

46
I.4.2d z-plane I.4.2e z-plane I.4.2f z-plane
y 4 y 4 y 4

2 2 2

-4 -2 2 4 -4 -2 2 4 -4 -2 2 4
-2 x -2 x -2 x
-4 -4 -4

I.4.2d w-plane I.4.2e w-plane I.4.2f w-plane


y 4 y 4 y 4

2 2 2

-4 -2 2 4 -4 -2 2 4 -4 -2 2 4
-2 x -2 x -2 x
-4 -4 -4

47
I.4.3
(a) Give a brief description of the function z 7 ! w = z 3 , considered
as a mapping from the z plane to the w- plane. (Describe what
happens to w as z traverses a ray emanating from the origin, and as
z traverses a ray a circle centered at the origin.) (b) Make branch
cuts and dene explicitly three branches of the inverse mapping.
(c) Describe the construction of the Riemann surface of z 1=3 .

Solution
(a)
The function w = f (z) = z 3 . For z = rei , we have z 3 = r3 ei3 . The radical
rays at angle are mapped to rays at angle 3 , that is, arg w = 3 arg z. The
magnitude of a complex number is cubed, jwj = jzj3 . Circles, centered at the
origin of radius r, are mapped to cocentric circles with radius r3 .
(b)
We make branch cuts at ( 1; 0],

1=3 n 1=3
p
z = w1=3 = jwj ei arg w = jwj ei Arg w+i2 = ei2 n=3
jwjei Arg w=3

we choose
g (z) = z 1=3 = r1=3 ei =3 , < < .
Sheet 1 : Take f1 (z) = g (z),
Sheet 2 : Take f2 (z) = e2 i=3 g (z),
Sheet 3 : Take f3 (z) = e4 i=3 g (z).

(c) Top edge of cut on sheet 1 to bottom edge of cut on sheet 2. Top edge of
cut on sheet 2 to bottom edge of cut on sheet 3. Top edge of cut on sheet 3
to bottom edge of cut on sheet 1. The endpoints for f (z) is continuous on
surface

Spara

48
I.4.3a
I.2.1b I.2.1c

1 1

-1 1 -1 1

-1 -1

Spara

49
I.4.4
Describe how to construct thep Riemann surfaces for the 2=5
following functions
(a) w = z 1=4 ; (b) w = z i; (c) w = (z 1) :
Remark. To describe the Riemann surface of a multivalued function, begin
with one sheet for each branch of the function, make branch cuts so that the
branches are dened continuously on each sheet, and identify each edge of
a cut on one sheet to another appropriate edge so that the function values
match up continuously.

Solution
(a) Use four sheets, can make branch cuts along real axis from 1 to 0.
(b) Use two sheets, can make branch cuts along horizontal line from 1 + i
to i.
(c) Use ve sheets, can make branch cuts along real axis from 1 to 1.

50
I.5.1
Calculate and plot for ez for the following points z.
(a) 0 (c) (i 1) =3 (e) i=m; m = 1; 2; 3; : : :
(b) i + 1 (d) 37 i (f) m (i 1) m = 1; 2; 3; : : :

Solution

I.5.1e I.5.1f

1 0.2

-1 1 -0.4 -0.2 0.2

-1 -0.2

(a)

e0 = 1:
(b)

i+1
e = e i e1 = e:
(c)
p !
(i 1)3 =3 i=3 =3 1 3
e =e e =e + i = 0:351 + 0:006i:
2 2
(d)

e37 i = e36 i e i = 1:
(e)
i=m
We take the limit for the sequence e , m = 1; 2; 3; : : : as m ! 1, and
have

i=m
e ! 1;

51
as m ! 1. Because e i=m = 1 the sequence approaches its limit along a
circle with radius 1.
(f)
We take the limit for the sequence em(i 1) = e m emi , m = 1; 2; 3; : : : as
m ! 1, and have

m mi
e e ! 0;
m mi m
as m ! 1. Because je e j=e the sequence spiraling to its limit in
origo.

52
I.5.2
Sketch each of the following gures and its image under the expo-
nential map w = ez . Indicate the images of horizontal and vertical
lines in your sketch.
(a) the vertical strip 0 < Re z < 1,
(b) the horizontal strip 5 =3 < Im z < 8 =3,
(c) the rectangle 0 < x < 1, 0 < y < =4,
(d) the disk jzj =2,
(e) the disk jzj ,
(f) the disk jzj 3 =2.

Solution

I.5.2a z-plane I.5.2b z-plane I.5.2c z-plane


4 3
8

2 6 2

4
-4 -2 2 4 1
-2 2

-4 -4 -2 0 2 4 1 2 3

I.5.2a w-plane I.5.2b w-plane I.5.2c w-plane


4 4 3

2 2
2

-4 -2 2 4 -4 -2 2 4 1
-2 -2

-4 -4 1 2 3

53
I..5.2d z-plane I.5.2e z-plane I.5.2f z-plane
4 4 4

2 2 2

-4 -2 2 4 -4 -2 2 4 -4 -2 2 4
-2 -2 -2

-4 -4 -4

I..5.2d w-plane I.5.2e w-plane I.5.2f w-plane


20 100
4

2 10

-4 -2 2 4 -10 10 20 30 100
-2 -10
-4
-20 -100

54
I.5.3
Show that ez = ez .

Solution

ez = ex iy
= ex e iy
= ex (cos ( y) + i sin ( y)) = ex (cos y i sin y) =
= ex (cos y + i sin y) = ex (cos y + i sin y) = ex eiy = ex+iy = ez :

55
I.5.4
Show that the only periods of ez are the integral multiples of 2 i,
that is, if ez+ = ez for all z, then is an integer times 2 i.

Solution
Show that the only periods of ez are the integral muliples of 2 i that is, if
ez+ = ez for all z, then is an integer times 2 i.

ez = ez+ = ez e ) e = 1 ) = 2 mi

56
I.6.1
Find and plot log z for the following complex numbers z. Specify
the principal value. p
(a) 2 (b) i (c) 1 + i (d) 1 + i 3 =2

Solution
(a)
Suppose that n = 0; 1; 2; : : :

log 2 = log j2j + i Arg 2 + 2 ni = log 2 + i2 n:


(b)
Suppose that n = 0; 1; 2; : : :

log i = log jij + i Arg i + 2 ni = i =2 + i2 n:


(c)
Suppose that n = 0; 1; 2; : : :

log (1 + i) =
p
= log j1 + ij + i Arg (1 + i) + 2 ni = log 2 + i =4 + i2 n =
= log 2=2 + i =4 + i2 n:

(d)
Suppose that n = 0; 1; 2; : : :

p
1+i 3
log 2
=
p p
1+i 3 1+i 3
= log 2
+ i Arg 2
+ 2 ni =
= i=3 + i2 n:

57
I.6.1a I.6.1b I.6.1c
20 20 20

10 10 10

-1 1 2 -1 1 2 -1 1 2
-10 -10 -10

-20 -20 -20

I.6.1d
20

10

-1 1 2
-10

-20

58
I.6.2
Sketch the image under the map w = Log z of each of the following
gures.
(a) the right halv-plane Re z > 0,
(b) the half-disk jzj < 1, Re z > 0,
(c) the unit circle jzjp= 1, p
(d) the slit annulus e < jzj < e2 , z 62 ( e2 ; e),
(e) the horizontal line y = e,
(f) the vertical line x = e.

Solution
(b) We have a disk with radius less then 1, this means jzj < 1, thus log jzj
< 0 and is unbounded, therefore, it goes from 0 to 1. Since Re (z) > 0,
the polar angle is between 2 and 2 .
(d) Here, we have an annulus, the polar angle is from to , thus the argu-
ment is in this range. Since, the log function is the inverse of the exponential
map, circles in z plane are stright lines in the w plane (w = log z).
Therefore, the image p of this annulus under the log function is the rectangle,
bounded by x = log j ej = 12 and x = log je2 j = 2.

I.6.2a z-plane I.6.2b z-plane I.6.2c z-plane


3 4
2
2
2
1

-3 -2 -1 1 2 3 -2 2 -4 -2 2 4
-1
-2
-2 -2
-3 -4

59
I.6.2a w-plane I.6.2b w-plane I.6.2c w-plane
3

2 /2
1

-3 -2 -1 1 2 3
-1
/2
-2

-3

I.6.2d z-plane I.6.2e z-plane I.6.2f z-plane


8 4 4
6
4 2 2
2

-8 -6 -4 -2
-2 2 4 6 8 -4 -2 2 4 -4 -2 2 4
-4 -2 -2
-6
-8 -4 -4

I.6.2d w-plane I.6.2e w-plane I.6.2f w-plane


4 4 4

2 2 2

-4 -2 2 4 -4 -2 2 4 -4 -2 2 4
-2 -2 -2

-4 -4 -4

60
I.6.3
Dene explicitly a continuous branch of log z in the complex plane
slit along the negative imaginary axis, Cn [0; i1).

Solution

I.6.3

We have log z = log rei = log r + i . To avoid the negative imaginary axis
we chose =2 < < 3 =2. We use the branch

f rei = log r + i ; =2 < < 3 =2;


of log z.

61
I.6.4
How would you make a branch cut to dene a single-valued branch
of the function log (z + i 1)? How about log (z z0 )?

Solution
Any straight line cut from z0 to 1, in any direction, will do.

62
I.7.1
Find all values and plot them.
p (1 i)
(a) (1 + i)i (b) ( i)1+i (c) 2 1=2
(d) 1+i 3

Solution
(a)
Suppose that m; n = 0; 1; 2; : : :

(1 + i)i =
p p
= ei log(1+i) = ei(logj1+ij+i Arg(1+i)+i2 m)
= ei(log 2+i =4+i2 m)
=e 2 m
e =4 i log
e 2
= [m = n] =
p
= e2 n e =4 i log
e 2
:

(b)
Suppose that m; n = 0; 1; 2; : : :

( i)1+i =
= e(1+i) log( i)
= e(1+i)(logj ij+i Arg( i)+i2 m)
= e(1+i)( i =2+i2 m)
=
2 m =2 i( =2+2 m)
=e e e = [m = n] = e2 n e =2 i(
e =2 2 n)
=
= ie2 n e =2
:

(c)
Suppose that m = 0; 1; 2; : : :

1=2
2 =
log 2=2 (logj2j+i Arg 2+i2 m)=2 (log 2+i2 m)=2 log 2=2 i m
=e =e =e =e e =
p
= 1= 2:

(d)
Suppose that n = 0; 1; 2; : : :

63
p 1 i
1+i 3 =
p p p
i) log(1+i 3) i)(logj1+i 3j+i Arg(1+i 3)+i2 n)
= e(1 = e(1 =
= e(1 i)(log 2+i =3+i2 n)
= e2 n elog 2+ =3 i(
e log 2+ =3+2 n)
=
= e2 n elog 2+ =3 i(
e log 2+ =3)
:

64
I.7.2 h i
2i
Compute and plot log (1 + i) .

Solution
We rewrite the expression, suppose that k; m; n = 0; 1; 2; : : :

h i
log (1 + i)2i =
h i
log(1+i) 2i
= log e = log e2i log(1+i) = log e2i(logj1+ij+i Arg(1+i)+i2 k) =
h p i
= log e2i(log 2+i =4+i2 k) = log e 4 k e =2 ei log 2 = [k = m] = log e4 m e =2 i log 2
e =
= log e4 m
e =2 i log 2
e +i Arg e4 m
e =2 i log 2
e +i2 n = 4 m =2+i log 2+i2 n =
= =2 + i log 2 + 4 m + 2 in:

Thus the set is a square lattice.

I.7.2
20

10

-10 10 20
-10

-20

65
I.7.3
Sketch the image of the sector f0 < arg z < =6g under the map w =
z a for
(a) a = 32 (b) a = i (c) a = i + 2
Use only the principal branch of z a .

Solution

I.7.3a z-plane I.7.3b z-plane I.7.3c z-plane


5 5 5

-5 5 -5 5 -5 5

-5 -5 -5

I.7.3a w-plane I.7.3b w-plane I.7.3c w-plane


5 2

-5 5 -2 2

1
-5 -2

Some points on the y-axis is marked, and the point (1; 0) too.

66
I.7.4
Show that (zw)a = z a wa , where on the right we take all possible
products.

Solution
Let 2 (zw)a , and k; m; n = 0; 1; 2; : : :

=
= ea log(zw) = ea(logjzwj+i Arg(zw)+i2 n)
= ea(logjzj+logjwj+i Arg z+i Arg w+i2 m+i2 n)
=
a(logjzj+i Arg z+i2 (m+n)) a(logjwj+i Arg w) a a
=e e 2z w :

Conversely, if 2 z a wa , say

=
= ea log z ea log w = ea(logjzj+i Arg z+i2 m) a(logjwj+i Arg w+i2 n)
e =
= ea(logjzj+logjwj+i Arg z+i Arg w+i2 (m+n))
= ea(logjzwj+i Arg(zw)+i2 k+i2 (m+n))
=
a(logjzwj+i Arg(zw)+i2 (k+m+n))
=e =
= ea log(zw) 2 (zw)a

We have that (zw)a = z a wa as sets

67
I.7.5
i
Find ii . Show that is does not coincide with ii i = i 1 .

Solution
We rewrite the expression ii , suppose that k = 0; 1; 2; : : :

ii =
= ei log i = ei(logjij+i Arg(i)+i2 k)
= ei(i =2+i2 k)
=
2 k =2
=e :
i
Now we can nd ii we use that we have ii , suppose that k; m = 0; 1; 2; : : :

i
ii =
= ii
i
= ei log(i ) = ei(log(e
i 2 k =2
)) = ei(logje 2 k =2
j+i Arg(e 2 k =2
)+i2 m)
=
i( 2 k =2+i2 m) 2 m i( 2 k =2) 2 m
=e =e e = ie =
= ie2 m
:

Which not coincide with ii i = i 1


= 1=i = i which is smaller than
f ie2 m ; m = 0; 1; 2; : : :g.

68
I.7.6
Determine the phase factors of the function z a (1 z)b at the branch
points z = 0 and z = 1. What conditions on a and b guarantee that
z a (1 z)b can be dened as a (continuous) single-valued function
on Cn [0; 1]?

Solution
Phase factors is e2 ia at 0 and e2 ib at 1. Require e2 ia e2 ib = 1, or a + b = n,
where n 2 Z , to have a continuous singlevalued determination of z a (1 z)b
on Cn [0; 1].

69
I.7.7
Let x1 < x2 < < xn be n consecutive
p points on the real axis.
Describe the Riemann surface of (z x1 ) (z xn ). Show that
for n = 1 and n = 2 the surface is topologically a sphere with certain
punctures corresponding to the branch points and 1. What is it
when n = 3 or n = 4? Can you say anything for general n? (Any
compact Riemann surface is topologically a sphere with handles.
Thus a torus is topologically a sphere with one handle. For a given
n, how many handles are there, and where do they come from?)

Solution p p
For n = 1 and n = 2, the functions f1 p= z x1 and f2 = (z x1 ) (z x2 ),
and for a general n we have fn (x) = (z x1 ) (z x2 ) : : : (z xn ). To con-
stuct the Riemann Suface we use two sheets with slits [x1 ; x2 ] ; [x3 ; x4 ] ; : : :.
If n is odd, we also need a slit [xn ; +1) . Identify top edge of slit on one
sheet with bottom edge of slit on other sheet. If n = 1 and n = 2 the surface
is topologically a sphere with certain punctures corresponding to the branch
points and 1. If n = 3 or n = 4, surface is a torus. For a general even
n, fn (x) will be continuous on Cn ([x1 ; x2 ] [ [x3 ; x4 ] [ : : : [ [xn 1 ; xn ]), and
its Riemann surface will be a sphere with n2 1 holes. For a general odd n
though, fn (x) will be continuous on Cn ([x1 ; x2 ] [ [x3 ; x4 ] [ : : : [ [xn ; +1]),
its Riemann surface will be a sphere with n 2 1 holes.

70
I.7.8 p
Show that z 2 1=z can be dened as a (single-valued) continuous
function outside the unit disk, that is, for jzj > 1. Draw branch
cuts so that the function can be dened continuous o the branch
cuts. Describe the Riemann surface of the function.

Solution p
The function is z 1 1=z 3 . If jzj > 1, can use the principal value of the
square root to dene a branch of the function. There are branch points at
z = 0 and z 3 = 1, that is at 0; 1; e2 i=3 and e 2 i=3 . Make two branch cuts by
connecting any two pairs of point by curves; for instance, connect 0 to 1 by
a straight line, and the other cube roots of unity by a straight line or arc of
unit circle. The resulting two-sheeted surface with identication of cuts and
with points at innity is a torus.

71
I.7.9 q
Consider the branch of the function z (z 3 1) (z + 1)3 that is posi-
tive at z = 2. Draw branch cuts so that this branch of the function
can be dened continuously o the branch cuts. Describe the Rie-
mann surface of the function. To what value at z = 2 does this
branch return if it is continued continuously once counterclockwise
around the circle fjzj = 2g?

Solution
We have that the function is
p p
(z + 1) z 2 z (1 1=z 3 ) (1 + 1=z):
If jzj > 1, the second square root can be dened to be single-valued for
jzj > 1. The value of the function at z = 2 returns to thepnegative of their
initial value then we travers the circle jzj = 2 , because z does. We can
construct a Riemann Surface by making cuts at 1; 0; 1; e2 i=3 ; e 2 i=3 and
1. The function for the Riemann Surface is
q
(z + 1) z (z 1) (z + 1) (z e2 i=3 ) (z e 2 i=3 ):

72
I.7.10 q
Consider the branch of the function z (z 3 1) (z + 1)3 (z 1) that is
positive at z = 2. Draw branch cut so that this branch of the func-
tion can be dened continuously o the branch cuts. Describe the
Riemann surface of the function. To what value at z = 2 does this
branch return if it is continued continuously once counterclockwise
around the circle fjzj = 2g?

Solution
We have that the function is
q
z (1 1=z 3 ) (1 + 1=z)3 (1
4
1=z)
The branch that is positive for z = 2 is, it is dened continuously for jzj > 1.
Branch return to original value around circle jzj = 2. We can construct a
Riemann Surface by making cuts at 0; 1; e2 i=3 and e 2 i=3 .
The function for the Riemann Surface is
q
(z 1) (z + 1) z (z + 1) (z e2 i=3 ) (z e 2 i=3 ):

73
I.7.11 p
Find the branch points of 3 z 3 1 and describe the Riemann surface
of the function.

Solution

I.7.11a I.7.11b I.7.11c


1 1 1

-1 1 -1 1 -1 1

-1 -1 -1

We rewrite as follows
p q
3 3
z3 1= (z 1) (z e2 i=3 ) (z e 2 i=3 ):

This equation will have 3 branch points, which are the cube root of unity.
So the phase factor is
e2 i=3 . The Riemann surface is obtained ty pasting three sheets with the
corresponding branch cuts, we end up with a one hole torus.
Make two cuts, from 1 to e 2 i=3 , on each sheet. In this case the cuts share
common endpoint.
We nee three shets where f0 (z), f1 (z) = e2 i=3 f0 (z) and f2 (z) = e 2 i=3 f0 (z)
Note: Can use Riemannformula to see that surface is a torus.
Cheek by going around each little tip what phase change to, which of for
your sheet.

74
I.8.1
Establish the following addition formulae
(a) cos (z + w) = cos z cos w sin z sin w;
(b) sin (z + w) = sin z cos w + cos z sin w;
(c) cosh (z + w) = cosh z sinh w + sinh z sinh w;
(d) sinh (z + w) = sinh z cosh w + cosh z sinh w;

Solution
(a)
Using the denitions of sine and cosine functions given on page 29 in CA we
have,

cos (z + w) =
ei(z+w) + e i(z+w) 1
= = 2ei(z+w) + 2e i(z+w) =
2 4
1 i(z+w) i(z w)
= e +e + e i(z w) + e i(z+w) +
4
+ ei(z+w) ei(z w) e i(z w) + e i(z+w) =
eiz + e iz eiw + e iw eiz e iz eiw e iw
= =
2 2 2i 2i
= cos z cos w sin z sin w:

(b)
Using the addition formula (a), and using that sin z = cos (z =2) and
cos z = sin (z =2) we have,

sin (z + w) = cos z + w =
2
= cos z cos w sin z sin w =
2 2
= cos z sin w + sin z cos w = sin z cos w + cos z sin w:

(c)
Using the addition formula (a) and the formulas cos (iz) = cosh z and sin (iz) =
i sinh z given on page 30 in CA we have,

75
cosh (z + w) = cos (i (z + w)) = cos (iz) cos (iw) sin (iz) sin (iw) =
= cosh z cosh w i sinh z i sinh w = cosh z cosh w + sinh z sinh w:

(d)
Using the addition formula (b) and the formulas cos (iz) = cosh z and sin (iz) =
i sinh z given on page 30 in CA we have,

sinh (z + w) = i sin (i (z + w)) = i [sin (iz) cos (iw) + cos (iz) sin (iw)] =
= i [i sinh z cosh w + cosh z i sinh w] = sinh z cosh w + cosh z sinh w:

76
I.8.2
Show that jcos zj2 = cos2 x + sinh2 y, where z = x + iy. Find all zeros
and periods of cos z.

Solution.
Use trigonometric formulas from page 29 and 30 in CA we have,

cos z = cos (x + iy) = cos x cos (iy) sin x sin (iy) = cos x cosh y i sin x sinh y:

Now take the modulus squared, and use cosh2 y = 1 + sinh2 y,

jcos zj2 = cos2 x cosh2 y + sin2 x sinh2 y =


= cos2 x 1 + sinh2 y + sin2 x sinh2 y =
= cos2 x + cos2 x + sin2 x sinh2 y = cos2 x + sinh2 y:

The identity for jcos zj2 shows that the only zeros of cos z are the zeros of
cos z on the real axis, because

cos x = 0 , x = 2 + m ; m = 0; 1; 2; : : : ;
cos z = 0 ,
sinh y = 0 , y = 0:
Translation by any period of cos z sends zeros to zeros. Thus any period
is an integral multiple of , and since odd integral multiples are not periods,
the only periods of cos z are 2 n; 1 < n < 1.

77
I.8.3
Find all zeros and periods of cosh z and sinh z.

Solution
We have that cosh z = cos (iz), thus the zeros of cosh z are at z = i =2 +im ,
m = 0; 1; 2; : : :, and the periods of cosh z are 2 mi, m = 0; 1; 2; : : :.

We have that sinh z = i sin (iz), thus the zeros of sinh z are at z = m i,
m = 0; 1; 2; : : :, and the periods of cosh z are 2 mi, m = 0; 1; 2; : : :.

78
I.8.4
Show that

1 1 1 + iz
tan z= log ;
2i 1 iz
where both sides of the identity are to be interpreted as subsets of
the complex plane. In other words, show that tan w = z if and only
if 2iw is one of the values of the logarithm featured on the right.

Solution
Set z = tan w, we have that

sin w eiw e iw
e2iw 1
z = tan w = = = :
cos w i (eiw + e iw ) i (e2iw + 1)
Solve for e2iw
1 + iz
e2iw = ;
1 iz
and take logarithm and solve for w

1 1 + iz
w= log :
2i 1 iz
1
Because z = tan w then w 2 tan z and we have the identity

1 1 1 + iz
tan z= log :
2i 1 iz

79
I.8.5
Let S denote the two slits along the imaginary axis in the complex
plane, one running from i to +i1, the other from i to i1. Show
that (1 + iz) = (1 iz) lies on the negative real axis ( 1; 0] if and
only if z 2 S. Show that the principal branch

1 1 1 + iz
Tan z= Log
2i 1 iz
maps the slit plane CnS one-to-one onto the vertical strip fjRe wj < =2g.

Solution
I.8.5 z-plane I.8.5 I.8.5

i
-1
-i

1+iz
w1 = 1 iz
w2 = Log w1

I.8.5 w-plane

/2 /2

w = 2i1 w2 = 2i1 Log 11+iz


iz
We begin to show that 1+iz1 iz
2 ( 1; 0] if and only if z 2 ( i1; i] [ [i; i1).
Set
1 + iz
= w1
1 iz
and solve for z, thus

80
1 w1
z= i
1 + w1
1 w1
As w1 goes from 1 to 1, 1+w 1
i goes from i to i1 along iR, and when
1 w1
w1 goes from 1 to 0, 1+w1 i goes from i1 to i along iR.
Since we have that
1 + iz 1 w1
= w1 ) z = i
1 iz 1 + w1
the map is one to one.
Remark that the map makes correspondence between the interval [ i: i1]
in z plane and [ 1; 1] in w1 plane, and between interval [i; i1] in z
plane and [ 1; 0] in w1 plane.
We have that w2 = Log w1 maps Cn( 1; 0) onto fjIm wj < g, thus w = w2i2
maps jIm w2 j < onto jRe wj < 2 , se gures.
We have that the function

1 1 1 + iz
Tan z= Log
2i 1 iz
maps CnS onto jRe wj < 2 , where Tan 1 z is the principal branch for
tan 1 z. The other branches are given by fn (z) = Tan 1 z + n , 1 < n <
1.

81
I.8.6
1
Describe the Riemann surface for tan z.

Solution

I.8.6 z-plane I.8.6 I.8.5


+++++++++++
++++


____

i
++++++
_ _ _ _-1
__
-i
++++
____

_ _ _ _
_______

1+iz
w1 = 1 iz
w2 = Log w1

I.8.6 w-plane
___________

+++++++++++

/2 /2

1 1 1+iz
w= w
2i 2
= 2i
Log 1 iz

We have that

1 1 1 + iz
f0 (z) = Tan z= Log ;
2i 1 iz
other branches of tan 1 z are fn (z) = f0 (z) + n , where 1 < n < 1.
Use one copy of the double slit plane S for each integer n, and dene fn (z) =
Tan 1 z + n on the nth sheet to the (n + 1) th sheet along one of the cuts,
so that fn (z) and fn+1 (z) have same value at the junction.

82
Make innite many copies of S and call them Sn . Dene fn (z) = Tan 1 z+n
on Sn . Identify "+" side of cut on Sn to " " side of cut on Sn+1 . Then
fn on Sn continuous to fn+1 on Sn+1 , and fn maps Sn onto vertical strip
n 21 < Re z < n + 12 . Note that composite function is not dened at
i and 1 (endpoints of slits), and its image omits the sequence 12 + n,
1 < n < 1.

83
I.8.7 p
Set w = cos z and = eiz . Show that = w w2 1. Show that
h p i
1
cos w = i log w w2 1 ;
where both sides of the identity are to be interpreted as subsets of
the complex plane.

Solution
Set w = cos z and = eiz , we have that

eiz + e iz
+ 1=
w = cos z = = :
2 2
Solve for
p
=w w2 1;
and set = eiz and take the logarithm and solve for z
p
z= i log w w2 1 :
1
Because w = cos z then z 2 cos w we have the identity

1
p
cos w= i log w w2 1 :

84
I.8.8
Show that the vertical strip jRe (w)j < =2 is mapped by the function
z (w) = sin w one-to-one onto the complex z plane with two slits
( 1; 1] and [+1; +1) on the real axis. Show p that the inverse
function is the branch of sin 1 z = i Log iz + 1 z 2 obtained by
taking the principal value of the square root. Hint. First show
that the function 1 z 2 on the slit plane omits the negative real
axis, so that the principal value of the square root is dened and
continuous on the slit plane, with argument in the open interval
between =2 and =2.

Solution
Set w = x + iy, by formulas on page 30 in CA we have that

sin w = sin (x + iy) = sin (x) cos (iy) + cos (x) sin (iy) =
= sin x cosh (y) + i cos (x) sinh y:

If jRe wj = jxj < =2 then sin w is mapped on Cn ( 1; 1] [ [+1; +1),


because

Im (sin w) = 0 ) y = 0 ) sin w = sin (x)


and

1 < sin (x) < 1


for these x.
1
Let now z = sin w. The value of w is referred to as sin z, that is, the
complex number whose sine is z. Note that

eiw e iw
z= : (1)
2i
Now with = eiw and 1= = e iw
in (1), we have

1=
z= :
2i
Mulitplying the above by 2i and doing some rearranging, we nd that

85
2 2
2iz = 1 or 2iz 1 = 0:
With the quadratic formula, we solve this equation for and nd
p p
= iz + 1 z2 or eiw = iz + 1 z2
We now take the logarithm of both sides of the last equation and divide the
result by i to obtain
1 p
w= log iz + 1 z2
i
1
and, since w 2 sin z,

1
p
sin z= i log iz 1 z2 :

86
II 1 2 3 4 5 6 7 8 9 10 11 12 13 14 15 16 17 18 19
1 X X X X X X X X X X X X X X X X
2 X X X X X X
3 X X X X X
4 X X X X X
5 X X X X X X X X
6
7
8

1
II.1.1
Establish the following:
n 2np +5n+1
(a) lim n+1 = 1 (c) lim p = 2; p > 1
n!1 n!1 nn +3n+1
(b) lim 2n = 0 (d) lim z
= 0; z 2 C:
n!1 n +1 n!1 n!

Solution
(a)
n 1
lim = lim = 1:
n!1 n + 1 n!1 1 + 1=n

(b)

n 1=n
lim = lim = 0:
n!1 n2 + 1 n!1 1 + 1=n2

(c)

2np + 5n + 1 2 + 5n1 p
+n p
lim = lim = 2:
n!1 np + 3n + 1 n!1 1 + 3n1 p+n p

(d)
Let m be a positive integer so that m > 2 jzj and let n > m, then

zn jzjm jzjn m
0 = =
n! m! (m + 1) (m + 2) n
jzjm jzj jzj jzj jzjm jzj jzj jzj
=
m! m + 1 m + 2 n m! m m m
m m
jzj 1 1 1 jzj 1
= !0
m! 2 2 2 m! 2 mn

zn
as n ! 1, it follows by instngning that n!
! 0 as n ! 1 as was to be
shown.

2
II.1.2
For which values of z is the sequence fz n g1
n=1 bounded? For which
values of z does the sequence converge to 0?

Solution
The sequence is bounded for jzj 1, and the series converge to 0 for jzj < 1.

3
II.1.3
Show that fnn z n g converges only for z = 0.

Solution
If we choose
2
n>
jzj
then

2 < n jzj
and

jnn z n j = jn zjn > 2n ! 1


when n ! 1 for z 6= 0.

4
II.1.4
N!
Show that lim k = 1; k 0.
N !1 N (N k)!

Solution
We have that

N! N (N 1) : : : (N k + 1) 1 2 k 1
= = 1 1 ::: 1 !1
Nk (N k)! N N ::: N N N N

as N ! 1 (k is xed).

5
II.1.5
Show that the sequence
1 1 1
bn = 1 ++ + + log n; n 1;
2 3 n
is decreasing, while the sequence an = bn 1=n is increasing. Show
that the sequences both converge to the same limit . Show that
1
2
< < 35 . Remark. The limit of the sequence is called Eulers
constant. It is not known whether Eulers constant is a rational
number or an irrational number.

Solution
We have that

1 1 1 1
bn = 1 + + + + + log n;
2 3 n 1 n
1 1 1
an = 1+ + + + log n:
2 3 n 1
We trivially have that
Z n
1
log n = dt ((1))
1 t
An elementary estimation gives

Zk+1
1 1 1
< dt < ((2))
k+1 t k
k

We take the dierence an+1 an and by (1) and (2) we have that

Z
n+1
1 1 dt
an+1 an = log (n + 1) + log n = > 0;
n n t
n

thus an+1 > an , so the sequence an is increasing.


We take the dierence bn+1 bn and by (1) and (2) we have that

6
Z
n+1
1 1 dt
bn+1 bn = log (n + 1) + log n = < 0;
n+1 n+1 t
n

thus bn+1 < bn , so the sequence bn is decreasing.


We have after some investigation on the increasing sequence an that
49 1
an a7 = log 7 > ; n 7;
20 2
thus 21 < an , if n 7.
We have after some investigation on the decreasing sequence bn that
19 093 197 3
bn b22 = log 22 < ; n 22
5173 168 5
thus bn 3=5, if n 22.
The both sequences converge to the same limit because bn an = 1=n have
the limit 0 as n ! 1. And this limit is in the interval 12 < < 53 .

7
II.1.6
For a complex number , we dene the binomial coe cient "
choose n" by

( 1) ( n + 1)
= 1; = ; n 1:
0 n n!
Show the following
(a) The sequence is bounded if and only if Re 1.
n
(b) ! 0 if and only if Re > 1.
n
(c) If 6= 0; 1; 2; : : :, then ! 1.
n+1 n
(d) If Re 1, 6= 1, then > for all n 0.
n+1 n
(e) If Re a > 1 and is not an integer, then <
n+1 n
for n large

Solution
(a)
We have that

s
Y
n
+1 Y
1
2 (1 + Re ) (1 + Re )2 (1 + Im )2
= 1 = = 1 + + :
n k k k2 k2
k=1 k=1

Let

(1 + Re )2 (1 + Im )2 2 (1 + Re )
tn = +
k2 k2 k
P
If Re < 1, we have that tk > 0 and n
! 1 then n ! 1 because tk
diverge is n unlimited.
(Im )2
If Re = 1, we have that tk = k2
and n
! A then n ! 1, and
+1
P P (Im )2
A 6= 0 becauxe for all k we have that 1 k
> 1 and tk = k2
<1
so n is limited.

8
If Re > 1 we have that it exists k 2 N so that 1 < tk < 0 for all k > K.
This gives that n is a decreasing series with respect to n, and so limited.

The conclution is that n


is limited if and only if Re 1.

b)
We have that n ! 0 if and only if n ! 0. From (a) we see that
the only possibility for this is Re > 1. For Re > 1 we can see that
n
0 is decreasing. Let M = lim n . Then we hav that M = lim n =
n!1 n!1
n
lim = M so M = M () M = 0 so ! 0, then n ! 1 if
n!1 n n+1 n
and only if Re > 1.

c)
If 6= 0; 1; 2; : : :, then n
6= 0 and we can divide, to get
( 1):::( (n+1)+1)
n+1 (n+1)! n 1
= ( 1):::( n+1)
= = n 1 ! 1
n
n+1 1+ n
n!
as n ! 1.

d)
j(n+1)j j nj distance( ;n)
= = > 1, if Re 1, 6= 1
j(n)j n+1 distance( 1;n)

e)
Let be xed and Re > 1. Then
q
j(n+1)j n)2 +(Im )2 j j2 1
= (Re (n+1)2
<1 for n > :
j(n)j 2(1+Re )

Parts (a) and (b) seems to require some facts about products:
Q
n
1+ k
! 1 as n ! 1, >0
k=1
Qn
1 k
! 1 as n ! 1, >0
k=1
Qn
1 k2
! A 6= 0 as n ! 1
k=1

9
II.1.7
Dene x0 = 0, and dene by induction xn+1 = x2n + 14 for n 0. Show
that xn ! 12 .
Hint. Show that the sequence is bounded and monotone, and that any limit
satises x = x2 + 41 .

Solution
We rst prove by induction that for every integer n 0 we have
1
0 xn and xn xn+1 :
2
1 1
For n = 0, we have x0 = 0 and x1 = x20 + 4
= 4
and so
1 1
0 x0 and x0 = 0 = x1 :
2 4
Hence, the assertion holds for n = 0. Now suppose that the assertion holds
for n = k, that is,
1
0 xk and xk xk+1 :
2
1 1 1
For 0 x 2
,it is straightforward to show that x x2 + 4 2
. Since
1
0 xk 2
and xk+1 = x2k + 14 we have
2
1 1 1 1
0 xk xk+1 = x2k + + =
4 2 4 2
1
and hence 0 xk+1 2
, and
1
xk+1 x2k+1 + = xk+2 :
4
Hence, the inductive step holds and so the sequence fxn gn is bounded and
increasing. It follows by Bounded Monotone Sequence Theorem that it
converges, say xn ! x. Note that the real-valued function f given by
f (t) = t2 + 41 for t 2 R is continuous and therefore we have,
1
x = lim xn+1 = lim f (xn ) = f (x) = x2 + :
n!1 n!1 4
1
It follows that x = x2 + 4
and so x = 12 . Hence, xn ! 1
2
as required.

10
II.1.8
Show that if sn ! s, then jsn sn 1 j ! 0.

Solution
We use standard "=2 proof and let sn ! s.
Let " > 0, choose N such that

jsn sj < "=2


for n > N .
If n > N , then we have

jsn+1 sn j = jsn+1 s+s sn j jsn+1 sj + jsn sj < "=2 + "=2 = ":

Thus

jsn+1 sn j ! 0
as n ! 1.

11
II.1.9
Plot each sequence and determine its lim inf and lim sup.
(a) sn = 1 + n1 + ( 1)n (c) sn = sin ( n=4)
n
(b) sn = ( n) (d) sn = xn (x 2 R xed)

Solution
(a)
lim sup sn = 2 lim inf sn = 0
(b)
lim sup sn = 1 lim inf sn = 1
(c)
lim sup sn = 1 lim inf sn = 1
(d) 8
< +1; jxj > 1;
lim sup sn = 1; jxj = 1;
:
0; jxj < 1:
and 8
>
> 1; 1<x< 1;
>
>
< 1; x = 1;
lim inf sn = 0; jxj < 1;
>
>
>
> 1; x = 1;
:
+1; x > 1:

12
II.1.10
At what points are the following function continuous? Justify your
answer.
(a) z (c) z 2 = jzj
(b) z= jzj (d) z 2 = jzj3

Solution
(a)
continuous everywhere
(b)
Continuous except at z = 0, where it is not dened
(c)
Continuous for jzj =
6 0. It is not dened at z = 0, but it has a limit 0 as
z ! 0. If we dene the function to be 0 at z = 0, it is contionuous there.
(d)
Contionuous for jzj =
6 0. It has no limit as z ! 0

13
II.1.11
At what points does the function Arg z have a limit? Where is Arg z
continuous? Justify your answer.

Solution
Arg z have a limit at each point of Cn ( 1; 0]. It is continuous in Cn ( 1; 0].
It is discontinuous at each point of ( 1; 0]. Values of function ! f g at
points of ( 1; 0], the function may take any value in intervall [ ; ] as
z ! 0. The value is depending in that direktion we approach z = 0.

14
II.1.12
Let h (z) be the restriction of the function Arg z to the lower half-
plane fIm z < 0g. At what points does h (z) have a limit? What is
the limit?

Solution
No limit at 0, has limit at all other points of closed lower half-plane, limit
= at points of ( 1; 0).

15
II.1.13
For which complex values of does the principal value of z have
a limit as z tends to 0? Justify your answer.

Solution
Set z = rei = eLog r ei = eLog r+i and = Re + i Im , then

Re +i Im
z = eLog r+i = eLog r Re +i Log r Im +i Re Im
=
= eLog r Re e Im
ei(Log r Im + Re )
= rRe e Im
ei(Log r Im + Re )
;

thus

jz j = rRe e Im
:
Because e Im is bounded we look at rRe in three intervals
If Re < 0, rRe ! 1 as r ! 0 thus jz j have no limit as z ! 0, and not
z either.
If Re = 0, we get z = e Im , have no limit unless Im = 0, thus we have
limit if = 0.
If Re > 0, rRe ! 0 as r ! 0 thus jz j have limit 0 as z ! 0, and z ! 0
as z ! 0.
The conclution is z ! 0 if Re > 0, and z ! 1 if = 0, otherwise we
have no limit at 0.

16
II.1.14
Let h (t) be a continuous complex-valued function on the unit in-
terval [0; 1], and consider
Z 1
h (t)
H (z) = dt:
0 t z
Where is H (z) dened? Where is H (z) continuous? Justify your
answer.
R1 Hint. Use the fact if jf (t) g (t)j < " for 0 t 1, then
0
jf (t) g (t)j dt < ".

Solution
We have that H (z) is dened for z 2 Cn [0; 1]. If h (z1 ) = 0 for some
z1 2 [0; 1]. Then H (z) is also dened for z = z1 .
R1
Then H(z) = h(t) t z
dz is continuous for z 2 Cn [0; 1]. If zn ! z 2 Cn [0; 1],
0
then th(t)
zn
! h(t)
t z
uniformly for 0 t 1, so the used proof shows that
H (zn ) ! H (z).

17
II.1.15
Which of the following sets are open subsets of C? Which are close?
Sketch the sets.
(a) The punctured plane, Cn f0g .
(b) The exterior of the open unitdisk in the plane, fjzj 1g .
(c) The exterior of the closed unitdisk in the plane, fjzj > 1g .
(d) The plane with the open unitinterval removed, Cn (0; 1) .
(e) The plane with the closed unitinterval removed, Cn [0; 1] .
(f) The semidisk, fjzj < 1; Im (z) 0g .
(g) The complex plane, C.

Solution
(a) open (c) open (e) open (g) both open and closed
(b) closed (d) neither (f) neither
Note: Sketches are di cult to do reasonably.

18
II.1.16
Show that the slit plane Cn ( 1; 0] is star-shaped but not convex.
Show that the slit plane Cn [ 1; 1] is not star-shaped. Show that a
punctured disk is not star-shaped.

Solution
(a) Cn ( 1; 0] is starshaped, because we can see every point from z = 1.
To show that Cn ( 1; 0] is not convex we may choose any two points such
that the stright line segment joining them contains a point in ( 1; 0]. For
example take 1 + i and 1 i. These two points are joined by a vertical
line which contain 1. From this follows that Cn ( 1; 0] is not convex.
(b) Cn [ 1; 1] is not starshaped. Given any z we have that z can not bee
seen from a stright line between z and z that must contain 0. We also have
that if z belong to the domain then z belongs also to the domain.
(c) Same argument as in (b) shows that Cn f0g is not starshaped.

19
II.1.17
Show that a set is convex if and only if it is star-shaped with respect
to each of its points.

Solution (A. Kumjian)


Let D C, since the empty set trivially satises both conditions we will
assume that D 6= ?.
Suppose rst that D is convex. We must show that D is star-shaped with
respect to each of its points. So x z0 2 D, to show that D is star-shaped
with respect to z0 . We must show that for every point z 2 D the line segment
connecting z0 to z is contained in D. Let z 2 D be given, then since D is
convex it contains the line segment joining z0 and z. Hence, D is star-shaped
with respect to z0 , then, since z0 was chosen arbitrarily, D is star-shaped
with respect to each of its points.

Conversely, suppose that D is star-shaped with respect to each of its points.


To show that D is convex we must show that given two points in D, the
line segment joining them is also contained in D. So let z0 ; z1 2 D be given.
Then since D is star-shaped with respect to z0 , D contains the line segment
joining z0 and z1 . Hence, D is convex.

20
II.1.18
Show that the following are equivalent for an open subset U of the
complex plane.
(a)
Any two points of U can be joined by a path consisting of straight
linesegments parallel to the coordinate axis.
(b)
Any continuously dierentiable function h (x; y) on U such that rh =
0 is constant.
(c)
If V and W are disjoint open subsets of U such that U = V [ W ,
then either U = V or U = W . Remark. In the context of topological
spaces, this latter property is taken as denition of connectedness.

Solution
Show that the following equivalent for an open subset U of the complex plane.
(a) ) (b) Suppose rh = 0 in D. Fix z0 2 D. If z1 2 D, join by polygonal
curve. rh = 0 ) h is constant on each segment of curve ) h (z0 ) = h (z1 ).
) h is constant in D.
(b) ) (c) Suppose rh = 0, h not constant, say h (z0 ) = 0 for some z0 . Let
W = fh = 0g, V = fh 6= 0g. Evidently V is open. Since rh = 0, h is
constant in a neighborhood of each point, so fh = 0g is open and W is open.
W 6= ?, V 6= ?, W \ V = ?, W [ V = U
(c) ) (a) Suppose z0 2 U . Let D = points that can be joined to z0 by a
polygonal curves, intervals parallel to coordinate axis. Evidently D is open.
Also if can connected points near z0 2 U , then can connect to U , so U nD is
open. By (c), U nD must be empty, some D 6= ?:

21
II.1.19
Give a proof of the fundamental theorem of algebra along the fol-
lowing lines. Show that if p (z) is a nonconstant polynomial, then
jp (z)j attains its minimum at some point z0 2 C. Assume that
the minimum is attained at z0 = 0, and that p (z) = 1 + az m + ,
where m 1 and a 6= 0. Contradict the minimality by showing that
P "ei 0 < 1 for an appropriate choice of 0 .

Solution (K. Seip)


Set

p (z) = an z n + an 1 z n 1
+ + a0 ;
where we assume an 6= 0. Since

jp (z)j
lim n = jan j ;
jzj!1 jzj

9 R > 0 such that jp (z)j > ja0 j for all jzj > R. Thus jp (z)j attains
its minimum in jzj R. We may assume it is attained at z0 = 0.
Suppose a0 6= 0, say a0 = 1. Then p (z) = 1 + az m + higher order
1
1
terms, a 6= 0. Choose z = " a
m
(any m th root does the job).
Then

p (z) = 1 "m + O "m+1 :


Thus for " su ciently small " we have jp (z)j < 1, which is a contra-
diction.

22
Follow the suggestion.
Let

p (z) = a0 + a1 z + am+1 z m+1 + ::: + aN z N ; aN 6= 0:


Choose R so that

jaN j RN > ja0 j + ja1 j R + jam+1 j Rm+1 + ::: + jaN 1j R


N 1
:
Then p (z) 6= 0 for jzj = R, the term aN z N dominates and further jp (z)j >
jp (0)j for jzj = R.
Let z0 be a point in the disk jzj 6 R at which the continuous function for
jp (z)j attains a minimum. Then jz0 j < R, so that is a disk centred at z0 so
that jp (z)j > jp (z0 )j on the disk. Suppose p (z0 ) 6= 0. We can assume that
p (z0 ) = 1.
Write

p (z) = 1 + am P (z z0 )m + O (z z0 )m+1 ;
where am 6= 0. Suppose am = r0 ei 0 .
Conside
zm = z0 + "e i 0 =m
e i =m
; " > 0:
We have

p (zm ) = 1 + r0 ei 0 em e i 0
e i
+ O "m+1 = 1 r0 "m + O "m+1 :
Have p (zm ) 1 r0 "m < 1 for " > 0 small. Contradiction! We conclude
that p (z0 ) = 0.

23
II.2.1
Find the derivatives of the following function.
n
(a) z 2 1 (c) (z 2 1) (e) 1= (z 2 + 3) (g) (az + b) = (cz + d)
(b) z n 1 (d) 1= (1 z) (f) z= (z 3 5) (h) 1= (cz + d)2

Solution
n 1 2
(a) 2z (c) n (z 2 1) 2z (e) 2z= (z 2 + 3) (g) (ad bc) = (cz + d)2
2
(b) nz n 1
(d) 1= (1 z)2 (f) ( 2z 3 5) = (z 3 5) (h) 2c= (cz + d)3

24
II.2.2
Show that
1 zn nz n
1 + 2z + 3z 2 + + nz n 1
= :
(1 z)2 1 z

Solution
Use the geometric sum

1 z n+1
1 + z + z2 + z3 + + zn = ; z 6= 1:
1 z
Dierentiate both sides

1 + 2z + 3z 2 + + nz n 1 =
(n + 1) ( 1) z n (1 z) + (1 z n+1 )
= =
(1 z)2
nz n+1 nz n + z n+1 z n + 1 z n+1
= =
(1 z)2
1 zn nz n+1 nz n 1 zn nz n
= + = :
(1 z)2 (1 z)2 (1 z)2 (1 z)

25
II.2.3
Show from the denition that the functions x = Re z and y = Im z
are not complex dierentiable at any point.

Solution
Dierentiation f (z) = x = Re z (from the denition)

f (z + z) f (z) Re (z + z) Re (z)
lim = lim =
z!0 z z!0 z
Re z 1; if z= x
= lim = :
z!0 z 0; if z=i y

Dierentiation g (z) = y = Im z (from the denition)

g (z + z) g (z) Im (z + z) Im (z)
lim = lim =
z!0 z z!0 z
Im z 0; if z= x
= lim = :
z!0 z i; if z=i y

The functions x = Re z and y = Im z are not complex dierentiable at any


point, because the functions have dierent limit as z ! 0 through real and
imaginary axis.

26
II.2.4 (+ IV.8.2)
Suppose f (z) = az 2 + bz z + cz 2 , where a, b, and c are xed complex
numbers. By dierentiating f (z) by hand, show that f (z) is com-
plex dierentiable at z if and only if bz + 2cz = 0. Where is f (z)
analytic?

Solution
d
We will nd dz
f (z) using the limit denition

f (z + z) f (z)
=
z
2
a (z + z)2 + b (z + z) (z + z) + c(z + z) (az 2 + bz z + cz 2 )
= =
z
2
a z 2 +2z z + z2 + b z z + z z+z z + z z + c z 2 + 2z z + z (az 2 + bz z + cz 2 )
=
z
2 2
2az z + a z + bz z + bz z + b z z + 2cz z + c z
= =
z
2
z z z
= 2az + a z + bz + bz + b z + 2cz +c =
z z z
z z
= 2az + a z + bz + b z + c z + (bz + 2cz)
z z
z
We know z is not analytic at any open set of C and lim z
does not ex-
4z!0
d
ist. Therefore, unless bz + 2cz = 0, the derivative dz f (z) would not exist.
Therefore f (z) is analytic on C if b = c = 0, otherwise is the function not
analytic on any open set.

27
II.2.5
Show that if f is analytic on D, then g (z) = f (z) is analytic on the
reected domain D = fz : z 2 Dg, and g 0 (z) = f 0 (z).

Solution (A. Kumjian)


Let f be analytic on the domain D and dene g on D as above (note that
D is also a domain). For a complex-valued function ' dened near a point
z0 it is easy to show that

lim ' (z) exists i lim ' (z) exists;


z!z0 z!z0

and if either exists, then the two limits are complex conjugates of each other.
Let z 2 D be given. We rst show that g is dierentiable at z and that
g 0 (z) = f 0 (z).

1 1 1
lim (g (z + h) g (z)) = lim f z+h f (z) = lim (f (z + k) f (z)) = f 0 (z)
h!0 h h!0 h k!0 k

where equation ( ) follows using the substitution k = h and basic algebraic


properties of conjugation, note that conjugation is a continuous function so
h ! 0 i h ! 0. Since f is analytic on the domain D, its derivative f 0
is continuous on D. Further, since g 0 (z) = f 0 (z) for all z 2 D , g 0 is the
composite of continuous functions and therefore is itself continuous. Hence,
g is analytic on D .

28
II.2.6
Let h (t) be a continuous complex-valued function on the unit in-
terval [0; 1], and dene
Z 1
h (t)
H (z) = dt; z 2 Cn [0; 1] :
0 t z
Show that H (z) is analytic and compute its derivative. Hint. Dif-
ferentiate by hand, that is, use the dening identity (2.4) of com-
plex derivative.

Solution
We use denition (2.4) of derivative to nd H 0 (z).

H 0 (z) =
Z 1 Z 1
H (z + z) H (z) 1 h (t) h (t)
= lim = lim dt dt =
z!0 z z!0 z
0 t z z 0 t z
Z 1
1 1 1
= lim h (t) =
z!0 z 0 t z z t z
Z 1 Z 1
h (t) h (t)
= lim = dt =
z!0 0 (t z z) (t z) 0 (t z) (t z)
Z 1
h (t)
= dt:
0 (t z)2

H 0 (z) is continuous in z, also by uniform convergence of integrand H (z) is


analytic for z 2 Cn [0; 1].

29
II.3.1
Find the derivatives of the following functions.
sin z sinh z
(a) tan z = cos z
(b) tanh z = cosh z
(c) sec z = 1= cos z

Solution
(a)

d d sin z cos z cos z sin z ( sin z) cos2 z + sin2 z 1


tan z = = 2
= 2
= :
dz dz cos z cos z cos z cos2 z
(b)

d d sinh z cosh z cosh z sinh z sinh z cosh2 z sinh2 z 1


tanh z = = 2 = 2 = :
dz dz cosh z cosh z cosh z cosh2 z
(c)
d d 1 1 sin z
sec z = = ( sin z) = = tan z sec z:
dz dz cos z cos2 z cos2 z

30
II.3.2
Show that u = sin x sinh y and v = cos x cosh y satisfy the Cauchy-
Riemann equations. Do you recognize the analytic function f =
u + iv? (Determine its complex form)

Solution
We have

@u @ @ @v
= sin x sinh y = cos x sinh y = cos x cosh y = ;
@x @x @y @y
@u @ @ @v
= sin x sinh y = sin x cosh y = cos x cosh y = :
@y @y @x dx
Hence, u and v satisfy Cauchy-Riemann equations, and now we calculate
f (z).

f (z) =
eix e ix ey e y eix + e ix ey + e y
= u + iv = +i =
2i 2 2 2
eix+y eix y e ix+y + e ix iy eix+y + eix y + e ix+y + e ix y
= i +i =
4 4
ix y ix+y i(x+iy) i(x+iy)
e +e e +e e + e iz
iz
=i =i =i =
2 2 2
= i cos z:

31
II.3.3
1 2 3 P L K
LLL
Show that if f and f are both analytic on a domain D, then f is
constant.

Solution (A. Kumjian)


Let D be a domain and let f be a complex valued function such that both f
and f are analytic on D. Then both Re f = 21 f + f and Im f = 2i1 f f
must also be analytic on D. Since Re f and Im f are also both real-valued,
it follows by the theorem on page 50, that both are constant. Hence, f =
Re f + i Im f is constant.

32
II.3.4
Show that if f is analytic on a domain D, and if jf j is constant,
then f is constant.
Hint. Write f = jf j2 =f .

Solution (with use of hint)


If f (z) = 0 for some z 2 D. Then f 0 on D, since jf j is constant. Assume
that f (z) 6= 0 for every z 2 D. Since f is analytic in D we have that
1=f and f = jf j2 =f (where jf j is constant) are analytic. From this follows
that Re f and Im f are analytic and real-valued and therefore constant. So
f = Re f + i Im f is constant.

33
II.3.4
Show that if f is analytic on a domain D, and if jf j is constant,
then f is constant.
Hint. Write f = jf j2 =f .

Solution
Set f = u + iv, where u and v are realvalued functions. Now suppose that
jf j = k is constant. We have that u2 + v 2 = k 2 . Dierentiate both sides with
respect to x respectively y
0 0
uux + vvx = 0;
0 0 (1)
uuy + vvy = 0:
0 0 0 0
We use Cauchy-Riemanns equations ux = vy and uy = vx in (1) and have
0 0
uux vuy = 0;
0 0 (2)
uuy + vux = 0:
In the simultaneusly systems of equations in (2) we rst rst multiply the
rst row with u and second row with v in (2) and add them together, and
0
have (u2 + v 2 ) ux = 0. And if we in the same system multiply the rst row
0
with v and the second row with u and add them we have (u2 + v 2 ) uy = 0.
Thus
0
(u2 + v 2 ) ux = 0;
0 (3)
(u2 + v 2 ) uy = 0:
If in (3) u2 + v 2 = 0, and we have that u = v = 0 and thus f is constant.
0 0
Suppose that u2 + v 2 6= 0. We have that (3) gives that ux = uy = 0, with
says that u is constant. Cauchy-Riemanns equations gives
0 0
0 = ux = v y ;
0 0 (4)
0 = uy = v y ;
thus by (4) f is constant.

34
II.3.5
If f = u + iv is analytic, then jruj = jrvj = jf 0 j.

Solution (K. Seip)


Because f = u + iv is analytic. Then

@u @u @u @v @v @v
ru = ; = ; = ;
@x @y @x @x @y @x
from which it follows that

jruj = jf 0 (z)j = jrvj :

35
II.3.6
If f = u + iv is analytic on D, then rv is obtained by rotating ru
by 90 . In Particular, ru and rv are orthogonal.

Solution (A. Kumjian)


Let 2 R then rotation of a vector (x; y) 2 R2 by the angle (about the
origin) is given by matrix multiplication:

x x cos sin x x cos y sin


7! A = = :
y y sin cos y x sin + y cos

Hence, rotation by 90 is given by the transformation (x; y) 7! ( y; x). By


the Cauchy-Riemann equations we have

@v @v @u @u
rv = ; = ; =A =2 ru:
@x @y @y @x
Thus, rv is obtained by rotating ru by 90 . It follows that ru and rv are
orthogonal.

Solution (D. Jakobsson)


Given since f is analytic, then
@u @v @u @v
(0.6) = and =
@x @y @y @x
Since ru = @u ; @u and rv = @x
@x @y
@v @v
; @y all we need to check is whether
rv rv = 0. To prove that we use (0.6).
@v @u @v @u @v @v @v @v
ru rv = + = =0
@x @x @y @y @x @y @y @x
We conclude that ru and rv are orthogonal.
Notice that the operation that sends ru ! rv is given by the counter-clock-
wise rotation matrix with = 2 . Keeping in mind (0.6), one can write the
following.

vx 0 1 vy cos 2 sin 2
vy
rv = = = =R ru
vy 1 0 vx sin 2 cos 2 vx 2

36
Remark.
We have that

@v @v @u @u
vx = ; vy = ; ux = ; uy = ;
@x @y @x @y
and

R ( =2) = A =2 :

37
II.3.7
Sketch the vector elds ru and rv for the following functions f = u + iv.
(a) iz (b) z 2 (c) 1=z
Solution

II.3.7a II.3.7a II.3.7a

y y 4
y 4 4

2 2 2

-4 -2 2 4 -4 -2 2 4 -4 -2 2 4
-2 x -2 x -2 x
-4 -4
-4

(a)

u= y ru = (0; 1)
f (z) = iz = y + ix ) )
v=x rv = (1; 0)
(b)

u = x2 y 2 ru = (2x; 2y)
f (z) = z 2 = x2 y 2 + 2ixy ) )
v = 2xy rv = (2y; 2x)
(c)
x cos
1 z x iy u = x2 +y 2 = r
f (z) = = 2 = 2 ) y sin
z jzj x + y 2 v = 2
x +y 2 = r
8 2 2 2 2
>
>
@u
= (xy2 +yx2 )2 = sin r2cos = cos r2
2
>
>
@x
< @u = 2xy
2 =
2 cos sin
2 = sin2 2
@y (x2 +y 2 ) r r
@v 2xy 2 cos sin sin 2
>
> = (x2 +y 2 )2
= r2
= 2
>
>
@x r
: @v
= y 2 x2
= sin2 cos2
= cos 2
@y (x2 +y 2 )2 r2 r2

1
ru = r2
( cos 2 ; sin 2 )
1
rv = r2
( sin 2 ; cos 2 )

38
II.3.8 (see III.4.2) (36)
Derive the polar form of the Cauchy-Riemann equations for u and
v,
@u
= 1r @v
@
; @u
@
= r @v
@r
:
@r
Check that for any integer m, the functions u rei = rm cos (m )
and v rei = rm sin (m ) satisfy the Cauchy-Riemann equations.

Solution
Set x = r cos and y = r sin , then take the partial derivative,
@x @x
@r
= cos @
= r sin
@y @y
@r
= sin @
= r cos
Use Cauchy-Riemanns equations and the derivative to get the rst equation

@u @u @x @u @y @u @u @v @v
= + = cos + sin = cos sin =
@r @x @r @y @r @x @y @y @x
1 @v @v 1 @v @x @v @y 1 @v
= r sin + r cos = + = ;
r @x @y r @x @ @y @ r@

Use Cauchy-Riemanns equations and the derivative to get the second equa-
tion

@u @u @x @u @y @u @u @v @v
= + = r sin + r cos = r sin r cos =
@ @x @ @y @ @x @y @y @x
@v @v @v @x @v @y @v
= r cos + sin = r + = r :
@x @y @x @r @y @r @r
we get
@u 1 @v
@r
= r@
;
@u
@
= r @v
@r
:
Set u rei = rm cos (m ) and v rei = rm sin (m ) and use it in the
Cauchy-Riemanns equations,

39
@u @ m 1 m 1 @ m 1 @v
= (r cos (m )) = mrm 1 cos (m ) = r m cos (m ) = (r sin m ) = ;
@r @r r r@ r@
@u @ m @ @v
= (r cos (m )) = mrm sin (m ) = rmrm 1 sin (m ) = r (rm sin (m )) = r :
@ @ @r @r

40
II.4.1
Sketch the gradient vector elds ru and rv for
(a) u + iv = ez (b) u + iv = Log z

Solution
a)
We have

u + iv = ez = ex cos y + iex sin y


and get

ru = (ex cos y; ex sin y) = ex (cos y; sin y)


(in black), and

rv = (ex sin y; ex cos y) = ex (sin y; cos y)


(in red).
b)
We have

u + iv = Log z = log r + i
and get

1!
ru = ur ;
r
1!
rv = u :
r

FIGURE II.4.1a NF FIGURE II.4.1b NF

41
II.4.2
Let a be a complex number a 6= 0, and let f (z) be an analytic branch
of z a on Cn ( 1; 0]. Show that f 0 (z) = af (z) =z. (Thus f 0 (z) = az a 1 ,
where we pick the branch of z a 1 that corresponds to the original
branch of z a divided by z.)

Solution
We may write the branch as follows

f (z) = z a = ea log z = ea(Log z+2 im)


= ea Log z+2 iam
= cea Log z ;
where c = e2 iam
for some m 2 Z. It follows by the chain rule that

a af (z)
f 0 (z) = cea Log z = :
z z
So, the desired result follows.

42
II.4.3 p
Consider the branch of f (z) = z (1 z) on Cn [0; 1] that has posi-
tive imaginary part at z = 2. What is f 0 (z)? Be sure to specify the
branch of the expression for f 0 (z).

Solution p
Set f (z) = z (1 z) (principal branch) and dierentiate
1 1 d 1 1 2z
f 0 (z) = p (z (1 z)) = p :
2 z (1 z) dz 2 z (1 z)
Branch of f (z) is i increasing for x > 0 large, thus f 0 (z) is i positive for
x > 0 large. p
Use branch of f 0 (z) = z (1 z) that it is i positive, i.e., same on f (z).
1 1 2z
f 0 (z) = :
2 f (z)

Solution (D. p
Jakobsson)
Given f (z) = z (1 z), we can dene w = f (z) and get w2 = z (1 z).
d d 2
(1 z) z = w
dz dz
dw
1 2z = 2w
dz
One gets
dp 1 2z
z (1 z) = p
dz 2 z (1 z)
and is dened on Cn [0; 1]. Take the principal branch with the positive imag-
inary part.

43
II.4.4
Recall that the principal branch of the inverse tangent function
was dened on the complex plane with two slits on the imaginary
axis by
1 1 1+iz
Tan = 2i
Log 1 iz
; z 62 ( i1; i] [ [i; i1) :
Find the derivative of Tan 1 z. Find the derivative of tan 1 z for
any analytic branch of the function dened on a domain D.

Solution
We have that

d 1 1 1 1 1 1 iz + 1 + iz 1
Tan z= i ( i) = = :
dz 2i 1 + iz 1 iz 2 (1 + iz) (1 iz) 1 + z2
1
Any two branches of tan z dier by a constant, so derivatives are same.

44
II.4.5 p
Recall that cos 1 (z) = i log z z 2 1 . Suppose g (z) is an ana-
lytic branch of cos (z), dened on a domain D. Find g 0 (z). Do
1

dierent branches of cos 1 (z) have the same derivative?

Solution
We have that

d 1
cos z=
dz
i 1 2 1=2
= p 1 z 1 2z =
z z 2 1 2
i z i
= p 1 p =p =
z z 2 1 z 2 1 z 2 1
1
=p :
1 z2

1
Derivatives of branches of cos z are not always the same.

45
II.4.6
Suppose h (z) is an analytic branch of sin 1 (z), dened on a domain
D. Find h0 (z). Do dierent branches of sin 1 (z) have the same
derivative?

Solution (A. Kumjian)


Regard h (z) as a local inverse of f (z) where f (z) = sin z. Then f 0 (z) =
cos z, so using the formula given in the statement of theorem on page 51 we
have
1 1
h0 (z) = 0 =
f (h (z)) cos h (z)
if cos h (z) 6= 0.
One local inverse of f satises h1 (0) = 0, while another local inverse of f
satises h2 (0) = . Using the formula above we see

0 1 0 1
h1 (0) = =1 and h2 (0) = = 1:
cos 0 cos
1
Hence, dierent branches of sin (z) do not necessarily have the same deriv-
ative.

46
II.4.7
Let f (z) be a bounded analytic function, dened on a bounded
domain D in the complex plane, and suppose that f (z) is one-one-
one. Show that the area of f (D) is given by
ZZ
2
Area (f (D)) = jf 0 (z)j dxdy:
D

Solution (K. Seip)


We have f : D ! C, jf (z)j M for some M < 1 when z 2 D. Since f is
assumed to be one-to-one, we may compute
ZZ
A (f (D)) = dudv
f (D)

by the change of variables (u (x; y) ; v (x; y)) ! (x; y), and since det Jf =
jf 0 (z)j2 , we get
ZZ
2
Area (f (D)) = jf 0 (z)j dxdy:
D

47
II.4.8
Sketch the image of the circle fjz 1j 1g under the map w = z 2 .
Compute the area of the image.

Solution

II.4.8 z-plane II.4.8 w-plane


y 4 y 4

2 2

-4 -2 2 4 -4 -2 2 4
-2 x -2 x
-4 -4

We have that f (z) = z 2 , that gives f 0 (z) = 2z and Jf (z) = 4 jzj2 .


The area of the image is
ZZ ZZ
Jf dxdy = 4 x2 + y 2 dxdy:
(x 1)2 +y 2 61

Set t = x 1, get

ZZ
4 t2 + 2t + 1 + y 2 dtdy =
t2 +y 2 61
2 3
ZZ ZZ
6 7
= 44 t2 + y 2 dtdy + 0 + dtdy 5 =
t2 +y 2 61 t2 +y 2 61
h i
=4 + =6 :
2

Solution (D. Jakobsson)


d
Given w = f (z) = z 2 , one can dierentiate f (z) and get dz f (z) = 2z =
2 (x + iy), we can use maple to integrate and get the following

48
Area (f (D)) =
ZZ Z 2 Z p1 (x 1)2
2
= 0
jf (z)j dxdy = p 4 jx + iyj2 dx dy =
2
D 0 1 (x 1)
Z p
2Z 1 (x 1) 2 Z 2 p 3=2
= p 4 x2 + y 2 dx dy = 8x2 2x x2 +8=3 2x x2 dx =
0 1 (x 1)2 0

=6

Change of variables

x = 1 + sin t
dx = cos tdt

Z =2 p 8 3=2
8 (1 + sin t)2 1 sin2 t + 1 sin2 cos tdt =
=2 3
Z !
=2
2 2 8 4
= 8 1 + 2sin|{z}t + sin t cos + 3 cos tdtdt =
=2
odd
Z =2
2
= 16 2 cos2 cos4 tdt =
0 3
Z =2
1
=8 2 (1 + cos 2t) 1 + 2 cos 2t + cos2 2t dt =
0 3
Z =2
5 4 1
=8 + cos 2t (1 + cos 4t) dt =
0 3 3 3
5 1 9
=8 =4 =6 :
2 3 6 6

49
II.4.9
Compute
ZZ
2
jf 0 (z)j dxdy;
D
2
for f (z) = z and D the open unit disk fjzj < 1g. Interpret your
answer in terms of areas.

Solution
By the result above
ZZ ZZ
0 2 2
jf (z)j dxdy = jf 0 (z)j dx dy = ;
jzj<1 jzj<1
x>0 x<0

thus
ZZ
2
jf 0 (z)j dx dy = 2 :
jzj<1

Remark.
The function maps the top and bottom halves of the unit disk one-to-one
onto the unit disk, so by Exersice II.4.7 the integrals on top and bottom
halves are each .

50
II.4.10
1 2 3 P L K

For smooth functions g and h dened on a bounded domain U , we dene the


Dirichlet form DU (g; h) by
ZZ
@g @h @g @h
DU (g; h) = + dx dy
U @x @x @y @y
Show that if z = f ( ) is a one-to-one analytic function from the bounded
domain V onto U , then

DU (g; h) = DV (g f; h f ) :
Remark. This shows that the Dirichlet form is a "conformal invariant".

Solution RR
Du (g; h) = rgrhdxdy Assume h; g ????
U
RR
Du (g; h) = r (g f ) r (h f )dudv =
RR U
g (u (x; y) ; v (x; y)) h (u (x; y) ; v (x; y)) dudv
U
@g(u(x;y);v(x;y)) @h(u(x;y);v(x;y))
@x @x
+
@g(u(x;y);v(x;y)) @h(u(x;y);v(x;y))
@y @y
=
@g @u @g @v @h @u @g @u @g @v
@u @x
+ @v @x @u @x
+ @h @v
@v @x
+ @u @y
+ @v @y
@h @u
@u @y
+ @h @v
@v @y
=
@g @h @u 2 @g @u @h @v @g @v @h @u @g @h @v 2
@u @u @x
+ @u @x @v @x
+ @v @x @u @x
+ @v @v @x
+
2 2
@g @h @u @g @u @h @v @g @v @h @u @g @h @v
@u @u @y
+ @u @y @v @y
+ @v @y @u @y
+ @v @v @y
=
2 2
@g @h @u 2 @g @h @v 2 @g @h @u @g @h @v
@u @u @x
+ @v @v @x
+ + =
! @u @u @y @v @v @y
!
2 2 2 2
@g @h @u @u @g @h @v @v
@u @u
+ + @v @v
+ +
@x @y @x @y
| {z } | {z }
jf 0 (z)j2 jf 0 (z)j2
@g @u @h @v @g @v @h @u @g @u @h @v @g @v @h @u
+ + +
@u @x @v @x @v @x @u @x @u @y @v @y @v @y @u @y
| {z }
Use C-R equations
RR RR
rgrh jf 0 (z)j2 dxdy = rgrhdudv
U U

51
II.5.1
Show that the following functions are harmonic, and nd its har-
monic conjugates:
(a) x2 y 2 (c) sinh x sin y (e) tan 1 (y=x) ; x > 0
2 2
(b) xy + 3x2 y y 3 (d) ex y cos (2xy) (f) x= (x2 + y 2 )

Solution
(a)
Set
0 00
ux = 2x uxx = 2
u (x; y) = x2 y2 ) 0 ) 00 ) 4u = 0;
uy = 2y uyy = 2
thus is the function u (x; y) is harmonic. Cauchy Riemann give us
0 0 0
vx = uy = 2y ) v (x; y) = 2xy + g (y) ) vy = 2x + g 0 (y)
0 0
vy = ux = 2x
Thus we have that

g 0 (y) = 0 ) g (y) = C;
thus

v (x; y) = 2xy + C:
(b)
Set

0 00
ux = y + 6xy uxx = 6y
u (x; y) = xy+3x2 y y 3 ) 0 ) 0 ) 4u = 0;
uy = x + 3x2 3y 2 uyy = 6y

thus is the function u (x; y) is harmonic. Cauchy Riemann give us

0 0 x2 0
vx = uy = x 3x2 + 3y 2 ) v (x; y) = 2
x3 + 3xy 2 + g (y) ) vy = 6xy + g 0 (y)
0 0
vy = ux = y + 6xy

Thus we have that

52
0 y2
g (x) = y ) g (y) = + C;
2
thus

y2 x2
v (x; y) = + 3xy 2 x3 + C:
2 2
In paranthesis we remark that the analytic function f (z) = u + iv is given
by

y2 x2
f (z) = xy +3x2 y y 3 +i + 3xy 2 x3 + C = iz 2 (z + 1=2)+iC:
2 2

(c)
Set

0 00
ux = cosh x sin y uxx = sinh x sin y
u (x; y) = sinh x sin y ) 0 ) 0 ) 4u = 0;
uy = sinh x cos y uyy = sinh x sin y

thus the function u (x; y) is harmonic. Cauchy Riemann give us

0 0 0
vx = uy = sinh x cos y ) v (x; y) = cosh x cos y + g (y) ) vy = cosh x sin y + g 0 (y)
0 0
vy = ux = cosh x sin y

Thus we have that

g 0 (y) = 0 ) g (y) = C;
thus

v (x; y) = cosh x cos y + C


In paranthesis we remark that the analytic function f (z) = u + iv is given
by

53
f (z) = sinh x sin y i (cosh x cos y + C) =
ex e x eiy e iy ex + e x eiy + e iy
= i + iC =
2 2i 2 2
i
= ex e x eiy e iy + ex + e x eiy + e iy + iC =
4
ex+iy + e (x+iy)
= i + iC = i cosh z + iC:
2
(d)
Set

0 2 2 2 2
2 y2 ux = 2xex y cos (2xy) 2yex y sin (2xy)
u (x; y) = ex cos (2xy) ) 0 2 2 2 2 )
uy = 2yex y cos (2xy) 2xex y sin (2xy)
00 2 2
uxx = 2ex y (cos 2xy + 2x2 cos 2xy 2y 2 cos 2xy 4xy sin 2xy)
) 0 2 2 ) 4u = 0;
uyy = 2ex y (cos 2xy + 2x2 cos 2xy 2y 2 cos 2xy 4xy sin 2xy)

thus the function u (x; y) is harmonic. Cauchy Riemann give us

8 0 0 2 2 2 2 2 y2
< vx = uy = 2yex y cos (2xy) + 2xex y sin (2xy) ) v (x; y) = ex sin (2xy) + g (y) )
0 2 2 2 2
) v = 2yex y sin (2xy) + 2xex y cos (2xy) + g 0 (y)
: 0 y 0 2 2 2 2
vy = ux = 2xex y cos (2xy) 2yex y sin (2xy)

Thus we have that

g 0 (y) = 0 ) g (y) = C;
thus
2 y2
v (x; y) = ex sin (2xy) + C
In paranthesis we remark that the analytic function f (z) = u + iv is given
by
2 y2 2 y2 2
f (z) = ex cos (2xy) + i ex sin (2xy) + C = ez + iC:
(e)

54
Set

1
u (x; y) = tan (y=x) ; x > 0 )
8
> 0 1 y y
< ux = =
=) 1 + (y=x)2 x2 x2 + y2 =)
> x
: u0y = 2
x + y2
8
> 00 2xy
>
< uxx =
(x2 + y 2 )2
) 2xy ) 4u = 0;
>
> 0
: uyy =
(x2 + y 2 )2

thus the function u (x; y) is harmonic. Cauchy Riemann give us

8
> 0 0 x 1 0 y
< vx = uy = ) v (x; y) = a 2
log (x2 + y 2 ) + g (y) ) vy = + g 0 (y)
x2 + y 2 2
x +y 2

> 0 0 y
: v y = ux = 2
x + y2
Thus we have that

g 0 (y) = 0 ) g (y) = C;
thus
1
v (x; y) = log x2 + y 2 + C:
2
(f)
Set

8 8
> x2 y 2 > 00 3y 2 x2
> 0
< ux = > u
< xx = 2x
x (x2 + y 2 )2 ) (x2 + y 2 )3
u (x; y) = ) ) 4u = 0;
x2 + y2 >
> 0 2xy >
> 0 3y 2 x2
: uy = : uyy = 2x 2
(x2 + y 2 )2 (x + y 2 )3

thus is the function u (x; y) is harmonic. Cauchy Riemann give us

55
8
> 0 0 2xy y 0 x2 y 2
>
< vx = uy = 2 ) v (x; y) = + g (y) ) vy = 0
2 + g (y)
2 2 2 2
(x + y ) 2 2
(x + y ) (x + y )
2 2
>
> 0 0 x y
: v y = ux =
(x2 + y 2 )2

Thus we have that

g 0 (y) = 0 ) g (y) = C;
thus
y
v (x; y) = +C
(x2 + y2)
In paranthesis we remark that the analytic funktion f (z) = u + iv is given
by

x y z z 1
f (z) = +i +C = 2 + iC = + iC = + iC:
x2 + y 2 (x2 + y 2 ) jzj zz z

56
II.5.2
Show that if v is a harmonic conjugate for u, then u is a harmonic
conjugate for v.

Solution
If f (z) = u (x; y) + iv (x; y) is analytic then if (z) = v (x; y) iu (x; y) is
analytic. Thus u is a harmonic conjugate of v.

57
II.5.3
Dene u (z) = Im (1=z 2 ) for z 6= 0, and set u (0) = 0.
(a) Show that all partial derivatives of u with respect to x exist
at all points of the plane C, as do all partial derivative of u with
respect to y.
2 2
(b) Show that @@xu2 + @@yu2 = 0.
(c) Show that u is not harmonic on C.
@2u
(d) Show that @x@y does not exist at (0; 0).

Solution
(a)
We rst dene

8
< 1 z2 (x iy)2 2xy
Im 2 = Im 4 = Im 2 = (x; y) 6= (0; 0)
u (z) =
:
z jzj 2 2
(x + y ) (x + y 2 )2
2

0 (x; y) = (0; 0)

For all (x; y) 6= (0; 0), the n th derivative of u (z) will be (x2f+y
(x;y)
2 )n+2 with

f (x; y) a polynomial of degree n + 2 in (x; y). At (0; 0), the function itself is
0, thus u (x; y) 2 C1 (x). By symmetry u (x; y) 2 C1 (y).
(b)
We have

@2u @2u @u 3x2 y 2 @u 3y 2 x2


+ 2 = 2y + 2x =
@x 2 @y @x (x2 + y 2 )3 @y (x2 + y 2 )3
x2 y 2 x2 y 2
= 24xy + 24xy =0
(x2 + y 2 )4 (x2 + y 2 )4
(c)
Notice that the function u (x; y) is not harmonic because the function itself,
its rst and second derivative are not continuous at the origin. The limit of
u (x; y) as (x; y) ! (0; 0) does not exist, for instance u n1 ; n1 goes to 1 as
n ! 1 whereas we dened the function u to be 0 at the origin.

1 1 n2
lim u ; = lim = 1:
n!1 n n n!1 2

58
(d)
2
@u 2x (x2 + y 2 ) + 2xy 2 (x2 + y 2 ) 2y 2x3 + 6xy 2
= =
@y (x2 + y 2 )4 (x2 + y 2 )3
thus

3 2
@2u ( 6x2 + 6y 2 ) (x2 + y 2 ) ( 2x3 + 6xy 2 ) 3 (x2 + y 2 ) 2x 6x4 36x2 y 2 + 6y 4
= = :
@x@y (x2 + y 2 )6 (x2 + y 2 )4

We have
(
6x4 36x2 y 2 +6y 4
@2u (x2 +y 2 )4
(x; y) 6= (0; 0)
=
@x@y 0 (x; y) = (0; 0)
@2u 6=x4 ; x 6= 0
(x; 0) = :
@x@y 0; x=0
@2u
This partial derivative is not continuous at x = 0, so @x@y
does not exist at
(0; 0).
.

59
II.5.4
Show that if h (z) is a complex-valued harmonic function (solution
of Laplaces equation) such that zh (z) is also harmonic, then h (z)
is analytic.

Solution
First partial derivative for zh (z) is,

@ (zh) @z @h
= h+z ;
@x @x @x
@ (zh) @z @h
= h+z :
@y @y @y
Second derivative for zh (z) is,

@ 2 (zh) @2z @z @h @z @h @2h


= h + + + z ;
@x2 @x2 @x @x @x @x @x2
@ 2 (zh) @2z @z @h @z @h @2h
= h + + + z :
@y 2 @y 2 @y @y @y @y @y 2
We have z = x+iy, thus @z=@x = 1, @z=@y = i, and @ 2 z=@x2 = @ 2 z=@y 2 = 0,
therefore

@ 2 (zh) @h @2h
= 2 + z ;
@x2 @x @x2
@ 2 (zh) @h @2h
= i2 + z :
@y 2 @y @y 2
Add the equations

@ 2 (zh) @ 2 (zh) @h @h @2h @2h


2
+ 2
=2 + 2i +z + :
@x @y @x @y @x2 @y 2
Because h (z) and zh (z) is harmonic, then both @ 2 (zh) =@x2 +@ 2 (zh) =@y 2 =
0 and @ 2 (h) =@x2 + @ 2 (h) =@y 2 = 0, we have
@h @h
= i :
@x @y

60
Write h = u + iv, get

@u @v @u @v @v @u
+i = i +i = i :
@x @x @y @y @y @y
Now we take real and imaginary parts and get
@u @v @u @v
@x
= @y
and @y
= @x
:
Thus yields Cauchy-Riemanns equations for u + v, then h (z) is analytic as
was to be shown.

61
II.5.5
Show that Laplaces equation in polar coordinates is

@ 2 u 1 @u 1 @2u
+ + = 0:
@r2 r @r r2 @ 2
Solution
Let x = r cos and y = r sin , and get partial derivatives
@x @x
= cos ; = r sin ;
@r @
@y @y
= sin ; = r cos :
@r @
Taking the rst derivative of u with respect to r
@u @u @x @u @y @u @u @u @u
= + = cos + sin = cos + sin :
@r @x @r @y @r @x @y @x @y
Taking the second derivative of u with respect to r

@2u
=
@r2
@ 2 u @x @ 2 u @y @ 2 u @x @ 2 u @y
= cos + cos + sin + sin =
@x2 @r @y@x @r @x@y @r @y 2 @r
@ 2 u @x @2u @2u @ 2 u @y
= cos + sin cos + cos sin + sin =
@x2 @r @y@x @x@y @y 2 @r
@2u @2u 2
2 @ u
= cos2 + 2 sin cos + sin :
@x2 @x@y @y 2
Taking the rst derivative of u with respect to
@u @u @x @u @y @u @u
= + = ( r sin ) + r cos :
@ @x @ @y @ @x @y
Taking the second derivative of u with respect to

62
@2u
=
@ 2
@ 2 u @x @ 2 u @y @u
= 2
( r sin ) + ( r sin ) + ( r cos ) +
@x @ @y@x @ @x
@ 2 u @x @ 2 u @y @u
+ (r cos ) + 2 (r cos ) + ( r sin ) =
@x@y @ @y @ @y
@2u @2u @u
= 2
( r sin ) ( r sin ) + (r cos ) ( r sin ) + ( r cos ) +
@x @y@x @x
@2u @2u @u
+ ( r sin ) (r cos ) + 2 (r cos ) (r cos ) + ( r sin ) =
@x@y @y @y
@2u @2u 2
2 @ u @u @u
= r2 sin2 2
2r 2
sin cos + r 2
cos 2
r cos r sin :
@x @x@y @y @x @y
Combining these partial derivatives, one gets

@ 2 u 1 @u 1 @2u
+ + =
@r2 r @r r2 @ 2
@2u @2u @2u 1 @u @u
= cos2 + 2 sin cos + sin2 + cos + sin +
@x2 @x@y @y 2 r @x @y
1 @2u @2u @2u @u @u
+ 2 r2 sin2 2r2 sin cos + r2 cos2 r cos r sin =
r @x2 @x@y @y 2 @x @y
@2u 2
@ u 2
@ u 1 @u 1 @u
= cos2 + 2 sin cos + sin2 + cos + sin +
@x2 @x@y @y 2 r @x r @y
@2u @2u @2u 1 @u 1 @u
+ sin2 2 sin cos + cos2 cos sin =
@x2 @x@y @y 2 r @x r @y
@2u @2u @2u @2u
= sin2 + cos2 + sin 2
+ cos 2
= + :
@x2 @y 2 @x2 @y 2
Because u is a harmonic function we have @ 2 u=@x2 + @ 2 u=@y 2 = 0 and we
have,

@ 2 u 1 @u 1 @2u @2u @2u


+ + = + = 0:
@r2 r @r r2 @ 2 @x2 @y 2

63
II.5.6
Show using Laplaces equation in polar coordinates that log jzj is
harmonic on the punctured plane Cn f0g.

Solution
Set z = rei , we have u (r; ) = log jzj = log r. We can compute the Laplacian
of log jzj without worrying about the origin, i.e. r 6= 0 because this case is
taken out, thus we have

@ 2 u 1 @u 1 @u2 1 11
log r = + + = + + 0 = 0:
@r2 r @r r2 @ 2 r2 r r

64
II.5.7
Show that log jzj has no conjugate harmonic function on the punc-
tured plane Cn f0g, though it does have a conjugate harmonic func-
tion on the slit plane Cn ( 1; 0).

Solution (A. Kumjamin)


Set D := Cn ( 1; 0] and let u : D ! R be given by u (z) := log jzj, it is
easily veried that u (z) = Re Log z and this shows that u is harmonic on D
by the main theorem in section II.5. Moreover, it follows by Cauchy-Riemann
equations that a harmonic conjugate v is given by
v (z) := Im Log z = Arg z for z 2 D:
Any other harmonic conjugate diers from v by a constant. Now if we set
D~ := Cn f0g and dene u~ : D ! R by u~ (z) := log jzj, it can easily be
shown by an argument similar to the one above that u~ is harmonic on D ~
even though it cannot be expressed as the real part of an analytic function
on D~ (the property of being harmonic is local). Suppose that u~ does have a
~ Then its restriction to D would have to
harmonic conjugate, say v~, on D.
be a harmonic conjugate of u and so it would be of the form z 7 ! Arg z + c
for some c 2 R. But such a function has no continuous extension to D. ~
This contradicts the fact that v~ is harmonic on D ~ (since this implies v~ is
~
continuous on D). Therefore, log jzj has no conjugate harmonic function on
the punctured plane Cn f0g.
Solution (K. Seip)
In Cn ( 1; 0], we know that any harmonic conjugate of log jzj must have
the form

v (x; y) = Arg z + c:
Such a function can not be made continuous in C since for x < 0, we have

lim v (x; y) = + c;
y!00+

and

lim v (x; y) = + c:
y!00

65
II.5.8
Show using Laplaces equation in polar coordinates that u rei =
log r is harmonic. Use the polar form of the Cauchy-Riemann
equations. (Exercise 3.8) to nd a harmonic conjugate v for u.
What is the analytic function u + iv?

Solution
Set
( (
0 00
ur = urr =
u rei = log r ) 0
r ) 00
r2
u = log r u =0
The function u rei = log r is harmonic because
1 1
u = urr + ur + 2 u = + = 0:
r r r2 r2
Cauchy Riemann equations in polar form (page 50) gives us

8
< 0 1 0 log r (log r)2 0
vr = u = ) v (r; ) = + g ( ) ) v = g0 ( )
: v 0 = rur0 = r 2
r

Thus we have that


2
g0 ( ) = ) g( ) = + C;
2
thus

(log r)2 2
v (r; ) = + + C:
2 2
We have that the analytic function f (z) = u + iv is given by

!
2
(log r)2 i i
f (z) = log r+i +C = (log r + i )2 +C = (log z)2 +iC:
2 2 2 2

66
II.6.1
Sketch the families of level curves of u and v for the following
functions f = u + iv.
(a) f (z) = 1=z (b) f (z) = 1=z 2 (c) f (z) = z 6
Determine where f (z) is conformal and where it is not conformal.

Solution
(a)
We have that
8
x
1 z x iy < u=
f (z) = = 2 = 2 ) x + y2
2

: v= y
z jzj x + y2
x + y2
2

Conformal except at z = 0, where it is not dened. (Conformal everywhere


if we view it as a mapping of the sphere).
We have

1 1 2 c 2 c2
u= ; (x + y 2 ) = x; x + y2 = ;
c c 2 4
which is circles centred on x axis and tangent to y axis.
We have

1 1 2 a 2 a2
v= ; (x + y 2 ) = y; x2 + y + = ;
a a 2 4
which is circles centred on y axis and tangent to x axis.
(b)
We have

1 z2 (x iy)2 x2 y 2 2ixy
f (z) = 2 = 4 = = )
z jzj (x2 + y 2 )2 (x2 + y 2 )2
( 2 2 2 2 cos(2 )
u = (xx2 +yy2 )2 = cos r2sin = r2
=) 2xy 2 cos sin sin(2 )
v = (x2 +y 2 )2 = r2
= r2

and

67
1 1
u= ; r2 = c cos (2 ) ; v= ; r2 = c sin (2 ) :
c c
Conformal everywhere except at z = 0.
(c)
We have that

u = r6 cos (6 )
f (z) = z 6 = r6 (cos (6 ) + i sin (6 )) )
v = r6 sin (6 )
Figure repeats itself. Rotate of fu = 0g by =12 is fv = 0g. Each grind
repeats itself every =6.
Conformal everywhere except at z = 0.

FIGURE II.6.1a NF FIGURE II.6.1b NF FIGURE II.6.1c NF

68
II.6.2
Sketch the families of level curves of u and v for f (z) = Log z = u+iv.
Relate your sketch to a gure in Section I.6.

Solution

II.6.2
/2
3/4 /4

3/4 /4
/2

We have

u = log jzj u = log r


f (z) = Log z = log jzj+i Arg z = log r+i ) ) :
v = Arg z v=

If u = constant and r = constant we have circles centred at 0, and if v =


constanat and = constanat we have rays issuing from 0, < < .
Refer to the Figure for the map w = Log z in section I.6. The sketch is of
the inverse images of horizontal and vertical lines in that Figure.

69
II.6.3
Sketch the families of level curves of u and v for the function f =
u + iv given by (a) f (z) = ez , (b) f (z) = e z , where is complex.
Determine where f (z) is conformal and where it is not conformal.

Solution
f (z) = ez = ex (cos y + i sin y) ) u = ex cos yv = ex sin y
u = const, x = log (c cos y) = log c log (cos y)
v = const, x = log (c sin y) = log c log (sin y)
Level curves are invariant under ????? in xdirection. Level curves are in-
variant under ????? in ydirection by . The function f (z) is conformal
everywhere.
f (z) = e z = e(a+ib)(x+iy) = eax by (cos (ay + bx) + i sin (ay + bx)) )
u = eax by cos (ay + bx)
v = eax by sin (ay + bx)
Figure repeat themselves
u = 0 at ay + bx = n + 2 , y = ab x + na + 2a . v = 0 at ay + bx = n ,
y = ab x + na
FIGURE II.6.3a NF FIGURE II.6.3b NF

70
II.6.4
Find a conformal map of the horizontal strip f A < Im z < Ag onto
the right half-plane fRe w > 0g. Hint. Recall the discussion of the
exponential function, or refer to the preceding problem.

Solution (A. Kumjian)

FIGURE II.6:4az FIGURE II.6:4aw


It is clearly intended that A > 0 so we will assume this. We dene the map
z
h : C ! C by h (z) = e 2A for all z 2 C. Then
z
h0 (z) = e 2A
2A
for all z 2 C. Hence, h is analytic with a nowhere vanishing derivative and
so h is conformal. Note that the exponential functions maps the horizontal
strip fz 2 C : =2 < Im z < =2g onto the right half-plane

fw 2 C : Re w > 0g = fw 2 C : =2 < Arg w < =2g :


Hence, h maps the horizontal strip fz 2 C : A < Im z < Ag onto the right
half-plane fw 2 C : Re w > 0g.

71
II.6.5
Find a conformal map of the wedge f B < arg z < Bg onto the right
half-plane fRe w > 0g. Assume 0 < B < .

Solution
Try w = z , multiply angels by 2B
, so eiB ! ei =2
,w=z =(2B)
does the trick

FIGURE II.6.5az FIGURE II.6.5aw

72
II.6.6
Determine where the function f (z) = z +1=z is conformal and where
it is not conformal. Show that for each w, there are at most two
values z for which f (z) = w. Show that if r > 1, f (z) maps the circle
fjzj = rg onto an ellipse, and that f (z) maps the circle fjzj = 1=rg
onto the same ellipse. Show that f (z) is one-to-one on the exterior
domain D = fjzj > 1g. Determine the image of D under f (z). Sketch
the images under f (z) of the circles fjzj = rg for r > 1, and sketch
also the images of the parts of the rays farg z = g lying in D.

Solution
f (z) = z + 1=z, f 0 (z) = 1 1=z 2 , f 0 (z) = 0 at z 2 = 1, z = 1.pNot dened
2
at z = 0, not conformal at z = 1. If f (z) = , then z = 4 =2.
If z = ei , then w = u + iv = cos + i sin + cos = i sin = , which
can be written u2 = ( + 1= )2 + v 2 = ( 1= )2 = 1, i.e. image is an ellipse,
replace by 1= and get same ellipse. Since f (D) = f (D), and f (z) is at
most twotoone, f (z) is onetoone on D. Since f (z) maps @D onto the
interval [ 2; 2], and f (z) maps onto C, the image of D is Cn [ 2; 2].

73
II.6.7
For the function f (z) = z + 1=z = u + iv, sketch the families of level
curves of u and v. Determine the images under f (z) of the top
half of the unit disk, the bottom half of the unit disk, the part
of the upper half-plane outside the unit disk, and the part of the
lower half-plane outside the unit disk. Hint. Start by locating
the images of the curves where u = 0, where v = 0, and where
v = 1. Note that the level curves are symmetric with respect to the
real and imaginary axis, and they are invariant under the inversion
z 7! 1=z in the unit circle.

Solution

74
II.6.8 ( From Hints and Solutions)
1 2 3 P L K

Consider f (z) = z + ei =z, where 0 < < . Determine where f (z)


is conformal and where it is not conformal and where it is not
conformal. Sketch the images under f (z) of the unit circle fjzj = 1g
and the intervals ( 1; 1] and [+1; +1) on the real axis. Show that
w = f (z) maps fjzj > 1g conformally onto the complement of a slit
plane in the w plane. Sketch roughly the images of the segments
of rays outside the unit circle farg z = ; jzj 1g under f (z). At
what angels do they meet the slit, and at what angles do they
approach 1?

Solution
f (z) = z + ei =z , f 0 (z) = 1 ei =z 2 , f 0 (z) = 0 at z 2 = ei . Not de-
ned at z = 0, not conformal at z = ei =2 . The expression f (z) =
ei =2 e i =2 z + 1= e i =2 z , shows that f (z) is a composition of the ro-
tation by =2, the function of Exercises 6 and 7, and a rotation by =2.
Thus f (z) maps fjzj > 1g onetoone onto the complement rotate of [ 2; 2]
by =2.

75
II.6.9
Let f = u + iv be a continuously dierentiable complex-valued func-
tion on a domain D such that the Jacobian matrix of f does not
vanish at any point of D. Show that if f maps orthogonal curves to
orthogonal curves, then either f or f is analytic, with nonvanishing
derivative.

Solution
Suppose f (0) = 0 and ux 6= 0 at 0. The tangents in the orthogonal directions
(1; t) and ( t; 1) are mapped to the tangents in the directions (ux ; vx ) +
t (uy ; vy ) and t (ux ; vx ) + (uy ; vy ).
The orthogonality of these directions for all t gives

h(ux ; vx ) + t (uy ; vy ) ; t (ux ; vx ) + (uy ; vy )i = 0;


can be rewritten

h(ux ; vx ) ; (uy ; vy )i+t (h(uy ; vy ) ; (uy ; vy )i h(ux ; vx ) ; (ux ; vx )i) t2 h(uy ; vy ) ; (ux ; vx )i = 0:

The systems of equations

(1) ux u y + v x v y = 0
(2) u2y + vy2 (u2x + vx2 ) = 0
most hold to get orthogonality. Put (1) into (2) and get

u2x u2y + vy2 u2x vx2 = vx2 vy2 + u2x vy2 u4x u2x vx2 =
= vx2 vy2 u2x + u2x vy2 u2x = vx2 + u2x vy2 u2x = 0:

If ux 6= 0 we are lead to ux = vy and uy = vx .


Tangent to curve s ! (s; ts) is tangent to image curve s + (u (s; ts) ; v (s; ts))
(1; 0) ! (ux ; vx ), (0; 1) ! (uy ; vy ), orthogonal ) (uy ; vy ) = C ( vx ; uy ) for
some. A ????? gives C = 1, same sign holds in ?????.
Either ux = vy , uy = vx ) conformal in neighborhood of 0, or ux = vy ,
uy = vx ) anti conformal in neighborhood of 0. ux = vy and uy = vx .
So Cauchy Riemann is satised for either f or f .

76
II.7.1
Compute explicitly the fractional linear transformations determined
by the following correspondences of triples
(a) (1 + i; 2; 0) 7 ! (0; 1; i 1) (e) (1; 2; 1) 7 ! (0; 1; 1)
(b) (0; 1; 1) 7 ! (1; 1 + i; 2) (f) (0; 1; i) 7 ! (0; 1; 1)
(c) (1; 1 + i; 2) 7 ! (0; 1; 1) (g) (0; 1; 1) 7 ! (0; 1; i)
(d) ( 2; i; 2) 7 ! (1 2i; 0; 1 + 2i) (h) (1; i; 1) 7 ! (1; 0; 1)

Solution

(w w0 ) (w1 w2 ) (z z0 ) (z1 z2 )
=
(w w2 ) (w1 w0 ) (z z2 ) (z1 z0 )
(a)
We have the points

z0 = 1 + i w0 = 0
z1 = 2 w1 = 1
z2 = 0 w2 = i 1
The mapping is

(w w0 ) (1 w2 =w1 ) (z z0 ) (z1 z2 )
= ;
(w w2 ) (1 w0 =w1 ) (z z2 ) (z1 z0 )
take limit as w1 ! 1, to obtain

w 0 (z (1 + i)) (2 0) 2iz + (2 2i)


= )w= :
w (i 1) (z 0) (2 (1 + i)) z 2
(b)
We have the points

z0 = 0 w0 = 1
z1 = 1 w1 = 1 + i
z2 = 1 w2 = 2
The mapping is

(w w0 ) (w1 w2 ) (z z0 ) (z1 =z2 1)


= ;
(w w2 ) (w1 w0 ) (z=z2 1) (z1 z0 )
take limit as z2 ! 1, to obtain

77
(w 1) ((1 + i) 2) z 0 2z (1 + i)
= )w= :
(w 2) ((1 + i) 1) 1 0 z (1 + i)
(c)
We have the points

z0 = 1 w0 = 0
z1 = 1 + i w1 = 1
z2 = 2 w2 = 1
The mapping is

(w w0 ) (w1 =w2 1) (z=z0 1) (z1 z2 )


= ;
(w=w2 1) (w1 w0 ) (z z2 ) (z1 =z0 1)
take limit as z0 ; w2 ! 1, to obtain

w 0 (1 + i) 2 i 1
= )w= :
1 0 z 2 z 2
(d)
We have the points

z0 = 2 w0 = 1 2i
z1 = i w1 = 0
z2 = 2 w2 = 1 + 2i
The mapping is

(w w0 ) (w1 w2 ) (z z0 ) (z1 z2 )
= ;
(w w2 ) (w1 w0 ) (z z2 ) (z1 z0 )
obtain

(w (1 2i)) (0 (1 + 2i)) (z ( 2)) (i 2)


= ) w = iz + 1:
(w (1 + 2i)) (0 (1 2i)) (z 2) (i ( 2))
(e)
We have the points

z0 = 1 w0 = 0
z1 = 2 w1 = 1
z2 = 1 w2 = 1

78
The mapping is

(w w0 ) (w1 =w2 1) (z z0 ) (z1 =z2 1)


= ;
(w=w2 1) (w1 w0 ) (z=z2 1) (z1 z0 )
take limit as z2 ; w2 ! 1, to obtain
w 0 z 1
= )w=z 1:
1 0 2 1
(f)
We have the points

z0 = 0 w0 = 0
z1 = 1 w1 = 1
z2 = i w2 = 1
The mapping is

(w w0 ) (w1 =w2 1) (z z0 ) (1 z2 =z1 )


= ;
(w=w2 1) (w1 w0 ) (z z2 ) (1 z0 =z1 )
take limit as z1 ; w2 ! 1, to obtain
w 0 z 0 z
= )w=
1 0 z i z i
(g)
We have the points

z0 = 0 w0 = 0
z1 = 1 w1 = 1
z2 = 1 w2 = i
The mapping is

(w w0 ) (1 w2 =w1 ) (z z0 ) (z1 =z2 1)


= ;
(w w2 ) (1 w0 =w1 ) (z=z2 1) (z1 z0 )
take limit as z2 ; w1 ! 1, to obtain
w 0 z 0 iz
= )w= :
w i 1 0 z 1
(h)

79
We have the points

z0 = 1 w0 = 1
z1 = i w1 = 0
z2 = 1 w2 = 1
The mapping is

(w w0 ) (w1 w2 ) (z z0 ) (z1 z2 )
= ;
(w w2 ) (w1 w0 ) (z z2 ) (z1 z0 )
obtain

(w 1) (0 ( 1)) (z 1) (i ( 1)) iz + 1
= )w= :
(w ( 1)) (0 1) (z ( 1)) (i 1) z+i

80
II.7.2
Consider the fractional linear transformation in Exercise 1a above,
which maps 1 + i to 0, 2 to 1, and 0 to i 1. Without referring to
an explicit formula, determine the image of the circle fjz 1j = 1g,
the image of the disk fjz 1j < 1g, and the image of the real axis.

Solution
It is obvious that the tripple (1 + i; 2; 0) lies on the circle jz 1j = 1 in the
z plane. From exercise II.7.1a we know that points in the w plane the
tripple is mapped on, i.e,

z0 = 1 + i w0 = 0
z1 = 2 w1 = 1
z2 = 0 w2 = i 1
The circle jz 1j = 1 is mapped to the straight line through 0 and i 1, i.e
Im w = Re w since tree points determines the "circle". By preservation of
orientation the disk goes to the half-plane to lower left of the straight line
through 0 and i 1.
Image of the real axis is "circle" orthogonal to image of jz 1j = 1, that must
be the straight line through i 1, with slope 1, i.e. the line Im w = Re w + 2.

II.7.2 z-plane II.7.2 w-plane

81
II.7.3
Consider the fractional linear transformation that maps 1 to i, 0
to 1 + i, 1 to 1. Determine the image of the unit circle fjzj = 1g,
the image of the open unit disk fjzj < 1g, and the image of the
imaginary axis. Illustrate with a sketch.

Solution
We have the points

z0 = 1 w0 = i
z1 = 0 w1 = 1 + i
z2 = 1 w2 = 1
The mapping is

(w w0 ) (w1 w2 ) (z z0 ) (z1 z2 )
= ;
(w w2 ) (w1 w0 ) (z z2 ) (z1 z0 )
obtain

(w i) ((1 + i) 1) (z 1) (0 ( 1)) i 1
= )w=
(w 1) ((1 + i) i) (z ( 1)) (0 1) z+i

By our mapping w = (i 1) = (z + i), we have that i on the unitcircle in


the z plane is mapped to 1 in the w plane. The circle jzj = 1 is mapped
to the straight line through i and 1, i.e Im w = Re w + 1 since tree points
determines the "circle". By preservation of orientation the disk goes to the
half-plane to upper left of the straight line through i and 1.
The tripple (1; 0; 1) on the real axis is mapped to the points (i; 1 + i; 1) in
the w plane, it must be so that p the real axis in the z plane is mapped
1 1 2
onto the circle w 2
+ 2
i = 2
in the w plane.
Imaginary axis must be mapped to a circle through 1+i, that is orthogonal to
image of real line, this is the straight line through 1+i and 0, i.e Im w = Re w.

82
II.7.3 z-plane II.7.3 w-plane

83
II.7.4
Consider the fractional linear transformation that maps 1 to i,
1 to 2i, and i to 0. Determine the image of the unit circle fjzj = 1g,
the image of the open unit disk fjzj < 1g, and the image of the
interval [ 1; +1] on the real axis. Illustrate with a sketch.

Solution
We have the points

z0 = 1 w0 = i
z1 = 1 w1 = 2i
z2 = i w2 = 0
The mapping is

(w w0 ) (w1 w2 ) (z z0 ) (z1 z2 )
= ;
(w w2 ) (w1 w0 ) (z z2 ) (z1 z0 )
obtain

(w ( i)) (2i 0) (z ( 1)) (1 i) ( 6 2i) z 2 + 6i


= )w=
(w 0) (2i ( i)) (z i) (1 ( 1)) 5z 3 + 4i

It is obvious that the tripple ( 1; 1; i) lies on the circle jzj = 1 in the z


plane. And we can see that thiese three points are mapped to three points
on the imaginary axis in the w plane, i.e Re w = 0, since tree points
determines the "circle". By preservation of orientation the disk goes to the
right half-plane.
The interval [ 1; 1] must be mapped to a circle through i and 2i that is
orthogonal to image of the unit circle, this is an arc of the circle w 21 i = 32 .
We have that w (0) = 65 25 i must be an point on the arc, thus the arc is the
circle w 21 i = 23 in the right half-plane from i to 2i.

84
II.7.4 z-plane II.7.4 w-plane

85
II.7.5
What is the image of the horizontal line through i under the frac-
tional linear transformation that interchanges 0 and 1 and maps 1
to 1 + i? Illustrate with a sketch.

Solution
We have the points

z0 = 0 w0 = 1
z1 = 1 w1 = 0
z2 = 1 w2 = 1 + i
The mapping is

(w w0 ) (w1 w2 ) (z z0 ) (z1 z2 )
= ;
(w w2 ) (w1 w0 ) (z z2 ) (z1 z0 )
obtain

(w 1) (0 (1 + i)) (z 0) (1 ( 1)) iz i
= )w=
(w (1 + i)) (0 1) (z ( 1)) (1 0) z i

The tripple (0; 1; 1) on the real axis is mapped to the points (1; 0; 1 + i) in
the w plane, it must be so pthat the real axis in the z plane is mapped onto
the circle w 1
2
+ 12 i = 22 in the w plane, since tree points determines
the "circle". The real axis and the line through i have one common point at
innity. The common point at innity for both real axis and the line through
i must be mapped to the same point in the w plane, we have that the point
is w (1) = i. The two lines in the z plane is parallell, thus the images of
the lines must go through i in the w plane and be parallell in that point,
thus the mapping of the horizontal line through i must be the line through
i, which is tangent line to circle at i, i.e. Im z = Re z + 1. Remark that the
image can not be a circle because w (i) = 1, so the image of the horizontal
line must contain the point at innity.

86
II.7.5 z-plane II.7.5 w-plane

87
II.7.6
Show that the image of a straight line under the inversion z 7! 1=z
is a straight line or circle, depending on whether the line passes
through the origin.

Solution
The image of the straight line ax+by = c is (kontrollera x iy i lsningshftet)
1 1 z 1
w= = = 2 = (x iy)
z zz jzj r2
where u = x=r2 and v = y=r2 . Get
ax + by c c
au bv = 2
= 2 = 2 = c u2 + v 2
r r x + y2
The image is the solution of c (u2 + v 2 ) au + bv = 0.
If c = 0 we have that

au + bv = 0;
this is a straight line through 0.
If c 6= 0 we have

c u2 + v 2 au + bv = 0;
which can be rewritten as
0s 12
2 2
a 2 b a 2 b
u + v+ =@ + A ;
2c 2c 2c 2c

it is a circle through 0.

Solution (K. Seip)


Set f (z) = z1 . Then 1 2 f (S) , 0 2 S, which proves the claim.

88
II.7.7
Show that the fractional linear transformation f (z) = (az + b) = (cz + d)
is the identity mapping z if and only if b = c = 0 and a = d 6= 0.

Solution
We have that
az + b
= z , az + b = cz 2 + dz , cz 2 + (d a) z b = 0:
cz + d
Put in values on z for intance 0 and 1 the expession becomes

(1) c (d a) = 0; z = 1:
(2) b=0 z=0
(3) c + (d a) = 0; z = 1:
From (2) we have b = 0 and if we add (1) and (3) we have c = 0.
Thus the fractional linear transformation f (z) is the identity mapping if and
only if b = c = 0, a = d 6= 0.

Solution
Set R = R [ f1g. Suppose f (z) = az+b
cz+d
maps R to R . Then 0 7! db 2 R ,
1 7! ac 2 R , 0 - ab 2 R , 1 - dc 2 R . At least one of theese rations
is dierent from 0 and 1, and since one of a; b; c; d can be chosen freely, the
result follows.

89
II.7.8
Show that any fractional linear transformation can be represented
in the form f (z) = (az + b) (cz + d), where ad bc = 1. Is this repre-
sentation unique?

Solution
If

w = ( z + )( z + )
divide each coe cient by the square root of

to obtain representation with

ad bc = 1:
Representation is not unique, as can multiply all coe cients by 1.

90
II.7.9
Show that the fractional linear transformations that are real on
the real axis are precisely those that can be expressed in the form
(az + b) = (cz + d), where a; b; c;and d are real.

Solution (K. Seip)


Set R = R[f1g. Suppose f (z) = az+b
cz+d
maps R to R . Then 0 7 ! db 2 R ,
1 7! ac 2 R , 0 - ab 2 R , 1 - dc 2 R . At least one of these is
dierent from 0 and 1, and since one of a; b; c; d can be chosen freely, the
result follows.

91
II.7.9
Show that the fractional linear transformations that are real on
the real axis are precisely those that can be expressed in the form
(az + b) = (cz + d), where a; b; c;and d are real.

Solution (A. Kumjian)


Set R = R [ f1g, note that a fractional linear transformation f : C ! C
is real on the real axis i f (R ) R (since f is continuous). For A = ac db
with det A 6= 0 we dene the fractional linear transformation fA : C ! C
by the formula fA (z) = (az + b) = (cz + d). It remains to prove that given a
fractional linear transformation f , we have f (R ) R i f = fA for some
2 2 matrix A with real entries.

Let f : C ! C be a fractional linear transformation. Suppose that f = fA


where A is a 2 2 matrix with real entries a; b; c and d as above. Then for
x 2 R we have
ax + b
f (x) = fA (x) = 2R :
cx + d
This is clear for x 2 R but requires a little work for x = 1. We have

az + b a=c if c 6= 0;
f (1) = lim =
z!1 cz + d 1 if c = 0:
Hence, f (R ) R .
Conversely, suppose that f (R ) R , we must show that f = fA for some 2
2 matrix with real entries. If A is a 2 2 matrix (with non-zero determinant),
then fA 1 = (fA ) 1 . Hence, f = fA for some 2 2 matrix A with real entries
i its inverse f 1 has the same property.
By the unlikeness part of the theorem on page 64, f is completely determined
by the three extended real numbers x0 = f (0), x1 = f (1) and x1 = f (1)
(recall that we have assumed f (R ) R ). So it su ces to show that
the fractional linear transformation g for which g (x0 ) = 0, g (x1 ) = 1 and
g (x1 ) = 1 has the requisite properties, because we must have g = f 1
(again by the uniqueness part of the theorem).
There are four cases to consider. The rst case is when x0 ; x1 ; x1 6= 1 and
the remaining three cases are x0 = 1; x1 = 1 and x1 = 1. The formula
for g (z) in the four cases is given as follows (in the rst case k = xx11 xx10 2 R):

92
x x0 x1 x1
k if x0 ; x1 ; x1 6= 1; if x0 = 1;
x x1 x x1
x x0 x x0
if x1 = 1; if x1 = 1:
x x1 x1 x0
In each case g (z) has the desired form. Hence, f = fA for some 2 2 matrix
A with real entries as required.

93
II.7.10
Suppose the fractional linear transformation (az + b) = (cz + d) maps
R to R, and ad bc = 1. Show that a; b; c; and d are real or they are
all pure imaginary.

Solution
Because R is mapped on R and the orientation is perserved, f (x) will be
either increasing or decreasing.

Case 1 : Suppose f (x) is increasing, then f 0 (x) = 1= (cx + d)2 > 0 for all
x ) c; d are real. f (z) = (az + b) = (cz + d), then ax + b is real for all x 2 R,
so a; b are also real.

Case 2 : Suppose f (x) is decreasing, then f 0 (z) = 1= (cz + d)2 < 0 for all
x ) c; d are pure imaginary. f (z) = (az + b) = (cz + d), then ax + b are pure
imaginary for all x 2 R, so a; b are also pure imaginary.

94
II.7.11
Two maps f and g are conjugate if there is h such that g = h f h 1 .
Here the conjugating map h is assumed to be one-to-one, with
appropriate domain and range. We can think of f and g as the
"same" map, after the change of variable w = h (z). A point z0 is
a xed point of f if f (z0 ) = z0 . Show the following. (a) If f is
conjugate to g, then g is conjugate to f .
(b) If f1 is conjugate to f2 and f2 to f3 , then f1 is conjugate to f3 .
(c) If f is conjugate to g, then f f is conjugate to g g, and more
generally, the m fold composition f f (m times) is conjugate
to g g (m times).
(d) If f and g are conjugate, then the conjugating function h maps
the xed points of f to xed points of g. In particular, f and g
have the same number of xed points.

Solution
(a)
If f is conjugate to g then

1 1
g=h f h )f =h g h
thus g is conjugate to f
(b)
If f1 = h f2 h 1 and f2 = g f3 g 1
we get

1 1 1
f1 = h g f3 g h = (h g) f3 (h g) :
(c)

m times
z }| {
g=h f h 1 theng g : : : g =
=h f h 1 h f h 1
::: h f h 1
=
m times
z }| {
1
=h f f ::: f h

(d)
g=h f h 1 , f (z0 ) = z0 , and w0 = h (z0 ), then

95
1
g (w0 ) = h f h (w0 ) = h (f (z0 )) = h (z0 ) = w0 :
We have that f and g have the same number of xed points since every point
of g is mapped to a xed point of f by the function h 1 .

96
II.7.12
Classify the conjugacy classes of fractional linear transformations
by establishing the following
(a)
A fractional linear transformation that is not the identity has either
1 or 2 xed points, that is, points satisfying f (z0 ) = z0 .
(b)
If a fractional linear transformation f (z) has two xed points, then
it is conjugate to the dilation z 7! az with a 6= 0, a 6= 1, that is, there
is a fractional linear transformation h (z) such that h (f (z)) = ah (z).
Is a unique? Hint. Consider a fractional linear transformation that
maps the xed points to 0 and 1.
(c)
If a fractional linear transformation f (z) has exactly one xed
point, then it is conjugate to the translation 7! + 1. In other
words, there is a fractional linear transformation h (z) such that
h (f (h 1 ( ))) = + 1, or equivalently, such that h (f (z)) = h (z) + 1.
Hint. Consider a fractional linear transformation that maps the
xed point to 1.

Solution
a) az+b
cz+d
= z , az + b = cz 2 + dz , cz 2 + (d a) z b = 0. If this is = 0
for every z, we get w (z) = z. It is not identically equall to zero, so it has
either 1 or 2 nite solutions. If c 6= 0, it has 2 solutions possibly one with
multiplicity and 1 is not a xed point. If c = 0, it has one nite solution
z1 = b= (d a), and z2 = 1 is a xed point, so they are two.
b) Make a change of variables h (z) = ( z + ) = ( z + ) that maps xed
points to 0 and 1 . Then h f h 1 is FLT with xed points at 0 and 1.
(h f h 1 ) (w) = aw for some a 6= 0, a 6= 1. f is conjugate to a dilation
Suppose aw is conjugate to Aw, i.e. 9h, (h (ah 1 )) (w) = Aw, h (az) =
Ah (z). h must map xed points to xed points, so either h (z) = cz or
h (z) = c=z. So either h (z) = cz ) h (az) = caz = Ah (z) = Acz ) A = a
or h (z) = c=z ) h (az) = c= (az) = Ah (z) = Ac=z ) A = 1=a. Thus a is
not unique.
c) Suppose h maps the xed points to 1, then h f h 1 has only one xed
point at 1, so (h f h 1 ) (w) = aw+b. Since this has no nite xed points,
aw +b = w has no solutions and a = 1, b 6= 0. Thus (h f h 1 ) (w) = w +b.

97
Now w + b is conjugate to w + 1, by a dilation g (w) = Aw. w ! w=A !
w=A + b ! w + Ab, take A = 1=b. So

1 1
g h f h g (w) = w + 1 ()
() (g h) f (g h) 1 (w) = w + 1 ()
() ((g h) f ) (z) = (g h) (z) + 1:

98
III 1 2 3 4 5 6 7 8 9 10 11 12 13 14 15 16 17 18 19
1
2
3
4
5
6
7
8

1
III.1.1 R 2
Evaluate y dx + x2 dy along the following paths from (0; 0) to
(2; 4),
(a) the arc of the parabola y = x2 ,
(b) the horizontal interval from (0; 0) to (2; 0), followed by the ver-
tical interval from (2; 0) to (2; 4), (c) the vertical interval from (0; 0)
to (0; 4), followed by the horizontal interval from (0; 4) to (2; 4).

Solution
(a)

Z Z 2 Z 2
2 2 x=t dx = dt; 0 t 2 2 2 72
y dx+x dy = = t dt+ t2 2t dt = :
y = t2 dy = 2t dt 0 0 5

(b)

Z Z 2 Z 2
2 2 x=t dx = dt 0 t 2 2
y dx + x dy = = 0 dt + t2 0 dt = 0
1
y=0 dy = 0 dt 0 0
Z Z 4 Z 4
2 2 x=2 dx = 0 dt 0 t 4 2
y dx + x dy = = t 0 dt + 22 dt = 16
y=t dy = dt 0 0
2
Z Z Z
2 2 2 2
y dx + x dy = y dx + x dy + y 2 dx + x2 dy = 16:
1 2

(c)

Z Z 4 Z 4
2 2 x=0 dx = 0 dx 0 t 4 2
y dx + x dy = = y 0 dt + 02 dt = 0
1
y=t dy = dt 0 0
Z Z 2 Z 2
x=t dx = dt 0 t 2
y 2 dx + x2 dy = = 42 dt + t2 0 dt = 32
y = 4 dy = 0dt 0 0
2
Z Z Z
y 2 dx + x2 dy = y 2 dx + x2 dy + y 2 dx + x2 dy = 32:
1 2

2
III.1.2 R
Evaluate xy dx both directly and using Greens theorem, where
R 1),
is the boundary of the square with vertices at (0; 0), (1; 0), (1;
and (0; 1).

Solution (A. Kumjian)

VII.1.2

1

4
3
2
1
1

Denote the square by D and note that xy dx = P dx + Q dy where P = xy


and Q = 0. Then P and Q are continuously dierentiable on D and = @D,
hence by Greens Theorem we have,

Z Z ZZ Z 1 Z 1
@Q @P 1
xy dx = P dx+Q dy = dx dy = x dx dy = :
@D D @x dy 0 0 2

Observe that = 1 + 2 + 3 + 4 where 1 and 3 are the bottom and


top of the square while 2 and 4 are the last two sides taken in the order
indicated by the order of the vertices in the statement of the problem (so the
boundary is oriented counter clockwise). Note that the path integrals on 2
and 4 are zero because the edges are vertical.

Z Z Z Z 1 Z 1
1 1
xy dx = xy dx + xy dx = x 0 dx x 1 dx = 0 = :
1 3 0 0 2 2

3
III.1.3 R
Evaluate @D x2 dy both directly and using Greens theorem, where
D is the quarter-disk in the rst quadrant bounded by the unit
circle an the two coordinate axes.

SolutionR
Evaluate @D x2 dy directly, set @D = 1 + 2 + 3.

Z Z 1
2 x=t dx = dt 0 t 1
x dy = = t2 0 dt = 0
1
y=0 dy = 0 dt 0
Z Z =2
2 x = cos t dx = sin t dt 0 t =2 2
x dy = = cos2 t cos t dt =
2
y = sin t dy = cos t dt 0 3
Z Z 1
x=0
dx = 0 dt 0 t 1
x2 dy = = 02 dt = 0
t y=1
dy = dt 0
3
Z Z Z Z
2 2 2 2
x dy = x dy + x dy + x2 dy =
@D 1 2 3
3
R
Now we evaluate @D x2 dy, this time using Greens theorem. In this case,
P (x; y) = 0 and Q (x; y) = x2 .

Z ZZ
2
x dy = 2x dxdy =
@D D
ZZ Z 1 Z =2
x = r cos dx dy = r dr d 0 r 1
=2 x dxdy = =2 r cos rdrd =
D y = r sin 0 =2 0 0
Z 1 Z =2
2 2
=2 r dr cos d = :
0 0 3

4
III.1.4 R
Evaluate y dx both directly and using Greens theorem, where
is the semicircle in the upper half-plane from R to R.

SolutionR
Evaluate y dx directly, set = 1 + 2.

Z Z R
x=t dx = dt R t R
y dx = = 0 dt = 0
1
y=0 dy = 0 dt R
Z Z
x = R cos t dx = R sin t dt 0 t R2
y dx = = R sin t R sin t dt =
2
y = R sin t dy = R cos t dt 0 2
Z Z Z 2
2 2 R
x dy = x dy + x2 dy =
@D 1 2
2
R
Now we evaluate y dx, this time using Greens theorem. In this case,
P (x; y) = y and Q (x; y) = 0.

Z ZZ
y dx = dx dy =
@D D
ZZ Z R Z
x = r cos dx dy = r dr d 0 r R
= dx dy = = r dr d =
D y = r sin 0 0 0
Z R Z 2
R
= r dr d = :
0 0 2

5
III.1.5 R R
Show that @D x dy is the area of D, while @D y dx is minus the area
of D.

Solution
Using
R in the two cases
For @D x dy, we have P = 0 and Q = x, using Greens theorem we have
Z Z Z
x dx = dx dy = Area D:
@D D
R
For @D
y dx, we have P = y and Q = 0, using Greens theorem we have
Z Z Z
y dx = dx dy = Area D:
@D D

6
III.1.6
Show that if P and Q are continuous complex-valued functions on
a curve , then
R P dx R Qdy
+ ; (z = x + iy)
z w z w
is analytic for w 2 Cn . Express F 0 (w) as a line integral over .

Solution
Dierentiate by hand, use uniform convergence of

1 1 1 1 1
= !
w z (w + w) z w (z w) (z (w + w)) (z w)2

as w ! 0, uniformly for z 2 . Get


R P dx R Qdx
F 0 (w) = 2 + ; z = x + iy
(z w) (z w)2

7
III.1.7
Show that the formula in Greens theorem is invariant under coor-
dinate changes, in the sense that if the theorem holds for a bounded
domain U with piecewise smooth boundary, and if F (x; y) is a
smooth function that maps U one-to-one onto another such domain
V and that maps the boundary of U one-to-one smoothly onto the
boundary of V , then Greens theorem holds for V . Hint. First note
the change of variable formulae for line and area integrals, given
by

Z Z
@ @
Pd = (P F) dx + dy ;
@V @x @y
ZZ Z@U
Z
Rd d = (R F ) det JF dx dy;
V U

where F (x; y) = ( (x; y) ; (x; y)), and where JF is the Jacobian ma-
trix
R of F . Use these formulae, with R = @P=@ . The summand
Q d is treated similarly.

Solution

ZZ ZZ
@P @P
d d = ( (x; y) ; (x; y)) det (JF ) dx dy =
@ @
V U

F (x; y) = ( (x; y) ; (x; y))

@F @ @
@x
= @x
+ @x
@F @ @
@y
= @y
+ @y

@ @ @ @
det JF (x; y) = =
@x @y @y @x
ZZ
@P @ @ @ @
= ( (x; y) ; (x; y)) dx dy
@ @x @y @y @x
U

Using Greens theorem

8
Z
Pd =
@V
Z Z
@ @ @ @
= (P F ) dx + dy = P ( (x; y) ; (x; y)) dx + dy =
@U @x @y @U @x @y
ZZ
@ @ @ @
= P + P dx dy =
U @y @x @x @y
ZZ
@ @ @2 @ @ @2
= P ( (x; y) ; (x; y)) P + P ( (x; y) ; (x; y)) +P dx dy =
@y @x @y@x @x @y @x@y
U
ZZ
@P @ @P @ @ @P @ @P @ @
= + + dx dy =
@ @y @ @y @x @ @x @ @x @y
U
ZZ
@P @ @ @P @ @
= + dx dy =
@ @y @x @ @x @y
U
ZZ
@P
= JF dx dy:
@
U

R
Similar argument as above hold for Qd
Here we use that
Z Z
@ @
Qd = (Q F ) dx + dy
@V @U @x @y
and replace R = @P=@ by R=@Q=@ to conclude that
Z ZZ
@Q
Qd = d d :
@V V @

9
III.1.8
Prove Greens theorem for the rectangle dened by x0 < x < x1 and
y0 < y < y1 (a) directly, and (b) using the result for triangles.

Solution

VII.1.8a VII.1.8b
III III

IV R II IV S II
T

I I

(a)
Make gure and set the linesegment between (x0 ; y0 ) and (x1 ; y0 ) to 1 and
proceed in this way in counterclockwise direction. We have
Z Z Z Z
Q dy = P dx = Q dy = P dx = 0:
1 2 3 4

Integrate around the rectangle

10
ZZ
@Q @P
dx dy =
R @x @y
Z y1 Z x1 Z x1 Z y1
@Q @P
= dx dy dy dx =
y0 x0 @x @y
Z y1 Zx0x1 y0

= [Q (x1 ; y) Q (x0 ; y)] dy [P (x; y1 )


P (x; y0 )] dx =
y0
Z y1 Z y1 Zx0x1 Z x1
= Q (x1 ; y) dy Q (x0 ; y) dy P (x; y1 ) dx + P (x; y0 ) dx =
y0 y0 x0 x0
Z x1 Z y1 Z x1 Z y1
= P (x; y0 ) dx+ Q (x1 ; y) dy P (x; y1 ) dx Q (x0 ; y) dy =
x0 y0 x0 y0
Z Z Z Z
= P dx+ Q dy + P dx+ Q dy =
1 2 3 4
Z
= (P dx + Q dy) :
@R

(b)
Write R = S [ T . The integrals on the diagonal are in dierent directions
and will cancel

Z
(P dx + Q dy) =
@R Z Z
= (P dx + Q dy) + (P dx + Q dy) =
@S @T
ZZ ZZ
@Q @P @Q @P
= dx dy + dx dy =
S @x @y T @x @y
ZZ
@Q @P
= dx dy:
R @x @y

11
III.2.1
Determine whether each of the following line integrals is indepen-
dent of path. If it is, nd a function h such that dh = P dx + Q dy.
If it is not, nd a closed path around which the integral is not
zero. (a) x dx + y dy, (b) x2 dx + y 5 dy, (c) y dx + x dy, (d) y dx x dy.

Solution
We compute the partial derivatve because if h exist we have that dh =
P dx + Qdy where @P=@y = @Q=@x.
(a)

P =x Q=y
@P @Q
@y
=0 =0
R x2
R@x y2
P dx = 2 + g (y) Qdy = 2
+ f (x)
x2 +y 2
)h= 2
(b)

P = x2 Q = y5
@P @Q
=0 =0
R@y 3
@x
R y6
P dx = x3 + g (y) Qdy = 6
+ f (x)
2x3 +y 6
)h= 6
(c)

P =y Q=x
@P @Q
=1 =1
R@y R@x
P dx = xy + g (y) Qdy = xy + f (x)
) h = xy
(d)

P =y Q= x
@P
@y
= 1 @Q
@x
= 1
The line integral is not independent of path, we use Greenstheorem
Z ZZ
(y dx x dy) = 2 dx dy = 2 :
jzj=1 jzj<1

12
III.2.2
Show that the dierential
ydx + xdy
; (x; y) 6= (0; 0) ;
x2 + y 2
is closed. Show that it is not independent of path on any annulus
centered at 0.

Solution
y x
P = x2 +y 2
Q= x2 +y 2
@P (x2 +y2 )+y(2y) y 2 x2 @Q (x2 +y2 ) x(2x) y 2 x2
@y
= (x2 +y 2 )2
= (x2 +y 2 )2 @x
= (x2 +y 2 )2
= (x2 +y 2 )2

Because @P=@y = @Q=@x;the dierential is closed and we can use Greens


Theorem

I
P dx + Q dy =
jzj=r
I I
y x 1
= dx+ 2 dy = 2 y dx + x dy =
x2 + y 2 x + y2 r
jzj=r jzj=r
ZZ
1 1
= 2 (1 + 1) dx dy = 2 2 r2 = 2 6= 0
r r
jzj<r

Thus the line integral is not independent of path.

13
III.2.3
Suppose P and Q are smooth functions on the annulus fa < jzj < bg
that Hsatisfy @P=@y = @Q=dx. Show directly using Greens theorem
that jzj=r P dx + Qdy is independent of the radius r, for a < r < b.

Solution (A. Kumjian)

III.2.3

2 D

Let r1 ; r2 be given so that a < r1 < r2 < b. We must prove that


I I
P dx + Qdy = P dx + Qdy:
jzj=r1 jzj=r2

Let D denote the annulus fz 2 C : r1 < jzj < r2 g. Observe that D is a


bounded domain with piecewise smooth boundary @D = 1 [ 2 where 1
denotes the circle fz 2 C : jzj = r1 g with clockwise orientation while 2 de-
notes the circle fz 2 C : jzj = r2 g with counter clockwise orientation. More-
over, both P and Q are continuously dierentiable on D = D [ @D. Hence,
Greens Theorem applies and we obtain
Z ZZ
@Q @P
P dx + Qdy = dxdy = 0;
@D D @x @y
and since, as noted above, @D consists of the two circles with opposite ori-
entation,
Z I I
P dx + Qdy = P dx + Qdy P dx + Qdy:
@D jzj=r2 jzj=r1
H
Hence, equation ( ) holds and jzj=r
P dx + Qdy is independent of the radius
r, for a < r < b.

14
III.2.4
Let P and Q be smooth functions on D satisfying @P=@y = @Q=@x.
Let 0 and 1 be two closed paths in D such that the straight line
segment Rfrom 0 (t) to 1 R(t) lies in D for every parameter value
t. Then P dx + Q dy = P dx + Q dy. Use this to give another
0 1
solution to the preceding exercise.

Solution
Use the theorem on page 81. Use the straight lines to deform 0 to 1 . Dene
s (t) = s 1 (t) + (1 s) 0 (t), 0 6 s 6 1, a1 6 t 6 b1 . The theorem on page
81 applies. Obtain the result for the annulus above by parameterizing the
circles jzj = r0 and jzj = r1 by 0 (t) = r0 e2 it , 1 (t) = r1 e2 it , 0 6 t 6 1. The
straight line segments joining 0 (t) to 1 (t) are radical are in the annulus,
so the rst part applies.

15
III.2.5
Let 0 (t) and 1 (t), 0 t 1, be paths in the slit annulus fa < jzj < bg n ( b; a)
from A to B. Write down explicitly a family of paths s (t) from A
to B in the slit annulus that deforms continuously to 1 .
Suggestion. Deform separately the modulus and the principal value
of the argument.

Solution
Write 0 (t) = r0 ei 0 (t) , 0 6 t 6 1 , when < 0 (t) < , r0 (t), 0 (t)
continuous. (Use the fact that (t) = Arg 0 (t) is continuous on the slit
annulus) Also 1 (t) = r1 ei 1 (t) .
Then consider

s (t) = [sr1 (t) + (1 s) r0 (t)] ei[s 1 (t)+(1 s) 0 (t)]


; 0 s 1:

This does the trick, it deform 0 to 1 continuously in D, and each s is a


path in D from A to B.
We see that s (t) belongs to the slit annulus for 0 s 1 and 0 t 1
since for every 0 5 1 we have

a < min (r0 (t) ; r1 (t)) sr1 (t) + (1 s) r0 (t) max (r0 (t) ; r1 (t)) < b

and

< min ( 0 (t) ; 1 (t)) s 1 (t) + (1 s) 0 (t) max ( 0 (t) ; 1 (t)) <

16
III.2.6
Show that any closed path (t), 0 t 1, in the annulus fa < jzj < bg
can be deformed continuously to the circular path (t) = (0) e2 imt ,
0 t 1, for some integer m. Hint. Reduce to the case where
j (t)j j (0)j is constant. Then start by nding a subdivision
0 = t0 < t1 < < tn = 1 such that arg (t) has a continuous deter-
mination on each interval tj 1 t tj .

Solution
We rst note that there are two cases. For the closed path (t) enclosing
the origin we can deform (t) to a curve with constant modulus. Suppose
j (t)j = r for 0 6 t 6 1 and (0) = (1) = 1 where a < r < b. Follow
hint, write (t) = rei j (t) , tj 1 6 t 6 tj . Note j (tj ) j+1 (tj ) is a integral
multiple of 2 , since (t) is continuous. Add multiples of 2 to the j s,
obtain (t) continuous for 0 6 t 6 1 such that (0) = 0 and (t) = rei (t) .
Note (1) = 2 m for some integer m. Deform by s (t) = rei[(1 s) (t)+2 sm] .
The second case is when (t) does not enclose the origin. For this case (t)
the counter of a region that lies entirely inside the annulus. Therefrom the
curve (t) may be deformed to any point of this region. In particular it
can be continuously deformed to the point (0) = (t) = (0) e2 imt when
m = 0.

17
III.2.7
Show that if 0 and 1 lie in dierent connected components of the
complement C nD of D in the extendedR complex plane, then there
is a closed path in D such that d 6= 0. Hint. The hypothesis
means that there are > 0and a bounded subset E of CnD such
that 0 2 E; and every point of E has distance at least 5 from every
point of CnD not in E. Lay down a grid of squares in the plane
with side length ;and let F be the union of the closed squares in
the grid that meet E or that border on a square meeting R E. Show
that @F is a nite union of a closed paths in D, and that @F d = 2 .

Solution
Proceed as in the hint.
R Assume that 0 Ris the centre of one of the squares
S0 in the grid. Then @S0 d = 2 , while @S d = 0 for any other square in
PR
the grid. Thus @Sj
d = 2 , where we sum over squares. S0 ; S1 ; : : : ; Sn
in F . If two squares in F are ?????, the corresponding integrals along the
edge cancel. Then this is the sum over the integrals over the edges of the
Sj s that have F on oneRside and CnF on the other, that is the edges that
form @F . We have thus @F d = 2 . Note that by construction, @F D.
Now note how edges @F can meet. Either a vertex in @F have one edge
coming in and one out, or it have two coming in and two out. By starting on
following edges, and making a left turn at vertices where 4 edges meet, we
must eventually end where we started, with a closed path. Three paths must
be disjoint,
P R except for vertices with Rfour edges. Call the paths 1 ; : : : ; m .
Since d = 2 , we must have d 6= 0 for one of the j s. (In fact,
j R
we will have d = 2 on the j s we get by moving from 0 to @F .)
j

18
III.3.1
For each of the following harmonic functions u, nd du, nd dv, and
nd v, the conjugate harmonic functions of u.
(a) u (x; y) = x y (c) u (x; y) = sinh x cos y
y
(b) u (x; y) = x3 3xy 2 (d) u (x; y) = x2 +y 2

Solution
(a)
Use Cauchy-Riemanns equations page 83.

u (x; y) = x y
du = dx dy
dv = dx + dy
v (x; y) = x + y
(b)
Use Cauchy-Riemanns equations page 83.

u (x; y) = x3 3xy 2
du = (3x2 3y 2 ) dx 6xy dy
dv = 6xy dx + (3x2 3y 2 ) dy
v (x; y) = 3x2 y y 3
(c)
Use Cauchy-Riemanns equations page 83.

u (x; y) = sinh x cos y


du = cosh x cos y dx sinh x sin y dy
dv = sinh x sin y dx + cosh x cos y dy
v (x; y) = cosh x sin y
(d)
Use Cauchy-Riemanns equations page 83.
y
u (x; y) = x2 +y 2
2xy x2 +y 2 y(2y) 2xy x2 y 2
du = (x2 +y 2 )2 dx + (x2 +y 2 )2
dy = (x2 +y 2 )2
dx + (x2 +y 2 )2
dy
2 2
dv = (xy2 +yx2 )2 dx 2xy
(x2 +y 2 )2
dy
x
v (x; y) = x2 +y 2

19
III.3.2
Show that a complex-valued function h (z) on a star-shaped domain
D is harmonic if and only if h (z) = f (z) + g (z), where f (z) and g (z)
are analytic on D.

Solution
Write h = u + iw where u, w are harmonic. By the Theorem on page 83
both u and w have harmonic conjugate in D, (since D is star-shaped), so
thus there are ', analytic such that u = Re ', w = Re .

u = (' + ') =2; w= + =2;


gives

1
h= 2
'+'+i +i = 12 [' + i ] + 1
2
'+i :
Take

f = 12 [' + i ] ; g = 12 [' i ];
then h (z) = f (z) + g (z).

To show the oposite direction. Assume that h (z) = f (z) + g (z) where f (z)
and g (z) are analytic on D. Then h = Re f + Re g + i (Im f + Im g) and
from Cauchy-Riemanns equations follows that

@ 2 (Re f ) @ 2 (Re g) @ 2 (Im f ) @ 2 (Im g)


+ +i +
@x2 @x2 dx2 @x2
@ 2 (Re f ) @ 2 (Re g) @ 2 (Im f ) @ 2 (Im g)
+ + + i =
@y 2 @y 2 @y 2 @y 2
@ 2 Im f @ 2 Im g @ 2 Re f @ 2 Re g @ 2 Im f @ 2 Im g
= + +i + +
@x@y @x@y @x@y @x@y @y@x @y@x
@ 2 Re f @ 2 Re g
+i =0
@y@x @y@x

so h (z) is harmonic.

20
III.3.3
Let D = fa < jzj < bg n ( b; a), an annulus slit along the negative
real axis. Show that any harmonic function on D has a harmonic
conjugate on D.
Suggestion. Fix c between a and b, and dene v (z) explicitly as a line
integral along the path consisting of the straight line from c to jzj
followed by the circular arc from jzj to z. Or map the slit annulus
to a rectangle by w = Log z.

Solution
Integrate the CR equations we have
Z z Z Z
1 @u @u @u @u
v (z) = dr + r d = dx + dy
z0 r@ @r @y @x
integrate along r interval, then interval. Perception is same as getting
from z0 to z1 , then from z1 to little disk. Integral is well-dened, continuous,
harmonic and is harmonic conjugate for u.

21
III.3.4 (From Hints and Solutions)
Let u (z) be harmonic on the annulus fa < jzj < bg. Show that there
is a constant C such that u (z) C log jzj has a harmonic conjugate
on the annulus. Show that C is given by
Z 2
r @u
C= rei d ;
2 0 @r
where r is any xed radius, a < r < b.

Solution
u has harmonic conjugate v1 on the annulus slit along ( b; a), and also a
harmonic conjugate v2 on the annulus slit (a; b). Since v1 v2 is constant
above the slit ( b; a), and also constant below the slit, v1 jumps by a
constant across the slit. Arg z also jumps by a constant across the slit. By
appropriate choice of C, v1 C Arg z is continuous across the slit ( b; a),
and u C log jzj has a harmonic conjugate v1 C log jzj on the annulus. For
the identity, use the polar form of the Cauchy-Riemann equations to convert
the rderivative of u to a derivative of v.

22
III.3.5
The ux of a function u across a curve is dened to be
Z Z
@u
ds = ru n ds;
@n
where n is the unit normal vector to and ds is arc length. Show
that if a harmonic function u on a domain D has a conjugate har-
monic function v on D, then the integral giving the ux is inde-
pendent of path in D. Further, the ux across a path in D from
A to B is v (B) v (A).

Solution
!
t = dx ; dy , !
ds ds
n = dy ds
; dx
ds
,
R R R @u
ru! n ds = @u dy
@x ds
@u dx
@y ds
ds = @y
dx + @u
@x
dy |{z}
=
R @v @v
R
@x
dx + @y dy = dv
( ) CR equations. If goes from P to Q, this is v (Q) v (P ), which shows
that the integral is independent of path in D.

23
III.4.1
Let f (z) be a continuous function on a domain D. Show that if
f (z) has the mean value property with respect to circles, as dened
above, then f (z) has the mean value property with respect to disks,
that is if z0 2 D and D0 is a disk
RR centered at z0 with area A and
1
contained in D, then f (z0 ) = A D0 f (z) dx dy.

Note: This MVP for area is not exactly what is dened as the MVP for
disks. What is then is that if h (z0 ) is the average of h (z) for any circle
fjz z0 j = rg, 0 < r < 1. Then h (z) is the average of h (z) over any disk
fjz z0 j = r0 g, 0 < r < r0 . Express dx dy in polar coordinates at z0 and
integrate rst with respect to .

Solution
Suppose that f satises the mean value property,
Z 2
1
f (z0 ) = f z0 + rei d
2 0
for all z0 2 D and r > 0 such that Br (z0 ) D. Let D0 = fz 2 C : jz z0 j Rg
D. We have, A = R2 and parametrizing in polar coordinates, and use the
mean value property with respect to circles,

ZZ ZZ
1 1 x = x0 + r cos dx dy = r dr d 0 r R
f (z) dx dy = f (z) dx dy =
A D0 R 2
D0 y = y0 + r sin 0 2
Z 2 Z R Z R Z 2 Z R
1 1 1
= 2
f z0 + rei r dr d = 2
f z0 + rei d r dr = 2 f (z0 ) r d
R 0 0 R 0 0 R2 0
Z R
2f (z0 ) R 1 r2
= r dr = 2 f (z0 ) = f (z0 ) :
R2 0 R 2 0

Another note: Its better to use something other than 1=A as A was already
used as the circle average.

24
III.4.2
Derive (4.2) from the polar form of the Cauchy-Riemann equations
(Exercise II.3.8).

Solution
The polar form of Cauchy-Riemanns equations (from Exercise II.3.8)
@u 1 @v @u @v
= ; = r :
@r r@ @ @r
Now we have
Z 2 Z 2
@u @v
r z0 + rei d = z0 + rei d =0
0 @r 0 @

25
III.4.3
A function f (t) on an interval I = (a; b) has the mean value property
if

s+t f (s) + f (t)


f = ; s; t 2 I:
2 2
Show that any a ne function f (t) = At + B has the mean value
property. Show that any continuous function on I with the mean
value property is a ne.

Solution (A. Kumjian)


Let I := (a; b) and let f : I ! R be given. Suppose rst that f is an a ne
function, that is, there are A; B 2 R such that f (t) = At + B for all t 2 I.
Now let s; t 2 I be given, then

s+t s+t 1 1 f (s) + f (t)


f =A +B = (As + B) + (At + B) = :
2 2 2 2 2

Hence, f has the mean value property.


Next suppose that f is continuous and that it has the mean value property.
We must prove that f is a ne. We claim that for any c; d 2 I, with c < d,
and t 2 [0; 1] we have

f (tc + (1 t) d) = tf (c) + (1 t) f (d) :


So let c; d 2 I be given with c < d. We will rst prove that equation ( )
holds for t = k=2m where k; m are nonnegative integers with k 2m . We
do this by induction on m. For m = 0, we have t = k = 0; 1 and the
assertion is obvious. Now suppose that the assertion holds for some integer
m 0 (that is, equation ( ) holds for t = k=2m for all k = 0; : : : ; 2m ).
This inductive step entails showing that equation ( ) holds for t = k=2m+1
for any nonnegative integer k with k 2m+1 . By the inductive hypothesis
we may assume k = 2j + 1 for some integer j = 0; : : : ; 2m 1 (for if k
were even we could rewrite t as j=2m for some integer j with 0 j 2m ).
Then setting t0 = j=2m and t1 = (j + 1) =2m , we have t = (t0 + t1 ) =2 and
1 t = ((1 t0 ) + (1 t1 )) =2, thus,

26
1
tc + (1 t) d = ((t0 c + (1 t0 ) d + (t1 c + (1 t1 ) d))
2
hence,

f (tc + (1 t) d) =
1
=f ((t0 c + (1 t0 ) d + (t1 c + (1 t1 )d) =
2
1
= (f (t0 c + (1 t0 ) d) + f (t1 c + (1 t1 ) d)) = by the mean value property
2
1
= (t0 f (c) + (1 t0 ) f (d) + t1 f (c) + (1 t1 ) f (d)) = by inductive hypothesis
2
= tf (c) + (1 t) f (d) :

So we have proved that equation ( ) holds for t = k=2m where k; m are


nonnegative integers with k 2m . Since such numbers are dense in [0; 1]
the claim follows by the continuity of f . It is now straightforward to verify
that for any c; d 2 I, with c < d, there are unique constants A and B such
that f (x) = Ax + B for all x 2 [c; d] (take A = (f (c) f (d)) = (c d) and
B = f (c) Ac ). The desired result now follows by observing that A and B
do not depend on c; d.

27
III.4.4
Formulate the mean value property for a function on a domain
in R3 , and show that any harmonic function has the mean value
property.
Hint. For A 2 R3 and r > 0, let Br be the ball of radius r centered
at A, with volume element d , and let @Br be its boundary sphere,
with area element d and unit outward normal vector n. Apply the
Gauss divergence theorem
ZZ ZZZ
F nd = r Fd
@Br Br

to F = ru.

Solution
Prove MV theorem for harmonic functions in Rn , as follows u (R! x)
! R R @u ! !
u 0 = 0 @r (r x ) dr. Let d = area measure, x = unit vector, get
R @u ! R @u ! R R R @2u !
@r
(R x ) d @r
(r x ) d =
0 0
(r x ) drd (!
S @r2
x ). Get directional
derivative of Strokes formula. Better: Apply strokes theorem to shell f 0 < < 1 g,
get
R R! R R !
::: V ! n d surface = : : : r V volume. Apply to ru = !
v,
R R R
VolumeR
get : : : ru! n d surface = : : : r2 ! n d volume = 0,
R R @u R R @u
::: @ d ::: @ d
! !
j x x0 j=R 1 R j x x0 j= 0
@u
@
: : : ud_ is constant.
!
j x x 0j=

28
III.5.1
Let D be a bounded domain, and let u be a real-valued harmonic
function on D that extends continuously to the boundary @D. Show
that if a u b, then a u b on D.

Solution
Since D [ @ is a compact set we have that u attains both maximum and
minimum values. Assume that u (z0 ) = c > b fore some z0 2 D. Then it
follows by the maximum principle that u c on D, but this is a contradiction
to the asumption that u is continuously extended to @D. Thereby u b on
D.
For u we have that b u a on @D. Assume that u (z0 ) = c0 > a
for seme z0 2 D. Then by the maximum principle u c0 on D, but this
contradicts that u is continuously extended to @D with u a on @D.
So u a on D, equivalently a u b on D.
This show that a u b on D.

29
III.5.2
Fix n 1, r > 0 and = ei' . What is the maximum modulus of
z n + over the disk fjzj rg? Where does z n + attain its maximum
modulus over the disk?

Solution (A. Kumjian)


We claim that the maximum modulus of z n + over the disk fz 2 C : jzj rg
is rn + and this is attained at = rei where = (2k + ')=n for k =
0; 1; : : : ; n 1. By the maximum modulus principle the maximum modulus
occurs on the boundary fz 2 C : jzj = rg. For z 2 C with jzj = r we have
by the triangle inequality
n
jz n + j jz n j + j j = rn + = (rn + ) ei' = j + j:
so the claim is proven.

30
III.5.3
Use the maximum principle to prove that the fundamental theorem
of algebra, that any polynomial p (z) of degree n 1 has a zero, by
applying the maximum principle to 1=p (z) on a disk of large radius.

Solution
It is su cient to show that any p (z) has one root, for by division we can
then write p (z) = (z z0 ) g (z), with g (z) of lower degree.
Note that if

p (z) = an z n + an 1 z n 1
+ + a0 ;
then as jzj ! 1, jp (z)j ! 1. This follows as
an 1 a0
p (z) = z n an + + + n :
z z
1
Assume p (z) is non-zero everywhere. Then p(z) is bounded when jzj R.
1 1
Also, p (z) 6= 0, so p(z) is bounded for jzj R by continuity. Thus, p(z) is a
bounded entire function, which must be constant. Thus, p (z) is constant, a
contradiction which implies p (z) must have a zero.

Solution (K. Seip)


If p (z) has no zeros, then 1=p (z) is an entire function. Also, if we denote by
m (R) the maximum of 1=p (z) on the circle jzj = R, then m (R) ! 0 when
R ! 1, unless p (z) is a constant. By the maximum principle j1=p (z)j
m (R) when jzj R, which means 1=p (z) = 0. This is impossible, and so
1=p (z) is not an entire function.

31
III.5.4
Let f (z) be an analytic function on a domain D that has no zeros
on D. (a) Show that if jf (z)j attains its minimum on D, then f (z) is
constant. (b) Show that if D is bounded, and if f (z) extends con-
tinuously to the boundary @D of D, then f (z) attains its minimum
on @D.

Solution
(a)
If jf (z)j attains its minimum on D, then j1=f (z)j attains its maximum on
D, which can only happens if f (z) is a constant, by the maximum principle.
(b)
Since D [ @D is compact and f (z) is continuous it follows that jf (z)j attains
both maximum and minimum value on D [ @D. Assume that jf (z)j does
not attain its minimum on @D. Then jf (z)j attains its minimum on D and
it follows by (a) that f is constant so jf (z)j attains its minimum on @D.

32
III.5.5
Let f (z) be a bounded analytic function on the right half-plane.
Suppose that f (z) extends continuously to the imaginary axis and
satises jf (iy)j M for all points iy on the imaginary axis. Show
that jf (z)j M for all z in the right half-plane.
Hint. For " > 0 small, consider (z + 1) " f (z) on a large semidisk.

Solution
Now we consider the function (z + 1) " f (z) in the right half-plane, because
f (z) is bounded here, we assume that jf (z)j C in this domain. On a
semidisk of radius r in the right half-plane we have

" C
(z + 1) f (z)
jr 1j"
For r = R su cient large, we have

" C
(z + 1) f (z) M
(R 1)"
for all jzj R, when Re z 0.
On the imaginary axis we have that
"
(z + 1) f (z) M
because jf (iy)j M for all points iy on the imaginary axis.
By the maximum principle, we have
"
(z + 1) f (z) M
for jzj R, when Re z 0.
Now let " ! 0, we have

jf (z)j M
for all z in the right half-plane.

Solution (K. Seip)


We assume that jf (z)j C for Re z > 0 and jf (iy)j M . For " > 0 set
"
F" (z) = (1 + z) f (z) :

33
Then

jF" (iy)j M
and

F" Rei C R "


M
for su ciently large R.
Thus for arbitary z, Re z > 0 we have

jF" (z)j M;
or

jf (z)j M (1 + jzj)" :
This holds for every " > 0, thus jf (z)j M.

34
III.5.6
Let f (z) be a bounded analytic function on the right half-plane.
Suppose that lim sup jf (z)j M as z approaches any point of the
imaginary axis. Show that jf (z)j M for all z in the half-plane.
Remark. This is a technical improvement on the preceding exercise
for students who can deal with lim sup (see Section V.1).

Solution
Set
"
g (z) = (z + 1) f (z) :
Then

jg (z)j 6 jf (z)j
becauce jz + 1j > 1 for all z in the right half-plane.
Take R large so that jg (z)j 6 M for jzj > R. The lim sup condition implies
that there is > 0 such that

jg (z)j 6 M + "; jzj < R; 0 < Re z 6 :


Apply the maximum principles to the domain fzj jzj < R; Re z > g, to ob-
tain jg (z)j 6 M + ". Then g (z) ! f (z) as " ! 0.

35
III.5.7
Let f (z) be a bounded analytic function on the open unit disk D.
Suppose there are a nite number of points on the boundary such
that f (z) extends continuously to the arcs of @D separating the
points and satises f ei M there. Show that jf (z)j M on D.
Hint. In the case that there is only one exceptional point z = 1,
consider the function (1 z)" f (z).

Solution
Let the points be a1 ; : : : ; an . Then (z aj )" , where 1 j n is analytic
"
(take analytic
Q branch). We have that jz aj j ! 0 as z ! aj for 1 j n.
Then [ (z aj )" ] f (z) = f" (z) satises f" (z) is continuous in D [ @D.
jf" (z)j 6 M on @D. jf" (z)j 6 M on D. Let " ! 0, obtain jf (z)j 6 M on
D.

Solution
Let C = supz2D jf (z)j. By denition jf (z)j C. Let z1 ; : : : ; zn 2 @D be all
the points for which f (z) does not extend continuously.
Now suppose that jf (z)j M for all z 2 @D except fz1 ; : : : ; zn g and to the
contrary M < C, then there is a z0 2 D such that jf (z0 )j = M + with
0< C M.
Let

g (z) = (z z1 )" : : : (z zn )" f (z)


where " is chosen so that 8z 2 D [ @D.
We have that

M + =2 M + =3
j(z z1 )" : : : (z zn )" j and j(z z1 )" : : : (z zn )" j
M+ M
Note that g (z) extends continuously to @D and that g (z) is analytic in D.
By the maximum modulus principle, and since

M + =2
jg (z0 )j = j(z0 z1 )" : : : (z0 zn )" j f (z0 ) (M + )
M+
we have that

36
M + =2 M + =3
(M + ) jg (z0 )j sup j(z z1 )" : : : (z zn )" j jf (z)j M
M+ z2@D M

which is a contradiction.

37
III.5.8
Let f (z) be a bounded analytic function on a horizontal strip in
the complex plane. Suppose that f (z) extends continuously to the
boundary lines of the strip and satises jf (z)j M there. Show
that jf (z)j M for all z in the strip. Hint. Find a conformal map
of the strip onto D and apply Exercise 7.

Solution
Assume strip is 2
< Im z < 2 , so it is mapped to right half-plane by
w = i log z, z = eiw . Then g (w) = f (eiw ) hold on right halfplane, 6 M
at all point of iR except at w = 0. Modify proof idea of Exercise III.5.5 to
get jgj 6 M on the halfplane.

38
III.5.9
Let D be an unbounded domain, D 6= C, and let u (z) be a harmonic
function on D that extends continuously to the boundary @D. Sup-
pose that u (z) is bounded below on D, and that u (z) 0 on @D.
Show that u (z) 0 on D. Hint. Suppose 0 2 @D, and consider
functions of the form u (z) + log jzj on D \ fjzj > "g.

Solution (From Hints and Solutions)


Let " > 0. Take > 0 such that u (z) > " for z 2 D, 0 < jzj < . Let
> 0. Take R > 0 so large that u (z) + log jzj > 0 for jzj > R. By the
maximum principle, u (z) + log jzj > " + log for z 2 D, < jzj < R,
hence for all z 2 D such that jzj > . Let ! 0, then let " ! 0, to obtain
u (z) > 0 on D.

Solution (K. Seip)


We may assume 0 2 @D. By continuity of u at 0, we can nd and so that
( ; > 0)

u (z) + log jzj "; z 2 D \ fjzj = g


for arbitary " > 0. Since u is bounded below, there is also R0 > 0 such that
for R > R0

u (z) + log jzj 0; z 2 D \ fjzj = Rg ;


Applying the maximum principle to u (z) log jzj in D \ f < jzj < Rg,
we get u (z) " log jzj. Since " and can be chosen arbitarily small, the
result follows.

39
III.5.10
Let D be a bounded domain, and let z0 2 @D. Let u (z) be a har-
monic function on D that extends continuously to each boundary
point of D except possibly z0 . Suppose u (z) is bounded below on
D, and that u (z) 0 for all z 2 @D, z 6= z0 . Show that u (z) 0 on
D.

Solution
D hold, z0 2 @D, u > M on D, u > 0 (@D) n fz0 g. The map ' (z) =
1= (z z0 ) maps D onto an unbounded domain, to which the result of Ex-
ercise 9 applies. Or: consider w (z) = u (z) + log (1= jz z0 j). Assume
jz z0 j 6 R on D, then take " > 0 small, assume R > 1. Then w (z) >
" log (1=R) at all points of (@D) n fz0 g, w (z) ! +1 as z ! z0 . By the
maximum principle, w (z) > " log (1=R) on D. Let " ! 0, get w (z) > 0 on
D.

40
III.5.11
Let E be a bounded set of integer lattice points in the complex
plane. A point m + ni of E is an interior point of E if its four
immediate neighbors m 1 + ni, m + ni i belong to E. Otherwise
m + ni is a boundary point of E. A function E is harmonic if its
value at any interior point of E is the average of its values at the
four immediate neighbors. Show that a harmonic function on a
bounded set of lattice points attains its maximum modulus on the
boundary of the set.

Solution
Suppose u (z) attains its maximum at m + ni, call it M . Then u (m + ni) is
the average of its values at its four neighbors, so all their values must coincide
with M. Consider fm + ni : u (m + ni) = M g. This is a nite set. It have a
point with largest m. This point have a neighbor in the boundary of E. u
attains value M at that point. u = c at some point of boundary of E.
For complexvalued case, follow same approach as in book. Remark: Should
just consider real-valued u, and show it attains its maximum and minimum
at boundary points of E.

41
III.6.1
Consider the uid ow with constant velocity V = (2; 1). Find the
velocity potential (z), the stream function (z), and the complex
velocity potential f (z) of ow. Sketch the streamlines of the ow.
Determine the ux of the ow across the interval [0; 1] on the real
axis and across the interval [0; i] on the imaginary axis.

Solution
V = (2; 1), V = r , = 2x + y, @@x = @@y , @@y = @@x ) = 2y x
f (z) = + i = 2z iz = (z i) z
Streamlines are straight line with slope 1=2, ux across [0; 1] = (1) (0) =
1, ux across [0; i] = (i) (0) = 2.

42
III.6.2
Fix real numbers and , and consider the vector eld given in
polar coordinates by

V (r; ) = ur + u ;
r r
where ur and u are the unit vectors in r and directions, respec-
tively.
(a)
Show that V (r; ) is the velocity vector eld of a uid ow, and nd
the velocity potential (z) of the ow.
(b)
Find the stream function (z) and the complex velocity potential
f (z) of the ow.
(c)
Determine the ux of the ow emanating from the origin. When
is 0 a source and when is 0 a sink?
(d)
Sketch the streamlines of the ow in the case = 1 and = 1.

Solution
a)
V (r; ) = r ur + r u
r = @@r ur + 1r @@ u = r ur + r u
Get @@r = r , = log r + h ( )
1@
r@
= h0 ( ) = r , h0 ( ) = , h ( ) = )V =r
= log r + is harmonic locally. is the velocity vector eld of a ow
b)
= Re ( log z + arg z) stream function = = Im ( log z + arg z) =
arg z log jzj
complex velocity of a point f (z) = ( i ) log z
c)
Flux = of around a circle centred at 2 , sin u : > 0, sin u : > 0.
d)
= 1, = 1. Note: and are only locally dened.

43
III.6.3
Consider the uid ow with velocity V = r , where (r; ) =
(cos ) =r. Show that the streamlines of the ow are circles and
sketch them. Determine the ux of the ow emanating from the
origin.

Solution
= cosr = r cos
r2
= Re 1
z
= Re z
2 = Im 1
z
= sin
r
R jzj
Since is single-valued, ux = jzj=" d = 0, and there is no source or sink
at 0. Set w = 1=z, z = 1=w, these streamlines are the curves Im (1=z) =
constant, which correspond to the curves Im w = constant get modied to
tangent to real line at 0 = z (1), and real axis. r = @@r ur + 1r @@ u =
cos
r2
ur sin
r2
u.

44
III.6.4
Consider the uid ow with velocity V = r , where

z 1
(z) = log :
z+1
Show that the streamlines of the ow are arcs of circles and sketch
them. Determine the ux of the ow emanating from each of the
singularities at 1.

Solution
is the composition of an analytic function w = (z 1) = (z + 1) and the har-
monic function log jwj , so is harmonic, and = Re log ((z 1) = (z + 1)).
The stream function is = ImH(log (z 1) = H(z + 1)) = arg ((z 1) = (z + 1))
, then is not single-valued. jz 1j=" d = jz 1j=" d arg (z 1) = 2 = ux
H H
emanating from +1. jz 1j=" d = jz 1j=" d arg (z + 1) = 2 = ux ema-
nating from 1. z = 1 is a source, z = 1 is a sink.
Set = (z 1) = (z + 1), w = log . The line Im w = constant in the w
plane correspond to rays issuing from 0 in the plane. (since = ew ), and
this correspond to arcs of circles from 1 to 1 in the zplane, since FLTs
maps circles to circles. Hence streamlines are arcs of circle from 1 to 1,
including the circle through 1.

45
III.6.5
Consider the uid ow in the horizontal strip f0 < Im z < g with
a sink at 0 and equal sources at 1. Find the stream function
(z) and the velocity vector eld V (z) of the ow. Sketch the
streamlines of the ow. Hint. Map the strip to a half-plane by
= ez and solve a Dirichlet problem with constant boundary values
on the three intervals in the boundary separating sinks and sources.

Solution
Consider uid ow in a horizontal strip. f0 < Im z < g with a sink 0, but
no source at 1 . Find the stream function (z). Sketch the ow lines.
Find the complex vector eld V (z). Hint : Map the half-plane by.
Map to half-plane = ez . A arg + B arg ( 1) + C, arg = arg ez = y,
Ay+C+B arg (ez 1), c3 = 0, requires A+ B+C = 0 ) C = (A + B),
A (y ) + B [arg (ez 1) ]. C1 = C2 require A + B B =
z
[ A B ], A [ (y ) + 2 [arg (e 1) ]] = A [ ( + y) + 2 arg (ez 1)]

46
III.6.6
For a uid ow with velocity potential (z), we dene the conju-
gate ow to be the ow whose velocity potential is the conjugate
harmonic function (z) of (z). What is the stream function of
the conjugate ow? What is the complex velocity potential of the
conjugate ow?

Solution
Stream function of conjugate ow in direction r is . Complex velocity
potential is g (z) = i = if (z) = i ( + i )

47
III.6.7
Find the stream function and the complex velocity potential of the
conjugate ow associated with the uid ow with velocity vector
ur =r. Sketch the streamlines of the conjugate ow. Do the particles
near the origin travel faster or slower than particles on the unit
circle?

Solution
Flow ur =r, = log jzj, = arg z. Complex velocity potential of conjugate
ow is i log z. Streamlines of the conjugate ow are = log jzj =
constant , r = jzj = constant, which are circles centered in origin. Speed
= jV (z)j = jf 0 (z)j = 1= jzj. Particles travel faster near z = 0.

48
III.6.8
Find the stream function of the conjugate ow of
1
V (r; ) =
( ur + u ) :
r
Sketch the streamlines of both the ow and the conjugate ow on
the same axis. (See Exercise 2d.)

Solution
This is the vector eld from 2d. Hence V = r ( log r + ). The velocity
potential is = log r + . The conjugate ow has velocity potential =
log r = Re ((i 1) log z). The stream function of the conjugate ow
is then Im ((i 1) log z) = + log r = . (The steam function of the
conjugate is always the negative of the velocity potential of the ow. See
Ex.6) Streamline of conjugate ow are orthogonal to the stream of the ow
(Exercise 2d) and velocity vector eld. Conjugate ow is the rotation by 90
of the velocity vector eld of the ow, so the origin is a sink of both ows in
this case.

49
III.7.1
Find the steady-state heat distribution u (x; y) in a laminar plate
corresponding to the half disk fx2 + y 2 < 1; y > 0g, where the semi-
circular top edge is held at temperature T1 and the lower edge
( 1; 1) is held at temperature T2 . Find and sketch the isothermal
curves for the heat distribution. Hint. Consider the steady-state
heat distribution for the full unit disk with the top held at temper-
ature T1 and the bottom temperature T3 , where T2 = (T1 + T3 ) =2.

Solution
(z) = 2 (Arg (1 + z) Arg (1 z)) has temp +1 on top, 1 on bottom
T1 = A + B
half and is 0 on the interval ( 1; 1). Take u = A + B, )
T2 = B
A = T/ 1 T2
, get u = (T1 T2 ) (z) + T2
B = T2
u = (T1 T2 ) 2 [Arg (1 + z) Arg (1 z)] + T2
The isothermal curves are the curves where u = konstant. these are the
curves

1+z
Arg = konsant
1 z
then

(1 + z) (1 z) = 1 + 2i Im z jzj2
and
Im z
= konstant.
1 jzj2

50
III.7.2
Find the potential function (x; y) for the electric eld for a con-
ducting laminar plate corresponding to the quater-disk fx2 + y 2 < 1; x > 0; y > 0g,
where the two edges on the coordinate axis are grounded (that is,
= 0 on the edges), and the semicircular edge is held at constant
potential V1 . Find and sketch the equipotential lines and the lines
of force for the electric eld. Hint. Use the conformal map = z 2
and the solution to the preceding exercise.

Solution
(z) = 2 (Arg (1 + w (z)) Arg (1 w (z))), 1 = V1 2 (Arg (1 + z 2 ) Arg (1 z 2 ))

51
III.7.3
Find the potential function (x; y) for the electric eld for a con-
ducting laminar plate corresponding to the unit disk where the
boundary quater-circles in each quadrant are held at a constant
voltages V1 ; V2 ; V3 and V4 . Hint. Map the disk to the upper half-
plane by w = w (z) and consider potential functions of the form
Arg (w a).

Solution
Let w = i (1 z) = (1 + z), w (1) = 0, w ( 1) = 1, w (i) = 1, w ( i) = 1.
z ! w (z) maps unit disk to upper halfplane. Suers to nd a har-
monic function u (w) in upper half-plane with boundary values constant
on each interval, as above. Any such function is a linear combination of
u 1 (w) = 1 Arg (w + 1), u0 (w) = 1 Arg w, u1 (w) = 1 Arg (w 1), and if
= A 1 u 1 + A0 u0 + A1 u1 + C, we get a system we can back solve.
8 8
>
> A 1 + A 0 + A 1 + C = V3 >
> C = V2
< <
A0 + A1 + C = V3 A1 = V1 V2
)
>
> A1 + C = V1 >
> A0 = V4 V1
: :
C = V2 A 1 = V3 V4
That does it-just plug in.
Example: V2 = 1, V1 = V3 = V4 = 0, this solution is 2 (z) = 1 1 Arg i 11+zz ,
and we P
can get other elementary solutions, 1 (z), 3 (z), 4 (z), and then
(z) = vj j (z).

52
III.7.4
Find the steady-state heat distribution in a laminar plate corre-
sponding to the vertical half-strip fjxj < =2; y > 0g, where the ver-
tical sides at x = =2 are held at temperature T0 = 0 and the
bottom edge ( =2; =2) on the real axis is held at temperature
T1 = 100. Make a rough sketch of the isothermal curves and the
lined of heat ow. Hint. Use w = sin z to map the strip to the
upper half-plane, and make use of harmonic functions of the form
Arg (w a).

Solution 8
< a+b+c=0
Try = a Arg (w + 1) + b Arg (w 1) + c, want a b + c = 100 )
:
8 a b+c=0
< a = 50
b = 50
:
c=0
(z) = 50 Arg (sin z + 1) 50 Arg (sin z 1).

53
III.7.5
Find the steady-state heat distribution in a laminar plate corre-
sponding to the vertical half-strip fjxj < =2; y > 0g, where the side
x = =2 is held at constant temperature T0 , the side x = =2 is
held at constant temperature T1 , and the bottom edge ( =2; =2)
on the real axis is insulated, that is, no heat passes through the
bottom edge, so the gradient ru of the solution u (x; y) is parallel
there to the x axis. Hint. Try linear functions plus constants.

Solution
u (x; y) = ax + b is harmonic. @u@y
=0
T1 T0
2
a + b = T0 a=
)
2
a + b = T1 b = T0 +T
2
1

u = 1 (T1 T0 ) x + T0 +T
2
1
.

54
III.7.6
Find the steady-state heat distribution in a laminar plate corre-
sponding to the upper half-plane fy > 0g, where the interval ( 1; 1)
is insulated, the interval ( 1; 1) is held at temperature T0 , and
the interval (1; 1) is held at temperature T1 . Make a rough sketch
of the isothermal curves and the lines of heat ow. Hint. Use the
solution to Exercise 5 and the conformal map from Exercise 4.

Solution
Let z = z (w) map the vertical half-strip to the upper half-plane with
z ( =2) = 1 and z ( =2) = 1, w = w (z) : upper halfplane ! vertical
semi strip. Take z = sin w, this has desired mapping property, w = sin 1 (z).
Then solution is composition of sine function with 2 (T1 T0 ) Re w + T0 +T
2
1
.
2 1 T0 +T1 1
Get u = (T1 T0 ) Re sin (z) + 2 , where we take branch sin z
with values in the vertical semi-strip. Note : This conformal map was used
in Exercise 4.

55
III.7.7
The gravitational eld near the surface of the earth is approxi-
mately constant, of the form F = ck, where k is the unit vector in
the z direction in (x; y; z) space and the surface of the earth is
represented by the plane where z = 0 (the at earth theory). Show
that F is conservative, and nd a potential function for F .

Solution
!
(x; y; z) = cz, r = c k
Since

@ @ @
r F = ; ; (0; 0; c) = (0; 0; 0) ;
@x @y @z
we have that F is conservative.

56
III.7.8
Show that the inverse square force eld F = ur =r2 on R3 is con-
servative. Find the potential function for F , and show that is
harmonic.

Solution
! !
r = ur2r = r , (x; y; z) = 1
r

57
III.7.9
For n 3, show that the function 1=rn 2 is harmonic on Rn n f0g.
Find the vector eld F that has this function as its potential.

Solution n
1 1 2 n 1
rn 2
= n 2 = u = r = (x21 + : : : + x2n ) 2

(1
x n)
2 +:::+x2 2
n
@u n
@xj
= 1 2 (x21 + : : : + x2n ) 2
2xj = (2 n) xj r n
n
@2u n n 1
@x2j
= (2 n) r + (2 n) xj 2
(x21 + : : : + x2n ) 2
2xj =
2 n (2 n)( n)x2j
rn
+ r n+2
2 n (2 n)n P 2
u = n rn r n+2
xj = 0 this shows that u is harmonic on Rn n f0g
P @u P x
F = u= e = (2rnn) xj ej = r2n n1 r j ej = (2rnn)~1ur
@xj j
!
F = (2r) rrn = rcnu 1
R P
F d = rnc 1 Area (s) = (n 2) Area(S)
rn 1

58
IV 1 2 3 4 5 6 7 8 9 10 11 12 13 14 15 16 17 18 19
1
2
3
4
5
6
7
8

1
IV.1.1
1 2 3 P L K F
KKK FFF
Let be the boundary of the triangle f0 < y < 1 x; 0 < x < 1g, with the
usual counterclockwise
R orientation.
R EvaluateR the following integrals
(a) Re z dz (b) Im z dz (c) z dz

Solution
a)
We begin do split the contur into three parts so that = 1 + 2 + 3.
And integrate along each side in the triangle

= 1 + 2 + 3

We parametrizize 1 = [0; 1], thus 1 (t) = t, for 0 t 1, then Re (t) = t


and dz = dt
Z Z Z 1
1
Re z dz = Re (t) dz = t dt = :
1 1 0 2
We parametrizize 2 = [1; i], thus 2 (t) = 1 t + it, for 0 t 1, then
Re (1 t + it) = 1 t and dz = ( 1 + i) dt, then
Z Z 1
1 1
Re z dz = (1 t) ( 1 + i) dt = + i:
2 0 2 2
We parametrizize 3 = [i; 0], thus 3 (t) = i (1 t), for 0 t 1, then,
Re (i (1 t)) = 0 and dz = i dt, then
Z Z 1
Re z dz = (0) ( i) dt = 0:
3 0

2
And now we take the parts in the integral together
Z Z Z Z
i
Re z dz = Re z dz + Re z dz + Re z dz =
1 2 3
2
b) For details see a)

Z Z 1
Im z dz = 0 dt = 0;
1 0
Z Z 1
1 1
Im z dz = t ( 1 + i) dt = + i;
0 2 2
Z2 Z 1
1
Im z dz = (1 t) ( i) dt = i:
3 0 2

thus
Z
1
Im z dz = :
2
c) For details see a)

Z Z 1
1
z dz = t dt = ;
0 2
Z Z 11
z dz = (1 t + it) ( 1 + i) dt = 1
2 0
Z Z 1
1
z dz = (i (1 t)) ( i) dt = :
3 0 2

thus
Z
z dz = 0:

3
IV.1.2
1 2 3 P L K
111 LLL
Absolutbeloppet i c) nagot a
r f el
Let be the unit circle fjzj = 1g, with the usual counterclockwise
orientation.
R m EvaluateR the following Rintegrals, for m = 0; 1; 2; : : :.
(a) z dz (b) z m dz (c) z m jdzj

Solution
Set z = ei , where 0 2 , and thus dz = iei d .
(a)

Z Z 2 Z 2 2
m im i i(m+1) ei(m+1)
z dz = e ie d = i e d = =
0 0 m+1 0
1 0; m 6= 1
= ei2 (m+1)
1 =
m+1 2 i m= 1:

(b)

Z Z 2 Z 2
m im i
z dz = e ie d = i ei(1 m)
d =
0 0
2
ei(1 m) 1 0; m 6= 1
= = ei2 (m 1)
1 =
1 m 0 1 m 2 i m = 1:

(c)

Z Z 2 2
m im eim 1 0; m 6= 0
z jdzj = e d = = ei2 m
1 =
0 im 0 im 2 m = 1:

4
IV.1.3
1 2 3 P L K
111
Let be the circle fjzj = Rg, with the usual counterclockwise ori-
entation.
R mEvaluate theR following integrals,
R m for m = 0; 1; 2; : : :.
(a) jz j dz (b) jz m j jdzj (c) z dz

Solution
Set z = Rei , where 0 2 , and thus dz = iRei d and jzj = R.
(a)

Z Z 2 Z 2
m m im i m+1 2
jz j dz = R e iRe d = iR ei d = Rm+1 ei 0
= Rm+1 ei2 1 = 0:
0 0

(b)

Z Z 2 Z 2
m
jz j jdzj = m im
R e iRe i
d =R m+1
d = Rm+1 [ ]20 = Rm+1 (2 0) = 2 Rm+1 :
0 0

(c)

Z Z 2 Z 2 2
m m im i m+1 i(1 m) m+1 ei(1 m)
z dz = R e iRe d = iR e d =R =
0 0 1 m 0
Rm+1 i2 (1 m) 0; m 6= 1; 0; m 6= 1;
= e 1 = m+1 = 2
1 m 2 iR ; m = 1; 2 iR ; m = 1:

5
IV.1.4
1 2 3 P L K
LLL
Show that if D is a bounded domain with smooth boundary, then
Z
z dz = 2i Area (D) :
@D

Solution
Substitute z = x iy, dz = dx + idy, and apply Greens theorem.

6
IV.1.5
1 2 3 P L K 138
111 KKK
Show that
I
ez
dz 2 e2 :
jz 1j=1 z+1

Solution
We begin with parametrizize the circle jz 1j = 1, as z = 1+eit = 1+cos t+
i sin t where 0 t 2 .
For the nominator in the integrand ez = (z + 1) we have

jez j = eRe z+i Im z = eRe z ei Im z = eRe z e2 ;


since Re z 2 on the circle.
And for the denominator in the integrad we have

j1 + zj = j2 + cos t + i sin tj Re (2 + cos t + i sin t) = 2 + cos t 1;

Thus

ez jez j e2
M= = = e2 :
1+z jz + 1j 1
Becauce the integrationcontour is a circle with radius 1 we have that L = 2 .
M L estimate gives
I
ez
dz 2 e2 :
jz 1j=1 z + 1

7
IV.1.6
1 2 3 P L K
111 KKK
Show that there is a strict inequality
I p log R
Log z
dz 2 2 ; R>e :
jzj=R z2 R

Solution
For the integrand Log z=z 2 , with the nominator Log z = log R + i with the
principalargument we have

p p p p
Log R log2 R + 2
log2 R + 2 log2 R + log2 R 2 log R
= 6 6 = :
R2 R2 R2 R2 R2

there the last inequalitys follows by given fact R > e that gives 2 < log2 R.
Becauce the integrationcontour is a circle with radius R we have that L =
2 R.
M L estimate gives
p
H Log z 2 log R p log R
jzj=R
dz 6 2 R = 2 2 ; R>e :
z2 R2 R

8
IV.1.7
1 2 3 P L K

Show that there is a strict inequality


I
zn 2 Rn+1
m
dz ; R > 1; m 1; n 1:
jzj=R z 1 Rm 1

Solution
R
If jf (z)j 6 m0 < m on an arc 0 of the circle, = 0 [ 1, then f (z) dz =

R R
f (z) dz + f (z) dz 6 m0 length ( 0 )+mlength ( 1 ) = (m0 m) length ( 0 )+
0 1

mlength ( ) < mlength ( )


R
Thus if f (z) dz = ML, then jf (z)j = M on . In this case, jz m j = Rn ,
1
zm 1
6 1
Rm 1
, } . )M=
ik=m
with strict inequality unless z = R e|2 {z Rn
Rm 1
,
th root of w
R
L = 2 R, get zn
jzj=R z m 1
dz < Rn
Rm 1
2 R= 2 Rn+1
Rm 1
, R > 1, m > 1, n > 0.

9
IV.1.8
1 2 3 P L K
LLL
Suppose the continuous function f ei on the unit circle satises
R
f ei M and jzj=1 f (z) dz = 2 M . Show that f (z) = cz for
some constant c with modulus jcj = M .

Solution H
Multiply f by a unimodular constant, assume jf (z)j 6 M , jzj=1 f (z) dz =
R2 i
R2
0
f e d = 2 M . Take real parts, have 0
Re f ei iei d = 2 M ,
Re f e ie d 6 f e
i i i
6 M . If have strict inequality somewhere,
R2
then 0 < 2 M . We conclude that Re f ei iei d M . Since f ei iei 6
M , we have Im f e ie di i
0. ) f e ie
i i
M , f ei ie i M ,
f (z) iM z. ) f (z) = cz for a constant, jcj = M .

10
IV.1.9
1 2 3 P L K

Suppose h (z) is a continuous function on a curve . Show that


Z
h (z)
H (w) = dz; w 2 Cn ;
z w
is analytic on the complement of , and nd H 0 (w).

Solution
For the analyticity, dierentiate
h by hand. (See i Exercise III.1.6). The Deriv-
R h(z) h(z)
ative is H 0 (w) = lim 1w z (w+ w) z w
dz =
w!1
R h(z) w R
lim 1w (z (w+ w))(z w)
dz = (zh(z)dz
w)2
w!1

11
IV.2.1
1 2 3 P L K K
111 LLL KKK 246
Evaluate the following integrals, for a path that travels from i
to i in the right half-plane, and also for a path from i to i
in theR left half-plane.R R R
(a) z 4 dz (b) ez dz (c) cos z dz (d) sinh z dz

Solution
The integrals are all independent of path and can be evaluated by nding a
primitive.
(a)
Z i
z5 ( i)5 ( i)5 2 5i
z 4 dz = = = :
5 i 5 5 5
(b)
Z
i
ez dz = [ez ] i =e i
e i
= 1 ( 1) = 0:

(c)

Z
i
cos z dz = [sin z] i =

ei( i)
e i( i)
ei( i)
e i( i)
= sin ( i) sin ( i) = =
2i 2i
2 (e e )
= =i e e :
2i
(d)

Z
i
sinh z dz = [cosh z] i = cosh ( i) cosh ( i) =

e i+e i
e i
+e ( i)
= = 0:
2 2

12
IV.2.2
1 2 3 P L K K
111 LLL 246 R
Using an appropriate primitive, evaluate 1=z dz for a path that
travels from i to i in the right half-plane, and also for a path
from i to i in the left half-plane. For each path give a precise
denition of the primitive used to evaluate the integral.

Solution
In right halfplane use f (z) = Log z = log jzj + i , where < < as a
primitive, get
Z
1
dz = [Log z] i i = log j ij + i log j i j i = i:
z 2 2
In left halfplane use f (z) = log0 z = log jzj + i , where 0 < < 2 as a
primitive, get
Z
1 3
dz = [log0 z] i i = log j ij + i log j i j + i = i:
z 2 2

13
IV.2.3
1 2 3 P L K
111 LLL KKK
Show that if m 6= 1, then z m has a primitive on Cn f0g.

Solution
z m+1
The primitive is m+1
, which is analytic for z 6= 0 and m 2 Z, m 6= 1.

14
IV.2.4
1 2 3 P L K
p
Let D = Cn ( 1; 1], and consider the branch of z 2 1 on D that is positive
on the interval (1; 1).
(a) p
Show that z + z 2 1 omits the negative real axis, that is, the range of the
function on D does not include any values in the interval ( 1; 0] on the real
axis.
(b) p p
Show that Log z + z 2 1 is a primitive for 1= z 2 1 on D.
(c)
Evaluate
Z
dz
p ;
z2 1
where is the path from 2i to +2i in D counterclockwise around the circle
jzj = 2.
(d)
Evaluate the integral above in the case is the entire circle jzj = 2, oriented
counterclockwise. (Note that the primitive is discontinuous at z = 2.)

Solution p
D = Cn ( 1; 1], z 2 1
a) p p
z + z 2 1 = t (t > 0) ) z 2 1 = t z, z 2 1 = t2 2z + z 2 ,
2z = t2 + 1, z = (t2 + 1) =2.
But values of the branch are > 0 on (1; 1) = D \ R. ) It assumes no
negative values.
b) p p
Log z + z 2 1 is the composition of z+ z 2 1 and Log w on Cn ( 1; 0],
d
p 1=2
so its analytic. By the chain role, dz Log z + z 2 1 = z+p1z2 1 1 + 21 (z 2 1) 2z =
p1 1+ p z = p 1 , where we use everywhere the constant branch
z+ z 2 1 z2 1 z2 1
p d a az a
of z 2 1, by ????? it on (1; 1) or using dz
z = z
c)

15
R p 2i p p
p dz = Log z + z 2 1 2i = Log 2i + i 5 Log 2i i 5 = i
z2 1 p
Use fact that p z 2 1 is positive imaginary
p on positive imaginary axis, and
also Log 2i + i 5 Log 2i i 5 ?????. d)
R dz p 2+i0
p
2
z 1
= Log z + z 2 1 2+i0
p
At 2 + 0t , haveLog z + z 2 1 !
p p
log 2 + 3 + i Arg z + z 2 1 at z = 2. As z ! 2 + i0, have
p p p p
arg z 2 p1 ! + , jz 2 1j ! 3. ) z + z 2 1 !p 2+( 1) 3 = 2 3,
2 2 2
arg z + z 1 ! . As z ! 2+i0, have arg z 1! , jz 1j !
p p p p
3. ) z + z 2 1 ! 2 + ( 1) 3 = 2 3, arg z + z 2 1 ! .
Values
R dz coincide ) p 2+i0
p
z2 1
= Log z + z 2 1 2+i0 = log jj log jj + i ( i) = 2 i

16
IV.2.5
1 2 3 P L K

Show that
R an analytic function f (z) has a primitive in D if and
only if f (z) dx = 0 for every closed path in D.

Solution (A. Kumjian)


Let f be an analytic function dened on a domain D. Suppose rst that f
has a primitive in D. Then by the Fundamental
R Theorem of Calculus for
Analytic Functions (Part I) we have f (z) dx = 0 for every closed path
2 D (since for anyRsuch path A = B).
Conversely, suppose f (z) dx = 0 for every closed path 2 D. By formula
(1.1) on p. 102, we have
Z Z
f (z) dx + if (z) dy = f (z) dx = 0

for every close path 2 D. It follows that the integral on the left is path
independent for paths which are not necessarily closed and hence, by the
Lemma on page 77, there is a continuously dierentiable function F on D
@ @
such that dF = f dx + if dy, that is, @x F = f and @y F = if . It follows that
the real and imaginary parts of F satisfy the Cauchy-Riemann equations:

@ @F @F @
Re F = Re = Re f = Im if = Im = Im F;
@x @x @y @y
@ @F @F @
Re F = Re = Re if = Im f = Im = Im F:
@y @y @x @x

Moreover, F 0 (z) = f (z) for all z 2 D. Hence, f has a primitive in D, namely


F.

17
IV.3.1
1 2 3 P L K
LLL
2
By integrating e z =2 around a rectangle with vertices R; it R;and
sending R to 1, show that
Z 1
1 2 2
p e x =2 e itx dx = e t =2 ; 1 < t < 1:
2 1

Use the known value of the integral for t = 0. Remark. This


2
shows that e x =2 is an eigenfunction of the Fourier transform with
eigenvalue 1. For more see the next exercise.

Solution
R RR R t ( R+iy)2 =2 R t (R+iy)2 =2 RR 2
z 2 =2 x2 =2
@D
e dz = R
e dx 0
e idy+ 0
e idy+ R
e (x+it) =2 dx =
0 p R1
2
e ( R+iy) =2 , 0, uniformly for 0 6 y 6 1 as R ! 1. 2 = 1 e x =2 dx =
2

R1 R1
e (x +2ixt t )=2 dx =
2 2
(x+it)2 =2
1R
e dx = 1
2 1 2
et =2 1 e x =2 e ixt dx
R1
) p12
2 2
1
e x =2 dx = e t =2

18
IV.3.2
1 2 3 P L K

We dene the Hermite polynomials Hn (x) and Hermite orthogonal functions


n (x) for n 0 by

2 dn x2 x2 =2
Hn (x) = ( 1)n ex e ; n (x) = e Hn (x) :
dxn
(a)
Show that Hn (x) = 2n xn + is a polynomial of degree n such that is even
when n is even and odd when n is odd.
(b)
By integrating the function

it)2 =2 dn 2
e(z n
e z
dz
around a rectangle with vertices R; it R and sending R to 1, show that
Z 1
1
p n (x) e
itx
dx = ( i)n n (t) ; 1 < t < 1:
2 1

Hint. Use the identity from Exercise 1, and also justify and use the identity
dn (x+it)2 1 dn (x+it)2
e = e :
dxn in dtn
(c)
00
Show that n x n + (2n + 1) n = 0.
(d) R R
00 00
Using n m dx = n n dx and (c), show that
Z 1

n (x) m (x) dx = 0; n 6= m:
1

Remark. This shows that the n s form an orthogonal system of eigenfuctions


for the (normalized) Fourier transform operator F with eigenvalues 1 and
i. Thus F extends to a unitary operator on square-integrable functions.
Further, F 4 is the identity operator, and the inverse Fourier transform is given
by (F 1 f ) (x) = (Ff ) ( x).

Solution

19
PartRA
2
dn z2
0 = @DR e(z it) =2 dz ne dz
R 1 (x it)2 =2 dn x2 R1 2 2 dn x2
! 1e dxne dx = 1 e t =2 e itx ex =2 dx ne dx =
R
t2 =2 1 itx n t2 =2 n^
e e ( 1) (x) dx = e ( 1) n (t)
R 1 R 1 x2 =2n dn (x+it)2 R 1 x2 =2 dn 2

(x+it)\@DR
! 1
e dx n e dx = 1
e d(it)n
e (x+it) dx =
n R1 2 2 n R1 2 2
( i) dtd n 1 ex =2 e (x+it) dx = ( i) dtd n 1 ex =2 e 2itx et dx =
Z 1
n 2 p n p
e x =2 e 2itx dx = 2 ( i)n dtd n e t = 2 e t =2 in n (t)
2 2 2
( i) dtd n e t
| 1 {z }
p
2 e (2t)2 =2

If we set these equal,pwe get


( i)n e t =2 ^ n (t) = 2 e t =2 in n (t), p12 ^ n (t) = ( i)n n (t)
2 2

PartRB
2 dn z2
0 = @DR ez =2 dz ne dz
R 1 x2 =2 d z2 R 1 x2 =2 dn (x+it)2 nR1 2 n
(x+it)2
1
e dx
e dx = 1
e dx n e dx = ( i) 1
ez =2 dtd n e dx
R 1 (x it)2 =2 d x2 R1 2 2

1
e dz n
e dx = 1 ex =2 dzdn e (x+it) dx =
R1 2 d 2 R1 2 2
e t =2 1 e itx ex =2 n e x dx + ( i)n 1 ex =2 dxdn e (x+it) dx
2

| dx
{z }
( 1)n n (x)
dn
R1
e t2 =2 ^ (t) = in n e + ( i)n
t2 d
et
2
e x2 =2
dxe 2ixt
n dt dxn 1
p p
^ n (t) = ( i)n (t) + ( i)n 2 d
et e
2 2t2
= ( i)n 2 d t2
e
dxn dxn
dn
x2 =2
Hn (x), Hn (x) = ( 1)n ex
2 x2
n (x) = e dxn
e =
x2 d x2 x2 x2
H0 (x) = 1, H1 (x) = e dx
e = e ( 2x) e = 2x
x2 d2 x2 x2 x2 x2
H2 (x) = e dx2
e =e 4x2 e 2e = 4x2 2
R1 R1 2 dn
Hn (x) = 2n xn + lower order polynomial. 1 n (x) e itx dx = 1 e x =2 ( 1)n ex dx
2 x2 ixt
n e e d
R 1 R 1
( 1)n 1 Dxn ex ex =2 ixt dx = 1 e x =2 Dxn ex =2 ixt dx =
2 2 2 2

R1 x2 n (x it)2 =2 t2 =2 R1 2
e dx = (i)n 1 e x Dtn e(x it) =2 et =2 dx =
2 2
e Dx e
1
2 R1 2 2 2 R1 2 2
et =2 in Dtn 1 e x e(x it) =2 dx = et =2 in Dtn 1 e x =2 e ixt e t =2 dx =
p 2 2 2
2 et =2 in Dtn e t =2 e t =2

20
IV.3.3
1 2 3 P L K

Let f (z) = c0 + c1 z + + cn z n be a polynomial.


(a)
If the ck s are real, show that
Z 1 Z 2 X
n
2 i d
f (x) dx f e = c2k :
1 0 2 k=0

Hint. For the rst inequality, apply Cauchys theorem th the function f (z)2
separately on the top half an the bottom half of the unit disk.
(b)
If the ck s are complex, show that
Z 1 Z 2 X
n
2 d
jf (x)j dx f e i
= jck j2 :
1 0 2 k=0

(c)
Establish the following variant of Hilberts inequality, that

X
n
cj ck X
n
jck j2 ;
j;k=0
j+k+1 k=0

with strict inequalityRunless the complex numbers c0 ; : : : ; cn are all zero. Hint.
1
Start by evaluating 0 f (x)2 dx.

Solution
(a)
R1 R R 2
f (x)2 dx = f (z)2 dz 6 f ei d
1 0
R1 R 1
R2 2
1
f (x)2 dx = f (z)2 dz 6 f ei d
R1 1
R 2 2
Add, get 2 1 f (x)2 dx =6 f ei d
R2 i 2 P 2
n
i 2 P
n
ik
Pn
0
f e d = 2 c k , by from f e = c k e cm e im
k=0 k=0 k=0
P P 2 P 2 P 2
(b) f (x)2 = ak x k + i b k x k = ak x k + bk xk
R1 2 R1 P k 2
R1 P k 2
P k
P
f (x) dx = a k x dx+ b k x dx = a k x + bk x k =
P
1
k
1 1
ck x

21
hR P R1 P i
1 d d
1
ak eik 2
+ 1
bk eik 2
" #1
R1 P 2 R1 P
n P
n
xj+k+1
P
n
cj ck
(c) 0 ( cj xj ) dx = 0 cj ck xj+k dx = cj ck j+k+1 = j+k+1
j;k=0 j;k=0 j;k=0
R1 R1 P 0
jj 6 0 jf (x)j dx 6 0 jf (x)j dx 6
2 2
jck j , for equality, must have2
R1 R1
0
jf (x)j = 0 jf (x)j , so f (x) = 0, for 1 6 x 6 0, and then f (x) 0
2 2

Suppose P (z) = a0 + a1 z + ::: + an z n is a polynomial. By integrating


P (z)2 log z, for an appropriate branch of log z, around a keyhole contour,
Pn R1 Pn R
6
aj ak 2
show that j+k+1
= P (x) dx jak j2 . P (z)2 log z dz = 0,
j;k=0 0 k=0 @D
R1 R1 R2 2
gives 0 P (z) log x dx 0 P (z)2 (log x + 2 i) dx+ 0 P ei
2
iei d = 0.
R1 R 2 R
Get 2 i 0 P (x)2 dx = i 0 P ei ei d . Note that P ei d , so latter
R2 2 R1 R2
integral is i 0 P ei ei ( ) d . Thus 0 P (x)2 dx = 0 P ei ei ( ) 2d
R1 R2 2 d Pn
) 0 P (x)2 dx 6 0
P ei
2
= jak j2
k=0

22
IV.3.4
1 2 3 P L K

Prove that a polynomial in z without zeros is constant (the fundamental


theorem of algebra) using Cauchys theorem, along the following lines. If
P (z) is a polynomial that is not a constant, write P (z) = P (0) + zQ (z),
divide by zP (z), and integrate around a large circle. this will lead to a
contradiction if P (z) has no zeros.

Solution
P (z) is a polynomial with no zeros. Assume P (0) = 1 . Write p (z) =
1 + zQ (z).
H 1
H P (z) H 1 Q(z) H 1
jzj=R z
dz = zP (z)
dz = zP (z)
+ P (z)
dz = zP (z)
dz2 = i
jzj=R jzj=R jzj=R
! 0 by ML-estimate if deg P > 1.
M = max zP1(z) 1
Rn+1
, L R, when n = det P
z=R
ML 1
Rn
! 0, if R ! 1, if n > 1.

23
IV.3.5
1 2 3 P L K

Suppose that D is a bounded domain with piecewise smooth boundary, and


that f (z) is analytic on D [ @D. Show that

Area (D)
sup jz f (z)j 2 :
z2@D Length (D)
Show that this estimate is sharp, and that
R in fact there exist D and f (z) for
which equality holds. Hint. Consider @D jz f (z)j dz, and use Exercise 4
in section 1.

Solution
R R
[z f (z)] dz = zdz = 2iArea (D)
R
@D @D

@D
[z f (z)] dz 6 sup jz f (z)j Length (@D)
z2@D
) sup jz f (z)j > Area(D)
2 Length(@D)
z2@D
To see its sharp, take D = D = unit disk, get sup jz f (z)j = 1
z2@D
Area(D)
2 Length(@D) =2 2
=1

24
IV.3.6
1 2 3 P L K

H fjzj Rg and analytic


Suppose f (z) is continuous in the closed disk
on the open disk fjzj < Rg. Show that jzj=R f (z) dz = 0. Hint.
Approximate f (z) uniformly by fr (z) = f (rz).

Solution (A. Kumjian)


H H
To prove jzj=R f (z) dz = 0 it su ces to show that jzj=R f (z) dz ", for
every " > 0. So let " > 0 be given. Since f is continuous in the closed disk
fjzj Rg, it must be uniformly continuous there (since the close disk is both
closed and bounded). Hence, there is a > 0 such that for all z1 ; z2 in the
closed disk, we have
"
jf (z1 ) f (z2 )j < if jz1 z2 j < :
2 R
For r 2 (0; 1), dene fr on the disk fz : jzj < R=rg by fr (z) = f (rz) for all
z such that jzj < R=r. Then since
H fr is analytic on a domain containing the
closed disk fjzj Rg, we have jzj=R f (z) dz = 0 by Cauchys theorem. Now
let r be chosen such that 1 =R < r < 1. Then for all z with jzj = R we
have jz zrj = R (1 r) < and so by the choice of we have
"
jf (z) fr (z)j = jf (z) f (rz)j < :
2 R
Hence we have

I I I
"
f (z) dz = (f (z) fr (z)) dz j(f (z) fr (z))j jdzj 2 R = ":
jzj=R jzj=R jzj=R 2 R

by the ML-estimate theorem on p. 105.

25
IV.4.1
1 2 3 P L K
111 LLL
Evaluate
H then following integrals, Husingezthe Cauchy integral formula:
(a) jzj=2 zz 1 dz; n 0 (e) jzj=1 zm dz; 1<m<1
H zn
H Log z
(b) jzj=1 z 2 dz; n 0 (f) jz 1 ij=5=4 (z 1)2 dz
H sin z
H
(c) jzj=1 z dz (g) jzj=1 z2 (z2dz 4)ez
H H
(d) jzj=1 cosh
z 3
z
dz (h) dz
jz 1j=2 z 2 (z 2 4)ez

Solution
We rearragnde the Cauchy integral formula. Let D be bounded domain with
piecewise smooth boundary @D and let f be an analytic function dened in
a domain which contains D. Then for any z0 2 D and positive integer m we
have,
Z
f (z) 2 i dm
dz = f (z) :
@D (z z0 )m (m 1)! dz m z=z0

(a)
The nominator has one zero at z = 1 inside jzj = 2, thus by by Cauchys
theorem,
H zn 2 i n
jzj=2
dz = [z ]z=1 = 2 i:
z 1 0!
(b)
The nominator has no zeros inside jzj = 1, thus by by Cauchys theorem,
H zn
jzj=1
dz = 0:
z 2
(c)
The nominator has one zero at z = 0 inside jzj = 1, thus by by Cauchys
theorem,
H sin z 2 i
jzj=1
dz = [sin z]z=0 = 0:
z 0!
(d)
The nominator has one tripel zero at z = 0 inside jzj = 1, thus by by
Cauchys theorem,

26
H cosh z 2 i d2
jzj=1
dz = cosh z = i:
z3 2! dz 2 z=0
(e)
The nominator has one tripel zero at z = 0 inside jzj = 1, thus by by
Cauchys theorem.
If m 0, then the integrand z m ez denes an analytic function on C, hence,
by Cauchys Theorem the integral must be zero.
If m 1, the nominator has a zero of order m at z = 0 inside jzj = 1, hence
by by Cauchys theorem.
I
ez 2 i dm 1 z 2 i
m
dz = m 1
e = :
jzj=1 z (m 1)! dz z=0 (m 1)!
Thus
I
ez 0 if m 0;
dz = 2 i
jzj=1 zm (m 1)!
if m 1:
(f)
Let f be the principal branch of the logarithm dened on the slit plane, that
is, f (z) = Log z for all z 6= 0 with Arg z 6= . Then f is analytic on a
domain jz 1 ij 5=4. The nominator has one double zero at z = 1 inside
jz 1 ij < 5=4, thus by by Cauchys theorem.
H Log z 2 i d
jz 1 ij=5=4 2 dz = (Log z) = 2 i:
(z 1) 1! dz z=1
(g)
The nominator has one double zero at z = 0 inside jzj = 1, thus by by
Cauchys theorem,
H dz 2 i d e z
i
jzj=1 2
= = :
z (z 2 4) ez 1! dz z2 4 z=0 2
(h)
The nominator has one zero at z = 0, and one zero at z = 2 inside jz 1j = 2,
thus by by Cauchys theorem,

I
dz 2 i 1 2 i 1 i i
= + = + 2:
jz 1j=2 z (z 2 4) ez 0! (z 2 4) ez z=0 0! z (z + 2) ez z=2 2 4e

27
IV.4.2
1 2 3 P L K
LLL
Show that a harmonic function is C 1 , that is, a harmonic function
has partial derivatives of all orders.

Solution
@ @
Write u = Re f , and note that @x u = Re f 0 , @y u = @u
@y
@v
= @x = Re (f 0 ).
@v
v = Im f = Re ( if ), @x = Re ( if ). Now take successive derivatives, get
m
@ u
@xp @y q
a multiple of Re f is Re (if 0 ).
0

28
IV.4.3
1 2 3 P L K
LLL
Use the Cauchy integral formula to derive the mean value property
of harmonic functions, that
Z 2
d
u (z0 ) = u z0 + ei ; z0 2 D;
0 2
whenever u (z) is harmonic in a domain D and the closed disk
jz z0 j is contained in D.

Solution H H2
f (z) d
Write f (z0 ) = jz z0 j= z z0
dz = 0
f z0 + ei 2
, which is the MVP.

29
IV.4.4
1 2 3 P L K

Let D be a bounded domain with smooth boundary @D, and let z0 2 D.


Using the Cauchy integral formula, show that there is a constant C such
that

jf (z0 )j D sup fjf (z)j : z 2 @Dg


for any function f (z) analytic on D [ @D. By applying this estimate to
f (z)n , taking nth roots, and letting n ! 1, show that the estimate holds
with C = 1. Remark. This provides an alternative proof of the maximum
principle for analytic function.

Solution R
Write f (z0 ) = 1
2 i
f (z)
@D z z0
dz . Letkf k = max jf (z)j, 1
jz z0 j
6 1
dist(z0 ;@D)
= d1 .
z2@D
By MLestimate, L = length @D, jf (z0 )j 6 2 1 d kf k L. Apply to f , get n

jf (z0 )jn 6 2 1 d kf n k L. Use kf n k = kf kn , take nth roots, get jf (z0 )j 6


L 1=n
kf k 2 d
. Let n ! 1, get jf (z0 )j 6 kf k.

30
IV.5.1
1 2 3 P L K
111 LLL
Denna beh
over N og f ixas till Lite
Show that if u is a harmonic function on C that is bounded above,
then u is constant.
Hint. Express u as the real part of an analytic function, and expo-
nentiate.

Solution
Let u be the real part of the analytic function f = u + iv, where we know
that u C, for some constant C. Then set g = eu+iv where g is an entire
function. Now we have that

jgj = eu+iv = eu eiv = jeu j eiv = eu :


We can se that becauce u is bounded above, so is g bounded above. Now
we now that g is both analytic and bounded, by Liouvilles theorem g is
constant. Thus f is constant.

31
IV.5.2
1 2 3 P L K
111 LLL
Show that if f (z) is an entire function, and there is a nonempty
disk such that f (z) does not attain any values in the disk, then
f (z) is constant.

Solution
If f (z) does not attain values in the disk jw cj < ", then 1= (f (z) c) is
a bounded entire function, hence constant by Liouvilles theorem, and f (z)
is constant.
Let w0 be the center of the disk that f omits. Then 1= (f (z) w0 )

32
IV.5.2
1 2 3 P L K
LLL
Show that if f (z) is an entire function, and there is a nonempty
disk such that f (z) does not attain any values in the disk, then
f (z) is constant.

Solution (A. Kumjian)


Let f be such an entire function. Then there is a disc D = Dr (z0 ) for some
z0 2 C and r > 0 such that f (z) 62 D for all z 2 C. That is, jf (z) z0 j r
for all z 2 C. We dene an entire function g by the formula
1
g (z) = for all z 2 C:
f (z) z0
It follows that jg (z)j 1=r for all z 2 C. Hence, g is a bounded entire
function an so by Liouvilles Theorem g must be a constant function. The
desired result now follows immediately.

33
If f does not attain values in the disk jw cj < ", then 1= (f c) is bounded,
hence constant by Liouvilles theorem, and f is constant.

34
IV.5.3
1 2 3 P L K
LLL
A function f (z) on the complex plane is double periodic if there
are two periods w0 and w1 of f (z) that do not lie on the same line
thought the origin (that is, w0 and w1 are linearly independent over
the reals, and f (z + w0 ) = f (z + w1 ) = f (z) for all complex numbers
z). Prove that the only entire functions that are doubly periodic
are the constants.

Solution
The "lattice" of points mw0 +nw1 , m; n 2 Z are the corners of parallelograms
that cover C, each parallelogram being a translate F + mw0 + nw1 of "the
fundamental region" F as shown in the gure:

Figure IV.5.3
Thus each 2 C can be written = z + mw0 + nw1 with z 2 F . Since we
assume f ( ) = f (z), it follows that f i bounded, thus a constant.

35
If jf (z)j 2 M for jzj 6 R. Suppose jw0 j, jw0 j 6 R. Let P = parallelogram
with vertices 0; w0 ; w1 ; w0 + w1 , and suppose for z 2 P . Any z 2 C have
form + mw0 + nw1 where 2 P . Thus jf (z)j = jf ( )j 6 M , so f (z) is
bounded and entire.

36
IV.5.4
1 2 3 P L K

Suppose that f (z) is an entire function such that f (z) =z n is bounded


for jzj R. Show that f (z) is a polynomial of degree at most n.
What can be said if f (z) =z n is bounded on the entire complex
plane?

Solution
Apply the Cauchy estimates for f (m+1) (z) to disk jz z0 j < R, and let
R ! 1, to obtain f (m+1) (z0 ) = 0. If f (z) is a polynomial of degree 6 m
such that f (z) =z m is bounded near 0, then f (z) = cz m .

37
IV.5.5
1 2 3 P L K

Show that if V (z) is the velocity vector eld for a uid ow in the entire
complex plane, and if the speed jV (z)j is bounded, then V (z) is a constant
ow.

Solution
Use V (z) = f 0 (z) (section III.6 p92), when f is entire. Get f 0 constant, by
Liouvilles theorem, i.e. V (z) is constant.

38
IV.6.1
1 2 3 P L K
LLL
Let L be a line in the complex plane. Suppose f (z) is a continuous
complex-valued function on a domain D that is analytic on DnL.
Show that f (z) is analytic on D.

Solution
Rotate and apply result in text.

39
IV.6.2
1 2 3 P L K

Let h (t) be a continuous function on the interval [a; b]. Show that the Fourier
transform
Z b
H (z) = h (t) e itz dt
a
is an entire function that satises

jH (z)j CeAjyj ; z = x + iy 2 C;
for some constants A; C > 0. Remark. An entire function satisfying such a
growth restriction is called an entire function of nite type.

Solution
The analyticity of H (z) follows from the theorem in the text. If jhj 6 M ,
take C = M (b a) and A = max (jaj ; jbj). We have je itz j = ety . jH (z)j 6
M (b a) max ety 6 M (b a) eAjyj
a t b

40
IV.6.3
1 2 3 P L K

Let h (t) be a continuous function on a subinterval [a; b] of [0; 1). Show


that the Fourier transform H (z), dened as above, is bounded in the lower
half-plane.

Solution

41
IV.6.4
1 2 3 P L K

Let be a smooth curve in the plane R2 , let D be a domain in the complex


plane, and let P (x; y; ) and Q (x; y; ) be continuous complex-valued func-
tions dened for (x; y) on and 2 D. Suppose that the functions depend
analytically on for each xed (x; y) on .
Show that
Z
G ( ) = P (x; y; ) dz + Q (x; y; ) dy

is analytic on D.

Solution
Reduce to Riemann integral and apply theorem in the text.

42
IV.7.1
1 2 3 P L K

Find an application for Goursats theorem in which it is not patently clear


by other means that the function in question is analytic.

Solution
Answer to this is not known (by me).

43
IV.8.1
1 2 3 P L K
LLL
Show from the denition that
@ @ @ @
@z
z = 1; @z
z = 0; @z
z = 0; @z
z = 1:

Solution
We use the rst-order dierential operators (page 124)

@ 1 @ @ @ 1 @ @
= i and = +i :
@z 2 @x @y @z 2 @x @y
We have that

@ 1 @ @ 1
z = i (x + iy) = (1 + 1) = 1
@z 2 @x @y 2
@ 1 @ @ 1
z = +i (x + iy) = (1 1) = 0
@z 2 @x @y 2
@ 1 @ @ 1
z = i (x iy) = (1 1) = 0
@z 2 @x @y 2
@ 1 @ @ 1
z = +i (x iy) = (1 + 1) = 1
@z 2 @x @y 2

44
IV.8.2
1 2 3 P L K
LLL
Compute @@z (az 2 + bz z + cz 2 ). Use the result to determine where
az 2 + bz z + cz 2 is complex-dierentiable and where it is analytic.
(See Problem II.2.3)

Solution
By the Leibnitz rule and the results of Exercise 1, the z derivative is bz+2cz.
The function is complex dierentiable at z if and only if bz + 2cz = 0. If
b = c = 0, the function is entire. Otherwise this locus is either a point f0g
or a straight line through the origin, and there is no open set on which the
function is analytic.

45
IV.8.3
1 2 3 P L K
LLL
Show that the Jacobian of a smooth function f is given by
2 2
@f @f
det Jf = :
@z @z

Solution
@u @v
@x @x @u @v @u @v
Jf = det @u @v = @x @y @y @x
@y @y
2 2
@f 2 @f 2 @(u+iv)
@z @z
= 1
4 @x
i @(u+iv)
@y
1
4
@(u+iv)
@x
+ i @(u+iv)
@y
2 2 2 2
1 @u @v @v @u 1 @u @v @v @u
= 4 @x
+ @y
+ @x
+ @y 4 @x @y
+ @x
+ @y
h i
1
= 4
2 @u @v
@x @y
@v @u
2 @x @y
+ 2 @u @v
@x @y
@v @u
2 @x @y
= @u @v
@x @y
@v @u
@x @y
= det Jf

46
IV.8.4
1 2 3 P L K
LLL
Show that

@2 @2 @2
+ = 4 :
@x2 @y 2 @z@ z
Deduce the following, for a smooth complex-valued function h.
(a) h is harmonic if and only if @ 2 h=@z@ z = 0.
(b) h is harmonic if and only if @h=@z is conjugate-analytic.
(c) h is harmonic if and only if @h=@ z = 0.
(d) If h is harmonic, then any mth order partial derivative of h is
a linear combination of @ m h=@z m and @ m h=@ z m .

Solution
@2 @ @ @ @ @2 2
@ 2
@ @2 @2 @2
4 @z@ z
= @x i @y @x
+ i @y = @x2
i @x@y i @y@x + @y 2
= @x2
+ @y 2
(a) clear
@2h @h
(b) h harmonic , @ z@z
=0, @z
is analytic
@2h @ @h
(c) h harmonic , @z@ z
=0, @z @z
= 0. Since @h
@z
= @@hz , this ????? @h
@z
is
@h
analytic , @ z is conjugate analytic.
@ k+l h
(d) If h is analytic, then @z k @ z l = 0 unless l = 0. Only non-vanishing deriva-
@mh k+l
tive is @zm = h (z). If h is conjugate analytic, ????? @@zk @zhl = 0 unless l = 0.
m

If h is harmonic, then h = analytic + conjugate analytic, so all derivatives


k l
of h vanish except for @@zhk = @@zhk .

47
R
(f) jz 1 ij=5=4 (zLog1)z2 dz = 21!i dz
d
Log zjz=1 = 2 i z1 z=1 = 2 i
R d e z ( e z )(z2 4) e z (2z)
(g) jzj=1 z2 (z2dz 4)ez = 2 i dz z2 4
=2 i (z 2 4)2
z=0 z=0
2 i4 i
= 16
= 2
R dz 1 1
(h) jz 1j=2 z(z 2 4)ez
=2 i (z2 4)ez
+ 2 i (z+2)ez =
z=0 z=2
1
2 i 4
+ 8e2 = i 12
1
+ 1
4e2

48
IV.8.5
1 2 3 P L K
LLL
With dz = dx idy, show that for a smooth function f (z) that
@f @f
df = dz + :
@z @z

Solution
df = @f
@x
dx + @f
@y
dy
@f @f
@z
dz + @z
dz = 12 @fi @f
@x @y
(dx + idy) + 21 @f
@x
+ i @f
@y
(dx idy) =
h i
1 @f 1 @f 1 @f 1 @f i @f @f @f @f
2 @x
dx + 2 @y
dy + 2 @x
dx + 2 @y
dy + 2 @x
dy @y
dx @x
dy + @y
dx = df

49
IV.8.6
1 2 3 P L K

Show that if D is a domain with smooth boundary, and if f (z) and g (z) are
analytic on D [ @D, then
Z ZZ
f (z) g (z) dz = 2i f (z) g 0 (z) dx dy:
@D D
Compare this formula with Exercise 1.4.

Solution R RR @ RR
By (8.4), @D f (z) g (z) dz = 2i D @E (f g) dz = 2i D f gdz
@g @f @g
Using the Leibnitz rule, obtain @@z (f g) = RRf @z = f g
R f @z + g @z =
0

To get Ex1.4, take f = 1, g (z) = z, get @D z dz = 2i D dxdt = 2iArea

50
IV.8.7
1 2 3 P L K
LLL
Show that the Taylor series expansion at z0 = 0 of a smooth function
f (z), through the quadratic terms, is given by

@f @f 1 @2f @2f
f (z) = f (0)+ (0) z+ (0) z+ 2
2
(0) z + 2 (0) jzj2 +o jzj3 :
@z @z 2 @z @z@ z

Solution
Since a smooth function f is a linear combination of the functions 1; z; z; z 2 ; z 2 ; jzj2
and a reminder term O jzj3 , it su ces to check the formula for these six
functions. The formula holds fro each of these. For instance, for jzj2 = z z
2 @2f
are harmonic. @f@z
(0) = 0, @f
@z
(0) = 0, @@ zf2 (0) = 0, f (0) = 0, @z@ z
(0) = 1.
3 3
The term O jzj has Taylor series 0 + O jzj . That ????? it.

51
IV.8.8
1 2 3 P L K
LLL
Establish the following version of the chain rule for smooth complex-
valued functions w = w (z) and h = h (w).

@ @h @w @h @ w
(h w) = + ;
@z @w @ z @w @z
@ @h @w @h @ w
(h w) = + :
@z @w @z @ w @z

Solution
Suppose w (0) = 0, h (0) = 0, write h (w) = aw + bw + O jwj2 , a = @w@h
(0),
@h 2 @w @w
b = @ w (0) and w (z) = w + w + O jzj , = @z (0), b = @ z (0) then
h (w (z)) = a z + z + O jzj2 + b z + z + O jzj2 + O jwj2 =
a z + a z + b z + b z + O jzj2 = a + b z + (a + b ) z + O jzj2
This gives,
@h w @h @w
@z
(0) = a + b = @w @z
+ @@hw @w
@z @ w=@ z
@h @w
= @w @z
+ @@hw @@wz
@h w @h @w @h @w @h @w @h @ w
@z
(0) = a + b = @w @z
+ @w
= @w @z
+ @ w @z
@z
|{z}
@ w=@z

52
IV.8.9
1 2 3 P L K
LLL
Show with the aid of the preceding exercise that if both h (w) and
w (z) are analytic, then (h w) (z) and (h w)0 (z) = h0 (w (z)) w0 (z).

Solution
If the functions h and w are analytic, then @h@ zw = 0 + 0 = 0 so h w is
analytic. (h w)0 (z) = @z
@ @h @w
h w = @w @z
+ 0 = h0 (w (z)) w0 (z)

53
IV.8.10
1 2 3 P L K

Let g (z) be a continuously dierentiable function on the complex plane that


is zero outside of som compact set. Show that
ZZ
1 @g 1
g (w) = dx dy; w 2 C:
C @z z w
Remark. If we integrate this formally by parts, we obtain
ZZ
1 @ 1
g (w) = g (z) dx dy:
C @z z w
Thus the "distribution derivative" of 1= ( (z w)) with respect to z is the
point mass w ("Dirac delta-function"), in the sense that it is equal to 0 away
from w, and it is innite at w in such a way that its integral (total mass) is
equal to 1.

Solution
Apply Pompeeius formula to a large disk fjzj < Rg.

54
V 1 2 3 4 5 6 7 8 9 10 11 12 13 14 15 16 17 18 19
1
2
3
4
5
6
7
8

1
V.1.1
1 2 3 P L K

(Harmonic Series) Show that


X
n
1
log n:
k=1
k
P1 1
Deduce that the series k=1 k does not converge. Hint. Use the
estimate
Z k+1
1 1
dx:
k k x

Solution
Let k be a nonnegative integer. Then since the function x 7! 1=x is decreasing
on the positive reals, we have for all x 2 [k; k + 1] that 1=x 1=k . Hence
locking at the oversum in the interval [k; k + 1]
Z k+1
1 1 1
((k + 1) k) = dx:
k k k x
Therefore, for n 1
X
n n Z
X k+1 Z n+1
1 1 1
dx = dx = log (n + 1) log n:
k=1
k k=1 k x 1 x
Where the last inequality follows from the fact that x 7 P
! log x is an increas-
ing function on the positive reals. And thus is the sum nk=1 k1 have not limit
as n ! 1 and does not converge.

2
V.1.2
1 2 3 P L K
P1
Show that if p < 1, then the series k=1 1=k p diverges. Hint. Use
Exercise 1 and the comparison test.

Solution
If k > 1 then, k p < k for 0 < p < 1, then
1 1
p
> :
k k
Thus we have
X1
1 X1
1
p
> :
k=1
k k=1
k
P
By the comparison test, we have that
Pthe since the sum nk=1 1=k diverge as
n ! 1 by Exercise 1, also the sum nk=1 1=k p diverge as n ! 1, which was
to be shown.

3
V.1.3
1 2 3 P L K
P1
Show that if p > 1, then the series k=1 1=k p converges to S, where

Xn
1 1
S p
< :
k=1
k (p 1) np 1

Rk
Hint. Use the estimate k1p < k 1 dx
xp
.

Solution
Let k be a nonnegative integer. Then since the function x 7! 1=x is decreasing
on the positive reals, we have for all x 2 [k 1; k] that 1=k 1=x. Becauce
p > 1, it follows that 1=k p 1=xp . Hence locking att the undersum in the
interval [k 1; k]
Z k
1 1 1
p
(k (k 1)) = p p
dx
k k k 1 x

Therefore

Xn n Z
X k Z n n
1 dx dx x p+1 1 n p+1
1
= 1+ = 1+ = 1+ 1+ :
k=1
kp k=1 k 1 xp 1 x
p p+1 1 p 1 p 1 p 1

Hence the serie converge to S, where


1 p
S 1+ = ;
p 1 p 1
since its terms are > 0.
Now we have that

XN Z N N
1 dx x p+1 n p+1
N p+1
n p+1

p
6 =1+ = !
k=n+1
k n xp p+1 n p 1 p 1
as N ! 1.
Thus we have

4
XN
1 Xn
1 n p+1
1
=S 6 =
k=n+1
kp k=1
k p p 1 (p 1) np 1

thus

XN
1 1
S :
k=n+1
kp (p 1) np 1

5
V.1.4
1 2 3 P L K

Show that the series


X1
( 1)k+1 1 1 1
=1 + +
k=1
k 2 3 4
converges. Hint. Show that the partial sums of the series satisfy
S2 < S 4 < S 6 < < S5 < S3 < S1 .

Solution
We dene the sum Sn by
Xn
( 1)k+1 1 1 1
Sn = =1 + + ::::
k=1
k 2 3 4
This is the alternating series test, which is standard signs alternate, and
terms ! 0, jtermsj # , so series converge.

6
V.1.5
1 2 3 P L K

Show that the series


1 1 1 1 1 1 1 1
1+ + + + + +
3 2 5 7 4 9 11 6
converges to 3S=2, where S is the sum of the series in Exercise 4.
(It turns out that S = log 2.) Hint. Organize the terms in the series
in Exercise 4 in groups of four, and relate it to the groups of three
in the above series.

Solution
We have the series S from exercise V.1.5 and arrange the terms in groups of
four
1 1 1 1 1 1 1 1 1 1 1
S=1 + + + + + + :::
2 3 4 5 6 7 8 9 10 11 12
And now we arrange the terms in groups of four

1 1 1 1 1 1 1 1 1 1 1
S= 1 + + + + + + :::
2 3 4 5 6 7 8 9 10 11 12

Now we rearrange the terms in groups of two, and divide the terms by 2

S 1 1 1 1 1 1
= + + + :::
2 2 4 6 8 10 12
Now add one group of four from the sum S with a group of two from the
sum S=2, and so on

3 1 1 1 1 1 1 1 1
S = 1+ 2 + + 2 + + 2 + :::+ =
2 3 4 5 7 8 9 11 12
1 1 1 1 1 1 1 1
= 1+ + + + + + :::
3 2 5 7 4 9 11 6

Since series S in Exercise 4 converge, thus also 3S=2 converge by ......

7
V.1.6
1 2 3 P L K

Show that
Z R
1
= [log (log k)]R
e = log (log R) log e (log e) ! 1
e k log k
P
diverges while 1 1
k=1 k(log k)2 converges.

Solution 1 (Comparison Test)


For the rst sum
n+1
2X
1
k=2n +1
k log k

there are 2n terms between k = 2n +1 and k = 2n+1 , each of the terms greater
1 1
than 2n+1 log 2n+1
, so these 2n terms have sum greater than 2(n+1) log 2
. Thus
series diverges, by comparison with the harmonic series. The other series is
treated similarly, using an upper estimate.

For the other sum


n+1
2X
1
k=2n +1
k (log k)2
there are 2n terms between k = 2n + 1 and k = 2n+1 , each of the terms less
1 1
than 2n+1 log 2n+1
, so these 2n terms have sum less than 2(n+1) log 2
. Thus series
converges, by comparison with the harmonic series.

Solution 2 (Integral Test)


Note that x log x and x (log x)2 are increasing for x > 1, so the terms of both
series are positive and decreasing. Vi can use Cauchys integralkriterium and
look att them corresponding integrals. Becauce of di cult with the integrand
we split the integral into two parts
Z R
1
= [log (log x)]R
e = log (log R) log (log e) ! 1;
e x log x

8
as R ! 1.
Z R R
1 1 1 1
= = ! 1;
e x (log x)2 log x e log e log R
as R ! 1.
We have that the series
X
1
1
k=1
k log k
diverges, while the series
X
1
1
k=1
k (log k)2
konverges.

9
V.1.7
1 2 3 P L K
P Pk=n 1
Show that the series ak converges if and only if k=m k(log k)2 converges.

Solution
P
n P
n
If Sn is the partial-sum ak , then Sn Sm 1 = ak . This follows imme-
k=1 k=m
P
n P
n
diately from ak = S n Sm 1 , when Sn = ak is the nth partial-sum of
k=m k=1
the series.
d xk kxk(k+x2k ) xk (2k)x2k 1
1
V.2.1 dx k+x2k
= 2 = 0 at k 2 xk 1 + kx3k 1 = 2kx3k 1 ,
(1+x
p
2k
)
k 2 xk 1 = k 3k 1 , k = x2k , k = xk . ) p
Function " for 0 6 x 6 k 1=2k , # for
x > k 1=2k , ! 0 as x ! 1. k+x 2k 6 k+k = = worst-case estimation. )
xk k 1
p
2 k
Converge to 0 uniformly.

10
V.2.1
1 2 3 P L K

Show that fk (x) = xk = k + xk converges uniformly to 0 on [0; 1).


Hint. Determine the worst-case estimator "k by calculus.
Solution
Suppose fk (x) converge pointwise to a function f (x) as k ! 1.
We have

xk
fk (x) = ;
k + x2k
thus

0 kxk 1
k + x2k xk (2k) x2k 1
fk (x) = ;
(k + x2k )2
0
there fk (x) = 0 for x1 = 0 and x2 = k 1=(2k) .
We have

x 0 k 1=(2k)
0
fk (x) 0 + 0
fk (x) % &
and the function is increasing for 0 x k 1=(2k) , and decreasing for x
for x k 1=2k , and fk (x) ! 0 as x ! 1. We have that fk (x) attains its
maximum for x = k 1(2k) .
We determine f (x) applying the works case estimator "k to the function
fk (x),
p
xk k 1
f (x) = lim lim = lim p = 0:
k!1 k + x2k k!1 k + k k!1 2 k

The series fk (x) = xk = k + xk converge uniformly to 0 on [0; 1).

11
V.2.2
1 2 3 P L K

Show that gk = xk = 1 + xk converges pointwise on [0; 1) but not


uniformly. What is the limit function? On which subsets of [0; 1)
does the sequence converge uniformly?

Solution
Suppose gk (x) converge pointwise to a function g (x) as k ! 1, we start to
show that is is an increasing function

0 kxk 1
xk kxk 1
kxk 1
gk (x) = = >0
(1 + xk )2 (1 + xk )2
also is the function gk (x) increasing, now we take the limit in three intervals.
If 0 x < 1, in this interval we have that xk ! 0 as k ! 1, and we have

xk 0
g (x) = lim k
= = 0:
k!1 1 + x 1+0
If x = 1, then

1k 1 1
g (1) = lim k
= = :
k!1 1 + 1 1+1 2
k
If x > 1, in this interval we have that x ! 0 as k ! 1, and we have

xk 1 1
g (x) = lim = lim = = 1:
k!1 1 + xk k!1 1=xk + 1 0+1
We also have that gk (x) ! g (x), where
8
< 0 for 0 x < 1;
1
g (x) = for x = 1;
: 2
1 for x > 1
Convergence is uniform on [0; 1 "] for any " > 0. Not uniform on [0; 1],
because limit function is not continuous at 1. Convergence is uniform on
[1 + "; +1] for any " > 0.

12
V.2.3
1 2 3 P L K

Show that fk (z) = z k =k converges uniformly for jzj < 1. Show that
0
fk (z) does not converge uniformly for jzj < 1. What can be said
0
about uniform convergence of fk (z)?

Solution
Suppose fk (z) converge pointwise to a function f (z) as k ! 1 in the interval
jzj 1, in this domain we have that z k =k 1=k thus

zk 1
f (z) = lim lim = 0:
k!1 k k!1 k

Thus fk (z) converges uniformly for jzj < 1.


0 0
Now suppose fk (z) converge pointwise to a function f (z) as k ! 1 in the
interval jzj 1, now we must split the interval when we take the limit

0 0 for jzj < 1;


f (z) = lim z k 1
=
k!1 1 for jzj = 1:
0
If jzj = 1, then fk (z) = 1 so the series can not converge uniformly for
0
jzj < 1. But for any " > 0 the series fk (z) = z k 1 converges uniformly for
jzj 1 ".

13
V.2.4
1 2 3 P L K

Show that
X1
1 xk
k=1
k 2 1 + x2k
converges uniformly for 1 < x < +1.

Solution
We = k12 , where the sum
P1apply the Weierstrass M test (p. 135.)k with Mk 2k
k=0 Mk converges. First we show that x 1 + x for all 1 < x <
+1.
We have

xk 1 1 + x2k = 1 + x2k for jxj 1


xk x2k < 1 + x2k = 1 + x2k for jxj > 1
This establishes the fact that

xk
1
1 + x2k
for all 1 < x < +1.
Hence, for each k 1 and 1 < x < +1,

1 xk 1
= Mk :
k 2 1 + x2k k2
Thus convergence is uniform on 1 < x < +1 by the Weierstrass M test.

14
V.2.5
1 2 3 P L K
P1 xk
For which real numbers x does k 1+x2k
converge?

Solution
Suppose gk (x) converge pointwise to a function g (x) as k ! 1, and take
limit in three intervals
If x < 1, in this interval we have that xk ! 0 as k ! 1, and we have
X
1
1 xk X 1
1 X 1 1X 1
< < = xk
k=1
k 1 + x2k k=1
1=x k + xk
k=1
1=x k
k=1

which is a convergent series.


If x = 1, then xk = x2k = 1 then
X
1
1 xk X 1 1
=
k=1
k 1 + x2k k=1
2k
which is a divergent series.
If x > 1, in this interval we have that xk ! 1 as k ! 1, and we have
X
1
1 xk X11
1 X 1 1
= <
k=1
k 1 + x2k k=1
k 1=xk + xk k=1
xk
which is a convergent series.

15
V.2.6
1 2 3 P L K
P1 xk
Show that for each " > 0, the series k 1+x2k
converges uniformly
for x 1 + ".

Solution
We have

xk
fk (x) = ;
1 + x2k
thus

0 kxk 1
1 + x2k xk (2k) x2k 1
xk 1 x3k 1
fk (x) = =
(1 + x2k )2 (1 + x2k )2
0
there fk (x) = 0 for x1 = 0 and x2 = 1.
We have

x 0 1
0
fk (x) 0 + 0
fk (x) % &
and the function is increasing for 0 x 1, and decreasing for x 1, and
fk (x) ! 0 as x ! 1. We have that fk (x) attains its maximum for x = 1.
If x = 1, then xk = x2k = 1 then
X
1
1 xk X 1 1
=
k=1
k 1 + x2k k=1
2k
which is a divergent series.
If x > 1, in this interval we have that xk ! 1 as k ! 1, and we have

X
1
1 xk X
1
1 (1 + ")k X
1
1 1
= 2k
=
k=1
k 1 + x2k k=1
k 1 + (1 + ") k=1
k 1= (1 + ") + (1 + ")k
k

X1
1 1
k=1
k (1 + ")k

16
V.2.7
1 2 3 P L K
LLL
Let an be aPbounded sequence of complex numbers. Show that for each " > 0,
the series 1 n=1 an n
z
converges uniformly for Re z 1 + ". Here we choose
the principal branch of n z .

Solution
P1
an n z , jan j 6 C, Re z > 1 + ". Apply Weierstrass M-test, jan n z j 6
n=1 P P
Cn Re z 6 n1+" C
= Mn , Mn < 1 ) an n z converges uniformly for
Re z > 1 + ".

17
V.2.8
1 2 3 P L K
LLL
P zk
Show that k2
converges uniformly for jzj < 1.

Solution
Apply the Weierstrass M test with M = 1=k 2 in jzj 1, we nd that
X1
zk X1
jzjk X 1
1
= < ;
k=1
k2 k=1
k2 k=1
k2
which is a convergent series by Weierstass M test. Thus the series con-
verges uniformly for jzj < 1.

18
V.2.9
1 2 3 P L K
LLL
P zk
Show that k
does not converges uniformly for jzj < 1.

Solution
Suppose that we have uniform convergence for jzj < 1, then the series must
converge for jzj 1
Apply the Weierstrass M test with M = 1=k in jzj 1, we nd that
X1
zk X1
jzjk X 1
1
= 2
< ;
k=1
k k=1
k k=1
k
which is a divergent series by Weierstass M test. Thus the series do not
converge uniformly for jzj < 1.

19
V.2.10
1 2 3 P L K

Shoe that if a sequence of functions ffk (x)g converges uniformly on Ej for


1 j n, then the sequence converges uniformly on the union E = E1 [
E2 [ [ En .

Solution
Use Cauchy criterion. Let " > 0. Choose Nj such that jfn (x) fk (x)j < k
for m; k > Nj and x 2 Ej . Let N = max fN1 ; : : : ; Nn g. Then jfm (x) fk (x)j <
" for m; k > N and x 2 E1 [ : : : [ En . ) ffm g converges uniformly.

20
V.2.11
1 2 3 P L K

Suppose that E is a bounded subset of a domain D C at a positive distance


from the boundary of D, that is > 0 such that jz wj for all z 2 E
and w 2 CnD. Show that E can be covered by a nite number of closed
disks contained in D. Hint. Consider all closed disks with centers at points
(m + ni) =10 and radius =10 that meet E.

Solution
Follow the hint. There are only nitely many of the disks in ????? (because
E is bounded); and they cover E, because the collection of all disks centred at
(m + ni) =10 of radius =10 covers the complex plan, and they are contained
in D, because dist ("; @D) > .

21
V.2.12
1 2 3 P L K

Let f (z) be analytic on a domain D, and suppose jf (z)j M for all z 2 D.


Show that for each > 0 and m 1, f (m) (z) m!M= m for all z 2 D
whose distance from @D is at least . Use this to show that if ffk (z)g is a
sequence of analytic functions on D that converges to f (z) on D, then for
(m)
each m the derivatives fk (z) converge uniformly to f (m) (z) on each subset
of D at a positive distance from D.

Solution
Use the Cauchy estimates for f (m) (w) on a disk centred at w of radius <
d (w; @D), get f (m) (w) 6 m!m
m .

22
V.3.1
1 2 3 P L K
LLL
Find the radius of convergence of the following power series:
P
1 P1
3k z k
P
1
kk
(a) 2k z k (d) k
4 +5 k (g) 1+2k kk
zk
k=0 k=0 k=1
P1 P1
2k z 2k
P1
(b) k k
6k
z (e) k2 +k
(h) (log k)k=2 z k
k=0 k=1 k=3
P1
2 k
P1
z 2k
P1
k!z k
(c) k z (f) 4k k k
(i) kk
k=1 k=1 k=1

Solution
(a)
We apply the ratio test, thus

ak 2k 1 1
R = lim = lim k+1 = lim = :
k!1 ak+1 k!1 2 k!1 2 2

(b)
We apply the ratio test, thus

ak k 6k+1 6k 6
R = lim = lim k = lim = lim = 6:
k!1 ak+1 k!1 6 k + 1 k!1 k + 1 k!1 1 + 1=k

(c)
We apply the ratio test, thus

ak k2 k2 1
R = lim = lim 2 = lim 2
= lim = 1:
k!1 ak+1 k!1 (k + 1) k!1 k + 2k + 1 k!1 1 + 2=k + 1=k 2

(d)
We apply the ratio test, thus

ak 3k 4k+1 + 5k+1
R = lim = lim k =
k!1 ak+1 k!1 4 + 5k 3k+1
4 k+1 4 k+1
3k 5k+1 5
+1 5 5
+1 5
= lim = lim = :
k!1
3k+1 5k 4 k
+1 k!1
3 4 k
+1 3
5 5

23
(e)
We apply the ratio test, thus

ak 2k z 2k (k + 1)2 + k + 1 k+2 1
R = lim = lim 2 = lim =
k!1 ak+1 k!1 k + k 2k+1 z 2(k+1) k!1 k 2 jzj2
1 + 2=k 1 0; 2 jzj2 < 1
= lim = :
k!1 1 2 jzj2 1; 2 jzj2 > 1
p
The convergence radius R = 1= 2.

(f)
We apply the ratio test, thus

ak z 2k 4k+1 (k + 1)k+1 4 (k + 1) (k + 1)k


R = lim = lim k k = lim =
k!1 ak+1 k!1 4 k z 2(k+1) k!1 z2 kk
k
4 (k + 1) 1 + 1=k
= lim = 1:
k!1 jzj2 1

(g)
We apply the ratio test, thus

ak kk 1 + 2k+1 (k + 1)k+1 k k 1 + 2k+1 (k + 1)k+1


R = lim = lim = lim =
k!1 ak+1 k!1 1 + 2k k k (k + 1)k+1 k!1 (k + 1)k (1 + 2k k k ) (k + 1)
!
k
1=k 1 2k+1 (k + 1)k
= lim + =
k!1 1 + 1=k (1 + 2k k k ) (k + 1) 1 + 2k k k
!
k k
1=k 1 k+1 1
= lim +2 = 2:
k!1 1 + 1=k (1 + 2k k k ) (k + 1) k 1= (2k k k ) + 1

(h)
We apply the root test, thus

24
1 1 1
R= p = q = = 0:
lim k
jak j lim log k
(log k)k=2
k
k!1 lim k!1
k!1

(i)
We apply the ratio test, thus

k k
ak k! (k + 1)k+1 k+1 1
R = lim = lim k = lim = lim 1+ = e:
k!1 ak+1 k!1 k (k + 1)! k!1 k k!1 k

25
V.3.2
1 2 3 P L K
LLL
Determine for which z the following series converge.
P
1 P1 P
1
(a) (z 1)k (c) 2m (z 2)m (e) nn (z 3)n
k=1 m=0 n=1
P1
(z i)k P1
(z+1)m P1
2n
(b) k!
(d) m2
(f) n2
(z 2 i)n
k=0 m=0 n=3

Solution
(a)
We apply the ratio test for ak = 1, thus

ak 1
R = lim = lim = 1:
k!1 ak+1 k!1 1

hence the radius of convergence is R = 1. Therefore, the series converges for


all z satisfying jz 1j < 1 and diverges for all z satisfying jz 1j > 1. Now
consider z such that jz 1j = 1 then

jz 1j = 1:
P1
Since k=1 1 diverges, the power series diverges for such z also. Hence, the
given series converges for all z satisfying jz 1j < 1.

(b)
We apply the ratio test for ak = 1=k!, thus

ak 1 (k + 1)!
R = lim = lim = lim (k + 1) = 1:
k!1 ak+1 k!1 k! 1 k!1

Therefore, the series converges for all z in C.

(c)
We apply the ratio test for am = 2m , thus

am 2m 1 1
R = lim = lim m+1 = lim = :
m!1 am+1 m!1 2 m!1 2 2
hence the radius of convergence is R = 1=2. Therefore, the series converges
for all z satisfying jz 2j < 1=2 and diverges for all z satisfying jz 2j > 1=2.
Now consider z such that jz 2j = 1=2 then

26
m
m m m 1
j2 (z 2) j = 2 = 1:
2
P
Since 1 m=1 1 converges, the power series diverges for such z also. Hence,
the given series converges for all z satisfying jz 2j < 1=2.

(d)
We apply the ratio test for am = 1=m2 , thus

am 1 (m + 1)2 m2 + 2m + 1 1 + 2=m + 1=m2


R = lim = lim 2 = lim = lim = 1:
m!1 am+1 m!1 m 1 m!1 m2 m!1 1

hence the radius of convergence is R = 1. Therefore, the series converges for


all z satisfying jz + ij < 1 and diverges for all z satisfying jz + ij > 1. Now
consider z such that jz + ij = 1 then

(z + i)m 1m 1
2
= 2 = 2:
m m m
P1 2
Since m=1 1=m converges, the power series converges for such z also.
Hence, the given series converges for all z satisfying jz + ij 1.

(e)
We apply the ratio test for an = nn , thus

n
an nn nn n 1
R = lim = lim = lim n = lim = 0:
n!1 an+1 n!1 (n + 1)n+1 n!1 (n + 1) (n + 1) n!1 n+1 n+1

Therefore, the series diverges for all z satisfying jz 3j > 0. Now consider z
such that jz 3j = 0 then

jnn (z 3)n j = nn 0n = 0:
P
Since 1 n=1 0 converges, the power series converges for z = 3. Hence, the
given series only converges for z = 3.

(f)
We apply the ratio test for ak = 2k =k 2 , thus

27
ak 2k (k + 1)2 k 2 + 2k + 1 1 + 2=k + 1=k 2 1
R = lim = lim 2 k+1
= lim 2
= lim = :
k!1 ak+1 k!1 k 2 k!1 2k k!1 2 2

hence the radius of convergence is R = 1=2. Therefore, the series con-


verges for all z satisfying jz 2 ij < 1=2 and diverges for all z satisfying
jz 2 ij > 1=2. Now consider z such that jz 2 ij = 1=2 then
n
2n n 2n 1 1
(z 2 i) = 2 = :
n2 n 2 n2
P
Since 1 2
k=1 1=n converges, the power series converges for such z also. Hence,
the given series converges for all z satisfying jz 2 ij 1=2.

28
V.3.3
1 2 3 P L K
LLL
Find the radius of convergence of the following series.
P
1
n
(a) z 3 = z + z 3 + z 9 + z 27 + z 81 +
P p
n=0
(b) z = z 2 + z 3 + z 5 + z 7 + z 11 +
p prime

Solutions
(a)
(b)
1 1
R= p = = 1:
lim sup k
jak j 1
Neither series converges at z = 1, so R = 1.

29
V.3.4
1 2 3 P L K
P
Show that the function dened by f (z) = z n! is analytic on the open unit
disk fjzj < 1g. Show that jf (r )j ! +1 as r ! 1 whenever is a root of
unity. Remark. Thus f (z) does not extend analytically to any larger open
set than the open unit disk.

Solution
Cauchy-Hadamard formula gives R = 1 (radius of convergence) ) f (z) is
analytic for jzj < 1. Suppose is an N th root of unity, N = 1. Then
n!=N P
1
(r )n! = rn! N = rn! for n > N . Now rn! " 1 as r " 1, because it is
increasing, and the nite partial sums ! 1 as r ! 1. ) jf (r )j ! +1 as
r ! 1.

30
V.3.5
1 2 3 P L K
LLL
What functions are represented by the following power series?
P
1 P
1
(a) kz k (b) k2z k
k=1 k=1

Solution
(a) P
Dierentiate the geometric series 1 k
k=0 z , obtain

X
1
1 X
1
1
k
z = ) kz k 1
= :
k=0
1 z k=1
(1 z)2
Multiply by z obtain
X
1
z
kz k = :
k=1
(1 z)2
(b) P
Dierentiate the geometric series 1 k
k=0 z twice, obtain

X
1
1 X
1
1 X
1
2
k k 1
z = ) kz = 2 ) k (k 1) z k 2
= :
k=0
1 z k=1
(1 z) k=2
(1 z)3

Multiply by z 2 obtain
X
1
2z 2
k2 k z2 = :
k=2
(1 z)3
We get
X
1 X
1
2z 2 z 2z 2
2 k
k z = kz k + 3 = z+
k=2 k=2
(1 z) (1 z)2 (1 z)3

31
V.3.6
1 2 3 P L K
P P
Show that the series ak z k , the dierentiated series
P ak k+1 kak z k 1 , and the
integrated series k+1
z all have the same radius of convergence.

Solution p p
Can use Cauchy-Hadamard formula, and k k = 1. Then lim sup k jak j =
p q k!1
ja j
lim sup k k jak j = lim sup k k+1k
, so all series have the same radius of conver-
k!1 k!1
gence. Can also use the characterization of R as the radius of the largest
disk to which the function extends and ?????. This largest disk is clearly the
same for f , f 0 , and ????.

32
V.3.7
1 2 3 P L K

Consider the series


X
1
k
2 + ( 1)k zk :
k=0

Use the Cauchy-Hadamard formula to nd the radius of conver-


gence of the series. What happens when the ratio test is applied?
Evaluate explicitly the sum of the series.

Solution
k
We apply the Cauchy-Hadamard formula where ak = 2 + ( 1)k . First
observe that
r
p k
2 + ( 1)k = 2 + ( 1)k
k k
jak j =

Note that

3 if k is even,
2 + ( 1)k =
1 if k is odd.
Hence
p
k 3 if k is even,
jak j =
1 if k is odd.
Hence, since the lim sup of this sequence is 3 we have by the Cauchy-Hadamard
formula
1 1
R= p = :
lim sup k
jak j 3
Applying the ratio test we have
k
ak 2 + ( 1)k 3k if k is even,
= k+1
= 1
ak+1 if k is odd.
2 + ( 1)k+1 3k+1

33
Since the sequence does not converge, the ratio test is inconclusive, in that
limk!1 jak j = jak+1 j does not exist. However since jak j = jak+1 j 3, it does
converge for jzj < 1=3.
Then we have with jzj < 1=3

X
1
k
2 + ( 1)k zk =
k=0
X
1
2n 2n 2n
X
1
2n+1 2n+1
= 2 + ( 1) z + 2 + ( 1)2n+1 z =
k=0 k=0
X
1 X
1 X
1 X
1
2n 2n 2n+1 2n+1 2 n n
= 3 z + 1 z = 9z +z z2 =
n=0 n=0 n=0 n=0
1 z
= + :
1 9z 2 1 z 2
Note: The series is convergent because j9z 2 j ; jz 2 j < 1 since jzj < 1=3, and
that the series have singularities at z = 1=3 and z = 1.

34
V.3.7 p
Let L = lim sup k jak j. The denition
p of n ln supnpimplies that given " > 0,
9 > 0, 9N 3 for k > N we have k jak j 6 L+", but k jak j > L " for innitely
P
many k 0 s. If k > N , then jak j < (L + ")k . If jzj < 1= (L + "), then ak z k
converges. (L + ") jzj < 1, k z k 6 (L + ")k jzjk = ((L + ") jzj)k converges
by comparison with geometric series. ) R > 1= (L + ") ) jak j > (L ")k .
If jzj > L ", then ak z k > (L ")k (L 1")k for innitely many k 0 s. ), and
P
ak z k diverge. ) R 6 L1 " . Let " ! 0, get R 6 L1 .

35
V.3.8
1 2 3 P L K

Write out a proof of the Cauchy-Hadamard formula (3.4).

Solution

36
V.4.1
1 2 3 P L K
LLL
Find the radius of convergence of the power series for the following
functions, expanded about the indicated point.
(a) z 1 1 about z = i; (d) Log z about z = 1 + 2i;
1
(b) cos z about z = 0; (e) z 3=2 about z = 3;
1 z i
(c) cosh z
about z = 0; (f) z3 z
about z = 2i:

Solution
(d)
Since the values of Log z are unbounded in any neighborhood of the origin it
may not be extended to an analytic function on any open disk containing the
p
origin. But Log z is dened and analytic on the disk of radius j1 +
p2ij = 5
centered at 1 + 2i. Hence, the desired radius of convergence is 5 by the
second Corollary on p. 146.

p p
(a) R = 2 (b) R = =2 (c) R = =2 (d) R = 5 (e) R = 3 (f) R = 2

37
V.4.2
1 2 3 P L K
LLL
Show that the radius
p of convergence of the power series expansion of (z 2 1) = (z 3 1)
about z = 2 is 7.

Solution
z+1
Rewrite f (z) as . Singularities of f (z) are at e2 i=3
, and
(z e2 i=3 )(z+e2 i=3 ) p
distance from 2 to nearest singularity is 7.

38
V.4.3
1 2 3 P L K
LLL
Find the power series expansion of Log z about the point
p z = i 2. Show
that the radius of convergence of the series is R = 5. Explain why this
does not contradict the discontinuity of Log z at z = 2.

Solution

(From Hints and Solutions) p


Log z extends to be analytic for jz (i 2)j < 5, through the extension
does not coincide with Log z in the part of the disk in the lower halfplane.

39
V.4.4
1 2 3 P L K
LLL
Suppose f (z) is analytic at z = 0 and satises f (z) = z + f (z)2 . What is
the radius of convergence of the power series expansion of f (z) about z = 0?

Solution
(From Hints and Solutions) p
Near 0 the function coincides with one of the branches of 1 1 4z =2.
The radius of convergence of the power series of either branch is 1=4, which
is the distance to the singularity at 1=4.

40
V.4.5
1 2 3 P L K

Deduce the identity eiz = cos z + i sin z from the power series expansions.

Solution
P
1
in z n
P P P
1
( 1)m z 2m P
1
( 1)m z 2m+1
eiz = n!
= + = (2m)!
+i (2m+1)!
, where n = 2m
n=0 n even n odd m=0 m=0
in the rst and n = 2m + 1 in the second series.

41
V.4.6
1 2 3 P L K
LLL
Find the power series expansions of cosh z and sinh z about z = 0. What are
the radii of convergence of the series?

Solution
P
1
z 2n z2 z4
(a) cosh z = cos iz = (2n)!
=1+ 2!
+ 4!
+ :::
n=0
(iz)3 (iz)5 z3 z5
(b) sinh z = i sin (iz) = ( i) (iz) 3!
+ 5!
::: = z + 3!
+ 5!
+ :::
P
1
z 2n+1
= (2n+1)!
n=0

42
V.4.7
1 2 3 P L K

Find the power series expansion of the principal branch Tan 1 (z) of
the inverse tangent function about z = 0. What is the radius of con-
vergence of the series? Hint. Find it by integrating its derivative
(a geometric series) term by term.

Solution
We have

d 1 X 1
k
1
Tan z= = z2 ;
dz 1 + z2 k=0

thus
X
1
z 2k+1
Tan 1
z= ( 1)k :
k=0
2k + 1
And we have R = 1.

43
V.4.8
1 2 3 P L K

Expand Log (1 + iz) and Log (1 iz) in power series about z = 0. By


comparing power series expansions (see the preceding exercise),
establish the identity

1 1 1 + iz
Tan z= Log :
2i 1 iz
(See Exercise 5 in section I.8)

Solution

X
1
z k+1 X
1
z k+1
k+1
Log (1 + iz) = ( i) ; Log (1 iz) = ik+1 :
k=0
k+1 k=0
k+1

Thus

2i X 2j z 2j+1 X
1 1
1 1 + iz z 2j+1
Log = i = ( 1)j :
2i 1 iz 2i j=0 2j + 1 j=0
2j + 1

44
V.4.9
1 2 3 P L K

Let a be real, and consider the branch of z a that is real and positive on
(0; 1). Expand z a in a power series about z = 1. What is the radius of
convergence of the series? Write down the series explicitly.

Solution
f (z) = z a = ea Log z , Re z > 0. Assume a 6= 0; 1; 2; : : :. f (m) (z) = a (a 1) : : : (a m + 1) z a m

, f (m) (1) = a (a 1) : : : (a m + 1)
P
1 (m)
f (z) = am (z 1)m , am = f m!(1) = a(a 1):::(a m!
m+1)
= m a
, m > 1, and
m=0
P
1
a0 = a1 = a
0
. f (z) = a
m
(z 1)m . R = 1 since am+1
am
= a m
m+1
! 1 as
m=0
m ! 1. If a is an integer > 0 get R = 1.

45
V.4.10 (From Hints and Solutions)
1 2 3 P L K

Recall that for a complex number , the binomial coe cient " choose n" is
dened by

( 1) ( n + 1)
= 1; and = ; n 1:
0 n n!

Find the radius of convergence of the binomial series


X
1
zn:
n
n=0

Show that the binomial series represents the principal branch of the function
(1 + z) . For which does the binomial series reduce to a polynomial?

Solution
Use f (n) = ( 1) : : : ( n + 1) (1 + z) n and the formula for the coe -
cient of z n . The series reduces to a polynomial for = 0; 1; 2; : : :. Otherwise
radius of convergence is 1, which is distance to the singularity at 1. We
can obtain the radius of convergence for the series also from the ratio test,
an
an+1
= ( (1):::(1):::( n+1)
n+1)(
(n+1)!
n) n!
= n+1n ! 1 as n ! 1.

46
V.4.11
1 2 3 P L K

For xed n 0, dene the function Jn (z) by the power series


X
1
( 1)k z n+2k
Jn (z) = :
k=0
k! (n + k)!2n+2k
Show that Jn (z) is an entire function. Show that w = Jn (z) satises the
dierential equation

1 n2
w00 + w0 + 1 w = 0:
z z2
Remark. This is Bessels dierential equation, and Jn (z) is Bessels function
of order n.

Solution 1
P ( 1)k z n+2k zn
Jn (z) = k!(n+k)!2n+2k
= n!2n
+ O (z n+2 )
k=0
P1
( 1)k (n+2k)z n+2k 1
zn 1
Jn0 (z) = k!(n+k)!2n+2k
= (n 1)!2n
+ O (z n+2 )
k=0
P1
( 1)k (n+2k)(n+2k 1)z n+2k 2
zn 2
Jn00 (z) = k!(n+k)!2n+2k
= (n 2)!2n
+ O (z n )
k=0
( 1)k (n+2k)(n+2k 1)
Term multiplying z n+2k in z 2 Jn00 + zJn0 + (z 2 n2 ) Jn is k!(n+k)!2n+2k
+
( 1)k (n+2k) ( 1)k n2
k!(n+k)!2n+2k k!(n+k)!2n+2k
( 1)k
(k 1)!(n+k 1)!2n+2k 2
=
( 1)k
k!(n+k)!2n+2k
(n + 2k) (n + 2k 1) + (n + 2k) n2 4k (n + k)
=0
Should cheek separately to the sum terms of J0 + J1 , when we divide by 0
above, ???? work out. (constant term ????? J0 , z term for J1 ). It works,
because the case k = 0 we replace 1= (k 1)! by k=k! = 0. Ratio test gives
radius of convergence = 1. (Its not clear).

47
V.4.12
1 2 3 P L K
LLL
Suppose
P that the analytic function f (z) has power series expansion
n
an z . Show that if f (z) is an even function, then an = 0 for n
odd. Show that if f (z) is an odd function, then an = 0 for n even.

Solution
If f (z) is analytic
P1 in Dr n(0) then f ( z) is also analytic in the same region
n
and
P1 f ( z) = 2n+1n=0 ( 1) an z . If f is even then f (z) f ( z) = 0, and so
n=0 2a2n+1 z = 0. A power series is 0 i all of its contents are 0 (since
(n)
an = f (0) =n! which shows an = 0 for n odd. The result for f (z) odd is
analogous.

48
f (z) even, f ( z) = f (z) . Chain rule: f 0 ( z) = f 0 (z), f 00 ( z) = f 00 (z),
etc. f (k) ( z) = ( 1)k f (k) (z). f (k) (0) = ( 1)k f (k) (0) ) f (k) (0) = 0 for k
odd. ) ak = f (k) (0) =k! for k odd. If f (z) is odd, f ( z) = f (z), same
argument gives ak = 0 for k even. Alternatively, apply ever result to zf (z)
which is even.

49
V.4.13
1 2 3 P L K

Prove the following version of LHospitalss rule. If f (z) and g (z) are ana-
lytic, f (z0 ) = g (z0 ) = 0, and g (z) is not identically zero, then

f (z) f 0 (z)
lim = lim 0 ;
z!z0 g (z) z!z0 g (z)

in the sense that either both limits are nite and equal, or both limits are
innite.

Solution
P
1
Assume z0 = 0 , f (z) = ak z k = f 0 (0) z +O (z 2 ), g (z) = g 0 (0) z +O (z 2 ) =
k=1 8
0 (0)
< 1 if g (0) = 0 and f (0) 6= 0 or k < N
aN
g 0 (0) z+aN z n +O z N +1 . fg(z) (z)
! fg0 (0) if g 0 (0) 6= 0 ! if g (0) = 0 , f (0) 6= 0, k = N
: bN
0 if g (0) = 0 , f (0) 6= 0, k > N
0
f (z) f ( ) f (z)
0 (0)+ka z k 1 +O z k 0 0
! fg0 (0)
(0)
k
g 0 (z)
= 0 N 1 N , 0 if g 0 (0) 6= 0
8 g (0)+N bN z +O(z ) g (z)
< 1 g (0) = 0 and either f (0) 6= 0 or k < N
N aN aN
! = g (0) = 0 , f (0) 6= 0, k = N
: N bN bN
0 g (0) = 0 , f (0) 6= 0, k > N
Then the limits are equal in all cases.

50
V.4.14
1 2 3 P L K

Let f be continuous function on the unit circle T = fjzj = 1g. Show that
f can be approximated uniformly on T by a sequence of polynomials in z
if and only if f has an extension F that is continuous on the closed disk
fjzj 1g and analytic on the interior fjzj < 1g. Hint. To approximate such
an F , consider dilates Fr (z) = F (rz).

Solution
If f has an analytic extension F , then Fr (z) = F (rz) has power series
P P
1
F (z) = an z k , jzj < 1. Fr (z) = an rn z k , jzj < 1r . Fr converge uniformly
n=j
for jzj 6 1, to F (rz) and F (rz) ! f (z) uniformly for jzj = 1 as v " 1. For
" > 0, take r < 1 with jFr (z) F (z)j < " for jzj = 1, then take N such that
PN P
N
an rn z n Fv (z) 6 2" for jzj = 1. Then an rn z n F (z) 6 2" + 2" = "
n=0 n=0
for jzj = 1. Conversely if pn are polynomials, pn ! f uniformly for jzj = 1,
then jpn pm j ! 0 uniformly for jzj = 1 as n; m ! 1, so by the maximum
principle, jpn pm j ! 0 uniformly for jzj 6 1. ) fpn g converges uniformly
to same function F (z) for jzj 6 1 . Since pn (z) ! f (z) for jzj = 1,
F (z) = f (z) for jzj = 1. Since pn is analytic for jzj < 1, also F (z) is
analytic for jzj < 1.

51
V.5.1
1 2 3 P L K
LLL
Expand the following functions in power series about 1
2 2
(a) z21+1 (b) z3z 1 (c) e1=z (d) z sinh (1=z)

Solution
(b)
Denote the above function by f and dene g by

(1=w)2 w
g (w) = f (1=w) = 3 = for w 6= 0
(1=w) 1 1 w3
and g (0) = 0. Note that g is analytic on the disk D1 (0) with power series
representation

w X1
n
X
1
g (w) = = w w3 = w3n+1
1 w3 k=0 k=0
3
for all w 2 D1 (0) (note that jw j 1 if jwj 1). Hence, we have
X
1
3n+1
X
1
1
f (z) = g (1=z) = (1=z) =
k=0 k=0
z 3n+1
for all z with jzj > 1.
P
1 n
(a) 1
z 2 +1
=1 z2 + z4 ::: = ( 1)n z 2n z12 1+1=z
1
2 =
1 ( 1)
z 2 z 2n
=
n=0
P
1
( 1)n
z 2n+2
n=0
z2 1 1 1
P
1
1
P
1
1
(b) z3 1
= z 1 1=z 3
= z z 3n
= z 3n+1
n=0 n=0
1=z 2
P
1
1 1
(c) e = n! z 2n
n=0
1 1 1 1
P
1
1 1
(d) z sinh z
=z z
+ 3!z 3
+ 5!z 5
+ ::: = (2n+1)! z 2n
n=0

52
V.5.2
1 2 3 P L K

Suppose f (z) is analytic at 1, with series expansion (5.1). With the notation
f (1) = b0 and f 0 (1) = b1 , show that

f 0 (1) = lim z jf (z) f (1)j :


z!1

Solution 1
P 0 P
1 P
1
bk+1
f (z) = bk
zk
= f (1)+ f (1)
z
+ bk
zk
. z [f (z) f (1)] = f 0 (1)+ zk
!
k=0 k=2 k=1
P
1
f 0 (1) as z ! 1, since bk+1 wk ! 0 as w ! 0.
k=1

53
V.5.3
1 2 3 P L K

Suppose f (z) is analytic at 1, with series expansion (5.1). Let


0 be the smallest number such that f (z) extends to be analytic
for jzj > . Show that the series (5.1) converges absolutely for
jzj > and diverges for jzj < .

Solution (A. Kumjian)


Recall that f is analytic at 1 i there is a function at 0 such that f (z) =
g (w) with w = 1=z wherever f (z) makes sense. We may suppose that f (z)
is dened for jzj > and that f is analytic at these points. Then setting
R = 1= if > 0 and R = 1 if = 0, we have that g is analytic on
DR (z0 ) and cannot be extended to an analytic function on any larger open
disk centered at z0 . Hence, g has a power series representation
X
1
g (w) = bk w k for jwj < R;
k=0

with radius of convergence R. Thus, the series converges absolutely for jwj <
R and diverges for jwj > R. It follows that by substituting w = 1=z, we have

X1
bk
f (z) = for jzj > ;
k=0
zk

where the series converges absolutely for jzj > and diverges for jzj < .

54
V.5.4 (From Hints and Solutions)
1 2 3 P L K

Let E be a bounded subset of the complex plane C over which area integrals
can be dened, and set
ZZ
dx dy
f (w) = ; w 2 CnE;
E w z
where z = x + iy. Show that f (w) is analytic at 1, and nd a formula for
the coe cients of the power series of f (w) at 1 in descending powers of w.
Hint. Use a geometric series expansion.

Solution P n n+1
If jzj 6 M for z 2 E, and R > M , then 1= (w z) = z =w converges
uniformly for z 2 E and jwj > R . Integrate term by term, obtain f (w) =
P
1
bn
RR n
w n+1 , jwj > R, where b n = z dxdy.
N =0 E

55
V.5.5 (From Hints and Solutions)
1 2 3 P L K

Determine explicitly the function f (w) dened in Exercise 4, in the case that
E = fjwj 1g is the unit disk. Hint. There are two formulae for f (w), one
valid for jwj 1 and the other for jwj 1. Be sure thy agree for jwj = 1.

Solution
To nd the formula for jwj < 1 , break the integral into two pieces corre-
sponding toZ Zjzj > jwj and Z Z to jzj < jwj, and use geometric series. f (w) =
RR dxdy dxdy dxdy RR n 1
w z
= + = (1)+(2). If jwj < 1, get z dxdy =
D w z w z
jzj<jwj jzj>jwj
| {z } | {z }
(1) (2)
RR n i(n 1)
RR 1
r drd e = 0 , f (w) = w z
dxdy = w , jwj > 1.
RR P RR zn
RR
If jwj < 1, get (1) w1 dxdy
1 z=w
= w
1
w n dxdy = 1
w
dxdy = w1 jwj2 =
jzj<jwj jzj6jwj jzj6jwj
w
RR dxdy
RR 1 dxdy
P
1 RR dxdy
(2) w z
= z 1 w=z
= wn z n+1
=0
1>jzj>jwj n=0
=w; jwj > 1
f (w) =
w; jwj < 1

56
V.6.1
1 2 3 P L K
LLL
Calculate the terms through order seven of the power series expansion about
z = 0 of the function 1= cos z.

Solution
z2 z4 z6
1
cos z
=1+ 2! 4! 6!
+ O (z 8 ) + (: : :)2 + (: : :)3 + O (z)
2
1 + z2! + a4 z + a6 z 6 + O (z 8 )
4
2 3
z 4 : 4!1 + 2!1 = 14 24 1
= 24 5
, z6 : 2
2!4!
+ 2!1 = 18 1
24
= 1
12
sin z z3 z5 z7 z2 5 4 1 6
cos z
= z 3!
+ 5! 7!
+ ::: 1+ 2
+ 24 z 12
z + :::
z:1
z 3 : 21 1
3!
= 1
2
1
6
= 31
z 5 : 5!1 1
2
1
3!
+ 245 1
= 120 1
12
5
+ 24 16
= 120 4
= 30 = 2
15
1 5 2
z 7 : 12 24 3!
+ 215! 7!1 = 7 6 5 2 57! 7+3 7 1 =
420 175+21 1
7!
= 265
7!
= 7 653
432

57
V.6.2
1 2 3 P L K
LLL
Calculate the terms through order ve of the power series expansion about
z = 0 of the function z= sin z.

Solution
z z 1
sin z
=z z 3 =3!+z 5 =5! :::
= 1 z 2 =3!+z 4 =5! :::
z2 z4 6
=1+ 3! 5!
+ z7! : : : + (: : :) + (: : :)3 + O (z 2 ) =
2

1 + z 2 =6 + a4 z 4 + a6 z 6 + O (z 7 )
a4 = 5!1 + (3!) 1
2 = 36
1 1
120
= 103603 = 360
7

6(1 14)+4 5 7
a6 = 7!1 = 3!5! 2
+ (3!) 1
3 = 3!7!
= 1403!7!78 = 3!7!
62
= 32
z 2 7z 4
sin z
= 1 + z6 + 360 + O (z 6 )

58
V.6.3
1 2 3 P L K
LLL
Show that
ez 1 1 3 3 4 11 5
= 1 + z2 z + z z +
1+z 2 3 8 30
Show that the general term of the power series is given by

n 1 1 ( 1)n
an = ( 1) + + ; n 2:
2! 3! n!
What is the radius of convergence of the series?

Solution
We have

1 X 1 X
1
zk
= ( 1)k z k for all jzj < 1 and for all z 2 C:
1+z k=0 k=0
k!

Since the given function can be expressed as product of these two we have

1 z X
1
ez
= e = cn z n for all jzj < 1
1+z 1+z n=0

where

( 1)n ( 1)n 1 ( 1)n 2 ( 1)0


an = + + + +
0! 1! 2! n!
by formula (6.1). Hence, c0 = 1, c1 = 0 and for n 2 we have, since
0! = 1! = 1,

( 1)n 2
( 1)0 1 1 ( 1)n
an = ( 1)n 1
( 1 + 1)+ + + = ( 1)n + + :
2! n! 2! 3! n!

The radius of convergence is 1.

59
Solve ez = (1 + z) = a0 + a1 z + a2 z 2 + : : : by multiplying
9 by (1 + z) and com-
a0 = 1 >
>
= Recursively
z a 0 + a 1 = 1
paring with the coe cients of e . ) a0 = 1
a1 + a2 = 1=2 >
>
; an = 1=n! a n
an 1 + an = 1=n!
So an = n!1 1
(n 1)!
+ (n 1 2)! : : : + ( 1)n 0!1 (the two last terms cancel.) The
radius of convergence is 1 because the function has a pole at 1 . (In fact,
( 1)n an ! e 1 when n ! 1 .)

60
V.6.4
1 2 3 P L K

Dene the Bernoulli numbers Bn by

z z2 z4 z6
cot (z=2) = 1 B1 B2 B3 :
2 2! 4! 6!
Explain why there are no odd terms in this series. What is the radius of
convergence of the series? Find the rst three Bernoulli numbers.

Solution
z 2 4 6
2
cot (z=2) = 1 B1 z2! B2 z4! B3 z6! : : :,
cot (z=2) = cos(z=2)
sin(z=2)
is odd, so z2 cot (z=2) is even, so only even terms appear in
the series. The function has singularities at z=2 = , i.e., at 2 . Distance
from 0 to nearest singularity = 2 = radius of convergence.
w2 4 w6
1 + w4! +:::
Find the two Bernoullinumbers B1 +B2 . w cot w = = w cos w
sin w
2!
w2 4
6!
w6
=
1 3!
+ w5! 7!
+:::
h 2 4
i 2
2
w4 w2
1 w2! + w4! + O (w6 ) 1 + w6 120
+ 6
+ O (w6 )
w2 w4 w2 1 1
1 2
+ 24
1+ 6
+ 36 120
w4 + O (w6 ) =
1 1 7 1 1
1+ 6 2
w2 + 360 12
+ 24 w4 + O (w6 ) =
1 2 1 4 6
1 3
w 45
w + O (w )
2 4 4
cot w = 1 B1 w2! B2 2 4!w :::
) 2! = 3 , B1 = 6 , 4! B2 = 45
22 B 1 1 1 24 1
, B2 = 244!45 , B2 = 30
1

For B3 , must keep all powers ????? to and ????? w8 . Better to just ?????
for B1 and B2 .

61
V.6.5
1 2 3 P L K

Dene the Euler numbers En by

1 X En 1
= zn:
cosh z n=0
n!
What is the radius of convergence of the series? Show that En = 0 for n odd.
Find the rst four nonzero Euler numbers.

Solution 1
1
P En n 3
cosh z
= n!
z , cosh z has zeros at 2
i; 2
i; : : :. Distance from 0 to
n=0
nearest singularity is =2 = R . Since cosh z is even, En = 0 for n odd.
1 1 z2 z4 z6
cosh z
= (1+z2 =2!+z 4 =4!+:::) = 1 + E2 2! + E4 4! + E6 6! =
2
z2 z4 z2 z4 z6
1 2!
+ 4!
+ ::: + 2!
+ 4!
+ 6!
::: (: : :)3 , E0 = 1
E2 1 E4 2
2!
= 2!
, E2 = 1, 4!
= 4!1 + 2!1 = 1
4
1
24
= 5
24
, E4 = 5
E6 1 1 1
6!
= 6!
+ 2 2!4! (2!)3
, E6 = 61
6!
E6 = 1+6 5 33
= 29 6 5 3 = 29 90 = 61

62
V.6.6
1 2 3 P L K

Show that the coe cients of a power series "depend continuously" on the
function they represent, in the following sense. If ffm (z)g is a sequence of
analytic functions that converges uniformly to f (z) for jzj > , and
X
1 X
m
k
fm (z) = ak;m z ; f (z) = ak z k ;
k=0 k=0

then for each k 0, we have ak;m ! ak as m ! 1.

63
V.7.1
1 2 3 P L K
LLL
Find the zeros and orders of zeros of the following functions
z 2 +1
(a) z2 1
(d) cos z 1 (g) ez 1
(b) 1
z
+ z5 (e) cos zz 1
1
(h) sinh2 z + cosh2 z
(c) z 2 sin z (f) coszz2 1 (i) Log
z
z
(principal value)

Solution
(c)
Let f (z) = z 2 sin z. Then f has zeros at all integer multiples of i.e. z0 = k
for k 2 Z. Since sin z has a zero of order 1 at 0, so sin z = zh (z) for some
analytic function h with h (0) 6= 0. Thus, f (z) = z 3 h (z) and so f has a
zero of order three at 0. Note that f 0 (z) = 2z sin z + z 2 cos z. Hence, for any
other zero z0 = k where k 2 Zn f0g, we have

f 0 (k ) = 2k sin k + (k )2 cos k = ( 1)k k 6= 0:


Thus, f has a simple zero at z0 = k when k 6= 0.
(d)
Note that g (z) = cos z 1 = 0 i z = 2k for k 2 Z. We will show
that all the zeros are double zeros. First observe that g 0 (z) = sin z and
so g 0 (2k ) = sin 2k = 0 for all k 2 Z. But g 00 (z) = cos z and so
g 0 (2k ) = cos 2k = 1 for all k 2 Z. Hence, all zeros are double zeros.

64
(a) simple zeros at i, (b) simple zeros at e i=4 , e3 i=4 , (c) triple zero at 0,
simple zeros at n , n = 1; 2; : : :, (d) double zeros at n , n = 1; 2 : : :,
(h) simple zeros at i=4 + n i=2, n = 1; 2; : : :, (i) no zeros.
1
(b) z + z5 = (z + 1) z15 = (z z1 ) (z z2 ) (z z3 ) (z z4 ) z15 , where zj =
1 4

ei( =4+j=4 2 ) , these four points are simple zeros.


y y
(d) cos z = cos (x + iy) = cos x cos (iy) sin x sin (iy), cos (iy) = e 2+e =
e y ey ey e y
cosh y, sin (iy) = 2i
=i 2
= i sinh y, cos z = 1 ) sin x sinh y =
0 ) y = 0 or x = n . If x = n , then cos (n ) cosh (n ) = ( 1)n cosh (n ).
This is = 1 only if n = 0, since cos (n ) > 1 for n 6= 0. Get x = 0, y = 0.
If y = 0, then cos x = 1 only at x = 2n , n = 0; 1; 2; : : :. Solutions
d
are z = 0; 2 ; 4 ; : : :. Since dz (cos z 1) = sin z = 0 at these points,
d2
dz 2
(cos z 1) = cos z 6= 0 , at these points. The zeros are double zeros.
(e) cos zz 1 , has only zeros at z = 2n , n = 0; 1; 2; : : : . Simple poles at
z = 0 . At the points dz d
= cos zz 1 = z( sin z) z2(cos(z 1)) = cos(2n ) 1
(2n )2
for n 6= 0.
Other zeros are double. Double zeros at z = 2n , n = 0; 1; 2; : : :.
2
(g) ez 1 = z + z2! + : : :, thus a simple zero at 0. Since ez 1 is periodic, we
have simple zeros at i2 k, k 2 Z .
(i) Log z = 0 when jzj = 1 and Arg z = 0, thus at z = 1. We write
2 3
Log z = Log (1 (1 z)) = 1 z (1 2z) + (1 3z) + : : :, so we have a simple
zero at 1.

65
V.7.2 (From Hints and Solutions)
1 2 3 P L K

Determine which of the functions in the preceding exercise are analytic at


1, and determine the orders of any zeros at 1.

Solution
(a) analytic at 1 , (b) analytic at 1, simple zero, (c) (i) not analytic at
1.

66
V.7.3
1 2 3 P L K
LLL
Show that the zeros of sin z and tan z are all simple.

Solution
d
Zeros of sin z are at n , 1 < n < 1, since cos (n ) = dz
sin zjz=n =
1 6= 0, the zeros are simple.

67
V.7.4
1 2 3 P L K
LLL
Show that cos (z + w) = cos z cos w sin z sin w, assuming the corresponding
identity for z and w real.

Solution
cos (z + w) = cos z cos w + sin z sin w = F (z; w) is entire in z for each xed
w and entire in w for each xed z. F (z; w) = 0 for z; w real. Apply theorem,
with D = C, E = R. Get F (z; w) = 0, so cos (z + w) = cos z cos w
sin z sin w.

68
V.7.5
1 2 3 P L K

Show that
Z 1
r
zt2 +2wt 2 =z
e dt = ew ; z; w 2 C; Re z > 0;
1 z
where we take the principal branch of the square root. Compare the result
to Exercise IV.3.1. Hint. Show that the integral is analytic in z and w, and
evaluate it for z = x >p 0 and w real by making a change of variable and
using the known value for z = 1 and w = 0.

Solution
R1 at2 2bt
e dt, note the integral is improper, so a limit process may be used
1
R1 2 R1 s2 2b
p s
for analytic. Re a < 0. x > 0, eat 2bt
dt = p1 e x ds. F ( ) =
x
1 1
Z1
R1 s2
R1 2 2 (s =2)
e e s ds = e (s =2)
e =4
ds = e =4
e ds
1 1
1
| {z }
R1 s2
p 2 R1 xt2
p b2 =x
e e s ds = e =4
, e e 2bt
dt = p1 e , x real
x
1 1
R1 zt2 zbt
p 2 =z
e e dt = z
eb
1

69
V.7.6
1 2 3 P L K
LLL
Suppose f (z) is analytic on a domain D and z0 2 D. Show that if
f (m) (z0 ) = 0 for m 1, then f (z) is constant on D.

Solution
Suppose that f (m) (z0 ) = 0 for m and dene g on D by g (z) = f (z) f (z0 ).
Then it su ces to show that g is identically zero. Suppose not, then since
g (m) (z0 ) for m 0, the power series expansion for g is trivial and therefore g
is identically zero on a disk on nonzero radius centered at z0 but this violates
the rst theorem on page 156 that asserts that an analytic function which is
not identically zero only has isolated zeros. This result may also be proved
using the Uniqueness Principle (the second theorem on page 156).

70
V.7.7
1 2 3 P L K
LLL
Show that if u (x; y) is a harmonic function on a domain D such that all the
partial derivatives of u (x; y) vanish at the same point of D, then u (x; y) is
constant on D.

Solution

71
V.7.8
1 2 3 P L K

With the convention that the function that is identically zero has a zero of
innite order at each point, show that if f (z) and g (z) have zeros of order n
and m respectively at z0 , then f (z) + g (z) has a zero of order k min (n; m).
Show that strict inequality can occur here, but that equality holds whenever
m 6= n.

Solution

72
V.7.9
1 2 3 P L K
LLL
Show that the analytic function f (z) has a zero of order N at z0 , then
f (z) = g (z)N for some function g (z) analytic near z0 and satisfying g 0 (z) 6=
0.

Solution
(From Hints and Solutions)
Write f (z) = (z z0 )N h (z), where h (z) has a convergent power series and
h (z0 ) 6= 0. Take g (z) = (z z0 ) e(log(h(z)))=N for an appropriate branch of
the logarithm.

73
V.7.10
1 2 3 P L K

Show that if f (z) is a continuous function on a domain D such that f (z)N


is analytic on D for some integer N , then f (z) is analytic on D.

Solution
f (z)N analytic. Zeros of f (z) are isolated, for this need maximum principle.
f (z) is analytic, except possibly at the isolated points where f (z) = 0. (By
the Riemanns theorem, f (z) is analytic ????? there but dont have this yet)
Write f (z)N = (z z0 )m h (z), h (z) analytic, h (z) 6= 0. Then h (z) has N th
root near z0 , by preceding exercise. f (z)N = g (z)N (z z0 )m , (f (z) =g (z))N =
(z z0 )m . (z z0 )m=N is continuous in a neighborhood of z0 . f (z) =g (z) re-
turns to original value, doing circle around z z0 . m is an integral multiple
of N .

74
V.7.11
1 2 3 P L K

Show that if f (z) is a nonconstant analytic function on a domain D, then


the image under f (z) of any open set is open. Remark. This is the open
mapping theorem for analytic functions. The proof is easy when f 0 (z) 6= 0,
since the Jacobian of f (z) coincides with jf 0 (z)j2 . Use Exercise 9 to deal
with the points where f 0 (z) is zero.

Solution
If f 0 (z0 ) 6= 0, then f maps open disks centred at z0 onto open sets, so f (D)
contains a disk centred at f (z0 ).
If f 0 (z0 ) = 0, assume f (z0 ) = 0, write f (z) = (z z0 )N h (z), h (z0 ) 6= 0.
Then f has a N th root near z0 , f (z) = g (z)N , g 0 (z0 ) 6= 0. g covers a ?????
disk, so f cover disk centred at 0.

75
V.7.12
1 2 3 P L K

Show that the open mapping theorem for analytic functions implies the max-
imum principle for analytic functions.

Solutions
Clearly open mapping theorem ) strict maximum principle for analytic func-
tions, since a non constant analytic function cant attain maximum at z0 and
cover a disk centred at z0 . Then strict maximum principle ) maximum
principle.

76
V.7.13 (From Hints and Solutions)
1 2 3 P L K

Let fn (z) be a sequence of analytic functions on a domain D such that


fn (D) D, and suppose that fn (z) converges normally to f (z) on D.
Show that either f (D) D, or else f (D) consists of a single point on @D.

Solution
f (D) D [ @D. If f (z) is not constant, then f (D) is open, and f (D)
cannot contain any point of @D.

77
V.7.14
1 2 3 P L K

A set E is discrete if every point of E is isolated. Show that a closed discrete


subset of a domain D either is nite or can be arranged in a sequence fzk g
that accumulates only on f1g [ @D.

Solution
Let Kn = fz 2 D : d (z; @D) > 1=n; jzj 6 ng. En is compact, only nitely
many points of E belongs to Kn ????? points, shading with those in E \ K1 ,
the E \ (K2 nK1 ) ,?????

78
V8.1
1 2 3 P L K
p
Suppose that the principal branch of z 2 1 is continued analytically from
z = 2 around the gure eight path indicated above. What is the analytic con-
tinuation of the function at the end of the path? Answer the same question
1=3 1=3
form the functions (z 3 1) and (z 6 1) .

Solution
p
p z 2 1, phase change i at +1, 1 at 1, i at +1, returns to initial value.
3
z3 p 1, phase change ei =3 at +1, ei =3 at +1, returns to e2 i=3 times initial
value. 3 z 6 1, phase change ei =3 at +1, e 2 i=3 at 1, ei =3 at +1, returns
to initial value.

79
V8.2
1 2 3 P L K

Show that f (z) = Log z = (z 1) 12 (z 1)2 + has an analytic contin-


it
uation around the unit circle (t) = e , 0 t 2 . Determine explicitly
the power series ft for each t. How is f2 related to f0 ?

Solution P m 1
df
dz
= z1 = , f (m) (z) = ( 1)
zm
(m 1)!
P
1 f (m) eit
( ) m P
1 itm m
Series is m!
(z eit
) = f (eit
) + ( 1)m 1 e
m
(z eit )
m=0 m=1
P
1 itm
it m
ft (z) = it + ( 1)m 1 e m (z e ) , get f2 (z) = f0 (z) + 2 i .
m=1

80
V.8.3
1 2 3 P L K
p
Show that each branch of z can be continued analytically along any path
in Cn f0g, and show that the radius of convergencepof the power series ft (z)
representing the continuation is j (t)j. Show that z cannot be analytically
along path containing 0.

Solution

81
V.8.4
1 2 3 P L K
P
Let f (z) be analytic on a domain D, x z0 2 D, and let f (z) = an (z z0 )n
be the expansion of f (z) about z0 . Let
Z z X1
an
F (z) = f ( )d = (z z0 )n+1
z0 n=0
n + 1
be the indenite integral of f (z) for z near z0 . Show that F (z) can be
continued analytically along any path in D starting at z0 . What happens in
the case D = Cn f0g, z0 = 1, and f (z) = 1=z? What happens in the case
that D is star-shaped?

Solution

82
V.8.5
1 2 3 P L K

Show that the function dened by


X n
f (z) = z2 = z + z2 + z4 + z8 +
is analytic on the open unit disk fjzj < 1g, and that it cannot be extended
analytically to any larger open set. Hint. Observe that f (z) = z +f (z 2 ),
and that f (r) ! +1 as r ! 1.

Solution

83
V.8.6
1 2 3 P L K
P
Suppose f (z) = an z n , where an = 0 except for n in a sequence nk that
satises nk+1 =nk 1 + for some > 0. Suppose further that the series has
radius of convergence R = 1. Show that f (z) does not extend analytically
to any point of the unit circle. Remark. Such a sequence with large gaps
between successive nonzero terms is called a lacunary sequence. This result
is the Hadamard gap theorem. There is a slick proof. If f (z) extends ana-
lytically across z = 1, consider g (w) = f (wm (1 + w) =2), where m is a large
integer.
Show that the power series for g (w) has radius of convergence r > 1, and
that this implies that the power series of f (z) converges for jz 1j < ".

Solution

84
V.8.7
1 2 3 P L K
P
Suppose f (z) = an z n , where the series has radius of convergence R < 1.
Show that there is an angle such that f (z) does not have an analytic
continuation along the path (t) = tei , 0 t R. Determine the radius of
convergence of the power series expansion of f (z) about tei .

Solution

85
V.8.8
1 2 3 P L K

Let f (z) be analytic at z0 , and let (t), a t b, be a path such that


(a) = z0 . If f (z) cannot be continued analytically along , show that there
is a parameter value t1 such that there is an analytic continuation ft (z) for
a t < t1 , and the radius of convergence of the power series ft (z) tends to
0 as t ! t1 .

Solution

86
V.8.9
1 2 3 P L K

Let P (z; w) be a polynomial in z and w, of degree n in w. Suppose that


f (z) is analytic at z0 and satises P (z; f (z)) = 0. Show that if ft (z)
is any analytic continuation of f (z) along any path starting at z0 , then
P (z; ft (z)) = 0 for all t. Remark. An analytic function f (z) satises a
polynomial equation P (z; p f (z)) = 0 is called an algebraic function. For
instance, the branches of z are algebraic functions, since the satisfy z wn =
n

0.

Solution

87
V.8.10
1 2 3 P L K

Let D be the punctured disk f0 < jzj < "g, suppose f (z) is analytic at z0 2
D, and ew0 = z0 . Show that f (z) has an analytic continuation along any
path in D starting at z0 if and only if there is an analytic function g (w) in the
half-plane fRe w < log "g such that f (ew ) = g (w) for w near w0 . Remark.
If f (z) does not extend analytically to D but has an analytic continuation
along any path in D, we say that f (z) has a branch point at z = 0. For the
proof, use the fact that any path in D starting at z0 is the composition of
a unique path in the half-plane starting at w0 and the exponential function
ew .

Solution

88
VI 1 2 3 4 5 6 7 8 9 10 11 12 13 14 15 16 17 18 19
1
2
3
4
5
6
7
8

1
VI.1.1
1 2 3 P L K
111 LLL
Find all possible Laurent expansions centered at 0 of the following
functions.
(a) z21 z (b) zz+11 (c) (z2 1)(z
1
2 4)

Solution
(a)
In the region 0 < jzj < 1, we have

1X k X X
1 1 1
1 1 1 k 1
= = z = z = [k 1 = n] = zn:
z2 z z1 z z k=0 k=0 n= 1

In the regionjzj > 1, we have

1 X 1 X X
1 k 1 2
1 1 1 1 k 2
= 2 1 = 2 = 2 z =[ k 2 = n] = zn:
z2 z z 1 z
z k=0 z z k=0 n= 1

(b)
In the region jzj < 1 we have

z 1 2 2 X 1 X
1
=1 =1 =1 2 ( 1)n z n = 1 2 ( 1)n z n :
z+1 z+1 1 ( z) n=0 n=1

In the region jzj > 1 we have

2 X ( 1)k
1
z 1 2 2 1
=1 =1 1
=1 =
z+1 z+1 z1 z
z k=0 z k
X1
( 1)k X1
( 1)n 1 X1
( 1)n
=1 2 k+1
= [k + 1 = n] = 1 2 =1+2 :
k=0
z n=1
zn n=1
z n

(c)
From computation, we have

2
1 1 1 1 1
= +
(z 2 1) (z 2 4) 3z 2 1 3 z2 4
In the region jzj < 1, we have

1 1 1 1 1 1 1 1 1
= + = =
(z 2 1) (z 2 4) 3z 2 1 3z 2 4 31 z 2 12 1 z42
1 X 2n 1 X z 2n 1 X
1 1 1
= z = 4 4 n z 2n :
3 n=0 12 n=0 4n 12 n=0

In the region 1 < jzj < 2, we have

1 1 1 1 1 1 1 1 1
= + 2 = 1 z2
=
(z 2 1) (z 2 4) 3z 2 1 3z 4 3z 1
2
z2
34 1 4

1 X 1 1 X z 2n 1 X 2k 1 X z 2n
1 1 1 1
= = z
3z 2 k=0 z 2n 12 n=0 4n 3 k= 1 12 n=0 4n

In the region jzj > 2, we have

1 1 1 1 1
= + =
(z 2 1) (z 2 4) 3 z2 1 3 z2 4
1 1 1 1
= + =
3 z 2 1 z12 3 z 2 1 z42
1 X 1 1 X 4n
1 1
= + =
3z 2 n=0 z 2n 3z 2 n=0 z 2n
1X 1 1 X 4n 1X n
1 1 1
2(n+1)
= + = (4 1) z = [n + 1 = k] =
3 n=0 z 2(n+1) 3 n=0 z 2(n+1) 3 n=0

1 X
1
1 k
= 4 1 z 2k :
3 k= 1

3
VI.1.2
1 2 3 P L K
111 LLL
For each of the functions in Exercise 1, nd the Laurent expansion centered
at z = 1 that converges at z = 12 . Determine the largest open set on which
each series converges.

Solution
(a)
Partial fractions give us
1 1 1
= +
z2 z z z 1
Laurent series for 1=z

1 X
1
1 1 1 1 1
= = 1 = =
z z+1 1 z + 1 1 z+1 z + 1 k=0 (z + 1)k
X
1
1 X1
= k+1
= [k + 1 = n] = (z + 1)n
k=0
(z + 1) n= 1

Laurent series for 1=z 1

1 X (z + 1)n 1 X
1 1
1 1 1 1
= = = = (z + 1)n
z 1 z+1 2 2 1 z+1
2
2 n=0 2n 2n+1 n=0

Thus we have

1 X
1
1 for n 1
= an (z + 1)n ; where an = 1
z2 z 2n+1
for n 0
k= 1

The function 1= (z 2 z) converges on the set 1 < jz + 1j < 2.


(b)
We have
z 1 2
=1
z+1 z+1
The function (z 1) = (z + 1) converges on the set 0 < jz + 1j < 1.

4
(c)
Partial fractions give us

1 1 1 1 1 1 1 1 1
= + + =
(z 2 1) (z 2 4) 6 z 1 6 z + 1 12 z 2 12 z + 2
1 1 1 1 1 1 1 1
= + + =
6 z + 1 2 6 z + 1 12 z + 1 3 12 z + 1 + 1
1 1 1 1 1 1 1 1
= z+1 + z+1 1 =
12 1 2
6 z + 1 36 1 3
12 (z + 1) 1 + z+1
1 X (z + 1)n 1 1 1 X (z + 1)n 1 X ( 1)k
1 1 1
= + =
12 n=0 2n 6 z + 1 36 n=0 3n 12 k=0 (z + 1)k+1

1 X (z + 1)n 1 1 1 X (z + 1)n 1 X
1 1 n= 1
= + + ( 1)n (z + 1)n =
12 n=0 2n 6z +1 36 n=0 3n 12 1
1 X 1 X
1 1
1 1 1 1
( 1)n (z + 1)n + (z + 1) 1 + (z + 1)n ;
12 n=0 6 12 12 n=0 12 2n 36 3n

The function 1= ((z 2 1) (z 2 4)) converges on the set 1 < jz + 1j < 2.

5
VI.1.3
1 2 3 P L K
LLL
Recall the powe series for the Bessel function Jn (z), n 0, given in Exercise
V.4.11, and dene J n (z) = ( 1)n Jn (z). For xed w 2 C, establish the
Laurent series expansion
hw i X1
exp (z 1=z) = Jn (w) z n ; 0 < jzj < 1:
2 1

From the coe cient formula (1.4), deduce that


Z 2
1
Jn (z) = ei(z sin n ) ; z 2 C:
2 0
Remark. This Laurent expansion is called Schlmlich formula.

Solution
(a) f (z) = tan z, 3 < jzj < 4
sin z
f (z) = tan z = cos z
, singularities at 2
+ n ; n = 0; 1; 2; : : :
Simple poles at 2 + n
lim (z sin z
=2) cos z
= sin 2 lim cos zz =2
cos =2
= sin =2
sin =2
= 1
z! =2 z! =2
sin z z+ =2 sin( =2)
lim (z + =2) cos z
= sin 2
lim cos z cos( =2)
= sin( =2)
= 1
z! =2 z! =2
) Principal parts at 2 are z
1
=2
.
) f0 (z) = f (z) + z 1 =2 + z+1 =2 is analytic for jzj < 32
f1 (z) = z 1=2 z+1 =2 is analytic for jzj > 2 ; ! 0 at ?????
) f (z) = f0 (z) + 6 f1 (z) is the Laurent decomposition for 3 < jzj < 4, also
for 2 < jzj < 32 .
z+ 2 +z 2 2z 2z
P1 2 1
n
(b) f1 (z) = z 2 2 =4
= z 2 2 =4
= z 2 k=0 4 z 2 =
P1 2
n
1
2 n=0 4 z 2n+1
, converges for jzj > =2.
(c) f (z) is odd, so f0 (z) is odd, and a0 = a2 = 0.
H H H H
a1 = 21 i jzj=3 tan z 2
z
dz = 1
2 i jzj="
+ jz =2j=" jz+ =2j="
tan z
z2
dz
tan z sin z 1
H
At z 0, z2 , jzj=" = 2 i
tan z
z cos z z
1 1 4 1
H
At z 2
, z 2 z 2 z =2 2 z =2
, jzH =2j="
= 24 2 i
At z 2
, tanz2
z 1
z 2 z+ =2
1 4 1
2 z+ =2 , jz+ =2j="
= 24 2 i

6
3
(d) Since f0 (z) = f (z) f1 (z) has poles at 2
, otherwise is analytic for
jzj < 32 , the series converges for jzj < 32 .

7
VI.1.4
1 2 3 P L K
LLL
Suppose that f (z) = f0 (z) + f1 (z) is the Laurent decomposition of an an-
alytic function f (z) on the annulus fA < jzj < Bg. Show that if f (z) is
an even function, then f0 (z) and f1 (z) are even functions, and the Laurent
series expansion of f (z) has only even powers of z. Show that if f (z) is an
odd function, then f0 (z) and f1 (z) are odd functions, ant the Laurent series
expansion of f (z) has only odd powers of z.

Solution
H f (z) P
1
Use an = z n+1 dz, or denition of f0 (z)+f1 (z), f (z) = an z n , f ( z) =
jzj=r 1
P
( 1)n an z n . f even ) an = an for n odd ) an = 0 for n odd. )
f0 (z) + f1 (z) have only even ????? ?????.

8
VI.1.5
1 2 3 P L K
LLL
Suppose f (z) is analytic on the punctured plane D = Cn f0g. Show
that there is a constant c such that f (z) c=z has a primitive in D.
Give a formula for c in terms of an integral of f (z).

Solution
By the Laurent Expansion Theorem we have for all z 2 D and r > 0
X
1 Z
k 1 f( )
f (z) = ak z where ak = d :
k= 1
2 i Cr (0) ( z0 )k+1
1
R
Setting c = a 1 = 2 i Cr (0)
f ( ) d we have
X
f (z) c=z = ak z k :
k6= 1
R
Recall that an analytic function g has a primitive on a region D i g (z) dz =
0 for every piecewise smooth
P closed path in D. Let be such a path in D
and observe the series k6= 1 ak z k converges uniformly on any closed annulus
fz : r jzj sg with 0 < r < s and hence on . Hence, by the rst theorem
on page 126, we may integrate the series termwise we obtain:
Z Z X X Z X
k
f (z) c=zdz = ak z = ak z k dz = ak 0 = 0;
k6= 1 k6= 1 k6= 1

since z k has a primitive if k 6= 1. Hence, f (z) c=z has a primitive in D.

9
VI.1.6
1 2 3 P L K

Fix an annulus D = fa < jzj < bg, and let f (z) be a continuous function on
its boundary @D. Show that f (z) can be approximated uniformly on @D by
polynomials in z and 1=z if and only if f (z) has continuous extension to the
closed annulus D [ @D that is analytic on D.

Solution
Use the conformity theorem for polynomial approximation, V5 14. If f is
analytic, continuous ????? values, unit f = f0 +f1 , its Laurent decomposing.
Then f0 + f1 have continuous boundary values. Can approximate f0 (z)
uniformly by polynomials in z for f1 (z) uniformly by polynomials in 1=z.
(?????? change of variable w = 1=z ). That does it. If f = lim gn (z)
uniformly on @D, then by the maximum principle jgk gj j ! 0 uniformly
on D, so gk ! f uniformly on D, and clearly F n@D = f .

10
VI.1.7
1 2 3 P L K

Show that a harmonic function u on an annulus fA < jzj < Bg has a unique
expansion
X
1 X
u rei = an rn cos (n ) + bn rn sin (n ) + c log r;
n= 1 n6=0

which is uniformly convergent on each circle in the annulus. Show that for
each r, A < r < B, the coe cients an ; bn and c satisfy
Z
n n 1
an r + a nr = u rei cos (n ) d ; n 6= 0;
Z
n n 1
bn r + b nr = u rei sin (n ) d ; n 6= 0;
Z
1
a0 + c log r = u rei d :
2
Hint. Use a decomposition of the form u = Re f +c log jzj, where f is analytic
on the annulus. (See Exercise III.3.4.)
Solution
By III.3.4, we have u = Re f +c log jzj for some e:
Write itsPLaurent series
f (z) = 1 k
k= a ck z , A < jzj < B.
Then for Pck = k k , wek have
Re f P= 1 k= 1 k Re z + k Im z
k

u= 1 k k
k= 1 k r cos (k ) + k r sin (k ) + c log
The series converging uniformly on ????? The term w= 0 ?????
Multiply
R by cos n , ????? useR orthogonality, get for
R n 6= 0,
n 2 n
u (r; ) cos n d = n r cos n d + n r cos2 n d =
n n
( nr + nr ) R
) n rn + n r n = 1 u (r; ) cos n d , n 6= 0.
n n 1
R
The formula for n r nr R = u (r; ) sin n d , n 6= 0, is similar.
1
R R
If we just integrate, we get u (r; ) = 21 c log rd + 2 0 d = 0 +
c log r
Note:

11
VI.1.8 (ej) H
Multiply f by a unimodular constant, assume jf (z)j 6 M , f (z) dz =
jzj=1
R2 R2
f ei d = 2 M . Take real parts, have Re f ei iei d = 2 M,
0 0
Re f ei iei d 6 f ei 6 M . If have strict inequality somewhere,
R2
then < 2 M . We conclude that Re f ei iei d M . Since f ei iei 6
0
M , we have Im f ei iei d 0. ) f ei iei M , f ei ie i
M,
f (z) iM z. ) f (z) = cz for a constant, jcj = M .

VI.1.9 (ej)
For the analyticity, dierentiate
h by hand. (See i Exercise III.1.6). The Deriv-
R h(z) h(z)
ative is H 0 (w) = lim 1w z (w+ w) z w
dz
w!1
R h(z) w R
= lim 1w (z (w+ w))(z w)
dz = (zh(z)dz
w)2
w!1

12
VI.2.1
1 2 3 P L K
LLL
Find the isolated singularities of the following functions, and de-
termine where they are removable, essential, or poles. Determine
the order of any pole, and nd the principal part at each pole.
2 sin z
(a) z= (z 2 1) (d) tan z = cos z
(g) Log 1 z1
z
(b) zze
2 1 (e) z 2 sin z1 (h) (zLog1)z3
(i) e1=(z +1)
2z 2
(c) e 1 z
(f) cos z
z2 2 =4

Solution
(a) (J.R.J Groves)
We have that z = 1 is a pole of order 2 since (z2 z 1)2 = (zg(z)
1)2
with g (z) =
z
(z+1)2
and g (z) is analytic near z = 1. Similarly we have, z = 1 is a pole
h(z)
of order 2 since (z z1)2 = (z+1) 2 with h (z) =
z
(z 1)2
and h (z) is analytic near
z = 1.
(b) (J.R.J Groves)
z g(z) zez
We have that z = 1 is a pole of order 1 since zze 2 1 = z 1 with g (z) = z+1 ,

and g (z) is analytic near z = 1. Similarly, z = 1 is a pole of order 1 since


zez z
z2 1
= h(z)
z+1
with h (z) = zze 1 , and h (z) is analytic near z = 1.
(c) (A. Kumjian)
Note that the given function is analytic on the punctured plane D = Cn f0g.
It has an isolated singularity at z = 0. For z 6= 0 we have
!
e2z 1 1 X1
(2z)k 1 X 2k z k
1
1 X 2j+1 j
1
= 1+ = = z :
z z k=0
k! z k=1
k! z j=0
(j + 1)!

Since the function has a power series expansion for all z 6= 0, it extends to an
analytic function at z = 0. Hence, the function has a removable singularity
at 0.
(d) (J.R.J Groves)
We have that cos z has simple zeros at z = =2 + k for k 2 Z. So cos1 z has
simple poles at z = =2 + k for k 2 Z. Since sin z is analytic and dierent
sin z
to 0 at these points, tan z = cos z
has simple poles at z = =2 + k for k 2 Z.
(e) (A. Kumjian)

13
The function is analytic on the punctured plane D = Cn f0g. It has an
isolated singularity at z = 0. For z 6= 0 we have

1 X1
( 1)k 1
2k+1
2 2
z sin =z =
z k=0
(2k + 1)! z
X1
( 1)k 1 X1
( 1)k 1
= 2k 1
=z+ :
k=0
(2k + 1)! z k=1
(2k + 1)! z 1
2k

Hence z = 0 is an essential singularity for the function since there are an in-
nite number of nonzero terms in the Laurent series with negative exponents.
(e) (J.R.J Groves)
The function z 2 sin z1 has an isolated singularity at z = 0. Since

1 X1
( 1)n 1 X1
( 1)n 1
z 2 sin = z2 2n+1
= 2n 1
;
z n=0
(2n + 1)! z n=0
(2n + 1)! z
z = 0 is an essential singularity.
(f) (J.R.J Groves)
The function z 2 ( =2)2 = (z =2) (z + =2) has simple zeros at z = =2
and z = =2. The function cos z has a simple zero at z = =2. So we suspect
a removable singularity. Set w = z =2. Then

cos (z) cos (z) cos (w + =2) 1 sin w


= = = :
z2 ( =2)2 (z =2) (z + =2) w (w + ) w+ w

Since we know that sin (w) =w = 1 w2 =3! + : : :, we deduce that our function
has a removable singularity at w = 0 and so at z = =2. In a similar fashion,
it also has a removable singularity at z = =2.
(g) (J.R.J Groves)
The function Log (z) is analytic on the region Cn ( 1; 0], it follows that
Log 1 z1 is analytic on Cn [0; 1]. There are no isolated singularities.
(h) (A. Kumjian)
First observe that for z 2 C with jz 1j < 1 we have

1 1 X
1
= = ( 1)k (z 1)k :
z 1 + (z 1) k=0

14
Since Log z is a primitive of 1=z, its power series expansion centered at z0 = 1
my be obtained from that of 1=z by integrating termwise: we obtain (note
Log 1 = 0)
X1
( 1)k+1
Log z = (z 1)k ;
k=1
k
for jz 1j < 1. Hence, we have for 0 < jz 1j < 1

Log z 1 X1
( 1)k+1 k
X 1
( 1)k+1
= (z 1) = (z 1)k 3
=
(z 1)3 (z 1)3 k=1 k k=1
k
1 1 X1
( 1)j
= 2 + (z 1)j
(z 1) 2 (z 1) j=0
k + 3

Hence, z = 0 is a pole of order 2. The principal part is given by (z 11)2 2(z1 1) .


(h) (J.R.J Groves)
We have that z = 1 is a pole of order 2. This follows from the fact that
2 3
Log z = (z 1) (z 21) + (z 31) for jz 1j < 1 so that
Log z 1 1 1 1
3 = + :
(z 1) (z 1)2 2 z 1 3
(i) (J.R.J Groves)
There are isolated singularities at z = i and z = i. Moreover,

1 i 1 1
=
z2 +1 2 z+i z i
so that
1 i 1 i 1 i 1 X1 ( i)n 1
e z2 +1 = e 2 z+i e 2 z i = e2 z+i :
n
n=0 2 n! (z i)n
i 1 1
since e 2 z+i is analytic and non-zero at z = i, e z2 +1 has an essential singularity
at z = i. Similarly, it has an essential singularity at z = i.

15
(a) Double poles at 1 , principal parts (1=4) (z 1)2 .
z ez ez
(b) Singularity at z = 1, simple poles zze 2 1 = 2
1
z 1
+ z+1 . Principal part
1=2 1=2
z 1
at z = 1, z+1 at z = 1.
(d) tan z = sin z= cos z, poles at z = =2 + n , n = 0; 1; 2; : : :. cos z =
sin ( =2 + n ) (z =2 n ) + O (z =2 n )3
sin( =2+n )+O((z =2+n )2 )
tan z = sin( =2+n )(z =2 n )+O (:::)3 = z =21 n + O (z =2 n ) Poles
( )
are simple, principal part 1= (z =2 n ) .
2 2
(f) cos z= (z =4), singularities at =2 are both removable.
(g) Dened and analytic for 1 1=z 2 Cn [ 1; 0), 1=z 1 2 Cn [0; 1),
1=z 2 Cn [1; 1), z 2 Cn [0; 1]. No isolated singularities.
Rz R2 dw RP
(h) Isolated singularity at z = 1, Log z = dw w
= 1+w 1
= ( 1)n (w 1)n dw =
1
P
1 n
( 1) (z 1) n+1
(z 1) 2
(z 1) 3

n+1
Log z = (z 1) 2
+ 3
:::
n=0
Log z 1 1
(z 1)3
= (z 1)2 z(z 1)
+ analytic
Double pole, principal part (z 11)2 z(z1 2) at z = 1. (i) e1=(z +1) , singularities
2

at z = i. Values of 1= (z 2 + 1) approach 1 from all directions as z ! i,


so values of e1=(z +1) do not converge, and singularities are essential.?????
2

check e1(1 t ) = f (it) ! +1 as t " 1 or t # 1, e1(1 t ) = f (it) ! 0 as


2 2

t # 1 or t " 1, so no limit at i.

16
VI.2.2
1 2 3 P L K
LLL
Find the radius of convergence of the power series for the following functions,
expanded about the indicated point.
(a) zz4 11 ; about z = 3 + i; (c) sinz z ; about z = i;
cos z z2
(b) z2 2 =4 ; about z = 0; (d) sin3 z ; about z = i:

Solution
(a) Removable singularity at z = 1, poles at i; 1. R = 3 = ?????? to i
(nearest singularity).
(b) singularities at =2 are removable. p
(c) removable at z = 0, poles at ; 2 . R=j i j= 2.
2 2
(d) z = sin z, simple poles at z = 0, R = .

17
VI.2.3
1 2 3 P L K

Consider the function f (z) = tan z in the annulus f3 < jzj < 4g. Let f (z)
= f0 (z) + f1 (z) be the Laurent decomposition of f (z), so that f0 (z) is
analytic for jzj < 4, and f1 (z) is analytic for jzj > 3 and vanishes at 1. (a)
Obtain an explicit expression for f1 (z).
(b)
Write down the series expansion for f1 (z), and determine the largest domain
on which it converges.
(c)
Obtain the coe cients a0 ; a1 and a2 of the power series expansion of f0 (z).
(d)
What is the radius of convergence of the power series expansion for f0 (z)?

Solution
(a) tan z has two poles in the disk jzj < 4, simple poles at =2, principal
parts 1= (z =2). If f1 (z) = 1= (z =2) 1 (z + =2), then f0 (z) =
f (z) f1 (z) is analytic for jzj < 4, and f1 (z) is analytic for jzj > 3 and
! 0 as z ! 1. By uniqueness, f (z) = f1 (z) + f2 (z) is the Laurent
decomposition.
z+ =2+z =2
(b) Use geometric series. Converges for jzj > =2. f1 (z) = z2 2 =4 =
2z 2z
P 2 1
1 n
z2 2 =4 = z2 4 z2
= , converges for jzj > =2.
k=0
P
1 2
n
1
2 4 z 2n+1
n=0
(c) a0 = a2 = 0; a1 = 1 + 8= 2 . (d) Since f0 (z) = f (z) f1 (z) has poles at
3 =2, otherwise is analytic for jzj < 3, the series converges for jzj < 3 =2,
and R = 3 =2.

18
VI.2.4
1 2 3 P L K

Suppose f (z) is meromorphic on the disk fjzj < sg, with only a nite number
of poles in the disk. Show that the Laurent decomposition of f (z) with
respect to the annulus fs " < jzj < sg has the form f (z) = f0 (z) + f1 (z),
where f1 (z) is the sum of the principal parts of f (z) at its poles

Solution
Since f1 (z) is a sum of principal parts, with poles inside fjzj 6 "g, f1 (z)
is analytic for jzj > ", and f1 (z) ! 0 as z ! 1. Further f0 (z) =
f (z) f1 (z) is analytic at ????? ??????, hence for jzj < 5. By the uniqueness
of the Laurent decomposition, f is f0 (z) + f1 (z) = f (z). Use the uniqueness
of the Laurent decomposition.

19
VI.2.5
1 2 3 P L K

By estimating the coe cients of the Laurent series, prove that if z0 is an


isolated singularity of f , and if (z z0 ) f (z) ! 0 as z ! z0 , then z0 is
removable. Give a second proof based on Moreas theorem.

Solution
Suppose that z0 is an isolated singularity of f and that (z z0 )N f (z) is
bounded near z0 . Then by Riemanns Theorem on removable singularities
(see p. 172), (z z0 )N f (z) has a removable singularity at z0 , the Laurent
expansion is of the following form: there is a r > 0 so that

N
X
1
(z z0 ) f (z) = ak (z z0 )k
k=0

for all 0 < jz z0 j < r. And hence, we have


X
1 X
1
f (z) = ak (z z0 )k N
= aj+N (z z0 )j :
k=0 j= N

It follows that z0 is a pole of order at most N unless ak = 0 for all k =


0; 1; : : : ; N 1, in which case z0 is a removable singularity.

20
VI.2.6
1 2 3 P L K
LLL
Show that if f (z) is continuous on a domain D, and if f (z)8 is analytic on
D, then f (z) is analytic on D.

Solution

21
VI.2.7
1 2 3 P L K
LLL
Show that if z0 is an isolated singularity of f (z), and if (z z0 )N f (z)
is bounded near z0 , then z0 is either removable or a pole of order
at most N .

Solution
Suppose that z0 is an isolated singularity of f and that (z z0 )N f (z) is
bounded near z0 . Then by Riemanns Theorem on removable singularities
(see p. 172), (z z0 )N f (z) has a removable singularity at z0 , the Laurent
expansion is of the following form: there is r > 0 so that

N
X
1
(z z0 ) f (z) = ak (z z0 )k
k=0

for all 0 < jz z0 j < r. And hence, we have


X
1
k N
X
1
f (z) = ak (z z0 ) = aj+N (z z0 )j :
k=0 j= N

It follows that z0 is a pole of order at most N unless ak = 0 for all k =


0; 1; : : : ; N 1, in which case z0 is a removable singularity.

22
VI.2.8
1 2 3 P L K

A meromorphic function f at z0 is said to have order N at z0 if f (z) =


(z z0 )N g (z) for some analytic function g at z0 such that g (z0 ) 6= 0. The
order of the function 0 is dened to be +1. Show that
(a) order (f g; z0 ) = order (f; z0 ) + order (g; z0 ),
(b) order (1=f; z0 ) = order (f; z0 ),
(c) order (f + g; z0 ) min forder (f; z0 ) ; order (g; z0 )g.
Show that the inequality can occur in (c), but that equality hold in (c)
whenever f and g have dierent orders at z0 .

Solution

23
VI.2.9
1 2 3 P L K

Recall that "f (z) = O (h (z)) as z ! z0 " means that there is a constant
C such that jf (z)j C jh (z)j for z near z0 . Show that if z0 is an isolated
singularity of an analytic function f (z), and if f (z) = O ((z z0 )m ) as
z ! z0 , then the Laurent coe cients of f (z) are 0 for k < m, that is the
Laurent series of f (z) has the form

f (z) = am (z z0 ) + am+1 (z z0 )m+1 + :


Remark. This shows that the use of notation O (z m ) in section V.6 is con-
sistent.

Solution

24
VI.2.10
1 2 3 P L K
LLL
Show that if f (z) and g (z) are analytic functions that both have the same
order N 0 at z0 , then

f (z) f (N ) (z0 )
lim = (N ) :
z!z0 g (z) g (z0 )

Solution

25
VI.2.11
1 2 3 P L K
P
Suppose f (z) = ak z k is analytic for jzj < R, and suppose that f (z)
extends to be meromorphic for jzj < R + ", with only one pole z0 on the
circle jzj = R. Show that ak =ak+1 ! z0 as k ! 1.

Solution P
If f (z) = an z n is analytic for jzj < R, and is meromorphic for jzj < R + ",
with polesP of ?????6 N on the circle jzj = R, then an = O nN 1 . If
f (z) = an z n as only one pole on the circle jzj = R, of ????? N , with
A( 1)N +1 kN
f (z) = A= (z z0 )N + lower order, then ak = (N 1)!z N +k+1
(1 + O (1=k)), so
0
PN
Al P
that ak =ak+1 ! z0 as n ! 1. Proof: Write f (z) = (z z0 )l
+ bk z k
P l=0
analytic, h (z) = bk z k analytic. We have jbk j 6 1=sk for some s >
P
1 j
R, bk = O 1= jz0 jk . z 1z0 = z10 1 z=z1
0
= 1
z0
z
zj
, ( 1)d (z(l z1)!)l =
0 0
j=0
1
P
1
j(j 1):::(j l+1)z j l

z0 z0l
j=l
1 ( 1)l+1 P
1
1
(z z0 )l
= (l 1)! z0k+l+1
(k + l) : : : (k + 1) z k
k=0
1 ( 1)l+1 P1
1
(z z0 )l
= (l 1)! z0k+l+1
(k + l) : : : (k + 1) z k
k=0

26
VI.2.12
1 2 3 P L K
LLL
Show that if z0 is an isolated singularity of f (z) that is not removable, then
z0 is an essential singularity for ef (z) .

Solution
Apply Arzela-Weierstrass theorem. Suppose z0 = 0, an isolated singularity
of f (z), not removable. 0 essential ) values of f (z) ???? on C as z ! 0,
! values of ef (z) ????? on C as z ! 0. 0 a pole ) f (z) ! 1, as z ! 0,
monotone, values of f (z) ?????? of 0 cover the extremum of some ??????
fjwj > Rg. In any neighbourhood of 0 there are values of ef (z) = ew that are
near 0 (e m ) and that are near to (e+m ), so ef (z) does not have a limit as
z ! 0. 0 is essential singularity of ef (z) .

27
VI.2.13 (From Hints and Solutions)
1 2 3 P L K

Let S be a sequence converging to a point z0 2 C, and let f (z) be analytic


on some disk centered at z0 except possibly at the points of S and z0 . Show
that either f (z) extends to be meromorphic on some neighborhood of z0 , or
else for any complex number L there is a sequence fwj g such that wj ! z0
and f (wj ) ! L.

Solution
Suppose values of f (z) do not cluster at L as z ! z0 . Then g (z) =
1= (f (z) L) is bounded for jz z0 j < "; z 6= zj . Apply Riemanns the-
orem rst to the zj `s for j large, then to z0 , to see that g (z) extends to be
analytic for jz z0 j < ", and f (z) is meromorphic there

28
VI.2.14
1 2 3 P L K

Suppose u rei is harmonic on a punctured disk f0 < r < 1g, with Laurent
series as in Exercise 7 of Section 1. Suppose > 0 is such that r u rei ! 0
as r ! 0. Show that an = 0 = bn for n .

Solution
P
1 P
1
u rei harmonic, 0 < r < 1. u = an rn cos n + bn rn sin n + c log r.
n=0 n6=0
Let m > , m > 0. Have am rm + a m r m = O (r ), am r2m + a m =
O (r +m ) as r ! 0. Let r ! 0, get a m = 0 for m > . Similarly b m = 0
for m > .

29
VI.2.15
1 2 3 P L K

Suppose u (z) is harmonic on the punctured disk f0 < jzj < g.


Show that if

u (z)
!0
log (1= jzj)
as z ! 0, then u (z) extends to be harmonic at 0. What can you say if
you know only that ju (z)j C log (1= jzj) for some xed constant C and
0 < jzj < ?

Solution
Assume u (z) = O (log (1= jzj)) R as z ! 0. Consider Laurent series as in
14. Have. For m > 1, get u rei d = O (log (1=r)) am rm + a m r m =
O (log (1=r)), am r2m + a m = O (rm log (1=r)), a m = 0 for all m > 0, and
b m = 0 for m > 0. u rei = Re f rei + c log r, when f is analytic, c =
u(r) Re f (rei ) c log r
constant. log(1=r) = log(1=r) + log(1=r) , c = 0, and u = Re f is analytic at
z = 0.

30
VI.3.1
1 2 3 P L K

Consider the functions in Exercise 1 of Section 2 above. Determine which


have isolated singularities at 1, and classify them.

Solution

31
VI.3.2
1 2 3 P L K

Suppose that f (z) is an entire function that is not a polynomial. What kind
of singularity can f (z) have at 1?

Solution

32
VI.3.3
1 2 3 P L K

Show that if f (z) is nonconstant entire function, then ef (z) has an essential
singularity at z = 1.

Solution

33
VI.3.4
1 2 3 P L K

Show that each branch of the following functions is meromorphic at 1, and


obtain the series expansion
p for each branch at
p 1.
2 5=2 3
(a) (z 1) (b) 3
(z 1) (c) z 2 1=z

Solution

34
VI.4.1
1 2 3 P L K
LLL
Find the partial fractions decompositions of the following functions.
(a) z21 z (c) (z+1)(z12 +2z+2) z 1
(e) z+1
z 2 +1 1 2
(b) z(z 2 1) (d) (z2 +1)2 (f) zz2 4z+3
z 6
(a)
1 A B
= +
z2 z z z 1

1
A = lim z f (z) = = 1
z!0 z 1 z=0
1
B = lim (z 1) f (z) = =1
z!1 z z=1

1 1 1
= +
z2 z z z 1
(b)

z2 + 1 A B C
2
= + +
z (z 1) z z 1 z+1

z2 + 1
A = lim z f (z) = = 1
z!0 z2 1 z=0
2
z +1
B = lim (z 1) f (z) = =1
z!1 z (z + 1) z=1
z2 + 1
C = lim (z + 1) f (z) = =1
z! 1 z (z 1) z= 1

z2 + 1 1 1 1
= + +
z (z 2 1) z z 1 z+1
(c)
1 A B C
= + +
(z + 1) (z 2 + 2z + 2) z+1 z+1+i z+1 i

35
1
A = lim z f (z) = =1
z! 1 (z 2 + 2z + 2) z= 1
1 1
B = lim (z + 1 + i) f (z) = =
z! 1 i (z + 1) (z + 1 i) z= 1 i 2
1 1
C = lim (z + 1 i) f (z) = =
z! 1+i (z + 1) (z + 1 + i) z= 1+i 2

1 1 1=2 1=2
=
(z + 1) (z 2 + 2z + 2) z+1 z+1+i z+1 i
(d) (/snider 106) man s?
1 A B C D
2 = 2 + + 2 +
(z 2 + 1) (z i) (z i) (z + i) z i

1 1
A = lim (z i)2 f (z) =
2 =
z!i (z + i) z=i 4
d 1 i
B = lim (z i)2 f (z) = 2 =
z!i dz (z + i) z=i 4
1 1
C = lim (z + i)2 f (z) = 2 =
z! i (z i) z= i 4
d 1 i
D = lim (z + i)2 f (z) = 2 =
z! i dz (z i) z= i 4

1 1=4 i=4 1=4 i=4


= 2 +
(z 2 + 1)2 (z i)2 (z i) (z + i) z i
(e)
z 1 A
= +B
z+1 z+1

A = lim (z + 1) f (z) = z 1jz= 1 = 2


z! 1
B = lim f (z) = 1
z!1

36
z 1 2
=1
z+1 z+1
(f)

z2 4z + 3 A B
= + +C
z2 z 6 z+2 z 3

z2 4z + 3
A = lim (z + 2) f (z) = = 3
z! 2 z 3 z= 2
z 2 4z + 3
B = lim (z 3) f (z) = =0
z!3 z (z + 1) z=3
C = lim f (z) = 1
z!1

z2 4z + 3 3
= +1
z2 z 6 z+2

37
VI.4.2
1 2 3 P L K
LLL
Use the division algorithm to obtain the partial fractions decomposition of
the following functions
3 9 z6
(a) zz2 +1
+1
(b) zz6 +11 (c) (z2 +1)(z 1)2
(a)

z3 + 1 z+1 A B
= z + = z + +
z2 + 1 z2 + 1 z+i z i

z+1 1 1
A = lim (z + i) f (z) = = + i
z! i z i z= i 2 2
z+1 1 1
B = lim (z i) f (z) = = i
z!i z + i z=i 2 2

z3 + 1 z+1 ( 1 + i) =2 (1 + i) =2
2
=z+ 2 =z+
z +1 z +1 z+i z i
(b)

z9 + 1 3
3 z +1 1 1 A B C
6
= z + 6
= z3+ 3 = z3+ 2
= z3+ + +
z 1 z 1 z 1 (z 1) (z + z + 1) z 1 z w z w

Because z 3 1 = 0 have tree roots z1 = 1, z2 = e2 i=3


, and z3 = e4 i=3
. Set
z2 = w, and we see that z3 = w.

1 1
A = lim (z 1) f (z) = =
z!1 z2 +z+1 z=1 3
1 w
B = lim (z w) f (z) = =
z!w (z 1) (z e 2 i=3 ) z=e2 i=3 3
1 w
C = lim (z w)2 f (z) = 2 i=3
=
z!w (z 1) (z e ) z=e 2 i=3 3

z9 + 1 1=3 w=3 w=3


6
= z3 + + + ; where w = e2 i=3
z 1 z 1 z w z w

38
Konstigt svar i boken?
1=3
z 1
+ zw=3w + zw=3w 1=3
z 1
+ w=3
z w
w=3
z+w
=
(c)

z6 z6
=
(z 2 + 1) (z 2 2z + 1) z4 2z 3 + 2z 2 2z + 1
z6 2z 3 z 2 + 2z 2
= z 2 + 2z + 2 +
z4 2z 3 + 2z 2 2z + 1 (z 2 + 1) (z 1)2
2z 3 z 2 + 2z 2 A B C D
2 = + + 2 +
(z 2 + 1) (z 1) z + i z i (z 1) z 1

2z 3 z 2 + 2z 2 1
A = lim (z + i)2 f (z) = =
z! i (z i) (z 1)2 z= i 4
2z 3 z 2 + 2z 2 1
B = lim (z i) f (z) = 2 =
z!i (z + i) (z 1) z=i 4
2z 3 z 2 + 2z 2 1
C = lim (z 1)2 f (z) = 2
=
z!1 (z + 1) z=1 2
3 2
d 2z z + 2z 2 5
D = lim (z 1)2 f (z) = =
z!1 dz (z 2 + 1) z=1 2

39
VI.4.3
1 2 3 P L K
LLL
Let V be the complex vector space of functions that are analytic on the
extended complex plane except possibly at the points 0 and i, where they
have poles of order at most two. What is the dimension of V ? Write down
explicitly a vector space basis for V .

Solution

40
VI.5.1
1 2 3 P L K

Show that if f (z) and g (z) have period w, then so do f (z) +g (z) and
f (z) g (z).

Solution

41
VI.5.2
1 2 3 P L K

Expand 1= cos (2 z) in a series of powers of e2 iz that converges in the upper


half-plane. Determine where the series converge absolutely and where it
converges uniformly.

Solution

42
VI.5.3
1 2 3 P L K

Expand tan z in a series of powers of exponentials eikz , 1 < k < 1, that


converges in the upper half-plane. Also nd an expansion of tan z as in an
exponential series that converges in the lower half-plane.

Solution

43
VI.5.4
1 2 3 P L K

Let f (z) be an analytic function in the upper half-plane that is periodic,


with real period 2 > 0. Suppose that there are A; C > 0 such that
jf (x + iy)j CeAy for y > 0. Show that
X
f (z) = an einz= ;
n A

where the series converges uniformly in each half-plane fy "g, for xed
" > 0.

Solution

44
VI.5.5
1 2 3 P L K

Suppose that 1 are periods of a nonzero doubly periodic function f (z), and
suppose that there are no periods w of f (z) satisfying 0 < jwj < 1. How
many periods of f (z) lie on the unit circle? Describe the possibilities, and
sketch the set of periods for each possibility.

Solution

45
VI.5.6
1 2 3 P L K

We say that w1 and w2 generate the periods of a double periodic function


if the periods of the function are precisely the complex numbers of the form
mw1 + nw2 where m and n are integers. Show that if w1 and w2 generate
the periods of a doubly periodic function f (z), and if 1 and 2 are complex
numbers, then 1 and 2 generate the periods of f (z) if and only if there
is a 2 2 matrix A with integer entries and with determinant 1 such that
A (w1 ; w2 ) = ( 1 ; 2 ).

Solution

46
VI.5.7
1 2 3 P L K

Let w1 and w2 be two complex numbers that do lie on the same line through
0. Let k 3. Show that the series
X
1
1
m;n= 1 (z (mw1 + nw2 ))k
converges uniformly on any bounded subset of the complex plane to dou-
ble periodic meromorphic function f (z), whose periods are generated by
w1 and w2 . Strategy. Show that the number of periods in any annulus
fN jzj N + 1g is bounded by CN for some constant C.

Solution

47
VI.6.1
1 2 3 P L K

Consider the continuous function f ei = j j, . Find the


i
complex Fourier series f e and show that it can be expressed as a cosine
series. Sketch the graphs of the rst three partial sums of the cosine series.
Discuss the convergence of the series. Does it converge uniformly? Partial
answer. The cosine series is

4 1 1
j j= cos + 2
cos 3 + 2 cos 5 + :
2 3 5

Solution

48
VI.6.2
1 2 3 P L K

Let f ei , < (the principal value of the argument). Find the


complex Fourier series of f ei and the sine series of f ei . Show that the
complex Fourier series diverges at = , while the sine series converges
at . Dierentiate the complex Fourier series term by term and determine
where the dierentiated series converges.

Solution

49
VI.6.3
1 2 3 P L K

Consider the continuous function f ei = 2 , . Find the


i
complex Fourier series of f e and show that it can be expressed as a cosine
series. Discuss the convergence of the series. Does it converge uniformly?
By substituting = 0, show that
2
1 1 1
=1 + + :
12 22 32 42

Solution

50
VI.6.4
1 2 3 P L K

Consider the continuous function f ei = 4 2 2 2 , . Find


i
the complex Fourier series of f e and show that it can be expressed as a
cosine series. Relate the Fourier series to the series of the function in Exercise
3.

Solution

51
VI.6.5
1 2 3 P L K
P P ik
Show that if ck converges absolutely, then ck e converges absolutely
for each , and the series converges uniformly for .

Solution

52
VI.6.6
1 2 3 P L K

Show that any function f ei on the unit circle with absolutely convergent
Fourier series has the form f ei = g ei + h (ei ), where g (z) and h (z)
are continuous functions on the unit circle that extend continuously to be
analytic on the open unit disk.

Solution

53
VI.6.7
1 2 3 P L K
P
If f ei ck eik , and the series converges uniformly to f ei , then
Z X1
2 d
f e i
= jck j2 :
2 k= 1

Remark. This is called Parsevals identity. Formula (6.6) show Parsevals


identity holds for a function f ei if and only if the partial sums of the
Fourier series of f ei converge to f ei in the sense of "mean-square" or
"L2 approximation".

Solution

54
VI.6.8
1 2 3 P L K

By applying Parsevals identity to the picewise constant function with series


(6.5), show that
2
1 1 1
=1+ 2
+ 2+ 2+ :
8 3 5 7
Use this identity and som algebraic manipulation to show that
2
1 1 1
=1+ 2
+ 2+ 2+ :
6 2 3 4

Solution

55
VI.6.9
1 2 3 P L K

By applying Parsevals identity to the function of Exercise 1, show that


4
1 1 1
=1+ 4
+ 4+ 4+ :
96 3 5 7
Use this identity and som algebraic manipulation to show that
4
1 1 1
=1+ 4
+ 4+ 4+ :
90 2 3 4

Solution

56
VI.6.10
1 2 3 P L K

If f (z) is analytic P
in some annulus containing the unit circle jzj = 1, with
Laurent expansion ak z k , then
I X
1
1
jf (z)j2 jdzj = jak j2 :
2 jzj=1 k= 1

Solution

57
VI.6.11
1 2 3 P L K

Let f ei be a continuous function on the unit circle, with Fourier series


P
ck eik . Show that f ei extends to be analytic on some annulus con-
taining the unit circle if and only if there exist r < 1 and C > 0 such that
jck j Crjkj for 1 < k < 1.

Solution

58
VI.6.12
1 2 3 P L K

Using the convergence theorem for Fourier series, prove that every continti-
nously function on the unit circle in the complex plane can be approximated
uniformly there by trigonometric polynomials, that is, by nite linear com-
binations of exponentials eik , 1 < k < 1. Strategy. First approximate
f eik by a smooth function.

Solution

59
VI.6.13
1 2 3 P L K

Let D be a domain bounded by a smooth boundary curve of length 2 . We


parametrize the boundary of D by arc length s, sot the P boundary is given by
iks
a smooth periodic function (s), 0 s 2 . Let ck e be the Fourier
series of (s).
(a) P 2
Show that k jck j2 = 1. Hint. Apply Parsevals identity to 0 (s) and use
j 0 (s)j = 1 for a curve parameterized by arc length.
(b) P
Show that the area of D is k jck j2 . Hint. Use Exercise IV.1.4.
(c)
Show that the area of D is , with equality if and only if D is a disk.
Remark. This proves the isoperimetric theorem. Among all smooth closed
curves of a given length, the curve that surrounds the largest area is a circle.

Solution

60
VI.1.14
1 2 3 P L K

Show that

Z X
n 2 Z X
n 2
X
n
d d
f e i ik
bk e = f e i ik
ck e + jbk ck j2 ;
k= m
2 k= m
2 k= m

for any choice of complex numbers bk , m k n. Remark. This


i
shows
Pn that the best mean-square approximate to f e by exponential sums
ik
m bk e , for xed m and n, is the corresponding partial sum of the Fourier
series.

Solution

61
VI.1.15
1 2 3 P L K

Show that a continously dierentiable function on the unit circle has an


absolutely convergent Fourier series. Strategy. Write the Fourier coe cients
ck of f ei as ak bk , where ak = 1=ik and bk is the Fourier coe cient of
the derivative.
qPUse Bessel
qP s inequality and the Cauchy-Schwarz inequality
P 2
k k j kj j k j2 .

Solution

62
VI.6.16
1 2 3 P L K

Let f ei be a continuous function on the unit circle. Suppose that f ei


is piecewise continuously dierentiable, in the sense that it has a continuous
derivative except at a nite number of points, at each of which the derivative
has limits from the left and from the right. Show that the Fourier series
f ei is absolutely convergent. Strategy. Cancel the discontinuities of the
derivative using translates of the function in Exercise 3, whose Fourier series
is absolutely convergent.

Solution

63
VI.6.17
1 2 3 P L K

Let f ei be a piecewise continuously dierentiable, in the sense that it is


continuously dierentiable except at a nite number of points, at each of
which both the function and its derivative have limits from the left and from
the right. Show that the Fourier series of f ei converges at each point, to
f ei if the function is continuous at ei , and otherwise to the average of
the limits of f ei from
P the ileft and from the right. Strategy. Show that
i i i
f e = f1 e + bj hj e , where f1 e satises the hypotheses of
Exercise 15, and each hj ei is obtained from the function of Exercise 2 by
change of variable 7 ! j.

Solution

64
VII 1 2 3 4 5 6 7 8 9 10 11 12 13 14 15 16 17 18 19
1 X X X X
2 X X X X X X X X X X X X
3 X X X X X X X
4 X X X X X X X X X X
5 X X X X X X
6 X X X X X X X
7 X X X
8 X

1
VII.1.1
Evaluate the following residues. h i
(a) Res z21+4 ; 2i (d) Res sin
z2
z
;0 (g) Res z
Log z
;1
1 cos z ez
(b) Res z 2 +4
; 2i (e) Res (h) Res
z2
;0 z 5n
;0
1 z +1
(c) Res z5 1
;1 (f) Res [cot z; 0] (i) Res zn 1
; wk

Solution
a)
By rule 4,

1 1 1 i
Res ; 2i = = = :
z2 +4 2z z=2i 4i 4
b)
By rule 4,

1 1 1 i
Res ; 2i = = = :
z2 +4 2z z= 2i 4i 4
c)
By rule 4,

1 1 1
Res ;1 = = :
z5 1 5z 4 z=1 5
d)
By rule 1,

sin z sin z
Res 2
; 0 = lim = 1:
z z!0 z

e)
By rule 2,
h cos z i d
Res ; 0 = lim cos z = sin zjz=0 = 0:
z2 z!0 dz

f)
By rule 3,
h cos z i cos z
Res [cot z; 0] = Res ;0 = = 1:
sin z cos z z=0

2
g)
By rule 3,

z z
Res ;1 = = 1:
Log z 1=z z=1

h)
By Laurent expansion

z z2 z3 z4
ez 1+ 1!
+ 2!
+ 3!
+ 4!
+ o (z 5 ) 1 1 1 1 1
= = + + + + + o (1) ;
z5 z5 z 5 1!z 4 2!z 3 3!z 2 4!z
hence

ez 1 1
Res 5
;0 = = :
z 4! 24
i)
We have poles of the form wk = e2 ik=n
, where k = 0; 1; 2; : : : n 1, remark
that wkn = 1, by rule 3,

zn + 1 zn + 1 wkn + 1 2 2wk 2e2 ik=n


Res ; wk = = n 1 = = = :
zn 1 nz n 1 z=wk nwk nwkn 1
n n

3
VII.1.2
Calculate the residue at each singularity in the complex plane of
the following functions.
z
(a) e1=z (b) tan z (c) (z2 +1)2 (d) z21+z

Solution
(a)
The function e1=z has a singularity at z = 0, by Laurent expansion
1 1
e1=z = 1 + + + ::::
z 2!z 2
Thus by denition

Res e1=z ; 0 = 1:
(b)
sin z
The function tan z = cos z
has isolated singularities at z = 2
+ n, 1<
n < 1, they are simple poles.
By rule 3,

sin z sin z
Res [tan z; =2 + n ] = Res ; =2 + n = = 1:
cos z sin z z= =2+n

(c)
z
The function (z 2 +1)2
has isolated singularities at z = i, they are double
poles.
By rule 2,

z d z (z + i)2 2z (z + i)
Res 2 ;i = = = 0;
(z 2 + 1) dz (z + i)2 z=i (z + i)4 z=i

z d z (z i)2 2z (z i)
Res 2; i = = = 0:
2
(z + 1) dz (z i)2 z= i (z i)4 z= i

(d)

4
1
The function z(z+1)
has isolated singularities at z = 0 and z = 1 they are
simple poles.
By rule 1,

1 1 1
Res ; 0 = lim = = 1;
z (z + 1) z!0 z + 1 z+1 z=0

1 1 1
Res ; 1 = lim = = 1:
z (z + 1) z! 1 z z z= 1

5
VII.1.3
Evaluate
R the followingR integrals using Rresidue theorem.
(a) jzj=1 sin
z2
z
dz (c) jzj=2 cosz z dz (e) jz 1j=1 z81 1 dz
R z R z4
R
(b) jzj=2 z2e 1 dz (d) jzj=1 sin z
dz (f) jz 1=2j=3=2 tanz z dz

Solution
(a)
Using the residue theorem and rule 1,

Z
sin z sin z sin z
2
dz = 2 i Res 2
; 0 = 2 i lim = 2 i (1) = 2 i:
jzj=1 z z z!0 z

(b)
Using the residue theorem and rule 3,

Z
ez
dz =
jzj=2 z2 1
ez ez
= 2 i Res ; 1 + Res ; 1 =
z2 1 z2 1
z
e ez e1 e 1
=2 i + =2 i =
2z z=1 2z z= 1 2
= 2 i sinh 1:

(c)
Using the residue theorem and rule 3,

Z h z i h z i
z
dz = 2 i Res ; + Res ; =
jzj=2 cos z cos z 2 cos z 2
!
z z
=2 i + =2 i =
sin z z= =2 sin z z= =2 2 2
= 2 2 i:

(d)
Using the residue theorem and rule 3,

6
Z
z4 z4 z4
dz = 2 i Res ;0 = 2 i = 0:
jzj=1 sin z sin z cos z z=0

(e)

VII.1.3e

|z - 1| = 1

Using the residue theorem with the contour in Figure VII.1.3e and rule 4,

Z
1 1 1 1
8
dz = 2 i Res ; 1 + Res ; ei =4
+ Res ;e i =4
=
jz 1j=1 z 1 z8 1 z8 1 z8 1
1 1 1 i
=2 i 7
+ 7 + = 1 + e i=4 + e i=4 =
8z z=1 8z z=ei =4 8z 7 z=e i =4 4
i 1 i 1+i i p
= 1+ p + p = 1+ 2 :
4 2 2 4

(f)
The function tanz z = sin z=z
cos z
has removable singularity at z = 0, and a isolated
singularity at z = 2 inside the domain jz 1=2j = 3=2. Using the residue
theorem and rule 3,

7
Z
tan z
dz =
jz 1=2j=3=2 z
!
sin z=z sin z=z sin z=z 2
= 2 i Res ; =2 i lim =2 i = (2 i) =
cos z 2 z! =2 sin z sin z z= =2

= 4i:

8
VII.1.4
1 2 3 P L K

Suppose P (z) and Q (z) are polynomials such that the zeros of Q (z)
are simple zeros at the points z1 ; : : : ; zm ; and deg P (z) < deg Q (z).
Show that the partial fractions decomposition P (z) =Q (z) is given
by

P (z) X P (zj ) 1
m
= 0 (z ) z
:
Q (z) j=1
Q j zj

Solution
Since the zeros of Q (z) are simple, the polynomial quotient P (z) =Q (z) has
at most simple poles. By Rule 3, the residue at zj is

P (z) =Q0 (z) :


Hence the principal part of P (z) =Q (z) at zj is

P (zj ) 1
:
Q0 (zj ) z zj
Since deg P (z) < deg Q (z), then

P (z)
!0
Q (z)
as z ! 1, and P (z) =Q (z) has no principal part at 1.
By partial fraction decomposition our polynomial quotient P (z) =Q (z) or
can be written as the sum of its principal parts at its pole,

P (z) X P (zj ) 1
m
= 0 (z ) z
:
Q (z) j=1
Q j zj

9
VII.1.5
1 2 3 P L K

Let f (z) be a meromorphic function on the complex plane that is


doubly periodic, and suppose that none of the poles of f (z) lie on
the boundary of the period parallelogram P constructed in Section
VI.5. By integrating f (z) around of P , show that the sum of the
residues at the poles of f (z) in P is zero. Conclude that there is no
doubly periodic meromorphic function with only one pole, a simple
pole, in the period parallelogram.

Solution R
Follow directions. @P = 0, the curve integrals over opposite sides cancel.
Thus the sum of the residues is zero. In particular, there can not be a single
nonzero residue.

10
VII.1.6
Consider the integral
Z i(z 1=2)2
e
2 iz
dz;
@DR 1 e
where DR is the parallelogram with vertices 12 (1 + i) R.
(a) p
Use the residue theorem to show that the integral is (1 + i) = 2.
(b)
By parameterizing the sides of the parallelogram, show that the
integral tends to
Z 1
2
(1 + i) e 2 t dt
1

as R ! 1.
(c)
Use (a) and (b) to show that
Z 1
s2
p
e ds = :
1

Solution
(a)

VII.1.6
2

3
z1
1

Set

11
Z i(z 1=2)2
e
I= 2 iz
dz;
@DR 1 e
i.e, we integrate
2
e i(z 1=2)
f (z) =
1 e 2 iz
along the parallelogram contour in Figure VII.1.6. The parallelogram has
vertices at 1=2 (1 + i) R.
Residue at a simple pole at z1 = 0, where by Rule 3,
" 2
# 2
e i(z 1=2) e i(z 1=2) e i=4
Res ; 0 = = :
1 e 2 iz 2 ie 2 iz 2 i
z=0
Using the Residue Theorem, we obtain that
Z i(z 1=2)2
e e i=4
2 iz
dz = 2 i ;
@DR 1 e 2 i
i.e.,
Z i(z 1=2)2
e 1+i
2 iz
dz = p :
@DR 1 e 2
(b)
1
Integrate along 1 parametrized by z = 2
+ (1 + i) t with R t R, and
let R ! 1. This gives

Z Z R i(z 1=2)2
e z = 12 + (1 + i) t
f (z) dz = dz = =
1 R 1 e 2 dz = (1 + i) dt
iz
Z R 2 Z R 2
e i((1+i)t) (1 + i) e 2 t
= 1
dt = (1 + i) 2 i(1+i)t
dt !
R 1 e 2 i( 2 +(1+i)t) R 1+e
Z 1 2
e 2 t
! (1 + i) 2 i(1+i)t
dt:
1 1+e

1
Now integrate along 2 parameterized by z = t+ 2
+ (1 + i) R with 0
t 1, and let R ! 1.

12
Z
f (z) dz =
2
Z i(z 1=2)2
e z = t + 12 + (1 + i) R
= dz = =
2
1 e 2 iz dz = dt
Z Z
e (t 2t)i e 2 (R Rt)
2 2
1 i( t+(1+i)R)2 1
e
= 2 i( t+1=2+(1+i)R)
dt = dt
0 1 e 0 1 e (2t 1 2R)i e2 R

e (t 2t)i e 2 (R Rt) (R2 Rt)


2 2 2
e
sup 1 sup =
0 t 1 1 e (2t 1 2R)i e2 R 0 t 1 j1 je (2t 1 2R)i e2 R jj

e 2 (R Rt) (R2 Rt)


2 2
e
= sup = sup ! 0:
0 t 1 j1 e2 Rj
0 t 1 e2 R 1
1
Integrate along 3 , parameterized by z = 2
+ (1 + i) t with R t R,
and let R ! 1. This gives

Z Z R i(z 1=2)2 1
e z = 2
+ (1 + i) t
f (z) dz = dzdz = =
3 R e1 2 dz = (1 + i) dt
iz
Z R 2 Z R 2 i(1+i)t 2 t2
e i( 1+(1+i)t) (1 + i) dt e e dt
= 1
= (1 + i) 2 i(1+i)t
!
R 1 e 2 i( 2 +(1+i)t) R 1+e
Z 1 2 i(1+i)t 2 t2
e e dt
! (1 + i) 2 i(1+i)t
dt:
1 1+e
1
Now integrate along 4 parameterized by z = t 2
(1 + i) R with 0 t 1,
and let R ! 1.

13
Z
f (z) dz =
4
Z i(z 1=2)2
e z = t 12 (1 + i) R
= dz = =
4
1 e 2 iz dz = dt
Z 1 (1+i)R)2 Z 1 (2R 2Rt+t2 2t+1)i 2 (R2 +R Rt)
e i(t 1
e e
= dz = dz =
0 1 e 2 i(t 1=2 (1+i)R)
0 1 e (2t 1 2R)i e 2 R

e (2R 2Rt+t2 2t+1)i


e 2 (R2 +R Rt)
e 2 (R2 +R Rt)
sup (2t 1 2R)i e 2 R
1 sup (2t 1 2R)i e 2 R jj
0 t 1 1 e 0 t 1 j1 je

e 2 (R2 +R Rt)
sup 2 Rj
! 0:
0 t 1 j1 e

Using the Residue Theorem and letting R ! 1, we obtain that

Z 1 2 t2 Z 1 2 i(1+i)t 2 t2
e e e dt
(1 + i) 2 i(1+i)t
dt + 0 + (1 + i) 2 i(1+i)t
dt + 0 =
1 1+e 1 1+e
e i=4
= 2 i ;
2 i
and therefore
Z 2 t2
1
e 1 + e 2 i(1+i)t 1+i
(1 + i) 2 i(1+i)t
dt = p ;
1 1+e 2
i.e.,
Z 1
2 t2 1+i
(1 + i) e dt = p :
1 2
(c)
Comparing real parts thus yields,
Z 1
1 2 t2
e dt = p ;
1 2
and changing variables as follows gives the desired result

14
Z 1
p Z 1
1 2 t2 s = p2 t 1 s2
p = e dt = =p e ds;
2 1 ds = 2 dt 2 1

i.e.,
Z 1
s2
p
e dt = :
1

15
VII.2.1
1 2 3 P L K

Show using residue theory that


Z 1
dx
2 2
= ; a > 0:
1 x +a a
Remark. Check the result by evaluating the integral directly, using
the arctangent function.

Solution

VII.2.1

z1

-R 1 R
z2

Set
Z 1
dx
I= ;
1 x 2 + a2
and integrate
1 1
f (z) = =
z2 + a2 (z ia) (z + ia)
along the contour in Figure VII.2.1.
Residue at a simple pole at z1 = ia, where by Rule 2,

1 1 i
Res ; ia = = :
z2 +a 2 2z z=ia 2a

16
Integrate along 1 , and let R ! 1. This gives
Z Z R Z 1
dx dx
f (z) dz = 2+a 2 ! 2 + a2
= I:
1 R x 1 x
Integrate along 2, and let R ! 1. This gives
Z
1
f (z) dz 2
R ! 0:
2
R a2 R
Using the Residue Theorem and letting R ! 1, we obtain that

i
I +0=2 i ;
2a
and therefore
Z 1
dx
= ; a > 0:
1 x2 + a2 a
Remark.
Check of the result by evaluating the integral directly gives, because the
integrand is positive the integral can be computed taking a single limit,
namely,

Z 1 Z b b
dx dx 1 x
= lim = lim arctan = ; a > 0:
1 x + a2 b!1
2
b
2
x +a 2 b!1 a a b a

17
VII.2.2
1 2 3 P L K

Show using residue theory that


Z 1
dx
= 3:
2 2 2 2a
1 (x + a )

Remark. Check the result by dierentiating the formula in the pre-


ceding exercise with respect to the parameter.

Solution

VII.2.2

z1

-R 1 R
z2

Set
Z 1
dx
I= ;
1 (x2 + a2 )2
and integrate
1 1
f (z) = =
(z 2 + a2 )2 (z ia) (z + ia)2
2

along the contour in Figure VII.2.2.


Residue at a double pole at z1 = ia, where by Rule 2,

1 d 1 2 i
Res 2 ; ia = lim 2 = = :
(z 2 + a2 ) z!ia dz (z + ia) (z + ia)3 z=ia
4a3

18
Integrate along 1,and let R ! 1. This gives
Z Z R Z 1
dx dx
f (z) dz = 2 ! = I:
(x 2 + a2 ) (x 2 +a2 )2
1 R 1

Integrate along 2, and let R ! 1. This gives


Z
1
f (z) dz R ! 0:
2
(R2 a2 )2 R3
Using the Residue Theorem and letting R ! 1, we obtain that that

i
I +0=2 i ;
4a3
and therefore
Z 1
dx
= :
1 (x2 + a2 )2 2a3
Remark.
From Exercise VII.2.1 we have the formula
Z 1
1
(1) 2 2
dx = ; a > 0:
1 x +a a
It is allowed to dierentiate both sides in the formula, because for every
compact interval with a 6= 0, both the integrand and its derivative are con-
tinuous, and the primitive of the derivative
R is uniformly bounded by a a
independent function h (x) such that h (x) < 1 on every such like interval.
We start by dierentiate the left hand side in the formula with respect to the
parameter a

Z 1 Z 1 Z 1
d 1 2a dx
(2) dx = dx = 2a ;
da 1 x + a2
2
1 (x + a2 )2
2
1 (x2 + a2 )2

and dierentiate the right hand side in the formula in the same way
d
(3) = :
da a a2
By (1) - (3), we have that

19
Z 1
dx
= :
1 (x2 + a2 )2 2a3

20
VII.2.3
Show using residue theory that
Z 1
x2 dx
2 = :
2
1 (x + 1) 2
Remark. Check the result by combining the preceding two exercises.

Solution

VII.2.3

z1

-R 1 R
z2

Set
Z 1
x2 dx
I= ;
1 (x2 + 1)2
and integrate

z2 z2
f (z) = =
(z 2 + 1)2 (z i)2 (z + i)2
along the contour in Figure VII.2.3.
Residue at a double pole at z1 = i, where by Rule 2,

z2 d z2 2z (z + i)2 2z 2 (z + i) i
Res ; i = lim = = :
(z 2 + 1)2 z!i dz (z 2 + i)2 (z + i)4 z=i
4

Integrate along 1, and let R ! 1. This gives

21
Z Z R Z 1
x2 dx x2 dx
f (z) dz = ! = I:
1 R (x2 + 1)2 1 (x2 + 1)2
Integrate along 2, and let R ! 1. This gives
Z
R2
f (z) dz R ! 0:
2
(R2 1)2 R
Using the Residue Theorem and letting R ! 1, we obtain that

i
I +0=2 i ;
4
and therefore
Z 1
x2 dx
= :
1 (x2 + 1)2 2
Remark.
We split the integrand using partial fraction, and put a = 1 in the formulas
from the preceding two exercises
Z 1 Z 1 Z 1
x2 dx dx dx
2 = 2 2 = = :
1 x +1 2 2
2 2
1 (x + 1) 1 (x + 1)

22
VII.2.4
Using residue theory, show that
Z 1
dx
4
=p :
1 x +1 2
Solution

VII.2.4

z2 z1

-R 1 z3 z4 R

Set
Z 1
dx
I= ;
1 x4+1
and integrate

1 1
f (z) = =
z4 +1 z 1+i
p z 1+i
p z 1 i
p z 1p i
2 2 2 2

along the contour in Figure VII.2.4.


Residue at a simple pole at z1 = 1+i
p , where by Rule 2,
2
p
1 1+i 1 2
Res 4 ; p = 3 = ( 1 i)
z +1 2 4z z=(1+i)=p2 8
1+i
Residue at a simple pole at z2 = p
2
, where by Rule 2,

23
p
1 1+i 1 2
Res 4 ; p = 3 p
= (1 i)
z +1 2 4z z=( 1+i)= 2 8
Integrate along 1, and let R ! 1. This gives
Z Z R Z 1
dx dx
f (z) dz = 4
! 4
= I:
1 R x +1 1 x +1

Integrate along 2, and let R ! 1. This gives


Z
1
f (z) dz 4
R ! 0:
2
R 1 R3
Using the Residue Theorem and letting R ! 1, we obtain that
p p !
2 2
I +0=2 i ( 1 i) + (1 i) ;
8 8
and therefore
Z 1
dx
=p :
1 x4+1 2

24
VII.2.5
Using residue theory, show that
Z 1
x2
dx = p :
0 x4 + 1 2 2

Solution

VII.2.5

z2 z1

-R 1 z3 z4 R

Set
Z 1 Z 1
x2 x2
I= dx ) 2I = dx
0 x4 + 1 1 x4 + 1
and integrate

z2 z2
f (z) = 4 =
z +1 z 1+i
p z 1+i
p z 1 i
p z 1p i
2 2 2 2

along the contour in Figure VII.2.5.


Residue at a simple pole at z1 = 1+i
p , where by Rule 3,
2
p
z2 1+i 1 2
Res 4 ; p = p
= (1 i) :
z +1 2 4z z=(1+i)= 2 8
1+i
Residue at a simple pole at z2 = p
2
, where by Rule 3,

25
p
z2 1+i 1 2
Res 4 ; p = p
= ( 1 i) :
z +1 2 4z z=( 1+i)= 2 8
Integrate along 1, and let R ! 1. This gives
Z Z R Z 1
x2 x2
f (z) dz = 4
dx ! 4
dx = 2I:
1 R x +1 1 x +1

Integrate along 2, and let R ! 1. This gives


Z
R2
f (z) dz R ! 0:
2
R4 1 R
Using the Residue Theorem and letting R ! 1, we obtain that
p p !
2 2
2I + 0 = 2 i (1 i) + ( 1 i) ;
8 8
and therefore
Z 1
x2
dx = p :
0 x4 + 1 2 2

26
VII.2.6
Show that
Z 1
x
dx = :
1 (x2 + 2x + 2) (x2 + 4) 10

Solution

VII.2.6

z1
z2

-R 1 R
z3
z4

Set
Z 1
x
I= dx
1 (x2 + 2x + 2) (x2 + 4)
and integrate

z z
f (z) = =
(z 2 + 2z + 2) (z 2 + 4) (z ( 1 + i)) (z 2i) (z ( 1 i)) (z + 2i)

along the contour in Figure VII.2.6.


Residue at a simple pole at z1 = 2i, where by Rule 3,

z z 1 1
Res ; 2i = 2 = i:
(z 2 2
+ 2z + 2) (z + 4) (z + 2z + 2) 2z z=2i 20 10

Residue at a simple pole at z2 = 1 + i, where by Rule 3,

27
z z 1 3
Res ; 1 + i = = + i:
(z 2 + 2z + 2) (z 2 + 4) (2z + 2) (z 2 + 4) z= 1+i 20 20

Integrate along 1, and let R ! 1. This gives

Z Z R
x
f (z) dz = dx !
R (x2 + 2x + 2) (x2 + 4)
1
Z 1
x
! 2 2
dx = I:
1 (x + 2x + 2) (x + 4)

Integrate along 2 , and let R ! 1. This gives


Z
R
f (z) dz 2
R ! 0:
2
(R 2R 2) (R2 4) R2
Using the Residue Theorem and letting R ! 1, we obtain that

1 1 1 3
I +0=2 i i+ + i
20 10 20 20
and therefore
Z 1
x
dx = :
1 (x2 + 2x + 2) (x2 + 4) 10

28
VII.2.7
1 2 3 P L K
LLL
Show that
Z 1 p
cos (ax) a= 2 a a
dx = p e cos p + sin p ; a > 0:
1 x4 + 1 2 2 2

Solution

VII.2.7

z2 z1

-R 1 z3 z4 R

Set
Z 1
cos (ax)
I= dx a > 0;
1 x4 + 1
and integrate

eiaz eiaz
f (z) = =
z4 + 1 z 1+i
p z 1+i
p z 1 i
p z 1p i
2 2 2 2

along the contour in Figure VII.2.7.


Residue at a simple pole at z1 = 1+i
p , where by Rule 3,
2

29
eiaz 1 + i eiaz
Res ; p = =
z4 + 1 2 4z 3 z=(1+i)=p2
p a=p2 p p a=p2 p p p
a= 2
p p p
a= 2
p
2e cos a= 2 2e sin a= 2 2e cos a= 2 2e sin a= 2
= i i+
8 8 8 8
1+i
Residue at a simple pole at z2 = p
2
, where by Rule 3,

eiaz
1+i eiaz
Res p =; =
z4 + 1
2 4z 3 p
z=( 1+i)= 2
p a= 2
p p p p
a= 2
p p p
a= 2
p p p
a= 2
p
2e cos a= 2 2e sin a= 2 2e cos a= 2 2e sin a= 2
= i i
8 8 8 8
Integrate along 1, and let R ! 1. This gives

Z Z R Z 1 Z 1 Z 1
eax cos (ax) sin (ax) sin (ax)
f (z) dz = dx ! dx+i dx = I+i dx
1 R x4 + 1 1 x4 + 1 1 x4 + 1 1 x4 + 1

Integrate along 2 , and let R ! 1. Because jeiaz j 1 in the upper half


plane if a > 0, this gives
Z
1
f (z) dz 4
R ! 0:
2
R 1 R3
Using the Residue Theorem and letting R ! 1, we obtain that

Z 1
p p
a= 2
p p p
a= 2
p !
sin (ax) 2e cos a= 2 2e sin a= 2
I+i dx = 2 i i i ;
1 x4 + 1 4 4

and therefore
Z 1 p
cos (ax) a= 2 a a
4
dx = p e cos p + sin p ; a > 0:
1 x +1 2 2 2

30
VII.2.8
Show that
Z 1
cos x
dx = :
1 (x2 + 1)2 e

Solution

VII.2.8

z1

-R 1 R
z2

Set
Z 1
cos x
I= dx
1 (x2 + 1)2
and integrate

eiz eiz
f (z) = =
(z 2 + 1)2 (z i)2 (z + i)2
along the contour in Figure VII.2.8.
Residue at a double pole at z1 = i, where by Rule 2,

eiz d eiz ieiz (z + i)2 2eiz (z + i) e 1


Res ; i = lim == = i
(z 2 + 1)2 z!i dz (z + i)2 (z + i)4 z=i
2

Integrate along 1, and let R ! 1. This gives

31
Z
f (z) dz =
1
Z 1 Z 1 Z 1
eix cos x sin x
= dx ! dx + i dx =
1 (x2 + 1)2 1 (x2 + 1)2 1 + 1)2
(x2
Z 1
sin x
=I +i 2 dx:
2
1 (x + 1)

Integrate along 2, and let R ! 1. Because jeiz j 1 in the upper half


plane, this gives
Z
1
f (z) dz R ! 0:
2
(R2 1)2 R3
Using the Residue Theorem and letting R ! 1, we obtain that
Z 1
sin x e 1
I +i 2 dx + 0 = 2 i i ;
2
1 (x + 1) 2
and therefore
Z 1
cos x
dx = :
1 (x2 + 1)2 e

32
VII.2.9
Show that
Z 1
sin2 x 1
dx = 1 :
1 x2 + 1 2 e2

Solution

VII.2.9

z1

-R 1 R
z2

Set
Z 1 Z 1
sin2 x 1 cos 2x
I= dx = dx
1 x2 + 1 1 2 (x2 + 1)
and integrate

1 ei2z 1 ei2z
f (z) = =
2 (z 2 + 1) 2 (z i) (z + i)
along the contour in Figure VII.2.9.
Residue at a simple pole at z1 = i, where by Rule 3,

1 ei2z 1 ei2z 1 2 1
Res ;i = = ie i
2 (z 2 + 1) 4z z=i 4 4
Integrate along 1 , and let R ! 1. This gives

33
Z
f (z) dz =
1
Z R Z 1 Z 1
1 ei2x 1 cos 2x sin 2x
= dx ! dx + i dx =
R 2 (x2 + 1) 1 2 (x2 + 1) 2
1 2 (x + 1)
Z 1
sin 2x
=I +i 2
dx
1 2 (x + 1)

Integrate along 2, and let R ! 1. Because jeiz j 1 in the upper half


plane, this gives
Z
2
f (z) dz R ! 0:
2
2(R2 1) R
Using the Residue Theorem and letting R ! 1, we obtain that
Z 1
sin 2x 1 2 1
I +i 2
dx + 0 = 2 i ie i ;
1 2 (x + 1) 4 4
and therefore
Z 1
sin2 x 1
dx = 1 :
1 x2 + 1 2 e2

34
VII.2.10
Show that
Z 1
cos (ax) e jajb
dx = ; 1 < a < 1; b > 0:
1 x 2 + b2 b
For which complex values of the parameters a and b does the in-
tegral exist? Where does the integral depend analytically on the
parameters?

Solution
The integral exists only for a real, and Re b 6= 0 or cos (iab) = 0. It depends
analytically on b for Re b 6= 0. It depends analytically on a in the intervals
1 < a < 0 and 0 < a < 1.
Case 1: a < 0

VII.2.10a

z1
1
-R R
z2

Set
Z 1
cos ax
I= dx; a < 0; b > 0
1 x 2 + b2
and integrate

eiaz eiaz
f (z) = =
z 2 + b2 (z ib) (z + ib)
along the contour in Figure 7.2.10a. (The lower half-plane)
Residue at a simple pole at z2 = ib, where by Rule 3,

35
eiaz eiaz eab
Res ; ib = = i
z 2 + b2 2z z= ib 2b
Integrate along 1 and let R ! 1. This gives

Z
f (z) dz =
1
Z R Z 1 Z 1
eiax cos (ax) sin (ax)
= dx ! dx + i dx =
R x 2 + b2 1 x 2 + b2 2
1 x +b
2
Z 1
sin (ax)
=I +i 2 2
dx
1 x +b

Integrate along 2 and let R ! 1. Because jeiaz j 1 in the lower half plane
if a < 0, this gives
Z
1
f (z) dz R ! 0:
2
R 2 b2 R
Using the Residue Theorem and letting R ! 1, we obtain that

eab
I +0= 2 i i ;
2b
and hence

eab
(1) I= ; a < 0:
b

Case 2: a > 0

36
VII.2.10b

z1

-R 3 R
z2

Set
Z 1
cos ax
I= dx; a > 0; b > 0;
1 x2 + b2
and integrate

eiaz eiaz
f (z) = =
z 2 + b2 (z ib) (z + ib)
along the contour in Figure VII.2.10b. (The upper half-plane)
Residue at a simple pole at z1 = ib, where by Rule 3,

eiaz eiaz e ab
Res ; ib = = i
z 2 + b2 2z z=ib 2b
Integrate along 3 and let R ! 1. This gives

Z
f (z) dz =
3
Z R Z 1 Z 1
eiax cos (ax) sin (ax)
= dx ! dx + i dx =
R x 2 + b2 1 x 2 + b2 2
1 x +b
2
Z 1
sin (ax)
=I +i 2 2
dx
1 x +b

37
Integrate along 4 and let R ! 1. Because jeiaz j 1 in the upper half plane
if a > 0, this gives
Z
1
f (z) dz R ! 0:
4
R 2 b2 R
Using the Residue Theorem and letting R ! 1, we obtain that

e ab
I +0=2 i i ;
2b
and hence

e ab
(2) I= ; a > 0:
b
By (1) and (2) we can concluede that
Z 1
cos (ax) e jajb
2 2
dx = ; 1 < a < 1; b > 0:
1 x +b b

38
VII.2.11
1 2 3 P L K

Evaluate
Z 1
cos x
dx:
1 (x2
+ a2 ) (x2 + b2 )
Indicate the range of the parameters a and b.

Solution
The integral converges absolutely if a2 ; b2 62 ( 1; 0], a; b 62 iR. It also
converges absolutely for certain points on the imaginary axis, so that ia
are zeros of cos x, ib are zeros of cos x, but they are not the same zeros.
The integrand, is an even function of a and b, so it su ces to evaluate it for
a; b in the right half-plane. For this, we can assume that a; b > 0, and extend
the formula by analyticity. We can also assume that a 6= b, and get other
case in this limit. So we may as well assume that 0 < a < b.

VII.2.11

z2
z1

-R 1 R
z3
z4

R1
The integral 1 (x2 +acos x 2 2
2 )(x2 +b2 ) dx converges absolutely if a ; b 2
= ( 1; 0] i.e.
a; b 2= iR. It also converges absolutely for certain points on imaginary axis,
so that ia are zeros of cos x, ib are zeros of cos x, but they are not same
zeros. (Problem should may be simplied to avoid thus points.)
The integrand, is an even function of a and b, so it su ces to evaluate it for
a; b in the right half-plane. For this, we can assume that a; b > 0, and extend
the formula by analyticity. We can assume that a 6= b, and get other case in
this limit. So we may as well assume 0 < a < b.

39
Case 1: (a; b > 0; a 6= b)
Set
Z 1
cos x
I= dx
1 (x2 + a2 ) (x2 + b2 )
and integrate

eiz eiz
f (z) = =
(z 2 + a2 ) (z 2 + b2 ) (z ia) (z ib) (z + ia) (z + ib)
along the contour in Figure VII.2.11.
Residue at a simple pole at z1 = ai, where by Rule 3,

eiz eiz e a
Res ; ia = =
(z 2 + a2 ) (z 2 + b2 ) 2z (z 2 + b2 ) z=ai 2a (b2 a2 ) i
Residue at a simple pole at z1 = ib, where by Rule 3,

eiz eiz e b
Res ; ib = =
(z 2 + a2 ) (z 2 + b2 ) 2z (z 2 + a2 ) z=bi 2b (b2 a2 ) i
Integrate along 1 and let R ! 1. This gives

Z Z R
eix
f (z) dz = 2 2 2 2
dx !
R (x + a ) (x + b )
1
Z 1 Z 1
cos x sin x
! 2 2 2 2
dx + i 2 2 2 2
dx =
1 (x + a ) (x + b ) 1 (x + a ) (x + b )
Z 1
sin x
=I +i 2 2 2 2
dx
1 (x + a ) (x + b )

Integrate along 2 and let R ! 1. Because jeiz j 1 in the upper half plane,
this gives
Z
1
f (z) dz R ! 0:
2
(R2 a2 ) (R2 b2 ) R3
Using the Residue Theorem and letting R ! 1, we obtain that
a b
e e
I +0=2 i ;
2a (b2 a2 ) i 2b (b2 a2 ) i

40
and therefore
Z 1
cos x be a ae b
dx = :
1 (x2 + a2 ) (x2 + b2 ) ab (b2 a2 )
Case 2: (a; b > 0; a = b)
From Case 1 we have

Z 1
cos x be a ae b (b a) e a
+a e a
e b
dx = = :
1 (x2 + a2 ) (x2 + b2 ) ab (b2 2
a) ab (b + a) (b a)

If we let b ! a, we get
Z 1
cos x
dx = 3 (1 + a) :
1 (x2 + a2 )2 2a
For a = 1, we have
Z 1
cos x
dx = ;
1 (x2 + 1)2 e
which thus yields the answer from Exercise 8.

41
VII.2.12
Let Q (z) be a polynomial of degree m with no zeros on the real
line, and let f (z) be a function that is analytic in the upper half-
plane and across the real line. Suppose there is b < m 1 such that
jf (z)j jzjb for z in the upper half-plane, jzj > 1. Show that
Z 1 X
f (x) f (z)
dx = 2 i Res ; zj ;
1 Q (x) Q (z)
summed over the zeros zj of Q (z) in the upper half-plane.

Solution

VII.2.12

2
zj - 1 z3
zj
z2 z1

-R 1 R

Set
Z 1
f (x)
I= dx
1 Q (x)
and integrate

f (z)
f (z) =
Q (z)
along the contour in Figure VII.2.12.
The sum of the residues at the poles z1 ; z2 ; : : : ; zj in the upper half plane,
X f (z)
Res ; zj :
Q (z)

42
Integrate along 1, and let R ! 1. This gives
Z Z R Z 1
f (x) f (x)
f (z) dz = dx ! dx = I:
1 R Q (x) 1 Q (x)
Integrate along 2, and let R ! 1. This gives

Z Z
z = Rei f Rei f Rei
f (z) dz = d R
2
0 0 jQ (Rei )j jQ (Rei )j
Rb
! 0:
Rm (C + O (1=R)) R (C + O (1=R)) CR

Using the Residue Theorem and letting R ! 1, we obtain that


X f (z)
I +0=2 i Res ; zj
Q (z)
and therefore
Z 1 X
f (x) f (z)
dx = 2 i Res ; zj
1 Q (x) Q (z)
Q.E.D.

43
VII.3.1
Show using residue theory that
Z 2
cos 2
d =2 1 p :
0 2 + cos 3

Solution

VII.3.1

|z| = 1

z3 z2 z1

Z 2
cos
I= d :
0 2 + cos
A change of variables as in book page 203-204 gives

Z I 1 I
2
cos 2
z + z1 dz 1 z2 + 1
I= d = = dz:
0 2 + cos jzj=1 2 + 12 z + 1
z
iz i 3 2
jzj=1 z + 4z + z

Integrate

z2 + 1 z2 + 1
f (z) = = p p
z 3 + 4z 2 + z z z 2 3 z 2+ 3
along the unit circle contour in Figure VII.3.1.
Residue at a simple pole at z1 = 0, where by Rule 3,

z2 + 1 z2 + 1
Res ; 0 = = 1:
z 3 + 4z 2 + z 3z 2 + 8z + 1 z=0

44
p
Residue at a simple pole at z2 = 2+ 3, where by Rule 3,
p
z2 + 1 p z2 + 1 2 3
Res 3 ; 2+ 3 = 2 = :
z + 4z 2 + z 3z + 8z + 1 p
z= 2+ 3 3
Using the Residue Theorem, we thus obtain
p !
1 2 3
I=2 i 1 ;
i 3
and therefore
Z 2
cos 2
d =2 1 p :
0 2 + cos 3

45
VII.3.2
Show using residue theory that
Z 2
d 2
=p ; a > b > 0:
0 a + b sin a 2 b2

Solution

VII.3.2 (a = 7, b = 6)

|z | = 1

z1

z2

Set
Z 2
1
I= d :
0 a + b sin
A change of variables as in book page 203-204 gives

Z 2 I I
1 1 dz 2
I= d = 1 1
= dz
0 a + b sin jzj=1 a+b 2i
z z
iz jzj=1 bz 2 + 2azi b

Integrate

2 2
f (z) = = p p
bz 2 + 2azi b b z a a2 b2
i z a+ a2 b2
i
b b

along the unit circle contour in Figure


p VII.3.2.
a+ a2 b2
Residue at a simple pole at z1 = b
i, where by Rule 3,

46
p
2 a+ a2 b2 2 i
Res 2
; i = p = p :
bz + 2azi b b 2bz + 2ai z= a+ a2 b2
i a2 b2
b

Using the Residue Theorem, we thus obtain

1
I=2 i p i ;
a2 b2
and therefore
Z 2
1 2
d =p ; a > b > 0:
0 a + b sin a2 b2

47
VII.3.3
Show using residue theory that
Z h p i
sin2
d = a a2 1 ; a > 1:
0 a + cos

Solution

VII.3.3 (a = 2)

|z| = 1

z3 z2 z1

Set

Z Z
sin2 =2 t sin2 (2 t)
I= d = = dt =
0 a + cos d = dt 2 a + cos(2 t)
Z 2 Z 2
sin2 (2 t) sin2 t
= dt = dt:
a + cos(2 t) a + cos t

We have
Z 2
sin2
2I = d :
0 a + cos
A change of variables as in book page 203-204 gives

Z 2 I 1 1 2 I 2
sin2 2i
z z dz 1 (z 2 1)
2I = d = 1 1
= dz:
0 a + cos jzj=1 a+ 2
z+ z
iz 2i jzj=1 z 2 (z 2 + 2az + 1)

48
Integrate

2 2
(z 2 1) (z 2 1)
f (z) = 2 2 = p p
z (z + 2az + 1) z2 z a + a2 1 z a a2 1

along the unit circle contour in Figure VII.3.3.


Residue at a double pole at z1 = 0 , where by Rule 2,
" #
2 2
(z 2 1) d (z 2 1)
Res 2 2 ; 0 = lim = 2a:
z (z + 2az + 1) z!0 dz (z 2 + 2az + 1)

p
Residue at a simple pole at z2 = a + a2 1, where by Rule 3,

" 2
# 2
2 2
(z 2 1) =z p (z 2 1) =z p
Res 2 ; a + a2 1 = = 2 a2 1:
z + 2az + 1 2z + 2a p
z= a+ a2 1

Using the Residue Theorem, we thus obtain

1 p
2I = 2 i 2a + 2 a2 1 ;
2i
and therefore
Z p
sin2
d = a a2 1 ; a > 1:
0 a + cos

49
VII.3.4
Show using residue theory that
Z p
d
= 2:
1 + sin2

Solution

VII.3.4

|z | = 1

z4 z3 z1 z2

Set

Z
1 = t
I= d = =
1 + sin2 d = dt
Z 0 Z 2
1 1
= 2 dt = dt:
2 1+ sin ( t) 0 1+ sin2 t

We have
Z 2
1
I= d :
0 1 + sin2
A change of variables as in book page 203-204 gives

Z 2 I I
1 1 dz 4 z
I= d = = dz:
0 1 + sin2 jzj=1 1+ 1
z 1 2
iz i jzj=1 z4 6z 2 + 1
2i z

50
Integrate

z
f (z) = =
z4 6z 2 + 1
z
= p p p p
z 2 1 z 2+1 z 2+1 z 2 1

along the unit circle contour in Figure VII.3.4


p
Residues at a simple poles at z1;2 = 2 1 , where by Rule 3,

z p z 1
Res ; 2 1 = 3 = p :
z4 6z 2 + 1 4z 12z z=
p
( 2 1) 8 2
Using the Residue Theorem, we thus obtain

4 1 1
I=2 i p p ;
i 8 2 8 2
and therefore
Z p
1
d = 2:
1 + sin2

51
VII.3.5
Show using residue theory that
Z
1 r2 d
2
= 1; 0 r < 1:
1 2r cos + r 2

Solution

VII.3.5 (r = 1/2)

|z | = 1

z2 z1

Set

Z
1 r2 d = t
I= = =
1 2
2r cos + r 2 d = dt
Z 0
1 r2 dt
= 2
=
2 1 2r cos ( t) + r 2
Z 2
1 r2 dt
= 2
0 1 + 2r cos t + r 2
A change of variables as in book page 203-204 gives

52
Z 2
1 r2 dt
I= =
0 1 + 2r cos t + r2 2
I
1 1 r2 dz
= =
2 jzj=1 1 + 2r 21 z + z1 + r2 iz
I
1 1 r2
= dz:
2 i jzj=1 (r + z) (rz + 1)

Integrate

1 r2 1 r2
f (z) = =
(r + z) (rz + 1) r (z + r) (z + 1=r)

along the unit circle contour in Figure VII.3.4


Residue at a simple pole at z1 = r, where by Rule 3,

1 r2 1 r2
Res ; r = = 1:
r (z + r) (z + 1=r) 2rz + r2 + 1
z= r

Using the Residue Theorem, we thus obtain


1
I=2 i (1) ;
2 i
and therefore
Z
1 r2 d
2
= 1; 0 r < 1:
1 2r cos + r 2

53
VII.3.6
By expanding both sides of the identity
Z 2
1 d 1
=p
2 0 w + cos w 2 1
in a power series at 1, show that
Z 2
1 (2k)!
cos2k d = ; k 0:
2 0 22k (k!)2

Solution
Rewrite both sides in the given identity
Z 2 1=2
1 d 1
(1) I= = 1 ;
2 0 1 + cos
w
w2
where the integrand in the left side in the identity can be rewritten as

1 X 1
k cos
k
= ( 1) :
1 + cos
w k=0
wk
Now, we work on the left side of the identity

Z 2 X Z
1 1 k cos
k
1 X1 ( 1)k 2
I= ( 1) d = cosk d :
2 0 k=0 wk 2 k=0 w k 0
R2
Because 0 cosk d = 0 if k odd, we have
X1 Z 2
1 1
(2) I =1+ cos2k d :
k=1
2 0 w2k
Now, we use the power series expansion (MH p. 192.) for real and 1<
x < 1,

( 1) ( 1) ( 2)
(1 + x) = 1+ x+ x2 + x3 + + xn + ;
2! 3! n

we have

54
1=2
1
1 =
w2
1 1 1 3 2 1 3 5 3
2 1 2 2 1 2 2 2 1
=1+ + + +
1! w2 2! w2 3! w2
1 3 5 7 4 1 3 5 7 1 2k k
2 2 2 2 1 2 2 2 2 2 1
+ + + + =
4! w2 k! w2k
X1 1 3 (2k 1) 1
=1+ =
k=1 2k k! w2k
X1 1 2 3 4 5 : : : (2k 1) 2k 1
=1+ =
k=1 2k k! 2 4 6 : : : 2k w2k
X1 1 2 3 4 5 : : : (2k 1) 2k 1
1+ =
k=1 2k k! 2k (1 2 3 : : : k) w2k
X1 (2k)! 1 X1 (2k)! 1
=1+ = 1 + ;
k k
k=1 2 k!2 k! w 2k k=1 22k (k!)2 w 2k

i.e.,

1
1=2 X1 (2k)! 1
(2) 1 =1+ :
w2 k=1 22k (k!)2 w2k
Comparing the two power series expressions (2) and (3) of the both sides in
the identity (1) gives us together with the uniqueness theorem for the power
series
Z 2
1 (2k)!
cos2k d = ; k 0:
2 0 22k (k!)2

55
VII.3.7
Show using residue theory that
Z 2
d 2 w
2 = ; w 2 C n [ 1; 1] :
0 (w + cos ) (w2 1)3=2
Specify carefully the branch of the power function. Check you
answer by dierentiating the integral of 1= (w + cos ) with respect
to the parameter w.

Solution

VII.3.7 (w = 2)

|z| = 1

z2 z1

Set
Z 2
d
I=
0 (w + cos )2
A change of variables as in book page 203-204 gives

Z 2 Z 2 I
d 1 dz 4 z
I= = = 2 dz:
0 (w + cos )2 0 w+ 1
2
z+ 1
z
2
iz i 2
jzj=1 (z + 2wz + 1)

4z
Integrate f (z) = (z 2 +2wz+1)2 along the unit circle contour in Figure VII.3.7.
p
We choose branch of w 2
1 on Cn [ 1; 1] that is positive for w 2 (1; 1).
It su ces to check the identity for w > p
1.
Residue at a double pole at z1 = w + w2 1, where by Rule 2,

56
z p
Res 2 ; w + w2 1 =
(z 2 + 2wz + 1)
d z
= lim p p 2 =
z! w+ w2 1 dz z w w2 1
p 2 p
z w w2 1 2z z w w2 1
= p 4 =
z w w2 1 p
z= w+ w 1 2
p
z w w 2 1 2z
= p 3 =
z w w2 1 p
z= w+ w2 1
1 w
= :
4 (w 2 1)3=2

Using the Residue Theorem, we thus obtain


!
4 1 w
I=2 i ;
i 4 (w2 1)3=2
and therefore
Z 2
d 2 w
2 = ; w 2 C n [ 1; 1] :
0 (w + cos ) (w2 1)3=2
Check.
From Exercise VII.3.6 we have the formula
Z 2
1 d 1
(1) =p
2 0 w + cos w 2 1
It is allowed to dierentiate both sides in the formula, because for every
compact interval with w 2 C n [ 1; 1], both the integrand and its derivative
are continuous, and that the primitive of the derivative
R is uniformly bounded
by an w independent funtion h ( ) such that h ( ) d < 1 on every such
like interval on w.
We start by dierentiating the left hand side in the formula with respect to
the parameter w

57
Z 2 Z 2
d 1 d 1 d
(2) = ;
dw 2 0 w + cos 2 0 (w + cos )2
and dierentiate the right hand side in the formula in the same way
d 1 w
(3) p = :
dw w 2 1 (w2 1)3=2
By (1) - (3), we thus have that
Z 2
d 2 w
2 = :
0 (w + cos ) (w 2 1)3=2

58
VII.4.1
By integrating around the keyhole contour, show that
Z 1
x a
dx = ; 0 < a < 1:
0 1+x sin ( a)
Solution

VII.4.1

1
z1
-R 4 3 R

Set
Z 1
x a
I= dx
0 1+x
and integrate

z a jzj a e ia arg z
f (z) = =
1+z 1+z
a
along the keyhole contour in Figure VII.4.1. We make a branchcut for z
along the positive real axis, where 0 < arg z < 2 .
Residue at a simple pole at z1 = 1, where by Rule 3,
a
jzj e ia arg z a ia arg z ia
Res ; 1 = jzj e z= 1
=e :
1+z

59
Integrate along 1, and let R ! 1 and " ! 0+ . This gives

Z
f (z) dz =
1
Z a Z R
jzj e ia arg z z = xe0i x a
dz = = dx !
1+z dz = dx " 1+x
1
Z 1
x a
! dx = I:
0 1+x
Integrate along 2 and let R ! 1. This gives then 0 < a < 1
Z
R a 2
f (z) dz 2 R ! 0:
2
R 1 Ra
Integrate along 3, and let R ! 1 and " ! 0+ . This gives

Z
f (z) dz =
3
Z a Z "
jzj e ia arg z z = xe2 i x a e 2 ia
= dz = = dx !
3
1+z dz = dx R 1+x
Z 0 a 2 ia Z 1
x e x a
! dx = e 2 ia
dx = e 2 ia
I:
1 1 + x 0 1+x

Integrate along 4, and let " ! 0+ . This gives then 0 < a < 1
Z
" a
f (z) dz 2 " 2 "1 a
! 0:
4
1 "
Using the Residue Theorem and letting R ! 1 and " ! 0+ , we obtain that

2 ia ia
I +0 e I +0=2 i e :
ia
Multiplying with e , this yields

ia ia
e e I = 2 i;
and hence solving for I,

60
2 i
I= ia ia
= ;
e e sin ( a)
i.e.,
Z 1
x a
dx = ; 0 < a < 1:
0 1+x sin ( a)

61
VII.4.2
By integrating around the boundary of a pie-slice domain of aper-
ture 2 =b, show that
Z 1
dx
= ; b > 1:
0 1 + xb b sin ( =b)
Remark. Check the result by changing variable and comparing with Exercise
1.

Solution

VII.4.2 (b = 12)

3 2

z1
1 R

Set
Z 1
dx
I= ;
0 1 + xb
and integrate
1 1
f (z) = =
1+z b
1 + jzjb eib arg z
along the pie-slice of aperture 2 =b contour in Figure VII.4.2. We make a
branch cut for z b along the negative imaginary axis, where =2 < arg z <
3 =2.
Residue at a simple pole at z1 = e i=b , where by Rule 3,

1 i=b 1 1 i=b
Res ;e = = e :
1 + zb bz b 1
z=e i=b b

62
Integrate along 1, and let R ! 1 and " ! 0+ . This gives

Z
f (z) dz =
1
Z Z R
1 z = xe0i 1
= dz = = dx !
1+ jzjb eib arg z dz = dx " 1+x
b
1
Z 1
1
! dx = I:
0 1 + xb
Integrate along 2 and let R ! 1. This gives then b > 1
Z
1 2 R 2
f (z) dz ! 0:
2
Rb 1 b Rb 1

Integrate along 3, and let R ! 1 and " ! 0+ . This gives

Z
f (z) dz =
3
Z Z "
1 z = xe2 i=b 1
= dz = = e2 i=b
dx !
3 1+ jzjb eib arg z dz = e2 i=b dx R 1 + xb
Z 0 Z 1
1 1
! e2 i=b
dx = e 2 i=b
dx = e2 i=b
I:
1 1 + xb 0 1 + xb

Integrate along 4, and let " ! 0+ . This gives then b > 1


Z
1 2 " 2 "
f (z) dz b
! 0:
4
1 " b b
Using the Residue Theorem and letting R ! 1 and " ! 0+ , we obtain that

1
I +0 e2 i=b
I +0=2 i e i=b
:
b
i=b
Multiplying with e , this yields

i=b i=b 2 i
e e I= ;
b
and hence solving for I,

63
2 i
I= i=b i=b )
=
(e e b sin ( =b)
i.e.,
Z 1
dx
= ; b > 1:
0 1 + xb b sin ( =b)

Remark.
We begin with the rst integral I, we make the substitution xb = t and use
the integral from Exercise VII.4.1 where we choose a = 1 1=b. (Because
b > 1, then 0 < 1 1=b < 1 and the restriction that 0 < a < 1 for the integral
in Exercise VII.4.1 is satised.)

2 3
Z 1 xb = t
dx
I= dx = 4 x = t1=b 5=
0 1 + xb 1 1=b 1
dx = b t dt
Z 1 1=b 1 Z
1t 1 1 t (1 1=b)
= dt = dt =
0 b 1+t b 0 1+t
1
= = :
b sin ( (1=b 1)) b sin ( =b)

64
VII.4.3
By integrating around the keyhole contour, show that
Z 1 2
log x cos ( a)
dx = ; 0 < a < 1:
0
a
x (x + 1) sin2 ( a)
Remark. Check the result by dierentiating the identity in Exercise 1.

Solution

VII.4.3

1
z1
-R 4 3 R

Set
Z 1
log x
I= ;
0 xa (x + 1)
and integrate

log x log jzj + i arg z


f (z) = = a ia arg z
xa (x + 1) jzj e (z + 1)
along the keyhole contour in Figure VII.4.3, where the cimicircle 4 have
radius ". We make a branch cut for log z along the positive real axis, so that
0 < arg z < 2 .
Residue at a simple pole at z1 = 1, where by Rule 3,

65
log jzj + i arg z log jzj + i arg z ia
Res a ia arg z ; 1 = = ie :
jzj e (z + 1) jzja eia arg z z= 1

Integrate along 1, and let R ! 1 and " ! 0+ . This gives

Z
f (z) dz =
1
Z ZR
log jzj + i arg z z = xe0i log x
= dz = = dx !
jzja eia arg z (z + 1) dz = dx a
" x (x + 1)
1
Z 1
log x
! a
dx = I:
0 x (x + 1)

Integrate along 2and let R ! 1. This gives then 0 < a < 1


q
Z log2 R + (2 )2 2 log R
f (z) dz a (R
2 R ! 0:
2
R 1) R
Integrate along 3, and let R ! 1 and " ! 0+ . This gives

Z
f (z) dz =
3
Z Z "
log jzj + i arg z z = xe2 i log x + 2 i
= a ia arg z dz = = !
3
jzj e (z + 1) dz = dx R x e
a 2 ia (x + 1)
Z 0
log x + 2 i
! a 2 ia (x + 1)
dx =
1 x e
Z 1 Z 1
2 ia log x 2 ia dx
= e a
dx 2 ie a
=
0 x (x + 1) 0 x (x + 1)
= e 2 ia I 2 ie 2 ia J

Integrate along 4,and let " ! 0+ . This gives then 0 < a < 1
Z p
log2 " + 2
f (z) dz a
2 " 2 "1 a jlog "j ! 0:
4
" (1 ")
Using the Residue Theorem and letting R ! 1 and " ! 0+ , we obtain that

66
2 ia 2 ia ia
I +0 e I 2 ie J +0=2 i ie :
ia
Multiplying with e , this yields

ia ia ia
e e I 2 ie J= 2 2;
and thus

ia
2i sin ( a) I 2 ie J= 2 2:
Separating real and imaginary parts, we get simultaneous equations
2
2 sin ( a) J = 2
2 sin ( a) I 2 cos ( a) J = 0;
and hence the solutions

Z 1 2 Z 1
log x cos ( a) dx
I= dx = ; J= = ;
0
a
x (x + 1) sin2 ( a) 0 xa (x + 1) sin ( a)
i.e.
Z 1 2
log x cos ( a)
dx = ; 0 < a < 1:
0
a
x (x + 1) sin2 ( a)

Remark.
From the Exercise VII.4.1 we have the formula
Z 1
x a
(1) dx = ; 0 < a < 1:
0 1+x sin ( a)
It is allowed to dierentiate both sides in the formula, because for every
compact interval with 0 < a < 1, both the integrand and its derivative is
continuous. And that the primitive of the derivative is uniformly limited by
a constant on every compact interval on a, there 0 < a < 1.
We begin to dierentiate the left side in the formula with respect to the
parameter a

Z 1 Z 1 Z 1
d x a log x x a
log x
(2) dx = dx = dx;
da 0 1+x 1 1+x 1 xa (x + 1)

67
and dierentiate the right hand side in the formula in the same way

d cos ( a)
(3) = :
da sin ( a) sin2 ( a)
By (1) - (3), we have that
Z 1 2
log x cos ( a)
dx = :
0 xa (x + 1) sin2 ( a)

68
VII.4.4
For xed m 2, show that by integrating around the keyhole con-
tour that
Z 1
x a a (a + 1) (a + m 2)
m dx = ; 1 m < a < 1:
0 (1 + x) (m 1)! sin ( a)
Remark. The result can be obtained also by integrating the formula in Ex-
ercise 1 by parts.

Solution

VII.4.4

1
z1
-R 4 3 R

Set
Z 1
x a
I= dx
0 (1 + x)m
and integrate

z a jzj a e ia arg z
f (z) = =
(1 + z)m (1 + z)m
a
along the keyhole contour in Figure VII.4.4. We make a branchcut for z
along the positive real axis, where 0 < arg z < 2 .

69
From Sa/Snider page 310 we have

1 dm 1
Res [f (z); z0 ] = lim [(z z0 )m f (z)] :
z!z0 (m 1)! dz m 1

Residue at a pole of order m at z1 = 1

z a 1 dm 1
Res m ; 1 = lim z a =
(1 + z) z! 1 (m 1)! dz m 1
1
= lim ( a) ( a 1) ( a m + 2) z ( a (m 1)) =
z! 1 (m 1)!
1
= lim ( 1)m 1 a (a + 1) (a + m 2) jzj( a (m 1)) ei( a (m 1)) arg z =
z! 1 (m 1)!
1
= ( 1)m 1 a (a + 1) (a + m 2) j 1j( a (m 1)) ei( a (m 1)) =
(m 1)!
1
= ( 1)m 1 a (a + 1) (a + m 2) ( 1)m 1 e ia =
(m 1)!
a (a + 1) (a + m 2)
= e ia :
(m 1)!

Integrate along 1, and let R ! 1 and " ! 0+ . This gives

Z Z Z
jzj a e ia arg z z = xe0i R
x a
f (z) dz = dz = = dx !
(1 + z)m dz = dx " (1 + x)m
1 1
Z 1
x a
! dx = I:
0 (1 + x)m

Integrate along 2 and let R ! 1. This gives then 1 m < a < 1, where
m 2
Z
R a 2
f (z) dz 2 R ! 0:
2
(R 1)m Rm+a 1
Integrate along 3, and let R ! 1 and " ! 0+ . This gives

70
Z
f (z) dz =
3
Z Z " a 2 ia
jzj a e ia arg z z = xe2 i x e
m dz = = m dx !
3
(1 + z) dz = dx R (1 + x)
Z 0 a 2 ia Z 1
x e 2 ia x a
! m dx = e m dx = e 2 ia
I:
1 (1 + x) 0 (1 + x)

Integrate along 4, and let " ! 0+ . This gives then 1 m < a < 1, where
m 2
Z a
"
f (z) dz 2 " 2 "1 a
! 0:
4
(1 ")m
Using the Residue Theorem and letting R ! 1 and " ! 0+ , we obtain that

2 ia a (a + 1) (a + m 2) ia
I +0 e I +0=2 i e :
(m 1)!
ia
Multiplying with e , this yields

ia ia a (a + 1) (a + m 2)
e e I=2 i ;
(m 1)!
and hence solving for I,

2i a (a + 1) (a + m 2) a (a + 1) (a + m 2)
I= ia ia )
= ;
(e e (m 1)! (m 1)! sin ( a)
i.e.
Z 1
x a a (a + 1) (a + m 2)
m dx = ; 1 m < a < 1:
0 (1 + x) (m 1)! sin ( a)

Remark.
Set (this is our induction hypothesis)
Z 1
x a a (a + 1) (a + m 2)
Im (a) = m dx =
0 (1 + x) (m 1)! sin ( a)

71
From Exercise VII.4.1 we have
Z 1
x b
dx = ; 0 < b < 1:
0 1+x sin ( b)
Use Exercise VII.4.1 and integrate by part

Z 1 Z 1
x b 1
= dx = x b dx =
sin ( b) 0 1+x 0 1+x
1 Z 1 b+1
x b+1 1 x 1
= dx =
1 b 1+x 0 0 1 b (1 + x)2
Z 1
1 x b+1
= dx:
1 b 0 (1 + x)2

Set b = a + 1 and solve for the integral in the left side in the inequality above
Z 1
x a a
2 dx = a = ;
0 (1 + x) sin ( (a + 1)) sin ( a)
that is the same as I2 (a), thus our induction hypothesis is valid for m = 2.
Now suppose that our formula is valid for m = p, and integrate by parts,

b (b + 1) (b + p 2)
=
(p 1)! sin ( b)
Z 1 Z 1
x b 1
= p dx = x b dx =
0 (1 + x) 0 (1 + x)p
1 Z 1 b+1
x b+1 1 x p
= p dx =
1 b (1 + x) 0 0 1 b (1 + x)p+1
Z 1
p x b+1
= dx:
1 b 0 (1 + x)p+1

Set b = a + 1 and solve for the integral in the left side in the inequality above

72
Z 1
x a
dx =
0 (1 + x)p+1
a (a + 1) (a + 2) ((a + 1) + p 2)
= =
p (p 1)! sin ( (a + 1))
a (a + 1) (a + 2) (a + (p + 1) 2)
= ;
p! sin ( a)

that is the same as Ip+1 (a), thus our induction hypothesis is valid for m =
p + 1.
We have showed that the formula Im (a) is valid for all m > 2, that was to
be shown.

73
VII.4.5
(log z)2
By integrating a branch of (z+a)(z+b)
around the keyhole contour,
show that
Z 1
log x (log a)2 (log b)2
dx = ; a; b > 0; a 6= b:
0 (x + a) (x + b) 2 (a b)

Solution

VII.4.5 (a = 1, b = 2)

1
z2 z1
-R 4 3 R

Set
Z 1
log x
I= dx
0 (x + a) (x + b)
and integrate

(log z)2 (log jzj + i arg z)2


f (z) = =
(z + a) (z + b) (z + a) (z + b)
around the keyhole contour in Figure VII.4.5. We make a branchcut for log z
along the positive real axis, where 0 < arg z < 2 .
Residue at a simple pole at z1 = a, where by Rule 3,

74
" #
(log jzj + i arg z)2 (log jzj + i arg z)2 (log a + i )2
Res ; a = = :
(z + a) (z + b) (z + b) b a
z= a

Residue at a simple pole at z1 = b, where by Rule 3,

" #
(log jzj + i arg z)2 (log jzj + i arg z)2 (log b + i )2
Res ; b = = :
(z + a) (z + b) (z + a) a b
z= b

Integrate along 1, and let R ! 1 and " ! 0+ . This gives

Z
f (z) dz =
1
Z Z
(log jzj + i arg z)2 z = xe0i R
(log x)2
= dz = = dx !
1
(z + a) (z + b) dz = dx " (x + a) (x + b)
Z 1
(log x)2
! dx = J:
0 (x + a) (x + b)
Integrate along 2 and let R ! 1. This gives
Z
log2 R + (2 )2 4 log2 R
f (z) dz 2 R ! 0:
2
(R a) (R b) R
Integrate along 3, and let R ! 1 and " ! 0+ . This gives

Z
f (z) dz =
3
Z Z "
(log jzj + i arg z)2 z = xe2 i (log x + 2 i)2
= dz = = dx !
3
(z + a) (z + b) dz = dx R (x + a) (x + b)
Z 0 Z 1
(log x + 2 i)2 (log x + 2 i)2
! dx = dx =
1 (x + a) (x + b) 0 (x + a) (x + b)
Z 1 Z 1 Z 1
(log x)2 log x 4 2
= dx 4 i + dx =
0 (x + a) (x + b) 0 (x + a) (x + b) 0 (x + a) (x + b)
= J 4 iI+K

75
Integrate along 4, and let " ! 0+ . This gives
Z
log2 " + (2 )2 2 " log2 "
f (z) dz 2 " ! 0:
4
(a ") (b ") ab
Using the Residue Theorem and letting R ! 1 and " ! 0+ , we obtain that

!
(log jbj + i )2 (log jaj + i )2
J +0 J 4 iI+K + 0 = 2 i + ;
b a a b

and hence, setting imaginary parts equal,

(log a)2 (log b)2


I= ;
2 (a b)
i.e.
Z 1
log x (log a)2 (log b)2
dx = ; a; b > 0; a 6= b:
0 (x + a) (x + b) 2 (a b)

76
VII.4.6
Using residue theory, show that
Z 1 a
x log x sin ( a) a 2 cos ( a)
dx = ; 1 < a < 1:
0 (1 + x)2 sin2 ( a)

Solution

VII.4.6

1
z1
-R 4 3 R

Set
Z 1
xa log x
I= dx
0 (1 + x)2
and integrate

z a log z jzja eia arg z (log jzj + i arg z)


f (z) = =
(1 + z)2 (1 + z)2
along the keyhole contour in Figure VII.4.6. We make a branchcut for z a
and log z along the real axis, where 0 < arg z < 2 .
Residue at a double pole at z1 = 1, where by Rule 3,

77
jzja eia arg z (log jzj + i arg z)
Res ; 1 =
(1 + z)2
d a az a log z z a
= (z log z) = + =
dz z= 1 z z z= 1
a
z
= (a log z + 1) =
z z= 1
jzja eia arg z
= (a (log jzj + i arg z) + 1) =
z z= 1
ia
=e ( 1 ai) :

Integrate along 1, and let R ! 1 and " ! 0+ . This gives

Z
f (z) dz =
1
Z Z
jzja eia arg z (log jzj + i arg z) z = xe0i R
xa log x
= dz = = 2 dx !
(1 + z)2 dz = dx " (1 + x)
1
Z 1
xa log x
! dx = I:
0 (1 + x)2

Integrate along 2 and let R ! 1. This gives


q
Z Ra log2 R + (2 )2 2 log R
f (z) dz 2 R ! 0:
2
(R 1)2 R1 a
Integrate along 3, and let R ! 1 and " ! 0+ . This gives

78
Z
f (z) dz =
3
Z Z " a 2 ia
jzja eia arg z (log jzj + i arg z) z = xe2 i x e (log jxj + 2 i)
= 2 dz = = dx !
3
(1 + z) dz = dx R (1 + x)2
Z 0 a 2 ia Z 1 a
x e (log jxj + 2 i) 2 ia x (log jxj + 2 i)
! 2 dx = e dx =
1 (1 + x) 0 (1 + x)2
Z 1 a Z 1
2 ia x log x 2 ia xa dx
= e dx 2 ie =
0 (1 + x)2 0 (1 + x)2
= e2 ia I 2 ie2 ia J

Integrate along 4, and let " ! 0+ . This gives


q
Z " log2 " + (2 )2
a
f (z) dz 2 " 2 "1+a jlog "j ! 0:
4
(1 ")2
Using the Residue Theorem and letting R ! 1 and " ! 0+ , we obtain that

I +0 e2 ia
I 2 ie2 ia
J +0=2 i e ia
( 1 ai) ;
ia
multiplying with e , this yields

ia ia ia
e e I 2 ie J = 2 i( 1 ai) ;
and thus

ia
2i sin ( a) I 2 ie J = 2 2a 2 i:
Separating real and imaginary parts, we get simultaneous equations

2 sin ( a) J = 2 2 a
2 sin ( a) I 2 cos ( a) J = 2 ;
and hence the solutions

Z 1 Z 1
xa log x sin ( a) a 2 cos ( a) xa dx a
I= = ; J= 2 = ;
0 (1 + x)2 sin2 ( a) 0 (1 + x) sin ( a)

79
i.e.
Z 1
xa log x sin ( a) a 2 cos ( a)
dx = ; 1 < a < 1:
0 (1 + x)2 sin2 ( a)

80
VII.4.7
Show that
Z 1
xa 1
dx = ; 0 < a < b:
0 1 + xb b sin ( a=b)
Determine for which complex values of the parameter a the integral exists
(in the sense that the integral of the absolute value is nite), and evaluate
it. Where does the integral depend analytically on the parameter a?

Solution

VII.4.7 (b = 12)

3 2

z1
1 R

Set
Z 1
xa 1
I= ;
0 1 + xb
and integrate

za 1 jzja 1 ei(a 1) arg z


f (z) = =
1 + zb 1 + jzjb eib arg z
along the pie-slice, of aperture 2 =b, contour in Figure VII.4.7. We make
a branch cut for z a 1 along the negative imaginary axis , where =2 <
arg z < 3 =2.
Residue at a simple pole at z1 = e i=b , where by Rule 3,

81
za 1 i=b za 1
1 ia=b
Res ;e = b = e :
1 + zb bz 1
z=e i=b b
Integrate along 1, and let R ! 1 and " ! 0+ . This gives

Z
f (z) dz =
1
Z Z
jzja 1 i(a 1) arg z
e z = xe0i R
xa 1
= dz = = dx !
1 1 + jzjb eib arg z dz = dx " 1 + xb
Z 1
xa 1
! dx = I:
0 1 + xb
Integrate along 2 and let R ! 1. This gives then 0 < a < b
Z
Ra 1 2 R 2
f (z) dz ! 0:
2
Rb 1 b bRb a

Integrate along 3, and let R ! 1 and " ! 0+ . This gives

Z
f (z) dz =
3
Z Z
jzja 1 i(a 1) arg z
e z = xe2 i=b "
xa 1 e2 i(a 1)=b
= = = e2 i=b
dx !
3 1+ jzjb eib arg z dz = e2 i=b dx R 1 + xb
Z 0 Z 1
xa 1 e2 i(a 1)=b
2 i=b 2 ia=b xa 1
! e dx = e dx = e2 ia=b
I:
1 1 + xb 0 1 + xb

Integrate along 4, and let " ! 0+ . This gives then 0 < a < b
Z
"a 1 2 " 2 "a
f (z) dz ! 0:
4
1 "b b b
Using the Residue Theorem and letting R ! 1 and " ! 0+ , we obtain that

1
I +0 e2 ia=b
I +0=2 i e ia=b
;
b
ia=b
and multiplying with e , this yields

82
ia=b ia=b 2 i
e e I=
b
and hence solving for I,
2 i
I= ia=b ia=b )
= ;
b (e e b sin ( a=b)
i.e.,
Z 1
xa 1
dx = ; 0 < a < b:
0 1 + xb b sin ( a=b)
Now we suppose a and b complex and rewrite the nominator in the integrand

xa 1
= e(a 1) log x
= e(Re a 1) log x i Im a log x
e = xRe a 1 ei Im a log x :
The absolute value of the integrand is

xa 1 xRe a 1 ei Im a log x xRe a 1


= = ;
1 + xb 1 + xb 1 + xb
thus Re a 1 > 1 and Re a 1 b < 1. Thus the integrand depends
analytically on the parameter a when 0 < Re a < b.

83
VII.4.8
By integrating a branch of (log z) = (z 3 + 1) around the boundary of
an indented sector of aperture 2 =3, show that
Z 1 Z 1
log x 2 2 1 2
dx = ; dx = p :
0 x3 + 1 27 0 x3 + 1 3 3
Remark. Compare the results with those of Exercise 3 (after changing vari-
ables) and Exercise 2.

Solution

VII.4.8

2
3
z1
4
z2
1 R
z3

Set
Z 1 Z 1
log x 1
I= dx; J= dx
0 x3 + 1 0 x3 +1
and integrate

log z log jzj + i arg z


f (z) = =
z3 + 1 z e i=3
(z e i ) z e5 i=3

along the indented sector, of aperture 2 =3, contour in Figure VII.4.8. We


make a branchcut for log z along the negative imaginary axis, where =2 <
arg z < 3 =2.
Residue at a simple pole at z1 = e i=3 , where by Rule 3,

84
log jzj + i arg z i=3 log jzj + i arg z i 2 i=3
Res ;e = = e :
z3 + 1 3z 2 z=e i=3 9
Integrate along 1, and let R ! 1 and " ! 0+ . This gives

Z
f (z) dz =
1
Z Z R
log jzj + i arg z z = xe0i log x
= dz = = dx !
z3 + 1 dz = dx 3
" x +1
1
Z 1
log x
! dx = I:
0 x3 + 1
Integrate along 2,
and let R ! 1. This gives
q
Z log2 R + 23
2
2 R 2 log R
f (z) dz 3
! 0:
2
R 1 3 3R2
Integrate along 3, and let R ! 1 and " ! 0+ . This gives

Z
f (z) dz =
3
Z Z "
log jzj + i arg z z = xe2 i=3 log x + 2 i=3 2 i=3
= dz = 2 i=3 = e dx !
3
3
z +1 dz = e dx R x3 + 1
Z 0 Z 1
log x + 2 i=3 2 i=3 log x + 2 i=3 2 i=3
! e dx = e dx =
1 x3 + 1 0 x3 + 1
Z 1 Z
2 i=3 log x 2 i 2 i=3 1 1 2 i 2 i=3
= e 3
dx e 3
dx = e2 i=3 I e J:
0 x +1 3 0 x +1 3

Integrate along and let " ! 0+ . This gives


4,
q
Z log2 " + 23
2
2 " 2 " jlog "j
f (z) dz 3 ! 0:
4
1 " 3 3

Using the Residue Theorem and letting R ! 1 and " ! 0+ , we obtain that

85
2 i 2 i
I + 0 e2 i=3
I e i=3
J +0=2 i e 2 i=3
:
3 9
i=3
Multiplying with e , this yields
2
2 i=3 2
2i sin ( =3) I ie J= ;
3 9
and thus
p
p i 2 23
3Ii + J= : J
3 3 9
Separating real and imaginary parts, we get the simultaneous equations
p 2
3
p 3
J = 29
3I 3
J = 0;
and hence the solutions
Z 1 Z 1
log x 2 2 1 2
I= dx = ; J= dx = p :
0 x3 + 1 27 0 x3 +1 3 3
Remark.
We begin with the rst integral I, we make the substitution x3 = t and use
the integral from Exercise VII.4.3, where we choose a = 2=3.

2 3
Z 1 x3 = t
log x
I= dx = 4 x = t1=3 5=
0 x3 + 1 1 2=3
dx = 3 t dt
Z 1
1 log t 1 2 cos 23 2 2
= dt = = :
9 0 t2=3 (t + 1) 9 sin2 23 27

For the second integral J, we use the integral from Exercise VII.4.2, where
we choose b = 3.
Z 1
dx 2
J= b
= = p :
0 1+x 3 sin ( =3) 3 3

86
VII.4.9
By integrating around an appropriate contour and using the results
of Exercise 8, show that
Z 1
(log x)2 10 3
dx = p :
0 x3 + 1 81 3

Solution

VII.4.9

2
3
z1
4
z2
1 R
z3

Set
Z 1
(log x)2
I= dx
0 x3 + 1
Integrate
(logjzj+i arg z)2
(log z)2
f (z) = =
z3 + 1 z e i=3 (z e i) z e5 i=3

along the pie-slice contour with 0 < arg z < 2 =3 in Figure VII.4.9. We
make a branchcut for log z along the negative imaginary axis, where =2 <
arg z < 3 =2.
Residue at a simple pole at z1 = e i=3 , where by Rule 3,

" #
(log jzj + i arg z)2 i=3 (log jzj + i arg z)2 2
2 i=3
Res ;e = = e :
z3 + 1 3z 2 i=3
27
z=e

87
Integrate along 1, and let R ! 1 and " ! 0+ . This gives

Z
f (z) dz =
1
Z Z
(log jzj + i arg z)2 z = xe0i R
(log x)2
dz = = dx !
1
z3 + 1 dz = dx " x3 + 1
Z 1
(log x)2
! dx = I:
0 x3 + 1
Integrate along 2, and let R ! 1. This gives
Z 2
log2 R + 23 2 R 2 log2 R
f (z) dz ! 0:
2
R3 1 3 3R2
Integrate along 3, and let R ! 1 and " ! 0+ . This gives

Z
f (z) dz =
3
Z Z "
(log jzj + i arg z)2 z = xe2 i=3 (log x + 2 i=3)2 2 i=3
= dz = = e dx !
3
z3 + 1 dz = e2 i=3 dx R x3 + 1
Z 0 Z 1
(log x + 2 i=3)2 2 i=3 (log x + 2 i=3)2 2 i=3
! e dx = e dx =
1 x3 + 1 0 x3 + 1
Z 1 Z Z
2 i=3 (log x)2 4 i 2 i=3 1 log x 4 2 2 i=3 1 1
= e dx e dx+ e dx =
0 x3 + 1 3 0 x3 + 1 9 0 x3 + 1
Z Z
2 i=3 4 i 2 i=3 1 log x 4 2 2 i=3 1 1
= e I e dx+ e :
3 0 x3 + 1 9 0 x3 + 1
Integrate along 4, and let " ! 0+ . This gives
Z 2
log2 " + 23 2 " 2 " log2 "
f (z) dz ! 0:
4
1 "3 3 3

Using the Residue Theorem and letting R ! 1 and " ! 0+ , we obtain that

Z 1 Z 1
2 i=3 4 i 2 i=3 log x 4 2 2 i=3 1 2
2 i=3
I+0 e I e dx+ e +0 = 2 i e :
3 0 x3 + 1 9 0
3
x +1 27

88
i=3
multiplying with e , this yields

Z 1 2 Z 1
i=3 i=3 4 i log x i=3 4 1 i=3 2 3
e e I e dx+ e dx = i;
3 0 x3 + 1 9 0
3
x +1 27
and thus, substituting the values for the integrals from Exercise 9,

4 i i=3 2 2 4 2
i=3 2 2 3
2i sin ( =3) I e + e p = i:
3 27 9 3 3 27
And hence, setting imaginary parts equal,
p 4 3 12 3 2 3
3I+ + = ;
81 81 27
and therefore
Z 1
(log x)2 10 2
dx = p :
0 x3 + 1 81 3

89
VII.4.10
By integrating a branch of (log z) = (z 3 1) around the boundary of
an indented half-disk and using the result of Exercise 8, show that
Z 1
log x 4 2
dx = :
0 x3 1 27

Solution

VII.4.10

z2

6 2

-R 5 1 z 1 3 R
z3

Set
Z 1
log x
I= dx
0 x3 1
Integrate

log z log jzj + i arg z


f (z) = =
z3 1 (z 1) z e2 i=3 z e4 i=3

along the indented half-disk contour in Figure VII.4.10. We make a branchcut


for log z along the negative imaginary axis, where =2 < arg z < 3 =2.
Residue at a simple pole at z1 = 1, where by Rule 3,

log jzj + i arg z log jzj + i arg z


Res ;1 = = 0:
z 3 1 3z 2 z=1

Residue at a simple pole at z2 = e2 i=3


, where by Rule 3,

90
log jzj + i arg z 2 i=3 log jzj + i arg z 2 i 4 i=3
Res ;e = = e :
z3 1 3z 2 z=e2 i=3 9

Integrate along 1 and 3, and let R ! 1 and " ! 0+ . This gives

Z Z
f (z) dz + f (z) dz =
1
Z 3
Z
log jzj + i arg z log jzj + i arg z
= dz + dz =
1
z3 1 3
z3 1
z = xe0i
= =
dz = dx
Z 1 " Z R
log x log x
= 3
dx + 3
dx !
" x 1 1+" x 1
Z 1
log x
! dx = I:
0 x3 1

Integrate along 2, and let " ! 0+ . This gives


Z
f (z) dz ! i ( 0) 0 = 0:
2

Integrate along 4,
and let R ! 1. This gives
Z p
log2 R + 2 log R
f (z) dz 3
R ! 0:
4
R 1 R2
Integrate along 5, and let R ! 1 and " ! 0+ . This gives

91
Z
f (z) dz =
5
Z Z "
log jzj + i arg z z = xe i log x + i
= dz = = e id !
5
z 3 1 dz = e i dx R x 3 1
Z 0 Z 1
log x + i log x + i
! 3
e i dx = dx =
1 x 1 0 x3 + 1
Z 1 Z 1
log x 1
= dx i dx =
0 x3 + 1 0
3
x +1
Z 1 Z 1
log x 1
= 3
dx i 3
dx:
0 x +1 0 x +1

Integrate along and let " ! 0+ . This gives


6,

Z p
log2 " +
f (z) dz " " jlog "j ! 0:
6
1 "3

Using the Residue Theorem and letting R ! 1 and " ! 0+ , we obtain that

Z 1 Z 1
log x 1 2 i 4 i=3
I +0+0 dx i dx + 0 = 2 i e ;
0 x3 + 1 0 x3 +1 27

and thus, substituting the values for the integrals from Exercise 9,

2 2
2 4 2
4 i=3
I i p = e :
27
3 3 9
And hence, setting imaginary parts equal,

2 2 4 2
I+ = ;
27 18
and therefore
Z 1
log x 4 2
dx = :
0 x3 1 27

92
VII.5.1
Use the keyhole contour indented on the lower edge of the axis at
x = 1 to show that
Z 1
log x 2 2
dx = ; 0 < a < 1:
0 xa (x 1) 1 cos (2 a)

Solution

VII.5.1

1
-R 5 z 3 R
6 4

Set
Z 1
log x
I= dx
1 xa (x 1)
and integrate

log z (log jzj + i arg z)


f (z) = = a(logjzj+i arg z)
za (z 1) e (z 1)
along the contour in Figure VII.5.1. We make a branchcut for log z along
the positive imaginary axis, where 0 < arg z < 2 . Thus f (z) is analytic on
the keyhole domain. It extends analytically to (0; 1) from above, and the

93
apparent singularity at z = 1 is removable. However, the extension to (0; 1)
from below has a simple pole at z = 1.
Residue at a simple pole at z = 1, where by Rule 3,

(log jzj + i arg z) (log jzj + i arg z) 2 ia


Res ; 1 = = 2 ie :
ea(logjzj+i arg z) (z 1) ea(logjzj+i arg z) z=1

Integrate along 1, and let R ! 1 and " ! 0+ . This gives

Z
f (z) dz =
1
Z ZR
(log jzj + i arg z) z = xe0i log x
= dz = = dx !
ea(logjzj+i arg z) (z 1) dz = dx a
" x (x 1)
1
Z 1
log x
! dx = I:
0 xa (x 1)

Integrate along and let " ! 0+ . This gives when 0 < a < 1
2,
q
Z log2 R + (2 )2 2 log R
f (z) dz a
2 R ! 0:
2
R (R 1) Ra
Integrate along 3 and 5, and let R ! 1 and " ! 0+ . This gives

Z Z
f (z) dz+ f (z) dz =
3
Z 5
Z
(log jzj + i arg z) (log jzj + i arg z)
= a(logjzj+i arg z)
dz+ a(logjzj+i arg z) (z
dz =
3
e (z 1) 5
e 1)
Z 1+" Z "
z = xe2 i 2 ia (log x + 2 i) 2 ia (log x + 2 i)
= =e dx+e dx !
dz = dx R xa (x 1) 1 " x (x
a 1)
Z 0 Z 1
2 ia (log x + 2 i) 2 ia (log x + 2 i)
!e a
dx = e dx =
1 x (x 1) 0 xa (x 1)
Z 1 Z 1
2 ia log x 2 ia 1
= e a
dx 2 ie a
dx = e 2 ia I 2 ie 2 ia J:
0 x (x 1) 0 x (x 1)

Integrate along 4, use fractional residue formula and let " ! 0+ . This gives

94
Z
2 ia
f (z) dz ! i( 0) 2 ie = 2 2e 2 ia
:
4

Integrate along and let " ! 0+ . This gives 0 < a < 1


6,
q
Z log2 " + (2 )2
f (z) dz a (1
2 " 2 "1 a jlog "j ! 0:
6
" ")
Using the Residue Theorem and letting R ! 1 and " ! 0+ , we obtain that

2 ia 2 ia
I e I 2 ie J + 2 2e 2 ia
+ 0 = 0:
Multiplying with e2 ia
, this yields

Ie2 ia
I 2 iJ + 2 2
= 0:
We equate real parts

2
(cos (2 a) 1) I + 2 = 0;
and therefore
Z 1
log x 2 2
a
dx = ; 0 < a < 1:
0 x (x 1) 1 cos (2 a)

95
VII.5.2
Show using residue theory that
Z 1
sin (ax) a
2
dx = 1 e ; a > 0:
1 x (x + 1)

Hint. Replace sin (az) by eiaz , and integrate around the boundary of a half-
disk indented at z = 0.

Solution

VII.5.2

z1
2
-R 1 z2 3 R
z3

Set
Z 1
sin ax
I= dx
1 x (x2 + 1)
and integrate

eiaz eiaz
f (z) = =
z (z 2 + 1) z (z i) (z + i)
along the contour in Figure VII.5.2.
Residue at a simple pole at z1 = 0, where by Rule 3,

eiaz eiaz
Res ;0 = 2 = 1:
z (z 2 + 1) 3z + 1 z=0
Residue at a simple pole at z2 = i, where by Rule 3,

96
eiaz eiaz e a
Res ; i = = :
z (z 2 + 1) 3z 2 + 1 z=i 2
Integrate along 1 and 3, and let R ! 1 and " ! 0+ . This gives

Z Z
f (z) dz + f (z) dz =
1 3
Z Z
eiaz eiaz z = x
= dz + dz = =
1
2
z (z + 1) 3
2
z (z + 1) dz = dx
Z " Z R
eiax eiax
= 2
dx + 2
dx !
R x (x + 1) " x (x + 1)
Z 1 Z 1 Z 1
eiax cos ax sin ax
! 2
dx = 2
dx + i 2
dx =
1 x (x + 1) 1 x (x + 1) 1 x (x + 1)
Z 1
cos ax
= 2
dx + iI:
1 x (x + 1)

Integrate along 2, use fractional residue formula and let " ! 0+ . This gives
Z
f (z) dz ! i ( 0) 1 = i:
2

Integrate along 4 , and let R ! 1. Because jeiaz j 1 in the upper half


plane if a > 0, this gives
Z
1
f (z) dz R ! 0:
4
R (R2 1) R2

Using the Residue Theorem and letting R ! 1 and " ! 0+ , we obtain that
Z 1
cos ax e a
2
dx + iI i + 0 = 2 i ;
1 x (x + 1) 2
and hence, setting imaginary parts equal,

I= e a;
i.e.,

97
Z 1
sin ax a
dx = 1 e ; a > 0:
1 x (x2 + 1)

98
VII.5.3
Show using residue theory that
Z 1
sin (ax) 2
2 2 2
dx = ; a > 0:
1 x( ax)

Solution

VII.5.3

2 4 6
-R 1 z3 3 z1 5 z2 7 R

Set
Z 1
sin (ax)
I= dx
1 x ( 2 a2 x 2 )
and integrate

eiaz eiaz
f (z) = =
z ( 2 a2 z 2 ) a2 z (z =a) (z + =a)
along the contour in Figure VII.5.3.
Residue at a simple pole at z1 = 0, where by Rule 3,

eiaz eiaz 1
Res ;0 = = :
z ( 2 a2 z 2 ) 2 3a2 z 2 z=0
2

Residue at a simple pole at z2 = =a, where by Rule 3,

eiaz eiaz 1
Res ; = = :
z ( 2 a2 z 2 ) a 2 3a2 z 2 z= =a 2 2

99
Residue at a simple pole at z3 = =a, where by Rule 3,

eiaz eiaz 1
Res ; = = :
z ( 2 a2 z 2 ) a 2 3a2 z 2 z= =a 2 2

Integrate along 1; 3; 5 and 7, and let R ! 1 and " ! 0+ . This gives

Z Z Z Z
f (z) dz + f (z) dz + f (z) dz + f (z) dz =
1 3 5 7
Z Z
eiaz eiaz
= dz + dz+
1
z ( 2 a2 z 2 ) 3
z ( 2 a2 z 2 )
Z iaz Z
e eiaz
+ 2
dz + dz =
5
z( a2 z 2 ) 7
z ( 2 a2 z 2 )
z = x
= =
dz = dx
Z =a " Z "
eiax eiax
= dx + dx+
R x ( 2 a2 x 2 ) =a+" x (
2 a2 x 2 )
Z =a " Z R
eiax eiax
+ dx + dx !
" x ( 2 a2 x 2 ) =a+" x (
2 a2 x2 )
Z 1
eiax
! 2
dx =
1 x( a2 x 2 )
Z 1 Z 1
cos (ax) sin (ax)
= 2 2 2
dx + i 2
dx =
1 x( ax) 1 x( a2 x 2 )
Z 1
cos (ax)
= 2
dx + iI:
1 x( a2 x 2 )

Integrate along 2, use fractional residue formula and let " ! 0+ . This gives
Z
1 1
f (z) dz ! i ( 0) 2
= i:
2
2 2

Integrate along 4, use fractional residue formula and let " ! 0+ . This gives
Z
1 1
f (z) dz ! i ( 0) 2 = i:
4

100
Integrate along 6,use fractional residue formula and let " ! 0+ . This gives
Z
1 1
f (z) dz ! i ( 0) 2
= i:
6
2 2
Integrate along 8 , and let R ! 1. Because jeiaz j 1 in the upper half
plane if a > 0, this gives
Z
1
f (z) dz R ! 0:
2
R (a R2
2 2) a2 R 2

Using the Residue Theorem and letting R ! 1 and " ! 0+ , we obtain that
Z 1
cos ax 1 1 1
2
dx + iI i i i = 0;
1 x (x + 1) 2 2
and hence, setting imaginary parts equal
1 1 1
I = 0;
2 2
i.e.,
Z 1
sin (ax) 2
2 2 2
dx = ; a > 0:
1 x( ax)

101
VII.5.4
Show using residue theory that
Z 1
1 cos x
dx = :
0 x2 2

Solution

VII.5.4

2
-R 1 z1 3 R

Set
Z 1 Z 1
1 cos x 1 cos x
I= dx ) 2I = dx
0 x2 1 x2
and integrate

1 eiz
f (z) =
z2
along the contour in Figure VII.5.4.
Residue at a double pole at z1 = 0, where by Rule 2,

1 eiz d
Res ; 0 = lim 1 eiz = ieiz z=0
= i
z2 z!0 dz

Integrate along 1 and 3, and let R ! 1 and " ! 0+ . This gives

102
Z Z
f (z) dz + f (z) dz =
1 3
Z Z
1 eiz 1 eiz
= dz + dz =
1
z2 3
z2
z = x
= =
dz = dx
Z " Z R Z 1
1 eix 1 eix 1 eix
dx + dx ! dx =
R x2 " x2 1 x2
Z 1 Z 1 Z 1
1 cos x sin x sin x
2
dx i 2
dx = 2I i 2
dx:
1 x 1 x 1 x

Integrate along 2, use fractional residue formula and let " ! 0+ . This gives
Z
f (z) dz ! i ( 0) ( i) = :
2

Integrate along 4 , and let R ! 1. Because jeiaz j 1 in the upper half


plane if a > 0, this gives
Z
2 2
f (z) dz 2
R ! 0:
4
R R

Using the Residue Theorem and letting R ! 1 and " ! 0+ , we obtain that
Z 1
sin x
2I i 2
dx + 0 = 0;
1 x

and hence, setting imaginary parts equal

2I = 0;
i.e.,
Z 1
1 cos x
dx = :
0 x2 2

103
VII.5.5
By integrating (e 2iz 1) =z 2 over appropriate indented contours
and using Cauchys theorem, show that
Z 1
sin2 x
2
dx = :
1 x

Solution

VII.5.5

4
-R 3 z1 1 R

Set
Z 1 Z 1
sin2 x 1 cos 2x
I= dx = dx
1 x2 1 2x2
and integrate

ei2z 1
f (z) =
z2
along the contour in Figure VII.5.5.
Residue at a double pole at z1 = 0, where by Rule 2,

ei2z 1 d i2z
Res ; 0 = lim e 1 = 2iei2z z=0
= 2i
z2 z!0 dz

Integrate along 1 and 3, and let R ! 1 and " ! 0+ . This gives

104
Z Z
f (z) dz + f (z) dz =
1 3
Z Z
ei2z 1 ei2z 1 z = x
= dz + dz = =
1
z2 3
z2 dz = dx
Z " i2x Z R i2x Z 1 i2x
e 1 e 1 e 1
= dx + dx ! dx =
R x2 " x2 1 x2
Z 1 Z 1 Z 1
cos 2x 1 sin 2x sin 2x
= dx + i dx = 2I + i dx:
1 x2 1 x2 1 x2

Integrate along 2, use fractional residue formula and let " ! 0+ . This gives
Z
f (z) dz ! i ( 0) 2i = 2 :
2

Integrate along 4 , and let R ! 1. Because jeiaz j 1 in the upper half


plane if a > 0, this gives
Z
2 2
f (z) dz 2
R ! 0:
4
R R

Using the Residue Theorem and letting R ! 1 and " ! 0+ , we obtain that
Z 1
sin 2x
2I i 2
dx + 2 + 0 = 0;
1 2x

and hence, setting real parts equal,

2I + 2 = 0;
i.e.,
Z 1
sin2 x
dx = :
1 x2

105
VII.5.6
By integrating a branch of (log z) = (z 3 1) around the boundary of
an indented sector of aperture 2 =3, show that
Z 1
log x 4 2
dx = :
0 x3 1 27
Remark. See also Exercise 4.10.

Solution

VII.5.6

5 4
6
z2
7 8 2
1 z1 3 R

z3

Set
Z 1
log x
I= dx
0 x3 1
and integrate

log z log jzj + i arg z


f (z) = =
z3 1 (z e0i ) z e2 i=3 z e4 i=3

along the contour in Figure VII.5.6. We make a branchcut for log z along
the negative imaginary axis, where =2 < arg z < 3 =2.
Residue at a simple pole at z1 = 1, where by Rule 3,

log jzj + i arg z log jzj + i arg z


Res ;1 = = 0:
z 3 1 3z 2 z=1

Residue at a simple pole at z2 = e2 i=3


, where by Rule 3,

106
log jzj + i arg z 2 i=3 log jzj + i arg z 2 i 2 i=3
Res ;e = = e :
z3 1 3z 2 z=e2 i=3 9
+
Integrate along 1 and 3, and let R ! 1 and " ! 0 . This gives

Z Z
f (z) dz+ f (z) dz =
1 3
Z Z
log jzj + i arg z log jzj + i arg z z = xe0i
dz+ dz = =
1
z3 1 3
z 3 1 dz = dx
Z 1 " Z R Z 1
log x log x log x
= 3
dx+ 3
dx ! dx = I:
" x 1 1+" x 1 0 x3 1

Integrate along 2, use fractional residue formula and let " ! 0+ . This gives
Z
f (z) dz ! i ( 0) 0 = 0:
2

Integrate along 4,
and let R ! 1. This gives
q
Z log2 R + 23
2
2 R 2 log R
f (z) dz 3
! 0:
4
R 1 3 R2
Integrate along 5 and 7, and let R ! 1 and " ! 0+ . This gives

Z Z
f (z) dz + f (z) dz =
5 7
Z Z
log jzj + i arg z log jzj + i arg z z = xe2 i=3
= dz+ dz = =
5
z3 1 7
z3 1 dz = e2 i=3 dx
Z 1+" Z "
log x + 2 i=3 2 i=3 log x + 2 i=3 2 i=3
= e dx + e dx !
R x3 1 1 " x3 1
Z 0 Z 1 Z
2 i=3 log x + 2 i=3 2 i=3 log x 2 i 2 i=3 1 1
!e dx = e dx e dx =
1 x3 1 0 x3 1 3 0 x3 1
Z
2 i=3 2 i 2 i=3 1 1
= e I e dx:
3 0 x3 1
Integrate along 6, use fractional residue formula and let " ! 0+ . This gives

107
Z 2
2 i 2 i=3 2
f (z) dz ! i( 0) e = e2 i=3
:
6
9 9

Integrate along and let " ! 0+ . This gives


8,
q
Z log2 " + 23
2
2 " 2 " jlog "j
f (z) dz 3 ! 0:
8
1 " 3 3

Using the Residue Theorem and letting R ! 1 and " ! 0+ , we obtain that
Z
2 i=3 2 i 2 i=3 1 1 2 2 2 i=3
I +0 e I e dx + e = 0:
3 0 x3 1 9
2 i=3
Multiplying with e , this yields
p ! Z
1 3 2 i 1 1 2 2
I i I dx + = 0:
2 2 3 0 x3 1 9
We equate real parts

3 2 2
I+ = 0;
2 9
and hence
Z 1
log x 4 2
dx = :
0 x3 1 27

108
VII.6.1
Integrate 1= (1 x2 ) directly, using partial fractions, and show that
Z 1
dx
PV = 0:
0 1 x2
Show that
Z 1 Z 1
dx dx
= +1; = 1:
0 1 x2 1 1 x2

Solution
Partial fractions gives

1 1 1 1
= +
1 x2 2 1+x 1 x

Z 1 Z 1 " Z 1
dx dx dx
PV = lim + =
0 1 x2 "!0 0 1 x2 1+" 1 x2
1 1
= lim [log (1 + x) log (1 x)]10 "
+ [log j1 + xj log j1 xj]1
1+" =
"!0 2 2
!
1 " 1
1 1+x 1 1+x
= lim log + log =
"!0 2 1 x 0 2 1 x 1+"

1 2 " 2+"
= lim log log =
"!0 2 " "
1 2 "
= lim log = 0:
"!0 2 2+"

Inspection gives

Z 1 Z 1 "
dx dx 1
= lim = lim+ [log (1 + x) log (1 x)]10 " =
0 1 x2 "!0+ 0 1 x 2 "!0 2
1 "
1 1+x 1 2 "
= lim+ log = lim+ log = +1 :
"!0 2 1 x 0 "!0 2 "

109
and the second integral

Z 1 Z 1
dx dx 1
= lim = lim+ [log (1 + x) log (1 x)]1
1+" =
1 1 x2 "!0+ 1+" 1 x 2 "!0 2
1
1 1+x 1 2+"
= lim+ log = lim+ log ! 1:
"!0 2 1 x 1+" "!0 2 "

110
VII.6.2
Obtain the principal value in Exercise 1 by taking imaginary parts
of the identity (5.4) p.211 in the preceding section and making a
change of variable

Solution
Identity (5.4)

Z 1 Z 1 " Z 0 Z
log x log jxj + i log jxj + i log z
dx+ dx+ dx+ dz = 0:
0 x2 1 1 x2 1 1+" x2 1 C" z
2 1

Taking imaginary part of (5.4), get


Z 1 " Z 0 Z
log z
dx + dx + Im dz = 0:
1 x2 1 1+" x
2 1 C" z2 1
Note
Z
log z
( ) Im dz ! 0;
C" z2 1
as " ! 0, by estimation on p. 211.
Making the change of variables, x ! x, and by using ( ) we get

Z 1 " Z 0
x = x
dx + dx = =
1 x2 1 1+" x2 1 dx = dx
Z 1 Z 1 "
dx + dx ! 0 as " ! 0;
1+" x2 1 0 x2 1

which gives that


Z 1
dx
PV = 0:
0 1 x2
Note. For a linear change of variables, we can take the limit either before or
after change of variable. Otherwise we can not do this.

111
VII.6.3
By integrating around the boundary of an indented half-disk in the
upper half-plane, show that
Z 1
1 a
PV 2
dx = 2
; 1 < a < 1:
1 (x + 1) (x a) a +1

Solution

VII.6.3 (a = 2)

z2
2

-R 1 z1 3 R
z3

Set
Z 1
1
I = PV dx
1 (x2 + 1) (x a)
and integrate
1 1
f (z) = =
(z 2 + 1) (z a) (z i) (z + i) (z a)
along the contour in Figure VII.6.3.
Residue at a simple pole at z1 = a, where by Rule 1,

1 1 1
Res ;a = = :
(z 2 + 1) (z a) z2 +1 z=a a2 +1
Residue at a simple pole at z1 = i, where by Rule 3,

112
1 1 1 a
Res ;i = 2 = + i:
(z 2 + 1) (z a) 3z 2az + 1 z=i 2 (a2 + 1) 2 (a2 + 1)

Integrate along 1 and 3, and let R ! 1 and " ! 0+ . This gives

Z Z
f (z) dz + f (z) dz =
1 3

Za " ZR
1 1
= 2
dx + dx !
(x + 1) (x a) (x2 + 1) (x a)
R a+"
Z 1
1
! dx = I:
1 (x2 + 1) (x a)

Integrate along 2, and let " ! 0+ . This gives


Z
1
f (z) dz ! i( 0) = i:
2
a2 +1 a2 + 1

Integrate along 4, and let R ! 1. This gives


Z
1
f (z) dz R ! 0:
4
(R2 1) (R jaj) R2
Using the Residue Theorem and letting R ! 1 and " ! 0+ , we obtain that

1 a
I i=2 i + i ;
a2 + 1 2 (a2 + 1) 2 (a2 + 1)
i.e.,
Z 1
1 a
PV dx = :
1 (x2 + 1) (x a) a2 +1

113
VII.6.4
Suppose m 2 and a1 < a2 < < am . By integrating around the
boundary of an indented half-disk in the upper half-plane, show
that
Z 1
1
PV dx = 0:
1 (x a1 ) (x a2 ) (x am )

Solution

VII.6.4

1 2 m - 1 m
a1 a2 a m - 1 am
-R R

Set
Z 1
1
I = PV dx
1 (x a1 ) (x a2 ) (x am )
and integrate
1 1
f (z) = = Qm
(z a1 ) (z a2 ) (z am ) k=1 (z ak )
along the semicircular contour indented at z = aj with small semicircles of
radii ", see Figure VII.6.4.
Residue at a simple pole at zj = aj ; 1 j m, where by Rule 3,

1 1 1
Res Qm ; aj = Qm = Qm :
k=1 (z ak ) k=1;k6=j (z ak )
z=aj k=1;k6=j (aj ak )

114
Integrate along the union of line segments on real axis denoted by L, and
let R ! 1 and " ! 0+ . This gives

Z Z a1 " X1 Z ak+1 "


m Z R
f (z) dz = f (z) dz+ f (z) dz+ f (z) dz =
L R k=1 ak +" am +"
Z a1 " m 1 Z
X ak+1 " Z R
1 1 1
= Qm dz+ Qm dz+ Qm dz =
R k=1 (z ak ) k=1 ak +" k=1 (z ak ) am +" k=1 (z ak )
z = x
= =
dz = dx
Z a1 " X1 Z ak+1 "
m Z R
1 1 1
= Qm dx+ Qm dx+ Qm dx !
R k=1 (x ak ) k=1 ak +" k=1 (x ak ) am +" k=1 (x ak )
Z 1
1
! Qm dx = I:
1 k=1 (x ak )

Integrate along 1; 2; : : : m, and let " ! 0+ . This gives


m Z
!
X X
m
1
f (z) dz ! i( 0) Qm :
j=1 j j=1 k=1;k6=j (aj ak )

Integrate along R, and let R ! 1, this gives


Z
1
f (z) dz Qm R ! 0:
R k=1 (z jak j) Rm 1

Using the Residue Theorem and letting R ! 1 and " ! 0+ , we obtain that
X
m
1
I i Qm = 0:
j=1 k=1;k6=j (aj ak )

Now, note that the the sums of the residues at a simple pole at zj = aj ;
1 j m, are real, and identifying the real part, we have
Z 1
1
PV dx = 0:
1 (x a1 ) (x a2 ) (x am )

115
VII.6.5
Show that
Z 1
xa 1 a
PV dx = cot ; 0 < a < b:
1 xb 1 b b
Hint. For b > 1 one can integrate a branch of z a 1 = z b 1 around a sector
of aperture 2 =b, indented at z = 1 and z = e2 i=b .

Solution

VII.6.5 (b = 3)

5 4
6
z2
7 8 2
1 z1 3 R

z3

Set
Z 1
xa 1
I = PV dx
1 xb 1
and integrate

za 1 jzja 1 ei(a 1) arg z


f (z) = b =
z 1 jzjb eib arg z 1
along the contour in Figure VII.6.5. We make a branchcut for z a 1
along the
negative imaginary axis, where =2 < arg z < 3 =2.
Residue at a simple pole at z1 = 1, where by Rule 3,

za 1 jzja 1 ei(a 1) arg z 1


Res ; 1 = = :
zb 1 bz b 1 b
z=1

116
Residue at a simple pole at z2 = e2 i=b
, where by Rule 3,

za 1 2 i=b jzja 1 i(a 1) arg z


e e2 i=b 2 i(a 1)=b
e
Res ;e = = :
zb 1 bz b 1 b
z=e2 i=b

Integrate along 1 and 3, and let R ! 1 and " ! 0+ . This gives

Z Z
f (z) dz+ f (z) dz =
1 3
Z Z
jzja 1 i(a 1) arg z
e jzja 1 i(a 1) arg z
e z = xe0i
= dz + dz =
=
1 jzjb eib arg z 1 jzjb eib arg z 1
3
dz = dx
Z 1 " Z R a 1 Z 1 a 1
xa 1 x x
= b
dx + b
dx ! dx = I:
" x 1 1+" x 1 0 xb 1

Integrate along 2, and let " ! 0+ . This gives


Z
1 i
f (z) dz ! i( 0) = :
2
b b

Integrate along 4, and let R ! 1. This gives when 0 < a < b


Z
Ra 1 2 R 2
f (z) dz ! 0:
4
Rb 1 b bRb a

Integrate along 5 and 7, and let R ! 1 and " ! 0+ . This gives

Z Z
f (z) dz+ f (z) dz =
5 7
Z Z
jzja 1 i(a 1) arg z
e jzja 1 i(a 1) arg z
e z = xe2 i=b
dz + = dz =
5 jzjb eib arg z 1 b ib arg z
7 jzj e 1 dz = e2 i=b dx
Z 1+" a 1 2 i(a 1)=b Z " a 1 2 i(a 1)=b
x e 2 i=b x e
= b
e dx + b
e2 i=b dx!
R x 1 1 " x 1
Z 0 a 1 2 i(a 1)=b Z 1 a 1
x e 2 i=b 2 i=b 2 i(a 1)=b x
! b
e dx = e e dx =
1 x 1 0 xb 1
= e2 i=b e2 i(a 1)=b I

117
Integrate along 6, and let " ! 0+ . This gives

Z
e2 i=b 2 i(a 1)=b
e e2 i=b 2 i(a 1)=b
e
f (z) dz ! i( 0) = i :
6
b b

Integrate along 8, and let " ! 0+ . This gives when 0 < a < b
Z
"a 1 2 " 2 "a
f (z) dz ! 0:
8
1 "b b b
Using the Residue Theorem and letting R ! 1 and " ! 0+ , we obtain that

i e2 i=b 2 i(a 1)=b


e
+ 0 e2
I i=b 2 i(a 1)=b
e I i = 2 i 0:
b b
Hence solving for I,

a
i 1 + e2 i=b 2 i(a 1)=b
e i 1 + e2 ia=b cos b a
I= = = a
= cot ;
b 1 e2 i=b e2 i(a 1)=b b 1 e2 ia=b b sin b
b b

i.e.,
Z 1
xa 1 a
PV dx = cot ; 0 < a < b:
1 xb 1 b b

118
VII.6.6
By integrating a branch of (log z) = z b 1 around an indented sec-
tor of aperture 2 =b, show that for b > 1,

Z 1 2 Z 1
log x 1
dx = ; PV dx = cot ( =b) :
0 xb 1 b2 sin2 ( =b) 0 xb 1 b

Solution

VII.6.6 (b = 3)

5 4
6
z2
7 8 2
1 z1 3 R

z3

Set
Z 1 Z 1
log x 1
I= dx J = PV dx
0 xb 1 0 xb 1
and integrate

log z log jzj + i arg z


f (z) = =
z b 1 jzjb eib arg z 1
along the contour in Figure VII.6.5. We make a branchcut for log z along
negative imaginary axis, where =2 < arg z < 3 =2.
Residue at a simple pole at z1 = 1, where by Rule 3,

log z log jzj + i arg z


Res b
;1 = = 0:
z 1 bz b 1 z=1

Residue at a simple pole at z2 = e2 i=b


, where by Rule 3,

119
log z 2 i=b log jzj + i arg z 2 ie2 i=b
Res ;e = = :
zb 1 bz b 1 z=e2 i=b b2
Integrate along 1 and 3, and let R ! 1 and " ! 0+ . This gives

Z Z
f (z) dz+ f (z) dz =
1 3
Z Z
log jzj + i arg z z = xe0i
log jzj + i arg z
= dz + dz = =
1 jzjb eib arg z 1 b ib arg z
3 jzj e 1 dz = dx
Z 1 " Z R Z 1
logx logx logx
= b
dx + b
dx ! dx = I:
" x 1 1+" x 1 0 xb 1

Integrate along 2, and let " ! 0+ . This gives


Z
f (z) dz ! i ( 0) (0) = 0:
2

Integrate along 4,
and let R ! 1. This gives when b > 1
q
Z log2 R + 2b
2
2 R 2 log R
f (z) dz b
! 0:
4
R 1 b bRb 1
Integrate along 5 and 7, and let R ! 1 and " ! 0+ . This gives

Z Z
f (z) dz+ f (z) dz =
5 7
Z Z
log jzj + i arg z log jzj + i arg z z = xe2 i=b
= dz + dz = =
b ib arg z
5 jzj e 1 b ib arg z
7 jzj e 1 dz = e2 i=b dx
Z 1+" Z "
logx + 2 i=b 2 i=b logx + 2 i=b 2 i=b
= b
e dx + e dx !
R x 1 1 " xb 1
Z 0 Z 1 Z 1
logx + 2 i=b 2 i=b 2 i=b logx 2 i 2 i=b 1
! b
e dx = e b
dx e dx =
1 x 1 0 x 1 b 0 xb 1
2 i 2
= e2 i=b
I e i=b
J:
b

120
Integrate along 6, and let " ! 0+ . This gives
Z
2 ie2 i=b
2 2 e2 i=b
f (z) dz ! i( 0) = :
6
b2 b2
Integrate along and let " ! 0+ . This gives when b > 1
8,
q
Z log2 " + 2b
2
2 " 2 " jlog "j
f (z) dz b
! 0:
8
1 " b b
Using the Residue Theorem and letting R ! 1 and " ! 0+ , we obtain that

2 i 2 2 2 e2 i=b
I +0 e2 i=b
I e i=b
J+ = 2 i 0:
b b2
2 i=b
Multiplying with e , this yields

2 i 2 i=b2 2
e J + 2 = 0: 1 I
b b
Separating real and imaginary parts, we get simultaneous equations
2 2
cos b
1 I + 2b2 = 0
sin 2b I 2b J = 0;
i.e.,

Z 1 2 Z 1
log x 1
I= dx = ; J = PV dx = cot ( =b) :
0 xb 1 b2 sin2 ( =b) 0 xb 1 b

121
VII.6.7
Suppose that P (z) and Q (z) are polynomials, deg Q (z) deg P (z)+2,
and the zeros of Q (z) on the real axis are all simple. Show that

Z 1 X X
P (z) P (z) P (z)
PV dx = 2 i Res ; zj + i Res ; xk ;
1 Q (z) Q (z) Q (z)

summed over the poles zj of P (z) =Q (z) in the open upper half-
plane and the poles xk of P (z) =Q (z) on the real axis. Remark.
In other words, the principal value of the integral is 2 i times the
sum of the residues in the upper half-plane, where we count the
poles on the real axis as being half in and half out of the upper
half-plane.

Solution

VII.6.7

R
zj - 1 z3
zj
z2 z1
1 2 m - 1 m
a1 a2 a m - 1 am
-R R

Use a semicircular contour indented at ak = xk with small semicircles of radii


", see Figure VII.6.7.
Get
Z
P (z) P (z) P (z)
! i( 0) Res ; xk = i Res ; xk
k
Q (z) Q (z) Q (z)
The condition deg Q deg P + 2 guarantees that the integral converges at
1: Otherwise we would have to use a residue at 1 (in this case deg Q =
deg P + 1).

122
Use the Residue Theorem, take limit, and the result follows
Set
Z 1
P (x)
I = PV dx
1 Q (x)
and integrate

P (z)
f (z) =
Q (z)
along the contour in Figure VII.6.7.
The sum of the residues at the simple poles ak = xk ; 1 k m on the real
axis is
X
m
P (z)
Res ; xk
k=1
Q (z)
The sum of the residues at the simple poles zk ; 1 k j inside the integra-
tion contour is
j
X P (z)
Res ; zj
k=1
Q (z)
Integrate along segments on real axis, and let R ! 1 and " ! 0+ . This
gives

Z Z a1 " X1 Z ak+1 "


m Z R
f (z) dz = f (z) dz+ f (z) dz+ f (z) dz =
R k=1 ak +" am +"
Z a1 " X1 Z ak+1 "
m Z R
P (z) P (z) P (z)
= dz+ dz+ dz =
R Q (z) k=1 ak +" Q (z) am +" Q (z)
z = x
= =
dz = dx
Z a1 " X1 Z ak+1 " P (x)
m Z R
P (x) P (x)
= dx+ dx+ dx !
R Q (x) k=1 ak +"
Q (x) am +" Q (x)
Z 1
P (x)
! dx = I:
1 Q (x)

123
Integrate along R, where we have deg Q (z) deg P (z) + 2 and let R ! 1.
This gives
Z
Rdeg P
f (z) dz R ! 0:
R
Rdeg Q R

Integrate along 1; 2; : : : m, and let " ! 0+ . This gives


m Z
X X
m
P (z)
f (z) dz ! i( 0) Res ; xk :
k=1 k k=1
Q (z)
Using the Residue Theorem and letting R ! 1 and " ! 0+ , we obtain that
j
!
Xm
P (z) X P (z)
I i Res ; xk =2 i Res ; zk ;
k=1
Q (z) k=1
Q (z)
i.e.,

Z 1 Xm Xm
P (x) P (z) P (z)
PV dx = 2 i Res ; zj + i Res ; xk :
1 Q (x) j=1
Q (z) k=1
Q (z)

124
VII.7.1
1 2 3 P L K
LLL
Show that
Z 1
jsin xj
dx = +1:
0 x
Hint: Show that the area under the mth arc of jsin xj =x is 1=m.

Solution

VII.7.1

2 3 4 5

For x in the interval (m 1) + 4


x m 4
, we have
1
jsin xj sin =p ;
4 2
and
1 1
:
x m
We estimate the integrand

jsin xj 1 1
p :
x 2m
We have
Z m =4
jsin xj 1 1 C
dx p = :
(m 1) + 4 x 2m m
Hence

125
Z m m Z
X k =4
jsin xj jsin xj 1 1
dx dx = C 1 + + : : : + ! +1
0 x k=1 (k 1) + 4 x 2 m

as m ! 1.
We have used the fact that the harmonic series 1 + 12 + 13 + 14 + + m1 tends
R m+1 dx
to innity as as m ! 1, by taking the oversum for the integral 1 x
,
Z m+1
1 1 1 dx
1+ + + + = log (m + 1) ! +1
2 3 m 1 x
as m ! 1.

126
VII.7.2
Show that
Z R
x3 sin x
lim dx = :
R!1 R (x2 + 1)2 2e

Solution

VII.7.2

z1

-R 1 R
z2

Set
Z 1
x3 sin x
I= dx
1 (x2 + 1)2
and integrate

z 3 eiz z 3 eiz
f (z) = =
(z 2 + 1)2 (z i)2 (z + i)2
along the contour in Figure VII.7.2.
Residue at a double pole at z1 = i, where by Rule 2,

z 3 eiz d z 3 eiz 1
Res 2 ; i = lim 2 = :
(z 2 + 1) z!i dz (z + i) 4e
Integrate along 1 , and let R ! 1. This gives

127
Z
f (z) dz =
1
Z Z R
z 3 eiz z = x x3 eix
= dz = = 2 dx !
1
(z 2 + 1)2 dz = dx 2
R (x + 1)
Z 1 Z 1 3 Z 1 3
x3 eix x cos x x sin x
! 2 dx = 2 dx + i 2 dx =
1 (x2 + 1) 2
1 (x + 1)
2
1 (x + 1)
Z 1 3
x cos x
= 2 dx + iI:
2
1 (x + 1)

Integrate along 2 , and let R ! 1. Use Jordans Lemma this gives


Z Z
R3 iz R3
f (z) dz e jdzj < ! 0:
2
(R2 1)2 2 (R2 1)2 R
Using the Residue Theorem and letting R ! 1, we obtain that
Z 1 3
x cos x 1
2 dx + iI = 2 i ;
2
1 (x + 1) 4e
and hence, setting imaginary parts equal,
i.e.,
Z 1 3
x sin x
2 dx = :
2
1 (x + 1) 2e

128
VII.7.3
Evaluate the limits
Z R
x sin (ax)
lim dx; 1 < a < 1:
R!1 R x2 + 1
Show that they do not depend continuously on the parameter a.

Solution

VII.7.3a

z1
1
-R R
z2

Case 1 a < 0:
Set
Z 1
x sin (ax)
I= dx
1 x2 + 1
and integrate

zeiaz zeiaz
f (z) = =
z2 + 1 (z i) (z + i)
along the contour in Figure VII.7.3a.
Residue at a simple pole at z2 = i, where by Rule 3,

zeiaz zeiaz ea
Res ; i = = :
z2 + 1 2z z= i 2
Integrate along 1, and let R ! 1. This gives

129
Z Z Z R
zeiaz z = xe0i xeiax
f (z) dz = dz = = dx !
1 1
z2 + 1 dz = dx R x2 + 1
Z 1 Z 1 Z 1
xeiax x cos (ax) x sin (ax)
! 2
dx = dx + i dx =
1 x +1 1 x2 + 1 2
1 x +1
Z 1
x cos (ax)
= dx + iI:
1 x2 + 1

Integrate along 2 , and let R ! 1. Use Jordans Lemma this gives


Z Z
R R
f (z) dz 2
eiaz jdzj < 2 ! 0:
2
R 1 2 R 1 R

Using the Residue Theorem and letting R ! 1, we obtain that


Z 1
x cos (ax) ea
dx + iI + 0 = 2 i ;
1 x2 + 1 2
and hence, setting imaginary parts equal,

I= ea :

Case 2 a = 0:
Z R Z R
x sin (ax) 0
lim dx = lim =0
R!1 R x2 + 1 R!1 R x2 +1
Case 3 a > 0:

130
VII.7.3b

z1

-R 3 R
z2

Set
Z 1
x sin (ax)
J= dx
1 x2 + 1
and integrate

zeiaz zeiaz
f (z) = =
z2 + 1 (z i) (z + i)
along the contour in Figure VII.7.3b
Residue at a simple pole at z1 = i, where by Rule 3,

zeiaz zeiaz e a
Res ; i = = :
z2 + 1 2z z=i 2
Integrate along 3, and let R ! 1. This gives

Z Z Z R
zeiaz z = xe0i xeiax
f (z) dz = dz = = dx !
3 3
z2 + 1 dz = dx R x2 + 1
Z 1 Z 1 Z 1
xeiax x cos (ax) x sin (ax)
! 2
dx = dx + i dx =
1 x +1 1 x2 + 1 2
1 x +1
Z 1
x cos (ax)
= dx + iJ:
1 x2 + 1
Integrate along 4, and let R ! 1. Use Jordans Lemma this gives

131
Z Z
R R
f (z) dz eiaz jdzj < ! 0:
4
R2 1 4
R2 1 R
Using the Residue Theorem and letting R ! 1, we obtain that
Z 1
x cos (ax) e a
dx + iI + 0 = 2 i ;
1 x2 + 1 2

and hence, setting imaginary parts equal,

I = e a:
and we have the solution
8
Z 1 < ea ; a < 0;
x sin (ax)
dx = 0 a=0
x2 + 1 :
1 e a; a > 0:
Not continuous at a = 0 becauce sin 0 = 0.

132
VII.7.4
By integrating z a 1 eiz around the boundary of a domain in the rst
quadrat bounded by the real and imaginary axes and a quater-
circle, show that

Z R
lim xa 1
cos xdx = (a) cos ( a=2) ; 0 < a < 1;
R!1 0
Z R
lim xa 1
sin xdx = (a) sin ( a=2) ; 0 < a < 1;
R!1 0

where (a) is the gamma function dened by


Z 1
(a) = ta 1 e t dt:
0
Remark: The formula for the sine integral holds also for 1 < a < 0. To see
this, integrate by parts.

Solution

VII.7.4

3
2

1 R

Set
Z R Z R
a 1
I = lim x cos xdx J = lim xa 1
sin xdx
R!1 0 R!1 0
where 0 < a < 1, and integrate

133
f (z) = z a 1 eiz = jzja 1 i(a 1) arg z iz
e e
along the contour in Figure VII.7.4
Integrate along 1 , and let R ! 1 and " ! 0+ . This gives

Z
f (z) dz =
1
Z Z R
z = xe0i
a 1 i(a 1) arg z iz
= jzj e e dz = = xa 1 eix dx !
dz = dx 0
1
Z 1 Z 1 Z 1
a 1 ix a 1
! x e dx = x cos xdx + i xa 1
sin xdx = I + iJ:
0 0 0

Integrate along 2 , and let R ! 1. Use Jordans Lemma this gives


Z Z
a 1
f (z) dz R eiz jdzj < Ra 1 ! 0:
2 2
R1 a
Integrate along 3, and let R ! 1 and " ! 0+ . This gives

Z
f (z) dz =
3
Z Z R
z = xe i=2
a 1 i(a 1) arg z iz
= jzj e e dz = i=2 = xa 1 e ia=2 e x dx !
dz = e dx 0
3
Z 1 Z 1
! xa 1 e ia=2 e x dx = e ia=2 xa 1 e x dx = e ia=2 (a) =
0 0
= (a) cos( a=2) (a) sin ( a=2) :

Using the Residue Theorem and letting R ! 1, we obtain that

I + iJ (a) cos( a=2) (a) sin ( a=2) = 0:


We equate real and imaginary parts

I= (a) cos( a=2) J= (a) sin ( a=2) :

Formulas for the Gamma function, Mathematics Handbook page 278

134
(z + 1) = z (z)

Z R Z R
a 1 a 1 R
x cos xdx = x sin x 0
(a 1) xa 2
sin xdx
0 0
Z R Z R
a 2 a 1 R
(a 1) x sin xdx = x sin x 0
xa 1
cos xdx
0 0

Z R Z R
a 2 a 1
(a 1) x sin xdx = R sin R xa 1
cos xdx
0 0

Z R Z R
a 2 Ra 1 sin R 1
x sin xdx = xa 1
cos xdx
0 a 1 a 1 0

Z R
sin R 1
xa 2
sin xdx = (a) cos ( a=2)
0 (1 a) R1 a a 1

Z R
(a + 1)
lim xa 1
sin xdx = cos (a + 1) =
R!1 0 a 2
= (a) [cos ( a=2) cos ( =2) sin ( a=2) sin ( =2)] =
= (a) sin ( a=2) :

135
VII.7.5
Show that
Z R Z R p
2 2
lim sin x dx = lim cos x dx = p ;
R!1 0 R!1 0 2 2
by integrating eiz around the boundary of the pie-slice domain determined
by 0 < arg z < =4 and jzj < R. Remark. These improper integrals are
called the Fresnel integrals. The identities can also be deduced from the
preceding exercise by changing variable.

Solution

VII.7.5

3
2

1 R

Set
Z R Z R
2
I = lim sin x dx J = lim cos x2 dx
R!1 0 R!1 0
where 1 < a < 0, and integrate
2
f (z) = eiz
along the contour in Figure VII.7.5
Integrate along 1 , and let R ! 1. This gives

136
Z Z Z R
z = xe0i
iz 2 2
f (z) dz = e dz = = eix dx !
dz = dx 0
1
Z 11 Z 1 Z 1
2
! eix dx = cos x2 dx + i sin x2 dx = J + iI:
0 0 0

Integrate along 2 as we can parametrize as z = Reit where 0 t =4,


and let R ! 1. Use that 4x= sin 2x for 0 x =4, this gives

Z Z
2 z = Reit
f (z) dz = eiz dz = =
2 2
dz = iReit dt
Z =4 Z =4
2 e2it 2
= eiR iReit dt R eiR (cos(2t)+i sin(2t)) dt =
0 0
Z =4 Z =4 h i =4
R2 sin 2t 4R2 t= 4R2 t=
=R e dt R e ds = e =
0 0 4R 0
R2
= 1 e ! 0:
4R 4R
i=4
Integrate along 3 as we can parametrize as z = xe where 0 x R,
and let R ! 1. This gives

Z Z Z R
iz 2 z = xe i=4 x2 i=4
f (z) dz = e dz = = e e dx !
dz = e i=4 dx 0
3 3
Z 1 Z
2 1 + i 1 x2
! e xe i=4
dx = p e dx =
0 2
Z 1 0
Z 1
1 x2 1 x2
= p e dx i p e dx:
2 0 2 0
We will use integral (41) from Mathematics Handbook page 177 with a = 1,
Z 1 r
ax2 1
e dx = :
0 2 a
and hence

137
Z Z 1 Z 1 p p
1 x2 1 x2
f (z) dz = p e dx ip e dx = p i p :
3
2 0 2 0 2 2 2 2

Using the Residue Theorem and letting R ! 1, we obtain that


p p
J + iI + 0 p i p = 0:
2 2 2 2
We equate real and imaginary parts
p p
I= p J= p :
2 2 2 2

138
VII.8.1
1 2 3 P L K

Evaluate the residue at 1 of the following functions.


3
(a) z2z 1 (c) zz2 +11 z n e1=z , n = 0; 1; : : :
(e) q
1 9
(b) (z2 +1)2 (d) zz6 +11 (f) zz ab

Note. There are two possibilities for (f), one for each branch of the square
root.
a)

z 1 1 1 1 1
= 1 = 1+ + 4 + ::: ;
z2 1 z 1 z2 z z 2 z
hence

z
Res ;1 = 1:
z2 1
b)

1 1
=O as z ! 1;
(z 2 + 1)2 z4
hence

1
Res ; 1 = 0:
(z 2 + 1)2
c)

z3 + 1 z (z 2 1) + z + 1 z 1
2
= 2
=z+ 2 + 2 =
z 1 z 1 z 1 z 1
1 1 1 1 1 1 1
=z+ 1 + 2 =z+ 1 + 2 + 4 + ::: + 2 ;
z 1 z2 z 1 z z z z 1

hence

z3 + 1
Res ;1 = 1:
z2 1

139
d)

z9 + 1 z 3 (z 6 1) + z 3 + 1 3 z3 + 1
= = z + ;
z6 1 z6 1 z6 1
hence

z9 + 1
Res ; 1 = 0:
z6 1
e)

1 1 1 1 1
z n e1=z = z n 1 + + 2
+ 3
+ 3
+ + ::: =
z 2!z 3!z 3!z 4!z 4
1 1 1
= zn + zn 1 + zn 2 + zn 3 + zn 4
+:
2! 3! 4!
1
We can see that the coe cient of z
is
1
;
(n + 1)!
hence
1
Res z n e1=z ; 1 = ; n = 0; 1; : : : :
(n + 1)!
f)
r s r
z a 1 a=z 1 aw
= = ;
z b 1 b=z 1 bw
analytic at w = 0:
1 1 aw 1=2 a(1 bw)+b(1 aw) 1 p1
coe cient of w is g 0 (0) = 2 1 bw (1 bw)2
= 2 1
(b a) =
w=0
b a
2
:

140
7.8.2
1 2 3 P L K

Show by integrating around the dogbone contour that


Z 1
x4 35
p dx = :
0 x (1 x) 128

Solution

VII.8.2

1 2

4 3

Set
Z 1
x4
I= p dx
0 x (1 x)
and integrate

z4 z4
f (z) = p =p p
z (1 z) jzjei arg z=2 j1 zjei arg0 (1 z)=2

along the contour in Figure VII.8.2.

141
VIII.8.2
arg z arg0 z

y y

0 0 0 0 0 0

-2 -1 1 2
x -2 -1 1 2
x
0 0 0 2 2 2

arg z arg0 (1 z)

y y

0 0 0 2 2 2 2

-2 -1 1 2
x -2 -1 1 2
x
0 0 0 0 0 0 0

4
The argument for the function f (z) = p pz
jzjei arg z=2 j1 zjei arg0 (1 z)=2

3/2 3/2 3/2 /2 /2

-2 -1 1 2
x
/2 /2 /2 0 /2 /2

Residue at a simple pole at z1 = 1 , where by result from Exercise VII.8.13


and Rule 1,

2 3
1
1 1 1
Res [f (z) ; 1] = Res f ;0 = Res 4 q w4
; 05 =
w2 w w2 1 1 1
w w

1 1 1 d4 1
= Res p = lim p =
w5 w 1 w!0 4! dz 4 w 1
35 35 35 35
= 9=2
= 9=2 i9 arg(w 1)=2
= e i=2 = i
128 (w 1) 128 (jw 1j) e 128 128

142
Integrate along 1, and let " ! 0+ . This gives
Z
f (z) dz =
1
Z 1 " Z 1 "
z4 x4
= p p dz = p p dx =
" jzjei arg z=2 j1 zjei arg0 (1 z)=2 " jxjei 0=2 j1 xjei 0=2
Z 1 " Z 1
x4 x4
= p dx ! p dx = I:
" x (1 x) 0 x (1 x)
Integrate along 2, and let " ! 0+ . This gives
Z
(1 + ")4
f (z) dz p 2 " 2 "9=2 ! 0:
2 (1 + ") "
Integrate along 3, and let " ! 0+ . This gives
Z
f (z) dz =
3
Z " Z "
z4 1
= p p dz = p p dx =
1 " jzjei arg z=2 j1 zjei arg0 (1 z)=2 1 " jxjei 0=2 j1 xjei2 =2
Z 1 " Z 1
x4 x4
= p dx ! p dx = I:
" x (1 x) 0 x (1 x)
Integrate along 4, and let " ! 0+ . This gives
Z
"4
f (z) dz p 2 " 2 "9=2 ! 0:
4 " (1 ")
Using the Residue Theorem and letting R ! 1 and " ! 0+ , we obtain that

35
I +0+I +0=2 i i ;
128
i.e.,
Z 1
x4 35
p dx = :
0 x (1 x) 128

143
7.8.3
1 2 3 P L K

Fix an integer n, positive or negative. Determine for which complex


values of the parameter a the integral
Z 1
xn
dx
a
0 x (1 x)1 a
converges, and evaluate it.

Solution. ( Obs fel i lsningen argumentet)


x 2 R+
1 x 2 R+
a = Re a + i Im a
xa = xRe a+i Im a = xRe a xi Im a
The integral is generalized in 0 and 1 " 2 R, x = 0
g (x) = xa1 n
f (x) 1
= (1 x) 1 a ! 1, as x ! 0
Rg(x)
" R"
0
g (x) dx < 1 , a n < 1 , 0
f (x) dx < 1 , a < n + 1.
The integral is generalized in 0 and 1 " 2 R, x = 0
1
g (x) = (1 x) 1 a

f (x) n
g(x)
= xxa ! 1, as x ! 1
R1 R"
"
g (x) dx < 1 , 1 a<1, 0
f (x) dx < 1 , a > 0.
Thus
R1
0
f (x) dx < 1 , 0 < a < n + 1.
Show that xi Im a begrnsad, set 2 [arg x]
xi Im a = ei Im a(log x+i ) = ei Im a log x = 1
On principal arg 0

Determination for which complex values of the parameter a the integral con-
verges

144
VII.8.3

1 2

4 3

Set
Z 1
xn
I= a dx
0 xa (1 x)1
and integrate
zn zn
f (z) = =
z a (1 z)1 a
jzja eai arg z j1 zj1 a e(1 a)i arg0 (1 z)

along the contour in Figure VII.8.3.

145
VIII.8.3
arg z arg0 z

y y

0 0 0 0 0 0

-2 -1 1 2
x -2 -1 1 2
x
0 0 0 2 2 2

arg z arg0 (1 z)

y y

0 0 0 2 2 2 2

-2 -1 1 2
x -2 -1 1 2
x
0 0 0 0 0 0 0

zn
The argument for the function f (z) = jzja eai arg z j1 zj
1 a (1 a)i arg (1 z)
e 0

2 2 2 2 2

-2 -1 1 2
x
0

Residue at a simple pole at z3 = 1 , where by result from Exercise VII.8.13


and Rule 1,

" #
1
1 1 1 wn
Res [f (z) ; 1] = Res f ;0 = Res ;0 =
w2 w w2 1 1 1 a
wa
1 w
p p
1jei arg(w 1)=2
2
w2 1 jw2
= Res ;0 = lim = ei =2
= i
w(1 + w2 ) w!0 w2 + 1

Integrate along 1, and let R ! 1 and " ! 0+ . This gives

146
Z
f (z) dz =
1
Z 1 " Z 1 "
zn zn
= = dx =
" jzja eai arg z j1 zj1 a e(1 a)i arg0 (1 z) " jzja eai 0 j1 zj1 a e(1 a)i 0
Z 1 "
zn
= dx !
" z a (1 z)1 a
Z 1
zn
! dx = I:
a
0 z (1 z)1 a

Integrate along 2, and let " ! 0+ . This gives


Z
(1 + ")4 2
f (z) dz 2 " ! 0:
4
(1 + ")a (1 (1 + "))1 a
" a

Integrate along 3, and let R ! 1 and " ! 0+ . This gives

Z
f (z) dz =
Z 3
" Z "
zn zn
= = dx =
1 " jzja eai arg z j1 zj1 a e(1 a)i arg0 (1 z) a ai 0
1 " jzj e j1 zj1 a e(1 a)i 2
Z 1 "
2 ia zn
= e dx !
" z a (1 z)1 a
Z 1
2 ia zn
! e dx = e2 ia I:
a
0 z (1 z)1 a

Integrate along 4, and let " ! 0+ . This gives


Z
"n
f (z) dz 1 a 2 " 2 "n+1 a
! 0:
4
"a (1 ")

Using the Residue Theorem and letting R ! 1 and " ! 0+ , we obtain that

(a 1) (a n)
I + 0 e2 ia
I +0=2 i ( 1)n :
n!

147
Solve for I, we obtain that

( 1)n 2 i (a 1) (a n)
( 1)n 2 i (a 1)n!(a n)
(a 1) (a n)
I= n!
= = ( 1)n ;
1 e2 ia e ia e ia sin ( a) n!
i.e.,

Z 1
xn (a 1) (a n) 0 < a < 1;
1 a dx = ( 1)n ;
0 xa (1 x) sin ( a) n! n = 0; 1; 2; : : :

148
VII.8.4
Show that
Z p
1
1 x2 p
dx = 2 1 :
1 1 + x2

Solution

VII.8.4

1 2

4 3

Set
Z 1
p
1 x2
I= dx
1 1 + x2
and integrate
p p p
1 z2 j1 + zjei arg (1+z)=2
j1 zjei arg0 (1 z)=2
f (z) = 2 = 2
z +1 z +1
along the contour in Figure VII.8.4.

149
VIII.8.4
arg z arg0 z

y y

0 0 0 0 0 0

-2 -1 1 2
x -2 -1 1 2
x
0 0 0 2 2 2

arg (1 + z) arg0 (1 z)

y y

0 0 0 0 2 2 2 2

-2 -1 1 2
x -2 -1 1 2
x
0 0 0 0 0 0 0 0

4
The argument for the function f (z) = p pz
jzjei arg z=2 j1 zjei arg0 (1 z)=2

3/2 3/2 /2 /2

-2 -1 1 2
x
/2 /2 0 0 /2 /2

Residue at a simple pole at z1 = i, where by Rule 3,

"p p #
j1 + zjei arg(1+z)=2 j1 zjei arg(1 z)=2
Res ;i =
(z i) (z + i)
p p
j1 + zjei arg(1+z)=2 j1 zjei arg(1 z)=2
= =
2z
z=i
p p p p
j1 + ijei arg(1+i)=2 j1 ijei arg(1 i)=2 j1 + ije(i =4)=2 j1 ije(i7 =4)=2
=
2i pp pp 2i
i
p p
2 2e 2( 1) 2 i
= = = =p
2i 2i 2i 2

150
Residue at a simple pole at z2 = i, where by Rule 3,

"p p #
j1 + zjei arg(1+z)=2 j1 zjei arg(1 z)=2
Res ; i =
z2 + 1
p p
j1 + zjei arg(1+z)=2 j1 zjei arg(1 z)=2
= =
2z
z= i
p i arg(1 i)=2
p i arg(1+i)=2
p ( i =4)=2
p
j1 ije j1 + ije j1 ije j1 + ije(i =4)=2
= = =
2i pp pp 2i
p
2 2 2 1
= = = p i:
2i 2i 2
Residue at a simple pole at z3 = 1 , where by result from Exercise VII.8.13
and Rule 1,

2 q 3
1
1 1 1 1 w2
Res [f (z) ; 1] = Res f ;0 = Res 4 ; 05 =
w2 w w2 1+ w12
p p
1jei arg(w 1)=2
2
w2 1 jw2
= Res ;0 = lim = ei =2
= i
w(1 + w2 ) w!0 w2 + 1
Integrate along 1, and let " ! 0+ . This gives

Z
f (z) dz =
Z 1 " p p p p
1
Z 1 "
j1 + zjei arg (1+z)=2
j1 zjei arg0 (1 z)=2
j1 + xjei 0=2 j1 xjei2 =2
= 2
dz = dx !
1+" z +1 1+" 1 + x2
Z 1 "
p
1 x2
! dx = I:
1+" 1 + x2
Integrate along and let " ! 0+ . This gives
2,
q
Z 1 (1 ")2
f (z) dz 2 " 2 "3=2 ! 0:
2
1 + (1 ")2

151
Integrate along 3, and let " ! 0+ . This gives

Z
f (z) dz =
Z 1+" p p p p
3
Z 1 "
j1 + zjei arg (1+z)=2
j1 zjei arg0 (1 z)=2
j1 + xjei 0=2 j1 xjei 0=2
= 2
dz = dx !
1 " z +1 1+" x2 + 1
Z 1 "
p
1 x2
! dx = I:
1+" 1 + x2

Integrate along 4, and let " ! 0+ . This gives


q
Z 1 (1 ")2
f (z) dz 2 " 2 "3=2 ! 0:
4
1 + (1 ")2

Using the Residue Theorem and letting R ! 1 and " ! 0+ , we obtain that

1 1
I +0 I +0=2 i p i+ p i i
2 2
i.e.,
Z p
1
1 x2 p
dx = 2 1 :
1 1 + x2

152
VII.8.5
1 2 3 P L K

Show that the sum of the residues of a rational function on the


extended complex plane is equal to zero.

Solution
I X
f (z) dz = 2 i Residues in nite plane;
jzj=R
I
f (z) dz = 2 i Res [f; 1]
jzj=R
H H P
Since = , we get 2 i Res = 0.

153
VII.8.6
1 2 3 P L K

Find the residue of z= (z 2 + 1) at each pole in the extended complex


plane, and check that the sum of the residues is zero.

Solution
Residue at a simple pole at z1 = i, where by Rule 3,

z z 1
Res ;i = = :
z2 +1 2z z=i 2
Residue at a simple pole at z1 = i, where by Rule 3,

z z 1
Res ; i = = :
z2 +1 2z z= i 2
For residue at 1, we have that

z2 1
a 1 = lim zf (z) = lim 2
= lim = 1:
z!1 z!1 z + 1 z!1 1 + 1=z 2

hence

z
Res ;1 = a 1 = 1;
z2 +1
and the sum of the residues is zero in the extended complex plane.

z a 1 1
(Since z 2 +1
= z
+O z2
as z ! 1).

154
VII.8.7
1 2 3 P L K

Find the sum of the residues of (3z 4 + 2z + 1) = (8z 5 + 5z 2 + 2) at its


poles in the (nite) complex plane.

Solution
For residue at 1, we have that

3z 5 + 2z 2 + z 3 + 2=z 3 + 1=z 4 3
a 1 = lim zf (z) = lim 5 2
= lim 3 4
= ;
z!1 z!1 8z + 5z + z z!1 8 + 5=z + 1=z 8
hence
3
Res [f; 1] =
:
8
We have that the sum of the residues in the complex nite plane is Res [f (z) ; 1],
thus the sum of residues in nite complex plane is 3=8.
a 1 1
f (z) = z
+O z2
as z ! 1, so again

155
VII.8.8
1 2 3 P L K

Fix n 1 and k 0. Find the residue of z k = (z n 1) at 1 by expand-


ing 1= (z n 1) in a Laurent series. Find the residue of z k = (z n 1)
at each nite pole, and verify that the sum of all the residues are
zero.

Solution.
zk
Let f (z) = zn 1
. At 1, we have

zk zk n
f (z) = = 1 = zk n
1+z n
+z 2n
+ : : : = zk n
+z k 2n
+z k 3n
+: : : :
zn 1 1 zn

Then Res [f; 1] = 1 if k mn = 1, thus k = mn 1 for some integer


m 1, and otherwise Res [f; 1] = 0.

Let wj be an nth root of unity, i.e wj = e2 ij=n where 0 j n 1. Then


f (z) has simple poles at w0 ; : : : ; wn 1 .
Residue at a simple pole at zj = wj , where by Rule 3 and that wjn = 1

zk z k+1 wjk+1 wjk+1


Res [f; wj ] = = = = :
nz n 1
z=wj nz n z=wj nwjn n
Let S be the sum of residues, then

1 X k+1
n 1
1 1 1 wn
S= wj = 1 + w + w2 + + wn 1
= = 0;
n j=0 n n 1 w

unless w = 1.
2 i
If w = 1, then wjk+1 = wj = 1 for every j and w = 1 , e n (k+1) = 1 ,
k+1
n
= m, thus k = mn 1 for some integer m 1.
Thus we have that the sum of the residues in the complex plane are zero.

156
VII.8.9
1 2 3 P L K

Show that f (z) is analytic at 1, then

Res [f (z) ; 1] = lim z (f (z) f (1)) :


z!1

Solution
Since f (z) is analaytic at 1, we have

a 1 1
f (z) = a0 + +O :
z z2
Then

a 1 1 1
z (f (z) f (1)) = z a0 + +O a0 =a 1 +O !a 1
z z2 z
as z ! 1.
From this follows that

lim z (f (z) f (1)) = a 1 = Res [f (z) ; 1] :


z!1

157
VII.8.10
1 2 3 P L K

Let D be an exterior domain. Suppose that f (z) is analytic on


D [ @D and at 1. Show that
Z
1 f( )
d = f ( ) f (1) ; z 2 D:
2 i @D z

Solution.
Cauchy integral formula for exterior domain. If D is an exterior domaina,
f (z) is analytic in D and att 1, then
Z
1 f( ) f( )
d = f (z) + Res ; 1 = f (z) f (1)
2 i @D z z
IIII

f( ) f( )
Res ;1 = lim = f (1) :
z !1 z
Or
a 1 a 2
f ( ) = a0 + + 2 + :::

f( ) a0 1
= +O 2
z z

Z Z
1 f( ) 1 f( )
d = f (z) + d =
2 i @D z 2 ijzj=R z
Z
1 a0
= f (z) d = f (z) a0 :
2 i jzj=R z

158
VII.8.11
1 2 3 P L K

If f (z) is not integrable at 1, we dene the principal value


Z 1 Z R
PV f (x) dx = lim PV f (x) dx:
1 R!1 R

Show that
8
Z 1 < i ; Im a > 0;
1
PV dx = 0; Im a = 0;
1 x a :
i ; Im a < 0:

Solution
Set a = a1 + a2 i, where a1 and a2 is xed. We compute the principal value
for the following 3 cases.
Case 1, Im a < 0 , a2 < 0. Integrate and let R ! 1. This gives

Z R Z R
dx dx R a1 a2 i
= = [log (x a1 a2 i)]RR = log =
R x a R x a1 a2 i R a1 a2 i
R a1 a2 i
= log + i arg (R a1 a2 i) i arg ( R a1 a2 i) !
R a1 a2 i
!0+i 0 i = i :

Case 2, Im a = 0 , a2 = 0. Integrate and let R ! 1 and " ! 0+ . This


gives

Z R Z R Z a1 " Z R
dx dx dx dx
= = + = [log jx a1 j]a1R " +[log jx a1 j]R
a1 +" =
R x a R x a1 R x a 1 a1 +" x a 1
R a1
= log " log jR + a1 j + log jR a1 j log " = log ! 0:
R + a1

Case 3, Im a > 0 , a2 > 0. Integrate and let R ! 1. This gives

159
Z R Z R
dx dx R a1 a2 i
= = [log (x a1 a2 i)]RR = log =
R x a R x a1 a2 i R a1 a2 i
R a1 a2 i
= log + i arg (R a1 a2 i) i arg ( R a1 a2 i) !
R a1 a2 i
!0+i 2 i =i :

We have that
8
Z 1 < i ; Im a > 0;
1
PV dx = 0; Im a = 0;
1 x a :
i ; Im a < 0:

160
VII.8.12
1 2 3 P L K

Suppose that P (z) and Q (z) are polynomials such that the degree
of Q (z) is strictly greater than the degree of P (z). Suppose that
the zeros x1 ; : : : ; xm of Q (z) on the real axis are all simple, and set
x0 = 1. Show that

Z 1 X X
P (x) P (x) P (x)
PV dx = 2 i Res ; zj + i Res ; zk ;
1 Q (x) Q (x) Q (x)

summed over the poles zj of P (z) =Q (z) in the open upper half-
plane and summed over the xk : s including 1. Hint. Use the preceding
exercise. See also Exercise 6.7

Solution.
1
By VII.8.11, formula holds for By Exercise b.7 (p. 215.) it holds if
h
z a i
P (x)
deg Q deg P + 2;in initial case Res Q(x) ; 1 = 0. By limiticity, it hods
form sums, hence whenever deg Q deg P + 1.

161
VII.8.13
1 2 3 P L K

Show that the analytic dierential f (z) dz transforms the change of variable
w = 1=z to f (1=w) dw=w2 . Show that the residue of f (z) at z = 1
coincides with that of f (1=w) =w2 at w = 0.

Solution P
We have that Res [f (z) ; 1] = a1 if f (z) = 11 aj z j , jzj > R.
We transform f (z) dz by the change of variable z = w1 , then dz = 1
w2
dw,
and f (z) dz = f w1 w12 dw.
Now we take the residue for f (1=w) dw=w2 at w = 0, we have that

1 X aj X
1 1
1 1 j 2
f = = ( aj ) w =
w w2 w2 1 wj 1

= a 4 w2 a 3w a 2 a 1w 1
a0 w 2
;

thus

1 1
Res f ;0 = a 1
w w2
We have that

1 1
Res [f (z) ; 1] = a1 = Res f ;0 :
w w2

162
VII.9.1
Show using residue theory that
Z 1
p
1 2 3
p
3
dx = :
0 x2 x3 3

Solution
Set
Z 1
1
I= p
3
dx
0 x2 x3
and integrate
Z 1
1 1
f (z) = p
3
dx = q p
0 z2 z3 3
jzj2 e2i arg z=3 3 j1 zjei arg0 (1 z)=3

along the contour in Figure VII.9.1.

163
VIII.9.1
arg z arg0 z

y y

0 0 0 0 0 0

-2 -1 1 2
x -2 -1 1 2
x
0 0 0 2 2 2

arg z arg0 (1 z)

y y

0 0 0 2 2 2 2

-2 -1 1 2
x -2 -1 1 2
x
0 0 0 0 0 0 0

The argument for the function f (z) = p


3
p1
jzj2 e2i arg z=3 3 j1 zjei arg0 (1 z)=3

4/3 4/3 4/3 2/3 /3 /3

-2 -1 1 2
x
2/3 2/3 2/3 0 /3 /3

Residue at a simple pole at z1 = 1 , where by result from Exercise VII.8.13


and Rule 1,

2 3
1 1 1 1
Res [f (z) ; 1] = Res f ;0 = Res 4 q ; 05 =
w2 w w2 3 1
1 1
w2 w

1 1 1 i=3
= Res p ;0 = lim p = e
w w 1
3
w!0 3
jw 1jei arg(w 1)=3

Integrate along 1, and let " ! 0+ . This gives

164
Z
f (z) dz =
1
Z 1 "
1
= q p dz =
" 3
jzj2 e2i arg z=3 3 j1 zjei arg0 (1 z)=3
Z 1 " Z 1 "
1 2 i=3 1
= q p dx = e p
3
dx !
" 3
jxj2 e2i 0=3 3 j1 xjei 2 =3 " x2 x3
Z 1
2 i=3 1 2 i=3
!e p
3
dx = e I:
0 x2 x3
Integrate along 2, and let " ! 0+ . This gives
Z
"4
f (z) dz p 2 " 2 "9=2 ! 0:
4 " (1 ")

Integrate along 3, and let " ! 0+ . This gives

Z
f (z) dz =
3
Z "
1
= q p dz =
1 " 3
jzj2 e2i arg z=3 3 j1 zjei arg0 (1 z)=3
Z " Z 1 "
1 1
= q p dx = p3
dx !
x 2 x 3
1 " 3
jxj2 e2i 0=3 3 j1 xjei 0=3 "
Z 1
1
! p3
dx = I:
0 x2 x3
Integrate along 4, and let " ! 0+ . This gives
Z
"4
f (z) dz p 2 " 2 "9=2 ! 0:
4 " (1 ")

Using the Residue Theorem and letting R ! 1 and " ! 0+ , we obtain that

165
2 i=3 i=3
e I +0 I +0=2 i e :
Solve for I, we obtain that
i=3
p
2 ie 2 i 2 3
I= 2 i=3
= i=3 i=3
= = ;
1 e e e sin ( =3) 3
i.e.,
Z 1
p
1 2 3
p
3
dx = :
0 x2 x3 3

166
VII.9.2
Show using residue theory that
Z 1
1
p dx = :
0 x x2 1 2

Solution

VII.9.2

3 4 1
-R 5 8 7 R

Set
Z 1
1
I= p dx
0 x x2 1
and integrate
1 1
f (z) = p = p p
z z2 1 i
z jz + 1je 0arg (z+1)=2 jz 1jei arg (z 1)=2

along the contour in Figure VII.9.2.

167
VIII.9.2
arg0 z arg z

y y

0 0 0 0 0 0

-2 -1 1 2
x -2 -1 1 2
x
2 2 2 0 0 0

arg0 (z + 1) arg0 (z 1)

y y

0 0 0 0 0 0

-2 -1 1 2
x -2 -1 1 2
x
2 2 2 2 0 0

1p
The argument for the function f (z) = p
z jz+1jei arg0 (z+1)=2 jz 1jei arg (z 1)=2

/2 /2 0 0

-2 -1 1 2
x
0 0 /2 /2

Residue at a simple pole at z1 = i, where by Rule 3,

" #
1
Res p p ;0 =
z jz + 1jei arg(z+1)=2 jz 1jei arg(z 1)=2

1 1
p p = i=2
= i
jz + 1jei arg(z+1)=2 jz 1jei arg(z 1)=2 e
z=0

Integrate along 1, and let R ! 1 and " ! 0+ . This gives

168
Z
f (z) dz =
1
Z R
1
= p p dz =
1+" z jz + 1jei arg0 (z+1)=2 jz 1jei arg (z 1)=2
Z R
1
= p p dx =
1+" x jx + 1jei 0=2 jx 1jei 0=2
Z R
1
= p dx !
1+" x x2 1
Z 1
1
! p dx = I:
1 x x2 1
Integrate along 2, and let " ! 0+ . This gives
Z
1
f (z) dz p R ! 0:
2
R R2 1 R

Integrate along 3, and let R ! 1 and " ! 0+ . This gives

Z
f (z) dz =
3
Z 1 "
1
= p p dz =
i
z jz + 1je 0arg (z+1)=2 jz 1jei arg (z 1)=2
R
Z 1 "
1
= p p dx =
x jx + 1je i =2 jx 1jei =2
R
Z R Z R
1 x= t 1
p dx = = p dt = I:
1 " x x
2 1 dx = dt 1 t t
2 1

Integrate along 4, and let " ! 0+ . This gives


Z
"4
f (z) dz p 2 " 2 "9=2 ! 0:
4 " (1 ")
Integrate along 5, and let R ! 1 and " ! 0+ . This gives

169
Z
f (z) dz =
5
Z R
1
= p p dz =
1 " z jz + 1jei arg0 (z+1)=2 jz 1jei arg (z 1)=2
Z R
1
= p p dx =
1 " x jx + 1jei =2 jx 1jei ( )=2
Z R Z R
1 x= t 1
p dx = = p dt = I:
1 " x x2 1 dx = dt 1 t t2 1

Integrate along 6, and let " ! 0+ . This gives


Z
1
f (z) dz p R ! 0:
6
R R2 1 R
Integrate along 7, and let R ! 1 and " ! 0+ . This gives

Z
f (z) dz =
7
Z 1+"
1
= p p dz =
R z jz + 1jei arg0 (z+1)=2 jz 1jei arg (z 1)=2
Z 1+"
1
= p p dx =
R x jx + 1jei 2 =2 jx 1jei 0=2
Z R
1
= p dx !
2
1+" x x 1
Z 1
1
! p dx = I:
1 x x2 1
Integrate along 8, and let " ! 0+ . This gives
Z
"4
f (z) dz p 2 " 2 "9=2 ! 0:
4 " (1 ")

Using the Residue Theorem and letting R ! 1 and " ! 0+ , we obtain that

170
I + 0 + I + 0 + I + 0 + I + 0 = 2 i ( i) ;
i.e.,
Z 1
1
p dx = :
0 x x2 1 2

171
VII.9.3
Show using residue theory that
Z 1
1
p dx = p
2 x2
1 (x + 1) 1 2

Solution

VII.9.3

8 2
7 1
-R 5 3 R
4

Set
Z 1
1
I= p dx
1 (x2 + 1) 1 x2
and integrate

1 1
f (z) = p = p p
(z 2 + 1) 1 x2 (z i) (z + i) j1 + zjei arg (1+z)=2 j1 zjei arg0 (1 z)=2

along the contour in Figure VII.9.3.

172
VIII.9.3
arg z arg0 z

y y

0 0 0 0 0 0

-2 -1 1 2
x -2 -1 1 2
x
0 0 0 2 2 2

arg (1 + z) arg0 (1 z)

y y

0 0 0 0 2 2 2 2

-2 -1 1 2
x -2 -1 1 2
x
0 0 0 0 0 0 0 0

p 1 p
The argument for the function f (z) =
(z i)(z+i) j1+zjei arg (1+z)=2 j1 zjei arg0 (1 z)=2

3/2 3/2 /2 /2

-2 -1 1 2
x
/2 /2 0 0 /2 /2

Residue at a simple pole at z1 = i, where by Rule 3,

" #
1
Res p p ;i =
(z i) (z + i) j1 + zjei arg(1+z)=2 j1 zjei arg(1 z)=2

1
p p =
j1 + zjei arg(1+z)=2 j1 zjei arg(1 z)=2 z=i
(z + i)
1 1
p p = p p
(i + i) j1 + zjei arg(1+i)=2 j1 zje i arg(1 i)=2 2i j1 + zje(i =4)=2 j1 zje(i7 =4)=2

1 1 1 1
= pp pp = p = p = p i
2i 2 2e i 2i 2( 1) 2 2i 2 2

173
Residue at a simple pole at z2 = i, where by Rule 3,

" #
1
Res p p ; i =
(z i) j1 + zjei arg(1+z)=2 j1 zjei arg(1 z)=2

1
= p p =
(z i) j1 + zjei arg(1+z)=2 j1 zjei arg(1 z)=2
z= i
1 1
p p = p p =
( i i) j1 + zjei arg(1 i)=2 j1 zjei arg(1+i)=2 2i j1 + zje( i =4)=2
j1 zje(i =4)=2

1 1 1 1
= pp pp = p = p = p i
2i 2 2e0i 2i 2 2 2i 2 2

Integrate along 1, and let " ! 0+ . This gives

Z
f (z) dz =
1
Z 1 "
1
= p p dz =
1+" j1 + zjei arg (1+z)=2 j1 zjei arg0 (1 z)=2
(z 2 + 1)
Z 1 " Z 1 "
1 1
= p p dx = p dx !
(z 2 + 1) j1 + xjei0=2 j1 xjei2 =2 2 x2
1+" 1+" (x + 1) 1
Z 1
1
p dx = I:
2 1 x2
1 (x + 1)

Integrate along 2, and let " ! 0+ . This gives


Z
"4
f (z) dz p 2 " 2 "9=2 ! 0:
4 " (1 ")

Integrate along 3, and let " ! 0+ . This gives

174
Z
f (z) dz =
1
Z 1+"
1
= p p dz =
1 " (z 2 + 1) j1 + zjei arg (1+z)=2 j1 zjei arg0 (1 z)=2
Z 1+" Z 1 "
1 1
= p p dx = p dx !
1 " (z 2 + 1) j1 + xjei 0=2 j1 xjei 0=2 1+" (x2 + 1) 1 x2
Z 1
1
! p dx = I:
1 (x2 + 1) 1 x2

Integrate along 4, and let " ! 0+ . This gives


Z
f (z) dz ! 0:
4

Integrate along 5, and let R ! 1 and " ! 0+ . This gives

Z
f (z) dz =
5
Z R
1
= p p dz =
1 " (z 2 + 1) j1 + zjei arg(1+z)=2 j1 zjei arg(1 z)=2
Z R Z R
1 1
= p p dx = p dx !
(z 2 + 1) j1 + xjei( )=2 j1 xjei0=2 2 1 x2 ( i)
1 " 1 " (x + 1)
Z 1
1
! p dx = J:
1 (x + 1) 1 x2 ( i)
2

Integrate along 6, and let R ! 1. This gives


Z
1 2
f (z) dz p 2 R ! 0:
6
(R2 1) R2 1 R3
Integrate along 7, and let R ! 1 and " ! 0+ . This gives

175
Z
f (z) dz =
7
Z 1 "
1
= p p dz =
R (z 2 + 1) j1 + zjei arg(1+z)=2 j1 zjei arg(1 z)=2
Z 1 " Z R
1 1
= p p dx = p dx !
R (z 2 + 1) j1 + xjei =2 j1 xjei2 =2 1 " (x2 + 1) 1 x2 ( i)
Z 1
1
! p dx = J:
1 (x2 + 1) 1 x2 ( i)

Integrate along 8, and let " ! 0+ . This gives


Z
"4
f (z) dz p 2 " 2 "9=2 ! 0:
4 " (1 ")

Using the Residue Theorem and letting R ! 1 and " ! 0+ , we obtain that

1 1
I +0 I J +0+J +0=2 i p i+ p i
2 2 2 2
i.e.,
Z 1
1
p dx = p :
1 (x2 + 1) 1 x2 2

176
VII.9.4
Show using residue theory that
Z 1
1 2
q dx = p :
1 3 (1 x) (1 + x)2 3

Solution
Set
Z 1
1
I= q dx
1 3 2
(1 x) (1 + x)
and integrate

1 1
f (z) = q = p q
3
(1 z) (1 + z)2 3
j1 zjei arg0 (1 z)=3 j1 + zj2 e2i arg (1+z)=3

along the contour in Figure VII.9.4.

177
VIII.9.4
arg0 z arg z

y y

0 0 0 0 0 0
x -2 -1 1 2
x
-2 -1 2 1 2 2 2 0 0 0

arg0 (1 z) arg (1 + z)

y y

2 2 2 2 0 0 0 0

-2 -1 1 2
x -2 -1 1 2
x
0 0 0 0 0 0 0 0

1
p
The argument for the function f (z) = p
3
j1 zjei arg0 (1 z)=3 j1+zj2 e2i arg (1+z)=3

4/3 4/3 2/3 2/3 /3 /3

-2 -1 1 2
x
2/3 2/3 0 0 /3 /3

Residue at a simple pole at z1 = 1 , where by result from Exercise VII.8.13


and Rule 1,

178
2 3
1 1 1 1
Res [f (z) ; 1] = Res 2
f ; 0 = Res 4 2 q ; 05 =
w w w 3 1 1 2
1 w
1+ w
2 3
1 1
= Res 4 q ; 05 =
w 3 2
(w 1) (w + 1)
2 3
1
lim 4 p q 5= e i=3
w!0 3
jw 1jei arg(w 1)=3 jw + 1j2 e2i arg(w+1)=3

Integrate along 1, and let " ! 0+ . This gives

Z
f (z) dz =
1
Z 1 "
1
= p q dz =
1+" 3
j1 zjei arg0 (1 j1 + zj2 e2i arg (1+z)=3
z)=3
Z 1 " Z 1 "
1 2 i=3 1
= p q dx = e q dx !
1+" 3
j1 xjei2 =3 j1 + xj2 e2i 0=3 1+" 3 (1 x) (1 + x)2
Z 1
2 i=3 1
!e q dx = e 2 i=3 I:
1 3 2
(1 x) (1 + x)

Integrate along 2, and let " ! 0+ . This gives


Z
"4
f (z) dz p 2 " 2 "9=2 ! 0:
4 " (1 ")

Integrate along 3, and let " ! 0+ . This gives

179
Z
f (z) dz =
3
Z 1+"
1
= p q dz =
1 " j1 zjei arg0 (1 z)=3 j1 + zj2 e2i arg (1+z)=3
3

Z 1+" Z 1 "
1 1
= p q dx = q dx !
1 " 3 i 0=3 2 2i 0=3 1+" 3 2
j1 xje j1 + xj e (1 x) (1 + x)
Z 1
1
! q dx = I:
1 3 (1 2
x) (1 + x)

Integrate along 4, and let " ! 0+ . This gives


Z
"4
f (z) dz p 2 " 2 "9=2 ! 0:
4 " (1 ")

Using the Residue Theorem and letting R ! 1 and " ! 0+ , we obtain that

2 i=3 i=3
e I +0 I +0=2 i e :
Solve for I, we obtain that
i=3
2 ie 2 i 2
I= 2 i=3
= i=3 i=3
= =p ;
e 1 e e sin ( =3) 3
i.e.,
Z 1
1 2
q dx = p :
1 3
(1 x) (1 + x)2 3

180
VII.9.5
Show using residue theory that
Z 1
1
q dx = p
4
:
0 (x + 1) 4 x (1 x) 3 2

Solution
Set
Z 1
1
I= q dx
3
0 (x + 1) 4 x (1 x)
and integrate

1 1
f (z) = q = p q
(z + 1) 4 z (1 z)3 (z + 1) 4 jzjei arg z=4 4
j1 zj3 e3i arg0 (1 z)=4

along the contour in Figure VII.9.5.

181
VIII.9.5
arg z arg0 z

y y

0 0 0 0 0 0

-2 -1 1 2
x -2 -1 1 2
x
0 0 0 2 2 2

arg z arg0 (1 z)

y y

0 0 0 2 2 2 2

-2 -1 1 2
x -2 -1 1 2
x
0 0 0 0 0 0 0

1
p
The argument for the function f (z) = p z=4 4
(z+1) 4 jzjei arg j1 zj3 e3i arg0 (1 z)=4

7/4 7/4 7/4 3/2 3/4 3/4

-2 -1 1 2
x
/4 /4 /4 3/4 3/4 3/4

Residue at a simple pole at z1 = 1, where by Rule 3,

182
2 3
1
Res 4 p q ; 15 =
(z + 1) 4
jzjei arg(z)=4 4
j1 zj3 e3i arg(1 z)=4

1
p q =
4
jzjei arg(z)=4 4 j1 zj3 e3i arg(1 z)=4
z= 1
1
= p q =
4
jzjei arg(z)=4 4
j1 zj3 e3i arg(1 z)=4
1 1 1 1 1 1
= p
4
= p
4
p + ip = p 4
+i p
8e i=4 8 2 2 2 2 242
Residue at a simple pole at z2 = 1 , where by result from Exercise VII.8.13
and Rule 1,

2 3
1 1 1 1
Res [f (z) ; 1] = Res f ;0 = Res 4 q ; 05 =
w2 w w2 1 4 1 1 3
w
+1 w
1 w
2 3
1
= lim 4 q ; 05 = 0
w!0 4 3
(1 + w) (w 1)
Residue at a simple pole at z2 = i, where by Rule 3,
Integrate along 1 , and let R ! 1 and " ! 0+ . This gives
Z
f (z) dz =
1
Z 1 "
1
= p q dz =
" (z + 1) 4
jzjei arg z=4 4 j1 zj3 e3i arg0 (1 z)=4
Z 1 " Z 1 "
1 1
= p q dx = i q dx !
" (x + 1) 4 jxjei 0=4 4 j1 xj3 e3i 2 =4 " (x + 1) 4 x (1 x)3
Z 1
1
!i q dx = iI:
3
0 (x + 1) 4 x (1 x)

183
Integrate along 2, and let " ! 0+ . This gives
Z
"4
f (z) dz p 2 " 2 "9=2 ! 0:
4 " (1 ")
Integrate along 3, and let R ! 1 and " ! 0+ . This gives

Z
f (z) dz =
3
Z "
1
= p q dz =
1 " (z + 1) 4
jzjei arg z=4 4
j1 zj3 e3i arg0 (1 z)=4
Z " Z 1 "
1 1
= p q dx = q dx !
1 " 4
(x + 1) jxjei 0=4 4
j1 xj3 e3i 0=4 " (x + 1) 4 x (1 3
x)
Z 1
1
! q dx = I:
3
0 (x + 1) 4 x (1 x)

Integrate along 4, and let " ! 0+ . This gives


Z
"4
f (z) dz p 2 " 2 "9=2 ! 0:
4 " (1 ")
Using the Residue Theorem and letting R ! 1 and " ! 0+ , we obtain that

1 1
iI + 0 I +0=2 i p
4
+i p +0 ;
2 2 242
i.e.,
Z 1
1
q dx = p
4
:
0 (x + 1) 4 x (1 x) 3 2

184
VIII 1 2 3 4 5 6 7 8 9 10 11 12 13 14 15 16 17 18 19
1
2
3
4
5
6
7
8

1
VIII.1.1
1 2 3 P L K
LLL
Show that z 4 + 2z 2 z + 1 has exactly one root in each quadrant.

Solution

VIII.1.1

Set p (z) = z 4 +2z 2 z +1, and compute 4 arg p (z) along the three segments
in the sector path in gure VIII.1.1. First of all p (z) have no real zeros.

The positive real axis 1 from 0 to R we parametizize as x : z = t; 0 t R,


2
and our function p (z) becomes p (t) = t4 + 2t2 t + 1 = (t2 + 1) t > 0.
Since p (x) > 0 for t 0, their is no change of argument on this line segment
on the real axis, thus 4 arg p (z) = 0.

The arc 2 from R to iR we parametizize as 2 : z = Reit , 0 t =2,


4 4it 2 2it it
and our function p (z) becomes p (t) = R e + 2R e Re + 1. Since
the variation in argument is determined by the dominating term R4 e4it for R
large the change of argument on this arc is 4 times the variation in argument
for the arc, and thus 4 arg p (z) 4 2 = 2 .

The positive imaginary axis 3 from iR to 0 we parametizize as 3 : z =


it; 0 t R, and our function p (z) becomes f (t) = (it)4 + 2 (it)2 it + 1
2
= (t2 1) it . We nd that the real part have a zero at t = 1, and the
imaginary part have a zero for t = 0. We make the following table and do
the sketches,

2
VIII.1.1 (R = 4) VIII.1.1 (R = 4)
t Re z Im z arg z
4
1 + 0
3
1 0 2
2
0 1 0 0
1

-2 -1 1 2 3 4
-1
-2

Thus when we move from iR to 0, the argument for p (z) remain in the 4-th
quadrant except for touching the imaginary axis at i and terminating at
1, and their is no change of argument on this line segment on the imaginary
axis, thus 4 arg p (z) 0.

Now we have that the total change of argument for p (z) is 2 , so we have
exactly one zero in the rst quadrat. Because that the roots come in complex
conjugate pairs it is plain that p (z) have a root in the fourth quadrant too.
2
Becauce that p (x) = (x2 + 1) x > 0 it is clear that p (z) have no real roots
and p (z) must have on has one zero in each quadrant.

3
VIII.1.2
1 2 3 P L K

Find the number of zeros of the polynomial p (z) = z 4 +z 3 +4z 2 +3z+2


in each quadrant.

Solution

VIII.1.2

Set p (z) = z 4 + z 3 + 4z 2 + 3z + 2, and compute 4 arg p (z) along the three


segments in the sector path in gure VIII.1.2. First of all p (z) have no real
zeros.

The positive real axis 1 from 0 to R we parametizize as x : z = t; 0 t R,


and our function p (z) becomes p (t) = t4 +t3 +4t2 +3t+2 > 0. Since p (x) > 0
for t 0, their is no change of argument on this line segment on the real
axis, thus 4 arg p (z) = 0.

The arc 2 from R to iR we parametizize as 2 : z = Reit , 0 t =2, and


4 4it 3 3it 2 2it it
our function p (z) becomes p (t) = R e + R e + 4R e + 3Re + 2. Since
the variation in argument is determined by the dominating term R4 e4it for R
large the change of argument on this arc is 4 times the variation in argument
for the arc, and thus 4 arg p (z) 4 2 = 2 .

The positive imaginary axis 3 from iR to 0 we parametizize as 3 : z =


it; 0 t R, and our function p (z) becomes f (t) = (it)4 + (it)3 + 4 (it)2 +
3it + 2 = t4 4t2 + 2 + i ( t3 + 3t) . We nd that the real part have a zero

4
p p p p
at t = 2 p 2 and t = 2 + 2, and the imaginary part have a zero for
t = 0 and t = 3. We make the following table and do the sketches,

t Re z Im z arg z VIII.1.2 (R = 10) VIII.1.2 (R = 10)


1
p + 0
p 4
p 2 + 2 0 2
2
p3 p 0
3
2 2 0 + 2 -4 -2 2 4
0 + 0 2 -2

-4

Thus when we move from iR to 0, the argument for p (z) on this line segment
on the imaginary axis changes so that, 4 arg p (z) 2 .

Now we have that the total change of argument for p (z) is 0, so we have
no zero in the 1st quadrant.
Since p (z) has real coe cients, the roots come in conjugate pairs (since there
are nor real roots), therefore there are no roots in the 4th quadras as well.
By symmetry, there are 2 roots each in the 2nd and 3rd quadrants.

5
VIII.1.3
1 2 3 P L K

Find the number of zeros of the polynomial p (z) = z 6 + 4z 4 + z 3 +


2z 2 + z + 5 in the rst quadrant fRe z > 0; Im z > 0g.

Solution

VIII.1.3

Set p (z) = z 6 + 4z 4 + z 3 + 2z 2 + z + 5, and compute 4 arg p (z) along the


three segments in the sector path in gure VIII.1.3. First of all p (z) have no
real zeros.

The positive real axis 1 from 0 to R we parametizize as x : z = t; 0 t R,


and our function p (z) becomes p (t) = t6 + 4t4 + t3 + 2t2 + t + 5 > 0. Since
p (t) > 0 for t 0, their is no change of argument on this line segment on
the real axis, thus 4 arg p (z) = 0.

The arc 2 from R to iR we parametizize as 2 : z = Reit , 0 t =2, and


our function p (z) becomes p (t) = R6 e6it +4R4 e4it +3R3 e3it +2R2 e2it +Reit +5.
Since the variation in argument is determined by the dominating term R6 e6it
for R large the change of argument on this arc is 6 times the variation in
argument for the arc, and thus 4 arg p (z) 6 2 = 3 .

The positive imaginary axis 3 from iR to 0 we parametizize as 3 : z =


it; 0 t R, and our function p (z) becomes f (t) = (it)6 + 4 (it)4 + (it)3 +
2 (it)2 + it + 5 = t6 + 4t4 2t2 + 5 + i ( t3 + t) . We nd that the real part
have a zero at t 1; 95, and the imaginary part have a zero for t = 0 and
t = 1. We make the following table and do the sketches,

6
VIII.1.3 (R = 3) VIII.1.3 (R = 3)
t Re z Im z arg z
1 1000
1; 95 0 2 -2 2 4 6 8 10
500
1 + 0 0 -2

0 + 0 0 -4
-1000 -500 500 1000
-500 -6

Thus when we move from iR to 0, the argument for p (z) on this line segment
on the imaginary axis changes so that, 4 arg p (z) .

Now we have that the total change of argument for p (z) is 4 , so we


have that p (z) has exactly two zeros in the rst quadrat. (None on real or
imaginary axis)

7
VIII.1.4
1 2 3 P L K
LLL
Find the number of zeros of the polynomial p (z) = z 9 +2z 5 2z 4 +z+3
in the right half-plane.

Solution

VIII.1.4

Set p (z) = z 9 + 2z 5 2z 4 + z + 3, and compute 4 arg p (z) along the three


segments in the sector path in gure VIII.1.4. First of all p (z) have no real
zeros.

The positive real axis 1 from 0 to R we parametizize as x : z = t; 0 t R,


and our function p (z) becomes p (t) = t9 + 2t5 2t4 + t + 3. Since p (x) > 0
for t 0, their is no change of argument on this line segment on the real
axis, thus 4 arg p (z) = 0.

The arc 2 from R to iR we parametizize as 2 : z = Reit , 0 t =2, and


9 9it 5 5it 4 4it it
our function p (z) becomes p (t) = R e + 2R e 2R e + Re + 3. Since
the variation in argument is determined by the dominating term R9 e9it for R
large the change of argument on this arc is 9 times the variation in argument
for the arc, and thus 4 arg p (z) 9 2 = 92 .
The positive imaginary axis 3 from iR to 0 we parametizize as 3 : z =
it; 0 t R, and our function p (z) becomes f (t) = (it)9 + 2 (it)5 2 (it)4 +
2
3 = 2t4 + 3 + iy (y 4 + 1) . We nd that the real part have a zero at
it + q
t = 4 32 , and the imaginary part have only a zero for t = 0. We make the
following table and do the sketches,

8
t Re z Im z arg z VIII.1.4 (R = 2) VIII.1.4 (R = 2)
1 + 600
q 2
400
30
4 3
2
0 + 2 200 20
0 + 0 0
-600-400-200 200 400 600 10
-200
-400
-600 -20 -10 10 20

Thus when we move from iR to 0, the argument is changed from =2 to 0, so


the charge of argument on this on the imaginary axis is , thus 4 arg p (z) =
=2.

Now we have that the total change of argument for p (z) is 4 , so we


have exactly two zeros in the rst quadrat. Because that the roots come
in complex conjugate pairs it is plain that p (z) has four zeros in the right
half-plane.

9
VIII.1.5
1 2 3 P L K

For a xed real number , nd the number of zeros z 4 +z 3 +4z 2 + z+3


satisfying Re z < 0. (Your answer depends on .)

Solution

VIII.1.5

Set p (z) = z 4 + z 3 + 4z 2 + z + 3, and compute 4 arg p (z) along the three


segments in the contuour in gure VIII.1.5.

The positive real axis 1 from 0 to R we parametizize as x : z = t; 0 t R,


2
and our function p (z) becomes p (t) = t4 + t3 + 4t2 + at + 3 = (t2 + 1) t
> 0. Since p (x) > 0 for t 0, their is no change of argument on this line
segment on the real axis, thus 4 arg p (z) = 0.

The arc 2 from R to iR we parametizize as 2 : z = Reit , 0 t =2, and


4 4it 3 3it 2 2it it
our function p (z) becomes p (t) = R e + R e + 4R e + aRe + 3. Since
the variation in argument is determined by the dominating term R4 e4it for R
large (independent of the value of a) the change of argument on this arc is 4
times the variation in argument for the arc, and thus 4 arg p (z) 4 2 = 2 .

The positive imaginary axis 3 from iR to 0 we parametizize as 3 : z =


it; 0 t R, and our function p (z) becomes f (t) = (it)4 + (it)3 + 4 (it)2 +
ait + 3 = (t2 1) (t2 p3) + i ( t3 + at). We nd that the real part have a
zero at t = 1 and t = p3 and the imaginary part have a zero for t = 0 and
and a second root t = a if a 0. We make the following table and do the
sketches,

10
a<0 VIII.1.5 (a = -4, R = 10) VIII.1.5 (a = -4, R = 10)
t Re z Im z arg z
1
p + 0
3 0 2
1 0 2
0 + 0 0

a=0 VIII.1.5 (a = 0, R = 10) VIII.1.5 (a = 0, R = 10)


t Re z Im z arg z
1
p + 0
3 0 2
1 0 2
0 + 0 0

0<a<1 VIII.1.5 (a = 1/2, R = 10) VIII.1.5 (a = 1/2, R = 10)


t Re z Im z arg z
1
p + 0
3 0 2
1 0
p 2
a + 0 0
0 + 0 0

11
a=1 VIII.1.5 (a = 1, R = 10) VIII.1.5 (a = 1, R = 10)
t Re z Im z arg z
1
p + 0
3 0 2
1 0 0
0 + 0 0

1<a<3 VIII.1.5 (a = 2, R = 10) VIII.1.5 (a = 2, R = 10)


t Re z Im z arg z
1
p + 0
3 0
p 2
a 0
3
1 0 + 2
0 + 0 2

a=3 VIII.1.5 (a = 3, R = 10) VIII.1.5 (a = 3, R = 10)


t Re z Im z arg z
1
p + 0
3 0 0
1 0 + 2
0 + 0 0

12
a>3 VIII.1.5 (a = 9, R = 10) VIII.1.5 (a = 9, R = 10)
t Re z Im z arg z
1 + 0
p
pa + 0 0
3 0 + 2
1 0 + 2
0 + 0 0

Thus when we move from iR to 0, the argument for p (z) only change if
1 < a < 3 and we have that 4 arg p (z) 2 , for all other value on a we
have that 4 arg p (z) 0. Remark that we can se that for a = 1 and a = 3
we have zeros on the imaginary axis.

Now we have that the total change of argument for p (z) is 2 , so we have
exactly one zero in the rst quadrat. Because that the roots come in complex
conjugate pairs it is plain that p (z) has one zero in each quadrant.

If 1 3, the total change of argument for p (z) is 0 and no zeros on


the imaginary axis, thus we must have four zeros in the open left half-plane.
If < 1 and > 3, the total charge of argument for p (z) is 2 and no
zeros of the imaginary axis, thus we must have two zeros in the open left
half-plane.
If = 1 and = 3, the total charge of argument for p (z) is 2 and we
have two zeros on the imaginary axis, thus we must have remaining two zeros
in the open left half-plane.

13
VIII.1.6
1 2 3 P L K

For a xed real number , nd the number of solutions of z 5 +2z 3 z 2 +z =


satisfying Re z > 0.

Solution

VIII.1.6

Set p (z) = z 5 + 2z 3 z 2 + z , and compute 4 arg p (z) along the two


segments in the contuour in gure VIII.1.6.

The arc 1 from iR to iR we parametizize as 2 : z = Reit , =2 t =2,


and our function p (z) becomes p (t) = R5 e5it +2R3 e3it R2 e2it +Reit . Since
the variation in argument is determined by the dominating term R5 e5it for R
large (independent of the value of ) the change of argument on this arc is 5
times the variation in argument for the arc, and thus 4 arg p (z) 5 2 = 52 .

The positive imaginary axis 2 from iR to iR we parametizize as 2 : z =


it; R t R, and our function p (z) becomes f (t) = (it)5 + 2 (it)3
2
(it)2 +pit = t2 p + it (t2 1) . We nd that the real part have a zero at
t= and t = and the imaginary part have a zero for t = 1, t = 0
and t = 1. We make the following table and do the sketch,

14
VIII.1.6 (a = 4, R = 4) VIII.1.6 (a = 4, R = 4)

t Re z Im z arg z
1

VIII.1.5 (a = 2, R = 10) VIII.1.5 (a = 2, R = 10)


t Re Im arg
1
p + 0 0
3 0
p 2
a 0
3
1 0 + 2
0 3 0 2

VIII.1.5 (a = 0, R = 10) VIII.1.5 (a = 0, R = 10)

t Re z Im z arg z
1

15
Thus when we move from iR to 0, the argument for p (z) remain in the 4-th
quadrant except for touching the imaginary axis at i and terminating at
1, and their is no change of argument on this line segment on the imaginary
axis, thus 4 arg p (z) = 0.

Now we have that the total change of argument for p (z) is 2 , so we have
exactly one zero in the rst quadrat. Because that the roots come in complex
conjugate pairs it is plain that p (z) has one zero in each quadrant.

16
VIII.1.7
1 2 3 P L K

For a xed complex number , show that if m and n are large integers, then
the equation ez = z + has exactly m + n solutions in the horizontal strip
f 2 im < Im z < 2 ing.

Solution

VIII.1.7

Set p (z) = z 4 +2z 2 z +1, and compute 4 arg p (z) along the three segments
in the contuour in gure VIII.1.7.
The positive real axis 1 from 0 to R we parametizize as x : z = t; 0 t R,
2
and our function p (z) becomes p (t) = t4 + 2t2 t + 1 = (t2 + 1) t > 0.
Since p (x) > 0 for t 0, their is no change of argument on this line segment
on the real axis, thus 4 arg p (z) = 0.

The arc 2 from R to iR we parametizize as 2 : z = Reit , 0 t =2,


and our function p (z) becomes p (t) = R4 e4it + 2R2 e2it Reit + 1. Since
the variation in argument is determined by the dominating term R4 e4it for R
large the change of argument on this arc is 4 times the variation in argument
for the arc, and thus 4 arg p (z) 4 2 = 2 .

The positive imaginary axis 3 from iR to 0 we parametizize as 3 : z =


it; 0 t R, and our function p (z) becomes f (t) = (it)4 + 2 (it)2 it + 1
2
= (t2 1) it . We nd that the real part have a zero at t = 1, and the
imaginary part have a zero for t = 0 on the positive imaginary axis. We
make the following table and do the sketch,

17
VIII.1.7
t Re Im arg 20
1 + 0
10
1 0 2
0 + 0 0 -20 -10 10 20
-10

-20

Thus when we move from iR to 0, the argument for p (z) remain in the 4-th
quadrant except for touching the imaginary axis at i and terminating at
1, and their is no change of argument on this line segment on the imaginary
axis, thus 4 arg p (z) = 0.

Now we have that the total change of argument for p (z) is 2 , so we have
exactly one zero in the rst quadrat. Because that the roots come in complex
conjugate pairs it is plain that p (z) has one zero in each quadrant.

18
VIII.1.8
1 2 3 P L K
LLL
Show that if Re > 1, then the equation ez = z + has exactly one
solution in the left half-plane.

Solution

VIII.1.8

Set p (z) = z 4 +2z 2 z +1, and compute 4 arg p (z) along the three segments
in the contuour in gure VIII.1.8.
The positive real axis 1 from 0 to R we parametizize as x : z = t; 0 t R,
2
and our function p (z) becomes p (t) = t4 + 2t2 t + 1 = (t2 + 1) t > 0.
Since p (x) > 0 for t 0, their is no change of argument on this line segment
on the real axis, thus 4 arg p (z) = 0.

The arc 2 from R to iR we parametizize as 2 : z = Reit , 0 t =2,


and our function p (z) becomes p (t) = R4 e4it + 2R2 e2it Reit + 1. Since
the variation in argument is determined by the dominating term R4 e4it for R
large the change of argument on this arc is 4 times the variation in argument
for the arc, and thus 4 arg p (z) 4 2 = 2 .

The positive imaginary axis 3 from iR to 0 we parametizize as 3 : z =


it; 0 t R, and our function p (z) becomes f (t) = (it)4 + 2 (it)2 it + 1
2
= (t2 1) it . We nd that the real part have a zero at t = 1, and the
imaginary part have a zero for t = 0 on the positive imaginary axis. We
make the following table and do the sketch,

19
VIII.1.8
t Re Im arg 20
1 + 0
10
1 0 2
0 + 0 0 -20 -10 10 20
-10

-20

Thus when we move from iR to 0, the argument for p (z) remain in the 4-th
quadrant except for touching the imaginary axis at i and terminating at
1, and their is no change of argument on this line segment on the imaginary
axis, thus 4 arg p (z) = 0.

Now we have that the total change of argument for p (z) is 2 , so we have
exactly one zero in the rst quadrat. Because that the roots come in complex
conjugate pairs it is plain that p (z) has one zero in each quadrant.

Set f (z) = ez z , Re > 1, and get f (iy) = cos y Re + i (sin y y)


i Im . Thus when we move from f ( iR) to f (iR), f (iy) remains in the left
half-plane and the change in arg f is .
When we move from f (iR) to f ( iR) along f Rei , 2 < < 32 , we cross
the real axis from the 4-th to the 1 st quadrant, thus the change in arg f is
again . (May also use Rouches Theorem.)

t Re Im arg VIII.1.4 (R = 2) VIII.1.4 (R = 2)


1
q + 2 100
30
4 3
2
0 + 2 50
20
0 + + 0
-100 -50 50 100 10
-50

-100 -20 -10 10 20

20
t Re Im arg VIII.1.4 (R = 2) VIII.1.4 (R = 2)
1
q + 2 400
30
4 3
2
0 + 2 200
20
0 + + 0
-400 -200 200 400 10
-200

-400 -20 -10 10 20

(it)4 (it)3 + 13 (it)2 it +36 = t4 + it3 13t2 it + 36

21
VIII.1.9
1 2 3 P L K

Show that if f (z) is analytic in a domain D, and if is a closed curve in D


such that the values of f (z) on lie in the slit plane Cn ( 1; 0] then the
increase in the argument of f (z) around is zero.

Solution

22
VIII.2.1
1 2 3 P L K
LLL
Show that 2z 5 + 6z 1 has one root in the interval 0 < x < 1 and
four roots in the annulus f1 < jzj < 2g.

Solution
Set rst p (z) = f1 (z) + h1 (z), where f1 (z) = 2z 5 and h1 (z) = 6z 1. On
the circle jzj = 2 we have,
jf1 (z)j = j2z 5 j = 2 25 = 64 and jh1 (z)j = j6z 1j 13, then jf1 (z)j >
jh1 (z)j on jzj = 2 so by Rouches Theorem p (z) has all 5 roots in the disk
jzj = 2.
Set now p (z) = f2 (z) + h2 (z), where f2 (z) = 6z and h2 (z) = 2z 5 1. On
the circle jzj = 1 we have, jf2 (z)j = j6zj = 6 and jh2 (z)j = j2z 5 1j 3,
then jf2 (z)j > jh2 (z)j on jzj = 1 so by Rouches Theorem p (z) has 1 roots
in the disk jzj = 1.
It follows that p (z) = 2z 5 + 6z 1 has 5 1 = 4 roots in the annulus
1 < jzj < 2. As any complex roots come in conjugate pairs that have the
same magnitude, the root in the disk jzj = 1 must be real, we have that
p (0) = 1 and p (1) = 7, thus this root must be in the interval 0 < x < 1.

23
VIII.2.2
1 2 3 P L K
LLL
How many roots does z 9 + z 5 8z 3 + 2z + 1 have between the circles
fjzj = 1g and fjzj = 2g?

Solution
Set rst p (z) = f1 (z)+h1 (z), where f1 (z) = z 9 and h1 (z) = z 5 8z 3 +2z+1.
On the circle jzj = 2 we have,
jf1 (z)j = jz 9 j = jzj9 = 29 = 512 and jh1 (z)j = jz 5 8z 3 + 2z + 1j 101,
then jf1 (z)j > jh1 (z)j on jzj = 2 so by Rouches Theorem p (z) has 9 roots
in the disk jzj = 2.
Set now p (z) = f2 (z) + h2 (z), where f2 (z) = z 9 + z 5 + 2z + 1 and h2 (z) =
8z 3 . On the circle jzj = 1 we have, jf2 (z)j = j 8z 3 j = 8 and jh2 (z)j =
jz 9 + z 5 + 2z + 1j 5, then jf2 (z)j > jh2 (z)j on jzj = 2 so by Rouches
Theorem p (z) has 3 roots in the disk jzj = 1.
Thus it follows that p (z) = z 9 + z 5 8z 3 + 2z + 1 has 9 3 = 7 roots in the
annulus 1 < jzj < 2.

24
VIII.2.3
1 2 3 P L K
LLL
Show that if m and n are positive integers, the the polynomial

z2 zm
p (z) = 1 + z +
+ + + 3z n
2! m!
has exactly n zeros in the unit disk.

Solution
2 m
Set p (z) = f (z) + h (z), where f (z) = 3z n and h (z) = 1 + z + z2! + + zm! .
On the circle jzj = 1 we have,
2 m
jf (z)j = j3z n j = 3 jzjn = 3 1n = 3 and jh (z)j = 1 + z + z2! + + zm!
P1 1
k=0 k! = e, then jf (z)j > jh (z)j on jzj = 1 so by Rouches Theorem
2 m
p (z) = 1 + z + z2! + + zm! + 3z n has exactly n roots in the unit disk jzj = 1.

25
VIII.2.4
1 2 3 P L K
LLL
Fix a complex number such that j j < 1. For n 1, show that
n z
(z 1) e has n zeros satisfying jz 1j < 1 and no other zeros in
the right half-plane. Determine the multiplicity of the zeros.

Solution
Set p (z) = f (z) + h (z), where f (z) = (z 1)n ez and h (z) = . On the
i
circle jz 1j = 1 which we parametrize by z = 1 + e , 0 2 we have,
n z i n 1+i 1+cos
jf (z)j = j(z 1) e j = 1 + e 1 e =e 1, then 0
2 and jh (z)j = j j = j j < 1 from the text, then jf (z)j > jh (z)j on
jz 1j = 1 so by Rouches Theorem p (z) = (z 1)n ez has n roots in
the disk jz 1j = 1. We have that f 0 (z) = n (z 1)n 1 ez + (z 1)n ez =
(n + z 1) (z 1)n 1 ez , thus the zeros of of f 0 (z) are at z = 1 and at
z = 1 n, where the only zeros in the right half-plane are at z = 1. Since
f (1) = , the zeros of f (z) must be simple unless = 0, in which case we
ge a zero of order n at z = 1.

***********
Hr r lite konstigt den derivarande satsen, hur funkar den med nollstllen
***********

26
VIII.2.5
1 2 3 P L K

For a xed satisfying j j < 1, show that (z 1)n ez + (z + 1)n has


n zeros in the right half-plane, which are all simple if 6= 0.

Solution

27
VIII.2.6
1 2 3 P L K

Let p (z) = z 6 + 9z 4 + z 3 + 2z + 4 be the polynomial treated in the example


in this section.
(a)
Determine which quadrants contain the four zeros of p (z) that lie inside the
unit circle.
(b)
Determine which quadrants contain the two zeros of p (z) that lie outside the
unit circle.
(c)
Show that the two zeros of p (z) that lie outside the unit circle satisfy
fjz 3ij < 1=10g.

Solution

28
VIII.2.7
1 2 3 P L K
LLL
Let f (z) and g (z) be analytic functions on the bounded domain D that
extend continuously to @D and satisfy jf (z) + g (z)j < jf (z)j + jg (z)j on
@D. Show that f (z) and g (z) have the same number of zeros in D, counting
multiplicity. Remark. This is a variant of Rouches theorem, in which the
hypotheses are symmetric in f (z) and g (z). Rouches theorem is obtained
by setting h (z) = f (z) g (z). For the solution of the exercise, see Exercise
9 in the preceding section.

Solution

29
VIII.2.8
1 2 3 P L K

Let D be a bounded domain, and let f (z) and h (z) be meromorphic functions
on D that extend to be analytic on @D. Suppose that jh (z)j < jf (z)j on
@D. Show by example that f (z) and f (z) + h (z) can have dierent numbers
of zeros on D. What can be said about f (z) and f (z) + g (z)? Prove your
assertion.

Solution

30
VIII.2.9
1 2 3 P L K

Let f (z) be a continuously dierentiable function on a domain D. Suppose


that for all complex constants a and b, the increase in the argument of f (z)+
az +b around any small circle in D on which f (z)+az +b 6= 0 is nonnegative.
Show that f (z) is analytic.

Solution

31
VIII.3.1
1 2 3 P L K
LLL
Let ffk (z)g be a sequence of analytic functions on D that converges normally
to f (z), and suppose that f (z) has a zero of order N at z0 2 D. Use Rouches
theorem to show that there exists > 0 such that for k large, fk (z) has
exactly N zeros counting multiplicity on the disk fjz z0 j < g.

Solution

32
VIII.3.2
1 2 3 P L K
LLL
Let S be the family of univalent functions f (z) dened on the open unit disk
fjzj < 1g that satisfy f (0) = 0 and f 0 (0) = 1. Show that S is closed under
normal convergence, that is, if a sequence in S converges normally to f (z),
then f 2 S. Remark. It is also true, but more di cult to prove, that S is
a compact family of analytic functions, that is, every sequence in S has a
normally convergent subsequence.

Solution

33
VIII.4.1
1 2 3 P L K
LLL
Suppose D is a bounded domain with piecewise smooth boundary. Let f (z)
be meromorphic and g (z) analytic on D. Suppose that both f (z) and g (z)
extend analytically across the boundary of D, and that f (z) 6= 0 on @D.
Show that
I Xn
1 f 0 (z)
g (z) dz = mj g (zj ) ;
2 i @D f (z) j=1

where z1 ; : : : ; zn are the zeros and poles of f (z), and mj is the order of f (z)
at zj .

Solution

34
VIII.4.2
1 2 3 P L K

Let f (z) be a meromorphic function on the complex plane that is doubly


periodic. Suppose that the zeros and the poles of f (z) are at the points
z1 ; : : : ; zn and at their translates by periods of f (z), and suppose no zj is a
translateP by a period of another zk . Let mj be the order of f (z) at zj . Show
that mj zj is a period of f (z). Hint. Integrate zf 0 (z) =f (z) around the
boundary of the fundamental parallelogram P constructed in Section VI.5.

Solution

35
VIII.4.3
1 2 3 P L K
LLL
Let ffk (z)g be a sequence of analytic functions on a domain D that converges
normally to f (z). Suppose that fk (z) attains each value w at most m times
(counting multiplicity) in D. Show that either f (z) is constant, or f (z)
attains each value w at most m times in D.

Solution

36
VIII.4.4
1 2 3 P L K

Let f (z) be an analytic function on the open unit disk D = fjzj < 1g.
Suppose there is an annulus U = fr < jzj < 1g such that the restriction of
f (z) to U is one-to-one. Show that f (z) is one-to-one on D.

Solution

37
VIII.4.5
1 2 3 P L K

Let f (z) = p (z) =q (z) be a rational function, where p (z) and q (z) are
polynomials that are relatively prime (no common zeros). We dene the
degree of f (z) to be the larger of the degrees of p (z) and q (z). Denote the
degree of f (z) by d.
(a)
Show that each value w 2 C, w 6= f (1), is assumed d times by f (z) on C.
(b)
Show that f (z) attains each value w 2 C d times on C (as always, counting
multiplicity).

Solution

38
VIII.4.6
1 2 3 P L K

Let f (z) be a meromorphic function on the complex plane, and suppose there
is an integer m such that f 1 (w) has at most m points for all w 2 C. Show
that f (z) is a rational function.

39
VIII.4.7
1 2 3 P L K

Let F (z; w) be a continuous function of z and w that depends analytically


on z for each xed w, and let F1 (z; w) denote the derivative of F (z; w) with
respect to z. Suppose F (z0 ; w0 ) = 0, and F1 (z0 ; w0 ) 6= 0. Choose such
that F (x; w0 ) 6= 0 for 0 < jz z0 j .
(a)
Show that there exists > 0 such that if jw w0 j < , there is a unique
z = g (w) satisfying jz z0 j < and F (z; w) = 0.
(b)
Show that
Z
1 F1 ( ; w)
g (w) = d ; jw w0 j < :
2 i j z0 j= F ( ; w)
(c)
Suppose further that F (z; w) is analytic in w for each xed z, and let F2 (z; w)
denote the derivative of F (z; w) with respect to w. Show that g (w) is ana-
lytic, and

g 0 (w) = F2 (g (w) ; w) =F1 (g (w) ; w) :


(d)
Derive the inverse function theorem given in this section, together with the
formula for the derivative of the inverse function, as a corollary of (a), (b),
and (c).
Remark. this is the implicit function theorem for analytic functions. Note
that a specic formula is given for the function g (w) dened implicitely by
F (g (w) ; w) = 0.

40
VIII.4.8
1 2 3 P L K

Let D be a bounded domain, and let f (z) be a continuous function on D[@D


that is analytic on D. Show that @ (f (D)) f (@D), that is, the boundary
of the open set f (D) is contained in the image under f (z) of the boundary
of D.

Solution

41
VIII.5.1
1 2 3 P L K

Find the critical points and critical values of f (z) = z + 1=z. Sketch the
curves where f (z) is real. Sketch the regions where Im f (z) > 0 and where
Im f (z) < 0.

Solution.

42
VIII.5.2
1 2 3 P L K

Suppose g (z) is analytic at z = 0, with power series g (z) = 2 + iz 4 + O (z 5 ).


Sketch and label the curves passing through z = 0 where Re g (z) = 2 and
Im g (z) = 0.

Solution

43
VIII.5.3
1 2 3 P L K

Find the critical points and critical values of f (z) = z 2 + 1. Sketch the set
of points z such that jf (z)j 1, and locate the critical points of f (z) on the
sketch.

Solution

44
VIII.5.4
1 2 3 P L K

Suppose that f (z) is analytic at z0 . Show that if the set of z such that
Re f (z) = Re f (z0 ) consists of just one curve passing through z0 , then
f 0 (z0 ) 6= 0. Show also that if the set of z such that jf (z)j = jf (z0 )j consists
of just one curve passing through z0 , then f 0 (z0 ) 6= 0.

Solution

45
VIII.5.5
1 2 3 P L K

How many critical points, counting multiplicity, does a polynomial of degree


m have in the complex plane? Justify your answer.

Solution

46
VIII.5.6
1 2 3 P L K

Find and plot the critical points and critical values of f (z) = z 2 + 1 and of
2
its iterates f (f (z)) = (z 2 + 1) + 1 and f (f (f (z))). Suggestion. Use the
chain rule.

Solution

47
VIII.5.7
1 2 3 P L K

Let f (z) be a polynomial of degree m. How many (nite) critical points does
the N fold iterate f f (N times) have? Describe them in terms of
the critical points of f (z).

Solution

48
VIII.5.8
1 2 3 P L K

We dene a pole of f (z) to be a critical point of f (z) of order k if z0 is a


critical point of 1=f (z) of order k. We dene z = 1 to be a critical point of
f (z) of order k if w = 0 is a critical point of g (w) = f (1=w) of order k. Show
that with this denition, a point z0 2 C is a critical point of order k for a
meromorphic function f (z) if and only if there are open sets U containing
z0 and V containing w0 = f (z0 ) such that each w 2 v, w 6= w0 , has exactly
k + 1 preimages in U . Remark. We say that f (z) is a (k + 1) sheeted
covering of f 1 (V n fw0 g) \ U over V n fw0 g.

Solution

49
VIII.5.9
1 2 3 P L K

Show that a polynomial of degree m, regarded as a meromorphic function on


C , has a critical point of order m 1 at z0 = 1.

Solution

50
VIII.5.10
1 2 3 P L K

Locate the critical points ana critical values in the extended complex plane of
the polynomial f (z) = z 4 2z 2 . Determine the order of each critical point.
Sketch the set of points z such that Im z 0.

Solution

51
VIII.5.11
1 2 3 P L K

Show that if f is a rational function, and if g is a fractional linear trans-


formation, then f and g f have the some critical points in the extended
complex plane C . What can be said about the critical values of g f ? What
can be said about the critical points and critical values f g?

Solution

52
VIII.5.12
1 2 3 P L K

Let f (z) = p (z) =q (z) be a rational function of degree d, so that p (z) and
q (z) are reactively prime, and d is the larger of the degrees of p (z) and
q (z). (See Exercise 4.5.) Show that f (z) has 2d 2 critical points, counting
multiplicity, in the extended complex plane C . Hint if deg p 6= deg q, then
the number of critical points of f (z) in the nite plane C is deg (qp0 q 0 p) =
deg p + deg q 1, while the order of the critical points at 1 is jdeg p deg qj
1.

Solution

53
VIII.5.13
1 2 3 P L K

Show that the set of solution points (w; z) of the equation z 2 2 (cos w)+1 = 0
consists of the graphs of two entire functions z1 (w) and z2 (w) of w. Specify
the entire functions, and determine where their graphs meet. Remark. The
solutions set forms a reducible one-dimensional analytic variety in C2 .

Solution

54
VIII.5.14
1 2 3 P L K

Let a0 (w) ; : : : ; am 1 (w) be analytic in a neighborhood of w = 0 and vanish


at w = 0. Consider the monic polynomial in z whose coe cients are analytic
functions in w,

P (z; w) = z m + am 1 (w) z m 1
+ + a0 (w) ; jwj < :
Suppose that for each xed w, 0 < jwj < , there are m distinct solutions of
P (z; w) = 0.
(a)
Show that the m roots of the equation P (z; w) = 0 determine analytic
functions z1 (w) ; : : : ; zm (w) in the slit disk fjwj < n ( ; 0]g. Hint. Use
the implicit function theorem (Exercise 4.7).
(b)
Glue together branch cuts to form an m sheeted (possibly disconnected)
surface over the punctured disk f0 < jwj < g on which the branches zj (w)
determine a continuous function.
(c)
Suppose that the indices are arranged so that for some xed k, 1 k n, the
continuation of zj (w) once around w = 0 is zj+1 (w) for 1 j k 1, while
the continuation of zk (w) once around w = 0 is z1 (w). Show that Q (z; w) =
(z z1 (w)) (z zk (w)) determines a polynomial in z whose coe cients
are analytic functions of w for jwj < . Show further that the polynomial
Q (z; w) is an irreducible factor of P (z; w), and that all irreducible factors of
P (z; w) arise from subsets of the zj (w)s in this way.
(d)
Show that if f (z) is an analytic function that has a zero of order m at
z = 0, and z1 (w) ; : : : ; zm (w) are solutions of the equation w = f (z), then
the polynomial P (z; w) = (z z1 (w)) (z zm (w)) is irreducible.

Solution

55
VIII.5.15
1 2 3 P L K

Consider monic polynomial in z of the form

P (z; w) = z m + am 1 (w) + + a0 (w) ;


where the functions a0 (w) ; : : : ; am 1 (w) are dened and meromorphic in
some disk centered at w = 0. Let P0 (z; w) and P1 (z; w) be two such poly-
nomials, and consider the following algorithm. Using the division algorithm,
nd polynomials A2 (z; w) and P2 (z; w) such that P0 (z; w) = A2 (z; w) P1 (z; w)+
P2 (z; w) and the degree of P2 (z; w) is less then the degree of P1 (z; w). Con-
tinue in this fashion, nding polynomials Aj+1 (z; w) and Pj+1 (z; w) such
that

Pj 1 (z; w) = Aj+1 (z; w) Pj (z; w) + Pj+1 (z; w)


and deg Pj+1 (z; w) < deg Pj (z; w), until eventually we reach

Pl (z; w) = 0; Pl 1 (z; w) = Al (z; w) Pl (z; w) :


Let D (z; w) be the monic polynomials in z obtained by dividing Pl (z; w) by
the coe cient of the highest power of z.
(a)
Show that D (z; w) is the greatest common divisor of P0 (z; w) and P1 (z; w),
in the sense that D (z; w) divides both P0 (z; w) and P1 (z; w), and each
polynomial that divides both P0 (z; w) and P1 (z; w) also divides D (z; w).
(b)
Show that there are polynomials A (z; w) and B (z; w) such that D = AP0
+BP1 .
(c)
Show, that if P0 (z; w) and P1 (z; w) are relatively prime (that is D (z; w) =
1), then there is " > 0 such that for each xed w, 0 < jwj < ", the polynomials
P0 (z; w) and P1 (z; w) have no common zeros.
(d)
Show that any polynomial P (z; w) as above can be factored as a product of
irreducible polynomials, and the factorization is unique up to the order of
the factors and multiplication of a factor by a meromorphic function in w.
(e)

56
Show that if the coe cients of P (z; w) are analytic at w = 0, then the
irreducible factors of P (z; w) can be chosen so that their coe cients are
analytic at w = 0.
(f)
Show that if P (z; w) is irreducible, then there is " > 0 such that for each
xed w, 0 < jwj < ", the roots of P (z; w) are distinct.
(g)
Show that the results of Exercise 14(a)-(c) hold without the supposition that
the solutions of P (z; w) = 0 are distinct.

Solution

57
VIII.6.1
1 2 3 P L K

Sketch the closed path (t) = eit sin (2t), 0 t 2t, and determine the
winding number W ( ; ) for each point not on the path.

Solution

58
VIII.6.2
1 2 3 P L K

Sketch the closed path (t) = e 2it cos t, 0 t 2 , and determine the
winding number W ( ; ) for each point not on the path

Solution

59
VIII.6.3
1 2 3 P L K

Let f (z) be analytic on an open set containing a closed path , and suppose
f (z) 6= 0 on . Show that the increase in arg f (z) around is 2 W (f ; 0).

Solution

60
VIII.6.4
1 2 3 P L K

Let D be a domain, and suppose z0 and z1 lie in the same connected com-
ponent of CnD.
(a)
Show that the increase in the argument of f (z) = (z z0 ) (z z1 ) around
any closed curve in D is an even multiple of 2 .
(b)
Show that (z z0 ) (z z1 ) has an analytic square in D.
(c)
Show by example that (z z0 ) (z z1 ) does not unnecessarily have an ana-
lytic cube in D.

Solution

61
VIII.6.5
1 2 3 P L K

Show that if is a piecewise smooth closed curve in the complex plane, with
trace , and if z0 62 , then
Z
1
dz = 0; n 2:
(z z0 )n

Solution

62
VIII.6.6
1 2 3 P L K

Let be a closed path in a domain D such that W ( ; ) = 0 for all 62 D.


Suppose that f (z) is analytic on D except possibly at a nite number of
isolated singularities z1 ; : : : ; zm 2 Dn . Show that
Z X
f (z) dz = 2 i W ( ; zk ) Res [f; zk ] :

Hint. Consider the Laurent decomposition at each zk , and use Exercise 5.

Solution

63
VIII.6.7
1 2 3 P L K

Evaluate
Z
1 dz
;
2 i z (z 2 1)
where is the closed path indicated in the gure. Hint. Either use Exercise
6, or proceed directly with partial fractions.

64
VIII.6.8
1 2 3 P L K

Let (t) and (t), a t b, be closed paths.


(a)
Show that if 2 C does not lie on the straight line segment between (t)
and (t), for a t b, then W ( ; ) = W ( ; ).
(b)
Show that if j (t) (t)j < j (t)j for a a t b, then W ( ; ) =
W ( ; ).

Solution

65
VIII.6.9
1 2 3 P L K

Let f (z) be a continuous complex-valued function on the complex plane


such that f (z) is analytic for jzj < 1, f (z) 6= 0 for jzj 1, and f (z) ! 1 as
z ! 1. Show that f (z) 6= 0 for jzj < 1.

Solution

66
VIII.6.10
1 2 3 P L K

Let K be a nonempty closed bounded subset of the complex plane, and let
f (z) be a continuous complex-valued function on the complex plane that
is analytic on CnK and at 1. Show that every value attained by f (z) on
C = C [ f1g is attained by f (z) somewhere on K, that is f (C ) = f (K).

Solution

67
VIII.7.1
1 2 3 P L K

Let f (z) be an entire function, and suppose g ( ) is analytic for in the open
upper hand lower half-planes and across the interval ( 1; 1) on the real line.
Suppose that
Z 1
f (x)
dx = g ( )
1 z

for in the upper half-plane. What is the value of the integral when is in
the lower half-plane? Justify your answer carefully.

Solution

68
VIII.7.2
1 2 3 P L K

Show that
Z 1
dx 1
= Log ; 2 Cn [ 1; +1] :
1 x +1
(Note that we use the principal branch of the logarithm here.) Reconcile this
result with your solution to Exercise 1.

Solution

69
VIII.7.3
1 2 3 P L K

Find the Cauchy integrals of the following functions around the unit circle
= fjzj =g, positively oriented.
(a) z (b) z1 (c) x = Re z (d) y = Im (z)

Solution

70
VIII.7.4
1 2 3 P L K

Suppose f (z) is analytic on an annulus f < jzj < g, and let f (z) = f0 (z)+
f1 (z) be the Laurent decomposition of f (z). (See Section VI.1.) Fix r
between and , and let F ( ) be the Cauchy integral of f (z) around the
circle jzj = r. Show that f0 ( ) = F ( ) for j j < r, and f1 ( ) = F ( ) for
j j > r. Show further that f0 ( ) = F ( ) and f1 ( ) = F+ ( ). Remark
The formula f (z) = f0 (z) + f1 (z) reects the jump theorem for the Cauchy
integral of f (z) around circles jzj = r.

Solution

71
VIII.7.5
1 2 3 P L K

Let be a piecewise smooth curve, and let F ( ) be the Cauchy integral (7.1)
of a continuous function f (z) on . Show that if g (z) is a smooth function
on the complex plane that is zero o some bounded set, then
Z ZZ
@g
g (z) f (z) dz = 2i F (z) dx dy:
C @z

Hint. Recall Pompeius formula (Section IV.8).

72
VIII.7.6
1 2 3 P L K

Determine whether the point z lie inside or outside. Explain.

Solution

73
VIII.7.7
1 2 3 P L K

A simple arc in C is the image of a continuous one-to-one function (t)


from a closed interval [a; b] to the complex plane. Show that a simple arc
in C has a connected complement, that is Cn is connected. You may
use the Tietze extension theorem, that a continuous real-valued function on
a closed subset of the complex plane can be extended to a continuous real-
valued function on the entire complex plane. Hint. Suppose z0 belongs to
a bounded complement of Cn . Find at continuous determination h (z) of
log (z z0 ) on , extend h (z) to a continuous function on C , and dene
f (z) = z z0 on the component Cn containing z0 , and f (z) = eh(z) on the
remainder of C n . Consider the increase in the argument of f (z) around
circles centered at z0 .

Prove the Jordan curve theorem for a simple closed curve by lling in the
following proof outline.
(a)
Show that each component Cn has boundary . Hint. For z0 = (t0 ) 2 ,
apply the preceding exercise to the simple arc n (I), where I is a small
open parameter interval containing t0 .
(b)
Prove the Jordan curve theorem in the case where contains a straight line
segment.
(c)
Show that for any z0 = (t0 ) 2 , any small disk D0 containing z0 , and
component U of Cn , there are points z1 = (t1 ) and z2 = (t2 ) such that
the image of the parameter segment between t1 and t2 is contained in D0 and
such that z1 and z2 can be joined by a broken line segment in U \ D0 .
(d)
With notation as in (b), let be the simple curve obtained by replacing the
segment of in D0 between z0 and z1 by the broken line segment in U \ D0
between them, and let be the simple closed curve in D0 obtained by the
following the segment of in D0 from z0 to z1 and returning to z0 along the
broken line segment. Show that W ( ; ) = 0 and W ( ; ) = W ( ; ) for
2 Cn , 6= D0 .
(e)

74
Using (b) and (d), show that Cn has at least two components and that
W ( ; ) = 1 for in each bounded component of Cn .
(f)
By taking U in (c) to be a bounded component of Cn , show that W ( ; ) =
0 for in any other component of Cn .

Solution

75
VIII.8.1
1 2 3 P L K

Which of the following domains in C are simply connected? Justify your


answers. (a) D = fIm z > 0g n [0; i], the upper half-plane with a vertical
slit from 0 to i. (b) D = fIm z > 0g n [i; 2i], the upper half-plane with a
vertical slit from i to 2i. (c) D = Cn [0; +1], the complex plane slit along
the positive real axis. (d) D = Cn [ 1; 1], the complex plane with an interval
deleted.

Solution

76
VIII.8.2
1 2 3 P L K

Show that a domain D in the extended complex plane C = C [ f1g is


simply connected if and only if its complement C nD is connected. Hint. If
D 6= C , move a point in the complement of D to 1. If D = C , rst deform
a given closed path to one that does not cover the sphere, then deform it to
a point by pulling along arcs of great circles.

Solution

77
VIII.8.3
1 2 3 P L K

Which of the following domains in C are simply connected? Justify your


answers. (a) D = C n [ 1; 1], the extended plane with an interval deleted,
(b) D = C n f 1; 0; 1g, the thrice-punctured sphere.

Solution

78
VIII.8.4
1 2 3 P L K

Show that a domain D in the complex plane is simply connected if and only
if any analytic function f (z) on D does not vanish at any point of D has an
analytic logarithm on D. Hint if f (z) 6= 0 on D, consider the function
Z z 0
f (z)
G (z) = dw:
z0 f (w)

Solution

79
VIII.8.5
1 2 3 P L K

Show that a domain D is simply connected if and only if any analytic function
f (z) on D that does not vanish at any point of D has an analytic square
root on D. Show that this occurs if and only if for any point z0 62 D the
function z z0 has an analytic square root on D.

Solution

80
VIII.8.6
1 2 3 P L K

Show that a domain D is simply connected if and only if each continuous


function f (z) on D that does not vanish at any point D has continuous
logarithm on D.

Solution

81
VIII.8.7
1 2 3 P L K

Let E be a closed connected subset of the extended complex plane C . Show


that each connected component C nE is simply connected.

Solution

82
VIII.8.8
1 2 3 P L K

Show that simple connectivity is a "topological property" that is, if U and


V are domains, and ' is a continuous map of U onto V such that ' 1 is also
continuous, then U is simply connected if and only if V is simply connected.

Solution

83
VIII.8.9
1 2 3 P L K

Suppose that f (z) is analytic on a domain D, and f 0 (z) has no zeros on D.


Suppose also that f (D) is simply connected, and that there is a branch g (w)
of f 1 that is analytic on w0 = f (z0 ) and that can be continued analytically
along any path in f (D) starting at w0 . Show that f (z) is one-to-one on D.

Solution

84
VIII.8.10
1 2 3 P L K

We
P dene an integral 1 cycle in D to be an expression of the form =
kj j , where 1 ; : : : ; m are closed paths in D and k1 ; : : : ; km are integers.
P
We dene the winding number of about to be W ( ; ) = kj W j ; ,
2 CnD. Show that if h ( ) is a continuous integer-valued function on
C nD such that h (1) = 0, then there is an integral 1-cycle on D such
that W ( ; ) = h ( ) for all 2 CnD.

Solution

85
VIII.8.11
1 2 3 P L K

An integral 1-cycle is homologous to zero i D if W ( ; ) = 0 whenever


62 D. Let U be a bounded domain whose boundary consists of a nite
number of piecewise smooth closed curves 1 ; : : : ; m , oriented positively with
respect to U , such that P
U together with its boundary is contained in D. Show
that the 1-cycle @U = j is homologous to zero in D.

Solution

86
VIII.8.12
1 2 3 P L K

Let D be a domain in C such that C nD consists of m + 1 disjoint closed con-


nected sets. Show that there are m piecewise smooth closed curves 1 ; : : : ; m
such thatPevery integral 1-cycle can be expressed uniquely in the form
= 0 + kj j , where the kj s are integers and 0 is homologous to zero in
D. Remark. The j s form a homology basis for D.

Solution

87

S-ar putea să vă placă și